modular arithmetic

45
Modular Arithmetic Dec 28

Upload: cody-donaldson

Post on 31-Dec-2015

72 views

Category:

Documents


0 download

DESCRIPTION

Modular Arithmetic. Dec 28. This Lecture. Basic rule of modular addition and modular multiplication. The Quotient-Remainder Theorem. For b > 0 and any a , there are unique numbers q ::= quotient( a , b ), r ::= remainder( a , b ), such that a = qb + r and 0  r < b. - PowerPoint PPT Presentation

TRANSCRIPT

Page 1: Modular Arithmetic

Modular Arithmetic

Dec 28

This Lecture

bull Basic rule of modular addition and modular multiplication

For b gt 0 and any a there are unique numbers

q = quotient(ab) r = remainder(ab) such

that

a = qb + r and 0 r lt b

0 b 2b kb (k+1)b

Given any b we can divide the integers into many blocks of b numbers

For any a there is a unique ldquopositionrdquo for a in this line

q = the block where a is in

a

r = the offset in this block

Clearly given a and b q and r are uniquely defined

-b

The Quotient-Remainder Theorem

Def a b (mod n) iff n|(a - b) iff a mod n = b mod n

Modular Arithmetic

eg 12 2 (mod 10)

107 207 (mod 10)

7 3 (mod 2)

7 -1 (mod 2)

13 -1 (mod 7)

-15 10 (mod 5)

Be careful a mod n means ldquothe remainder when a is divided by nrdquo

a b (mod n) means ldquoa and b have the same remainder when divided by nrdquo

12 mod 10 = 2

207 mod 10 = 7

7 mod 2 = 1

-1 mod 2 = 1

-1 mod 7 = 6

-15 mod 5 = 0

Fact a a mod n (mod n) as a and a mod n have the same remainder mod n

Fact if a b (mod n) then a = b + nx for some integer x

Lemma If a c (mod n) and b d (mod n) then

a+b c+d (mod n)

Modular Addition

When you try to understand a statement like this

first think about the familiar cases eg n=10 or n=2

When n=2 it says that if a and c have the same parity

and b and d have the same parity

then a+b and c+d have the same parity

When n=10 it says that if a and c have the same last digit

and b and d have the same last digit

then a+b and c+d have the same last digit

And the lemma says that the same principle applied for all n

Lemma If a c (mod n) and b d (mod n) then

a+b c+d (mod n)

Modular Addition

Example 1 13 1 (mod 3) 25 1 (mod 3)

=gt 12 + 25 (mod 3) 1 + 1 (mod 3) 2 (mod 3)

Example 2 87 2 (mod 17) 222 1 (mod 17)

=gt 87 + 222 (mod 17) 2 + 1 (mod 17) 3 (mod 17)

Example 3 101 2 (mod 11) 141 -2 (mod 11)

=gt 101 + 141 (mod 11) 0 (mod 11)

In particular when computing a+b mod n we can first replace

a by a mod n and b by b mod n so that the computation is faster

Lemma If a c (mod n) and b d (mod n) then

a+b c+d (mod n)

Modular Addition

a c (mod n) =gt a = c + nx for some integer x

b d (mod n) =gt b = d + ny for some integer y

To show a+b c+d (mod n) it is equivalent to showing that n | (a+b-c-d)

Consider a+b-c-d

a+b-c-d = (c+nx) + (d+ny) ndash c ndashd = nx + ny

It is clear that n | nx + ny

Therefore n | a+b-c-d

We conclude that a+b c+d (mod n)

Proof

Lemma If a c (mod n) and b d (mod n) then

ab cd (mod n)

Modular Multiplication

Example 1 9876 6 (mod 10) 17642 2 (mod 10)

=gt 9876 17642 (mod 10) 6 2 (mod 10) 2 (mod 10)

Example 2 10987 1 (mod 2) 28663 1 (mod 2)

=gt 10987 28663 (mod 2) 1 (mod 2)

Example 3 1000 -1 (mod 7) 1000000 1 (mod 7)

=gt 1000 1000000 (mod 7) -1 1 (mod 7) -1 (mod 7)

In particular when computing ab mod n we can first replace

a by a mod n and b by b mod n so that the computation is faster

Lemma If a c (mod n) and b d (mod n) then

ab cd (mod n)

Modular Multiplication

a c (mod n) =gt a = c + nx for some integer x

b d (mod n) =gt b = d + ny for some integer y

To show ab cd (mod n) it is equivalent to showing that n | (ab-cd)

Consider ab-cd

ab-cd = (c+nx) (d+ny) ndash cd

= cd + dnx + cny + n2xy ndash cd = n(dx + cy + nxy)

It is clear that n | n(dx + cy + nxy) Therefore n | ab-cd

We conclude that ab cd (mod n)

Proof

This Lecture

bull Applications Fast exponentiation and fast division test

Fast Exponentiation

1444 mod 713

= 144 144 144 144 mod 713

= 20736 144 144 mod 713

= 59 144 144 mod 713

= 8496 144 mod 713

= 653 144 mod 713

= 94032 mod 713

= 629 mod 713

20736 20736 mod 713

= 59 59 mod 713

= 3481 mod 713

= 629 mod 713

Because 20736 59 (mod 713)

Because 653 8496 (mod 713)

shortcut

Repeated Squaring

14450 mod 713

= 14432 14416 1442 mod 713

= 64848559 mod 713

= 242

1442 mod 713 = 59

1444 mod 713 = 1442 1442 mod 713= 5959 mod 713= 629

1448 mod 713= 14441444 mod 713= 629629 mod 713= 639

14416 mod 713= 14481448 mod 713= 639639 mod 713= 485

14432 mod 713= 1441614416 mod 713= 485485 mod 713= 648

Note that 50 = 32 + 16 + 2

Fast Division Test

Using the basic rules for modular addition and modular multiplication

we can derive some quick test to see if a big number is divisible by a small number

Suppose we are given the decimal representation of a big number N

To test if N is divisible by a small number n of course we can do a division to check

But can we do faster

If n = 2 we just need to check whether the last digit of N is even or not

If n = 10 we just need to check whether the last digit of N is 0 or not

If n = 5 we just need to check whether the last digit of N is either 5 or 0 or not

What about when n=3 When n=7 When n=11

Fast Division Test

A number written in decimal divisible by 9 if and only if

the sum of its digits is a multiple of 9

Example 1 9333234513171 is divisible by 9

9+3+3+3+2+3+4+5+1+3+1+7+1 = 45 is divisible by 9

Example 2 128573649683 is not divisible by 9

1+2+8+5+7+3+6+4+9+6+8+3 = 62 is not divisible by 9

Claim A number written in decimal is divisible by 9 if and

only if

the sum of its digits is a multiple of 9Hint 10 1 (mod 9)

Let the decimal representation of N be dkdk-1dk-2hellipd1d0

This means that N = dk10k + dk-110k-1 + hellip + d110 + d0

Note that di10i mod 9

= (di) (10i mod 9) mod 9

= (di) (10 mod 9) (10 mod 9) hellip (10 mod 9) mod 9

= (di) (1 mod 9) (1 mod 9) hellip (1 mod 9) mod 9

= di mod 9

i terms

Fast Division Test

Rule of modular multiplication

Let the decimal representation of n be dkdk-1dk-2hellipd1d0

This means that N = dk10k + dk-110k-1 + hellip + d110 + d0

Note that di10i mod 9 = di mod 9

Hence N mod 9 = (dk10k + dk-110k-1 + hellip + d110 + d0) mod 9

= (dk10k mod 9 + dk-110k-1 mod 9 + hellip + d110 mod 9 + d0 mod 9) mod

9

= (dk mod 9 + dk-1 mod 9 + hellip + d1 mod 9 + d0 mod 9) mod 9

= (dk + dk-1 + hellip + d1 + d0) mod 9

Hint 10 1 (mod 9)

Fast Division Test

Claim A number written in decimal is divisible by 9 if and

only if

the sum of its digits is a multiple of 9

Rule of modular addition

By previous slide

The same procedure works to test whether N is divisible by n=3

Fast Division Test

What about n=11

Hint 10 -1 (mod 11)

Let the decimal representation of N be d92d91d90hellipd1d0

Then N is divisible by 11 if and only if

d92-d91+d90hellip-d1+d0 is divisible by 11

Why Try to work it out before your TA shows you

What about n=7

Hint 1000 -1 (mod 7)

This Lecture

bull Multiplicative inverse

The multiplicative inverse of a number a is another number arsquo such that

a middot arsquo 1 (mod n)

Multiplication Inverse

Does every number has a multiplicative inverse in modular arithmetic

For real numbers every nonzero number has a multiplicative inverse

For integers only 1 has a multiplicative inverse

An interesting property of modular arithmetic is that

there are multiplicative inverse for integers

For example 2 5 = 1 mod 3 so 5 is a multiplicative inverse

for 2 under modulo 3 (and vice versa)

Multiplication Inverse

Does every number has a multiplicative inverse in modular arithmetic

Multiplication Inverse

What is the pattern

Case Study

Why 2 does not have a multiplicative inverse under modulo 6

Suppose it has a multiplicative inverse y

2y 1 (mod 6)

=gt 2y = 1 + 6x for some integer x

=gt y = frac12 + 3x

This is a contradiction since both x and y are integers

Necessary Condition

Claim An integer k does not have an multiplicative inverse under modulo n

if k and n have a common factor gt= 2 (gcd(kn) gt= 2)

Proof

This claim says that for k to have a multiplicative inverse modulo n

then a necessary condition is that k and n do not have a common factor gt= 2

Suppose by contradiction that there is an inverse krsquo for k such that

krsquok = 1 (mod n)

Then krsquok = 1 + xn for some integer x

Since both k and n have a common factor say cgt=2

then k=ck1 and n=cn1 for some integers k1 and n1

So krsquock1 = 1 + xcn1

Then krsquok1 = 1c + xn1

This is a contradiction since the LHS is an integer but the RHS is not

Sufficient Condition

What about if gcd(kn)=1

Would k always have an multiplicative inverse under modulo n

For example gcd(37) = 1 3middot5 1 (mod 7)

gcd(89) = 1

gcd(411) = 1 4middot3 1 (mod 11)

8middot8 1 (mod 9)

It seems that there is always an inverse in such a case but why

gcd(89) = 1 8s + 9t = 1 for some integers s and t

8s = 1 ndash 9t

8s 1 (mod 9)gcd(89) = spc(89)

Theorem If gcd(kn)=1 then have krsquo such that

kmiddotkrsquo 1 (mod n)

Proof Since gcd(kn)=1 there exist s and t so that sk + tn = 1

So tn = 1 - sk

This means n | 1 ndash sk

This means that 1 ndash sk 0 (mod n)

This means that 1 sk (mod n)

So krsquo = s is an multiplicative inverse for k

Sufficient Condition

gcd(kn)=spc(kn)

The multiplicative inverse can be computed by the extended Euclidean algorithm

Corollary k has a multiplicative inverse mod n if and only if gcd(kn)=1

This Lecture

bull Fermatrsquos little theorem

Cancellation

There is no general cancellation in modular arithmetic

Note that (mod n) is very similar to =

If a b (mod n) then a+c b+c (mod n)

If a b (mod n) then ac bc (mod n)

However if ac bc (mod n)

it is not necessarily true that a b (mod n)

For example 4middot2 1middot2 (mod 6) but 4 1 (mod 6)

3middot4 1middot4 (mod 8) but 3 1 (mod 8)

4middot3 1middot3 (mod 9) but 4 1 (mod 9)

Observation In all the above examples c and n have a common factor

Cancellation

Claim Assume gcd(kn) = 1 If imiddotk jmiddotk (mod n) then i j (mod n)

For example multiplicative inverse always exists if n is a prime

Proof Since gcd(kn) = 1 there exists krsquo such that kkrsquo 1 (mod n)

imiddotk jmiddotk (mod n)

=gt imiddotkmiddotkrsquo jmiddotkmiddotkrsquo (mod n)

=gt i j (mod n)

Remarks (Optional) This makes arithmetic modulo prime a field

a structure that ldquobehaves likerdquo real numbers

Arithmetic modulo prime is very useful in coding theory

In particular when p is a prime amp k not a multiple of p then gcd(kp)=1

If i j (mod p) then imiddotk jmiddotk (mod p)

Therefore

k mod p 2k mod p hellip (p-1)k mod p

are all different numbers

Fermatrsquos Little Theorem

Claim 1 Assume gcd(kn) = 1 If imiddotk jmiddotk (mod n) then i j (mod n)

Claim 2 Assume gcd(kn) = 1 If i j (mod n) then imiddotk jmiddotk (mod n)

For example when p=7 and k=3

3 mod 7 = 3 2middot3 mod 7 = 6 3middot3 mod 7 = 2 4middot3 mod 7 = 5 5middot3 mod 7 = 1 6middot3 mod 7 = 4

Notice that in the above example every number from 1 to 6 appears exactly once

In particular when p is a prime amp k not a multiple of p then gcd(kp)=1

If i j (mod p) then imiddotk jmiddotk (mod p)

Therefore

k mod p 2k mod p hellip (p-1)k mod p

are all different numbers

Fermatrsquos Little Theorem

Each of ik mod p cannot be equal to 0 because p is a prime number

Let ci = ik mod p

So 1 lt= c1 lt= p-1 1 lt= c2 lt= p-1 hellip 1lt = cp-1 lt= p-1

By the above we know that c1c2hellipcp-2cp-1 are all different

So for each i from 1 to p-1 there is exactly one cj such that cj = i

Therefore we have

(k mod p)middot(2k mod p)middothellipmiddot((p-1)k mod p) = c1middotc2middothellipmiddotcp-2middotcp-1 = 1middot2middot3hellipmiddot(p-2)middot(p-1)

Fermatrsquos Little Theorem

1 kp-1 (mod p)

Theorem If p is prime amp k not a multiple of p

For example when p=5 k=4 we have kp-1 mod p = 44 mod 5 = 1

ldquoProofrdquo

4middot3middot2middot1 [(4 mod 5) (2middot4 mod 5) (3middot4 mod 5) (4middot4 mod 5)] (mod 5)

[4 middot (2middot4) middot (3middot4) middot (4middot4)] (mod 5)

[44 middot (1middot2middot3middot4)] (mod 5)

Since gcd(1middot2middot3middot4 5)=1 we can cancel 1middot2middot3middot4 on both sides

This implies

1 44 (mod 5)

By the previous slide or direct calculation

Fermatrsquos Little Theorem

1 kp-1 (mod p)

Theorem If p is prime amp k not a multiple of p

Proof

1middot2middotmiddotmiddot(p-1) (k mod p middot 2k mod pmiddothellipmiddot(p-1)k mod p) mod p

(kmiddot2k middotmiddotmiddot (p-1)k) mod p

(kp-1)middot1middot2 middotmiddotmiddot (p-1) (mod p)

So by cancelling 1middot2 middotmiddotmiddot (p-1) on both sides applying

Claim 1

(we can cancel them because gcd(1middot2 middotmiddotmiddot (p-1) p)=1)

we have

1 kp-1 (mod p)

By 2 slides before

By the multiplication rule

Wilsonrsquos Theorem

Theorem p is a prime if and only if

(p-1) -1 (mod p)

First we consider the easy direction

If p is not a prime assume p gt= 5 (for p=4 3 2 (mod 4) )

Then p=qr for some 2 lt= q lt p and 2 lt= r lt p

If q ne r then both q and r appear in (p-1)

and so (p-1) 0 (mod p)

If q = r then p = q2 gt 2q (since we assume p gt 5 and thus q gt 2)

then both q and 2q are in (p-1)

and so again (p-1) 0 (mod p)

Wilsonrsquos Theorem

Theorem p is a prime if and only if

(p-1) -1 (mod p)

To prove the more interesting direction first we need a lemma

Lemma If p is a prime number

x2 1 (mod p) if and only if x 1 (mod p) or x -1 (mod p)

Proof x2 1 (mod p)

iff p | x2 - 1

iff p | (x ndash 1)(x + 1)

iff p | (x ndash 1) or p | (x+1)

iff x 1 (mod p) or x -1 (mod p)

Lemma p prime and p|amiddotb iff p|a or p|b

Wilsonrsquos Theorem

Theorem p is a prime if and only if

(p-1) -1 (mod p)

Letrsquos get the proof idea by considering a concrete example

10

1middot2middot3middot4middot5middot6middot7middot8middot9middot10 mod 11

1middot10middot(2middot6)middot(3middot4)middot(5middot9)middot(7middot8) mod 11

1middot-1middot(1)middot(1)middot(1)middot(1) mod 11

-1 mod 11

Besides 1 and 10 the remaining numbers are paired up into multiplicative inverse

Wilsonrsquos Theorem

Theorem p is a prime if and only if

(p-1) -1 (mod p)

ProofSince p is a prime every number from 1 to p-1 has a multiplicative inverse

By the Lemma every number 2 lt= k lt= p-2 has an inverse krsquo with knekrsquo

Since p is odd the numbers from 2 to p-2 can be grouped into pairs

(a1b1)(a2b2)hellip(a(p-3)2b(p-3)2) so that aibi 1 (mod p)

Therefore (p-1) 1middot(p-1)middot2middot3middotmiddotmiddotmiddotmiddot(p-3)middot(p-2) (mod p)

1middot(p-1)middot(a1b1)middot(a2b2)middotmiddotmiddotmiddotmiddot(a(p-3)2b(p-3)2) (mod p)

1middot(-1)middot(1)middot(1)middotmiddotmiddotmiddotmiddot(1) (mod p)

-1 (mod p)

This Lecture

bull Eulerrsquos phi function

Inclusion-Exclusion (n sets)

What is the inclusion-exclusion formula for the union of n sets

sum of sizes of all single setsndash sum of sizes of all 2-set intersections+ sum of sizes of all 3-set intersectionsndash sum of sizes of all 4-set intersectionshellip+ (ndash1)n+1 times sum of sizes of intersections of all n

sets

1 2 nA A A

1

12 1

( 1)n

ki

S nk i S

S k

A

Inclusion-Exclusion (n sets)

Inclusion-Exclusion (n sets)

sum of sizes of all single setsndash sum of sizes of all 2-set intersections+ sum of sizes of all 3-set intersectionsndash sum of sizes of all 4-set intersectionshellip+ (ndash1)n+1 times sum of sizes of intersections of all n

sets

|A1 [ A2 [ A3 [ hellip [ An|

We want to show that every element is counted exactly once

Consider an element which belongs to exactly k sets say A1 A2 A3 hellip Ak

In the formula such an element is counted the following number of times

Therefore each element is counted exactly once and thus the formula is correct

Euler Function

Given a number n how many numbers from 1 to n are relatively prime to n

When n is a prime number

then every number from 1 to n-1 is relatively prime to n

and so

When n is a prime power

then p 2p 3p 4p hellip n are not relatively prime to n

there are np = pc-1 of them

and other numbers are relatively prime to n

Therefore

Euler Function

Given a number n how many numbers from 1 to n are relatively prime to n

Suppose

Then p 2p 3p 4p hellip n are not relatively prime to n there are np of them

Also q 2q 3q 4q hellip n are not relatively prime to n and there are nq of them

Other numbers are relatively prime to n

Therefore

The numbers pq 2pq 3pq hellip n are subtracted twice and there are npq of them

So the correct answer is

Euler Function

Given a number n how many numbers from 1 to n are relatively prime to n

Let

Let S be the set of numbers from 1 to n that are not relatively prime to n

Let Ai be the set of numbers that are a multiple of piS = A1 [ A2 [ hellip [ An

For the intersection of k sets say A1 A2 A3hellip Ak

then every number in A1 Aring A2 Aring hellip Aring Ak is a multiple of p1p2hellippk

then |A1 Aring A2 Aring hellip Aring Ak| = np1p2hellippk

Euler Function

Given a number n how many numbers from 1 to n are relatively prime to n

Let

Let S be the set of numbers from 1 to n that are not relatively prime to n

Let Ai be the set of numbers that are a multiple of piS = A1 [ A2 [ hellip [ An

|A1 Aring A2 Aring hellip Aring Ak| = np1p2hellippk

When r=3 (only 3 distinct factors)

|A1 [ A2 [ A3|

= np1 + np2 + np3

- np1p2 ndash np1p3 ndash np2p3 + np1p2p3

|A1 [ A2 [ A3| = |A1| + |A2| + |A3|

ndash |A1 Aring A2| ndash |A1 Aring A3| ndash |A2 Aring A3|

+ |A1 Aring A2 Aring A3|

= n(1-p1)(1-p2)(1-p3)

Euler Function

Given a number n how many numbers from 1 to n are relatively prime to n

Let

Let S be the set of numbers from 1 to n that are not relatively prime to n

Let Ai be the set of numbers that are a multiple of piS = A1 [ A2 [ hellip [ An

|A1 Aring A2 Aring hellip Aring Ak| = np1p2hellippk

sum of sizes of all single setsndash sum of sizes of all 2-set intersections+ sum of sizes of all 3-set intersectionsndash sum of sizes of all 4-set intersectionshellip+ (ndash1)n+1 times sum of sizes of intersections of

n sets

|A1 [ A2 [ A3 [ hellip [ An|

|S| = |A1 [ A2 [ hellip [ An|

= n(1-p1)(1-p2)hellip(1-pn)

calculationshellip

  • Modular Arithmetic
  • PowerPoint Presentation
  • Slide 3
  • Slide 4
  • Slide 5
  • Slide 6
  • Slide 7
  • Slide 8
  • Slide 9
  • Slide 10
  • Slide 11
  • Slide 12
  • Slide 13
  • Slide 14
  • Slide 15
  • Slide 16
  • Slide 17
  • Slide 18
  • Slide 19
  • Slide 20
  • Slide 21
  • Slide 22
  • Slide 23
  • Slide 24
  • Slide 25
  • Slide 26
  • Slide 27
  • Slide 28
  • Slide 29
  • Slide 30
  • Slide 31
  • Slide 32
  • Slide 33
  • Slide 34
  • Slide 35
  • Slide 36
  • Slide 37
  • Slide 38
  • Slide 39
  • Slide 40
  • Slide 41
  • Slide 42
  • Slide 43
  • Slide 44
  • Slide 45
Page 2: Modular Arithmetic

This Lecture

bull Basic rule of modular addition and modular multiplication

For b gt 0 and any a there are unique numbers

q = quotient(ab) r = remainder(ab) such

that

a = qb + r and 0 r lt b

0 b 2b kb (k+1)b

Given any b we can divide the integers into many blocks of b numbers

For any a there is a unique ldquopositionrdquo for a in this line

q = the block where a is in

a

r = the offset in this block

Clearly given a and b q and r are uniquely defined

-b

The Quotient-Remainder Theorem

Def a b (mod n) iff n|(a - b) iff a mod n = b mod n

Modular Arithmetic

eg 12 2 (mod 10)

107 207 (mod 10)

7 3 (mod 2)

7 -1 (mod 2)

13 -1 (mod 7)

-15 10 (mod 5)

Be careful a mod n means ldquothe remainder when a is divided by nrdquo

a b (mod n) means ldquoa and b have the same remainder when divided by nrdquo

12 mod 10 = 2

207 mod 10 = 7

7 mod 2 = 1

-1 mod 2 = 1

-1 mod 7 = 6

-15 mod 5 = 0

Fact a a mod n (mod n) as a and a mod n have the same remainder mod n

Fact if a b (mod n) then a = b + nx for some integer x

Lemma If a c (mod n) and b d (mod n) then

a+b c+d (mod n)

Modular Addition

When you try to understand a statement like this

first think about the familiar cases eg n=10 or n=2

When n=2 it says that if a and c have the same parity

and b and d have the same parity

then a+b and c+d have the same parity

When n=10 it says that if a and c have the same last digit

and b and d have the same last digit

then a+b and c+d have the same last digit

And the lemma says that the same principle applied for all n

Lemma If a c (mod n) and b d (mod n) then

a+b c+d (mod n)

Modular Addition

Example 1 13 1 (mod 3) 25 1 (mod 3)

=gt 12 + 25 (mod 3) 1 + 1 (mod 3) 2 (mod 3)

Example 2 87 2 (mod 17) 222 1 (mod 17)

=gt 87 + 222 (mod 17) 2 + 1 (mod 17) 3 (mod 17)

Example 3 101 2 (mod 11) 141 -2 (mod 11)

=gt 101 + 141 (mod 11) 0 (mod 11)

In particular when computing a+b mod n we can first replace

a by a mod n and b by b mod n so that the computation is faster

Lemma If a c (mod n) and b d (mod n) then

a+b c+d (mod n)

Modular Addition

a c (mod n) =gt a = c + nx for some integer x

b d (mod n) =gt b = d + ny for some integer y

To show a+b c+d (mod n) it is equivalent to showing that n | (a+b-c-d)

Consider a+b-c-d

a+b-c-d = (c+nx) + (d+ny) ndash c ndashd = nx + ny

It is clear that n | nx + ny

Therefore n | a+b-c-d

We conclude that a+b c+d (mod n)

Proof

Lemma If a c (mod n) and b d (mod n) then

ab cd (mod n)

Modular Multiplication

Example 1 9876 6 (mod 10) 17642 2 (mod 10)

=gt 9876 17642 (mod 10) 6 2 (mod 10) 2 (mod 10)

Example 2 10987 1 (mod 2) 28663 1 (mod 2)

=gt 10987 28663 (mod 2) 1 (mod 2)

Example 3 1000 -1 (mod 7) 1000000 1 (mod 7)

=gt 1000 1000000 (mod 7) -1 1 (mod 7) -1 (mod 7)

In particular when computing ab mod n we can first replace

a by a mod n and b by b mod n so that the computation is faster

Lemma If a c (mod n) and b d (mod n) then

ab cd (mod n)

Modular Multiplication

a c (mod n) =gt a = c + nx for some integer x

b d (mod n) =gt b = d + ny for some integer y

To show ab cd (mod n) it is equivalent to showing that n | (ab-cd)

Consider ab-cd

ab-cd = (c+nx) (d+ny) ndash cd

= cd + dnx + cny + n2xy ndash cd = n(dx + cy + nxy)

It is clear that n | n(dx + cy + nxy) Therefore n | ab-cd

We conclude that ab cd (mod n)

Proof

This Lecture

bull Applications Fast exponentiation and fast division test

Fast Exponentiation

1444 mod 713

= 144 144 144 144 mod 713

= 20736 144 144 mod 713

= 59 144 144 mod 713

= 8496 144 mod 713

= 653 144 mod 713

= 94032 mod 713

= 629 mod 713

20736 20736 mod 713

= 59 59 mod 713

= 3481 mod 713

= 629 mod 713

Because 20736 59 (mod 713)

Because 653 8496 (mod 713)

shortcut

Repeated Squaring

14450 mod 713

= 14432 14416 1442 mod 713

= 64848559 mod 713

= 242

1442 mod 713 = 59

1444 mod 713 = 1442 1442 mod 713= 5959 mod 713= 629

1448 mod 713= 14441444 mod 713= 629629 mod 713= 639

14416 mod 713= 14481448 mod 713= 639639 mod 713= 485

14432 mod 713= 1441614416 mod 713= 485485 mod 713= 648

Note that 50 = 32 + 16 + 2

Fast Division Test

Using the basic rules for modular addition and modular multiplication

we can derive some quick test to see if a big number is divisible by a small number

Suppose we are given the decimal representation of a big number N

To test if N is divisible by a small number n of course we can do a division to check

But can we do faster

If n = 2 we just need to check whether the last digit of N is even or not

If n = 10 we just need to check whether the last digit of N is 0 or not

If n = 5 we just need to check whether the last digit of N is either 5 or 0 or not

What about when n=3 When n=7 When n=11

Fast Division Test

A number written in decimal divisible by 9 if and only if

the sum of its digits is a multiple of 9

Example 1 9333234513171 is divisible by 9

9+3+3+3+2+3+4+5+1+3+1+7+1 = 45 is divisible by 9

Example 2 128573649683 is not divisible by 9

1+2+8+5+7+3+6+4+9+6+8+3 = 62 is not divisible by 9

Claim A number written in decimal is divisible by 9 if and

only if

the sum of its digits is a multiple of 9Hint 10 1 (mod 9)

Let the decimal representation of N be dkdk-1dk-2hellipd1d0

This means that N = dk10k + dk-110k-1 + hellip + d110 + d0

Note that di10i mod 9

= (di) (10i mod 9) mod 9

= (di) (10 mod 9) (10 mod 9) hellip (10 mod 9) mod 9

= (di) (1 mod 9) (1 mod 9) hellip (1 mod 9) mod 9

= di mod 9

i terms

Fast Division Test

Rule of modular multiplication

Let the decimal representation of n be dkdk-1dk-2hellipd1d0

This means that N = dk10k + dk-110k-1 + hellip + d110 + d0

Note that di10i mod 9 = di mod 9

Hence N mod 9 = (dk10k + dk-110k-1 + hellip + d110 + d0) mod 9

= (dk10k mod 9 + dk-110k-1 mod 9 + hellip + d110 mod 9 + d0 mod 9) mod

9

= (dk mod 9 + dk-1 mod 9 + hellip + d1 mod 9 + d0 mod 9) mod 9

= (dk + dk-1 + hellip + d1 + d0) mod 9

Hint 10 1 (mod 9)

Fast Division Test

Claim A number written in decimal is divisible by 9 if and

only if

the sum of its digits is a multiple of 9

Rule of modular addition

By previous slide

The same procedure works to test whether N is divisible by n=3

Fast Division Test

What about n=11

Hint 10 -1 (mod 11)

Let the decimal representation of N be d92d91d90hellipd1d0

Then N is divisible by 11 if and only if

d92-d91+d90hellip-d1+d0 is divisible by 11

Why Try to work it out before your TA shows you

What about n=7

Hint 1000 -1 (mod 7)

This Lecture

bull Multiplicative inverse

The multiplicative inverse of a number a is another number arsquo such that

a middot arsquo 1 (mod n)

Multiplication Inverse

Does every number has a multiplicative inverse in modular arithmetic

For real numbers every nonzero number has a multiplicative inverse

For integers only 1 has a multiplicative inverse

An interesting property of modular arithmetic is that

there are multiplicative inverse for integers

For example 2 5 = 1 mod 3 so 5 is a multiplicative inverse

for 2 under modulo 3 (and vice versa)

Multiplication Inverse

Does every number has a multiplicative inverse in modular arithmetic

Multiplication Inverse

What is the pattern

Case Study

Why 2 does not have a multiplicative inverse under modulo 6

Suppose it has a multiplicative inverse y

2y 1 (mod 6)

=gt 2y = 1 + 6x for some integer x

=gt y = frac12 + 3x

This is a contradiction since both x and y are integers

Necessary Condition

Claim An integer k does not have an multiplicative inverse under modulo n

if k and n have a common factor gt= 2 (gcd(kn) gt= 2)

Proof

This claim says that for k to have a multiplicative inverse modulo n

then a necessary condition is that k and n do not have a common factor gt= 2

Suppose by contradiction that there is an inverse krsquo for k such that

krsquok = 1 (mod n)

Then krsquok = 1 + xn for some integer x

Since both k and n have a common factor say cgt=2

then k=ck1 and n=cn1 for some integers k1 and n1

So krsquock1 = 1 + xcn1

Then krsquok1 = 1c + xn1

This is a contradiction since the LHS is an integer but the RHS is not

Sufficient Condition

What about if gcd(kn)=1

Would k always have an multiplicative inverse under modulo n

For example gcd(37) = 1 3middot5 1 (mod 7)

gcd(89) = 1

gcd(411) = 1 4middot3 1 (mod 11)

8middot8 1 (mod 9)

It seems that there is always an inverse in such a case but why

gcd(89) = 1 8s + 9t = 1 for some integers s and t

8s = 1 ndash 9t

8s 1 (mod 9)gcd(89) = spc(89)

Theorem If gcd(kn)=1 then have krsquo such that

kmiddotkrsquo 1 (mod n)

Proof Since gcd(kn)=1 there exist s and t so that sk + tn = 1

So tn = 1 - sk

This means n | 1 ndash sk

This means that 1 ndash sk 0 (mod n)

This means that 1 sk (mod n)

So krsquo = s is an multiplicative inverse for k

Sufficient Condition

gcd(kn)=spc(kn)

The multiplicative inverse can be computed by the extended Euclidean algorithm

Corollary k has a multiplicative inverse mod n if and only if gcd(kn)=1

This Lecture

bull Fermatrsquos little theorem

Cancellation

There is no general cancellation in modular arithmetic

Note that (mod n) is very similar to =

If a b (mod n) then a+c b+c (mod n)

If a b (mod n) then ac bc (mod n)

However if ac bc (mod n)

it is not necessarily true that a b (mod n)

For example 4middot2 1middot2 (mod 6) but 4 1 (mod 6)

3middot4 1middot4 (mod 8) but 3 1 (mod 8)

4middot3 1middot3 (mod 9) but 4 1 (mod 9)

Observation In all the above examples c and n have a common factor

Cancellation

Claim Assume gcd(kn) = 1 If imiddotk jmiddotk (mod n) then i j (mod n)

For example multiplicative inverse always exists if n is a prime

Proof Since gcd(kn) = 1 there exists krsquo such that kkrsquo 1 (mod n)

imiddotk jmiddotk (mod n)

=gt imiddotkmiddotkrsquo jmiddotkmiddotkrsquo (mod n)

=gt i j (mod n)

Remarks (Optional) This makes arithmetic modulo prime a field

a structure that ldquobehaves likerdquo real numbers

Arithmetic modulo prime is very useful in coding theory

In particular when p is a prime amp k not a multiple of p then gcd(kp)=1

If i j (mod p) then imiddotk jmiddotk (mod p)

Therefore

k mod p 2k mod p hellip (p-1)k mod p

are all different numbers

Fermatrsquos Little Theorem

Claim 1 Assume gcd(kn) = 1 If imiddotk jmiddotk (mod n) then i j (mod n)

Claim 2 Assume gcd(kn) = 1 If i j (mod n) then imiddotk jmiddotk (mod n)

For example when p=7 and k=3

3 mod 7 = 3 2middot3 mod 7 = 6 3middot3 mod 7 = 2 4middot3 mod 7 = 5 5middot3 mod 7 = 1 6middot3 mod 7 = 4

Notice that in the above example every number from 1 to 6 appears exactly once

In particular when p is a prime amp k not a multiple of p then gcd(kp)=1

If i j (mod p) then imiddotk jmiddotk (mod p)

Therefore

k mod p 2k mod p hellip (p-1)k mod p

are all different numbers

Fermatrsquos Little Theorem

Each of ik mod p cannot be equal to 0 because p is a prime number

Let ci = ik mod p

So 1 lt= c1 lt= p-1 1 lt= c2 lt= p-1 hellip 1lt = cp-1 lt= p-1

By the above we know that c1c2hellipcp-2cp-1 are all different

So for each i from 1 to p-1 there is exactly one cj such that cj = i

Therefore we have

(k mod p)middot(2k mod p)middothellipmiddot((p-1)k mod p) = c1middotc2middothellipmiddotcp-2middotcp-1 = 1middot2middot3hellipmiddot(p-2)middot(p-1)

Fermatrsquos Little Theorem

1 kp-1 (mod p)

Theorem If p is prime amp k not a multiple of p

For example when p=5 k=4 we have kp-1 mod p = 44 mod 5 = 1

ldquoProofrdquo

4middot3middot2middot1 [(4 mod 5) (2middot4 mod 5) (3middot4 mod 5) (4middot4 mod 5)] (mod 5)

[4 middot (2middot4) middot (3middot4) middot (4middot4)] (mod 5)

[44 middot (1middot2middot3middot4)] (mod 5)

Since gcd(1middot2middot3middot4 5)=1 we can cancel 1middot2middot3middot4 on both sides

This implies

1 44 (mod 5)

By the previous slide or direct calculation

Fermatrsquos Little Theorem

1 kp-1 (mod p)

Theorem If p is prime amp k not a multiple of p

Proof

1middot2middotmiddotmiddot(p-1) (k mod p middot 2k mod pmiddothellipmiddot(p-1)k mod p) mod p

(kmiddot2k middotmiddotmiddot (p-1)k) mod p

(kp-1)middot1middot2 middotmiddotmiddot (p-1) (mod p)

So by cancelling 1middot2 middotmiddotmiddot (p-1) on both sides applying

Claim 1

(we can cancel them because gcd(1middot2 middotmiddotmiddot (p-1) p)=1)

we have

1 kp-1 (mod p)

By 2 slides before

By the multiplication rule

Wilsonrsquos Theorem

Theorem p is a prime if and only if

(p-1) -1 (mod p)

First we consider the easy direction

If p is not a prime assume p gt= 5 (for p=4 3 2 (mod 4) )

Then p=qr for some 2 lt= q lt p and 2 lt= r lt p

If q ne r then both q and r appear in (p-1)

and so (p-1) 0 (mod p)

If q = r then p = q2 gt 2q (since we assume p gt 5 and thus q gt 2)

then both q and 2q are in (p-1)

and so again (p-1) 0 (mod p)

Wilsonrsquos Theorem

Theorem p is a prime if and only if

(p-1) -1 (mod p)

To prove the more interesting direction first we need a lemma

Lemma If p is a prime number

x2 1 (mod p) if and only if x 1 (mod p) or x -1 (mod p)

Proof x2 1 (mod p)

iff p | x2 - 1

iff p | (x ndash 1)(x + 1)

iff p | (x ndash 1) or p | (x+1)

iff x 1 (mod p) or x -1 (mod p)

Lemma p prime and p|amiddotb iff p|a or p|b

Wilsonrsquos Theorem

Theorem p is a prime if and only if

(p-1) -1 (mod p)

Letrsquos get the proof idea by considering a concrete example

10

1middot2middot3middot4middot5middot6middot7middot8middot9middot10 mod 11

1middot10middot(2middot6)middot(3middot4)middot(5middot9)middot(7middot8) mod 11

1middot-1middot(1)middot(1)middot(1)middot(1) mod 11

-1 mod 11

Besides 1 and 10 the remaining numbers are paired up into multiplicative inverse

Wilsonrsquos Theorem

Theorem p is a prime if and only if

(p-1) -1 (mod p)

ProofSince p is a prime every number from 1 to p-1 has a multiplicative inverse

By the Lemma every number 2 lt= k lt= p-2 has an inverse krsquo with knekrsquo

Since p is odd the numbers from 2 to p-2 can be grouped into pairs

(a1b1)(a2b2)hellip(a(p-3)2b(p-3)2) so that aibi 1 (mod p)

Therefore (p-1) 1middot(p-1)middot2middot3middotmiddotmiddotmiddotmiddot(p-3)middot(p-2) (mod p)

1middot(p-1)middot(a1b1)middot(a2b2)middotmiddotmiddotmiddotmiddot(a(p-3)2b(p-3)2) (mod p)

1middot(-1)middot(1)middot(1)middotmiddotmiddotmiddotmiddot(1) (mod p)

-1 (mod p)

This Lecture

bull Eulerrsquos phi function

Inclusion-Exclusion (n sets)

What is the inclusion-exclusion formula for the union of n sets

sum of sizes of all single setsndash sum of sizes of all 2-set intersections+ sum of sizes of all 3-set intersectionsndash sum of sizes of all 4-set intersectionshellip+ (ndash1)n+1 times sum of sizes of intersections of all n

sets

1 2 nA A A

1

12 1

( 1)n

ki

S nk i S

S k

A

Inclusion-Exclusion (n sets)

Inclusion-Exclusion (n sets)

sum of sizes of all single setsndash sum of sizes of all 2-set intersections+ sum of sizes of all 3-set intersectionsndash sum of sizes of all 4-set intersectionshellip+ (ndash1)n+1 times sum of sizes of intersections of all n

sets

|A1 [ A2 [ A3 [ hellip [ An|

We want to show that every element is counted exactly once

Consider an element which belongs to exactly k sets say A1 A2 A3 hellip Ak

In the formula such an element is counted the following number of times

Therefore each element is counted exactly once and thus the formula is correct

Euler Function

Given a number n how many numbers from 1 to n are relatively prime to n

When n is a prime number

then every number from 1 to n-1 is relatively prime to n

and so

When n is a prime power

then p 2p 3p 4p hellip n are not relatively prime to n

there are np = pc-1 of them

and other numbers are relatively prime to n

Therefore

Euler Function

Given a number n how many numbers from 1 to n are relatively prime to n

Suppose

Then p 2p 3p 4p hellip n are not relatively prime to n there are np of them

Also q 2q 3q 4q hellip n are not relatively prime to n and there are nq of them

Other numbers are relatively prime to n

Therefore

The numbers pq 2pq 3pq hellip n are subtracted twice and there are npq of them

So the correct answer is

Euler Function

Given a number n how many numbers from 1 to n are relatively prime to n

Let

Let S be the set of numbers from 1 to n that are not relatively prime to n

Let Ai be the set of numbers that are a multiple of piS = A1 [ A2 [ hellip [ An

For the intersection of k sets say A1 A2 A3hellip Ak

then every number in A1 Aring A2 Aring hellip Aring Ak is a multiple of p1p2hellippk

then |A1 Aring A2 Aring hellip Aring Ak| = np1p2hellippk

Euler Function

Given a number n how many numbers from 1 to n are relatively prime to n

Let

Let S be the set of numbers from 1 to n that are not relatively prime to n

Let Ai be the set of numbers that are a multiple of piS = A1 [ A2 [ hellip [ An

|A1 Aring A2 Aring hellip Aring Ak| = np1p2hellippk

When r=3 (only 3 distinct factors)

|A1 [ A2 [ A3|

= np1 + np2 + np3

- np1p2 ndash np1p3 ndash np2p3 + np1p2p3

|A1 [ A2 [ A3| = |A1| + |A2| + |A3|

ndash |A1 Aring A2| ndash |A1 Aring A3| ndash |A2 Aring A3|

+ |A1 Aring A2 Aring A3|

= n(1-p1)(1-p2)(1-p3)

Euler Function

Given a number n how many numbers from 1 to n are relatively prime to n

Let

Let S be the set of numbers from 1 to n that are not relatively prime to n

Let Ai be the set of numbers that are a multiple of piS = A1 [ A2 [ hellip [ An

|A1 Aring A2 Aring hellip Aring Ak| = np1p2hellippk

sum of sizes of all single setsndash sum of sizes of all 2-set intersections+ sum of sizes of all 3-set intersectionsndash sum of sizes of all 4-set intersectionshellip+ (ndash1)n+1 times sum of sizes of intersections of

n sets

|A1 [ A2 [ A3 [ hellip [ An|

|S| = |A1 [ A2 [ hellip [ An|

= n(1-p1)(1-p2)hellip(1-pn)

calculationshellip

  • Modular Arithmetic
  • PowerPoint Presentation
  • Slide 3
  • Slide 4
  • Slide 5
  • Slide 6
  • Slide 7
  • Slide 8
  • Slide 9
  • Slide 10
  • Slide 11
  • Slide 12
  • Slide 13
  • Slide 14
  • Slide 15
  • Slide 16
  • Slide 17
  • Slide 18
  • Slide 19
  • Slide 20
  • Slide 21
  • Slide 22
  • Slide 23
  • Slide 24
  • Slide 25
  • Slide 26
  • Slide 27
  • Slide 28
  • Slide 29
  • Slide 30
  • Slide 31
  • Slide 32
  • Slide 33
  • Slide 34
  • Slide 35
  • Slide 36
  • Slide 37
  • Slide 38
  • Slide 39
  • Slide 40
  • Slide 41
  • Slide 42
  • Slide 43
  • Slide 44
  • Slide 45
Page 3: Modular Arithmetic

For b gt 0 and any a there are unique numbers

q = quotient(ab) r = remainder(ab) such

that

a = qb + r and 0 r lt b

0 b 2b kb (k+1)b

Given any b we can divide the integers into many blocks of b numbers

For any a there is a unique ldquopositionrdquo for a in this line

q = the block where a is in

a

r = the offset in this block

Clearly given a and b q and r are uniquely defined

-b

The Quotient-Remainder Theorem

Def a b (mod n) iff n|(a - b) iff a mod n = b mod n

Modular Arithmetic

eg 12 2 (mod 10)

107 207 (mod 10)

7 3 (mod 2)

7 -1 (mod 2)

13 -1 (mod 7)

-15 10 (mod 5)

Be careful a mod n means ldquothe remainder when a is divided by nrdquo

a b (mod n) means ldquoa and b have the same remainder when divided by nrdquo

12 mod 10 = 2

207 mod 10 = 7

7 mod 2 = 1

-1 mod 2 = 1

-1 mod 7 = 6

-15 mod 5 = 0

Fact a a mod n (mod n) as a and a mod n have the same remainder mod n

Fact if a b (mod n) then a = b + nx for some integer x

Lemma If a c (mod n) and b d (mod n) then

a+b c+d (mod n)

Modular Addition

When you try to understand a statement like this

first think about the familiar cases eg n=10 or n=2

When n=2 it says that if a and c have the same parity

and b and d have the same parity

then a+b and c+d have the same parity

When n=10 it says that if a and c have the same last digit

and b and d have the same last digit

then a+b and c+d have the same last digit

And the lemma says that the same principle applied for all n

Lemma If a c (mod n) and b d (mod n) then

a+b c+d (mod n)

Modular Addition

Example 1 13 1 (mod 3) 25 1 (mod 3)

=gt 12 + 25 (mod 3) 1 + 1 (mod 3) 2 (mod 3)

Example 2 87 2 (mod 17) 222 1 (mod 17)

=gt 87 + 222 (mod 17) 2 + 1 (mod 17) 3 (mod 17)

Example 3 101 2 (mod 11) 141 -2 (mod 11)

=gt 101 + 141 (mod 11) 0 (mod 11)

In particular when computing a+b mod n we can first replace

a by a mod n and b by b mod n so that the computation is faster

Lemma If a c (mod n) and b d (mod n) then

a+b c+d (mod n)

Modular Addition

a c (mod n) =gt a = c + nx for some integer x

b d (mod n) =gt b = d + ny for some integer y

To show a+b c+d (mod n) it is equivalent to showing that n | (a+b-c-d)

Consider a+b-c-d

a+b-c-d = (c+nx) + (d+ny) ndash c ndashd = nx + ny

It is clear that n | nx + ny

Therefore n | a+b-c-d

We conclude that a+b c+d (mod n)

Proof

Lemma If a c (mod n) and b d (mod n) then

ab cd (mod n)

Modular Multiplication

Example 1 9876 6 (mod 10) 17642 2 (mod 10)

=gt 9876 17642 (mod 10) 6 2 (mod 10) 2 (mod 10)

Example 2 10987 1 (mod 2) 28663 1 (mod 2)

=gt 10987 28663 (mod 2) 1 (mod 2)

Example 3 1000 -1 (mod 7) 1000000 1 (mod 7)

=gt 1000 1000000 (mod 7) -1 1 (mod 7) -1 (mod 7)

In particular when computing ab mod n we can first replace

a by a mod n and b by b mod n so that the computation is faster

Lemma If a c (mod n) and b d (mod n) then

ab cd (mod n)

Modular Multiplication

a c (mod n) =gt a = c + nx for some integer x

b d (mod n) =gt b = d + ny for some integer y

To show ab cd (mod n) it is equivalent to showing that n | (ab-cd)

Consider ab-cd

ab-cd = (c+nx) (d+ny) ndash cd

= cd + dnx + cny + n2xy ndash cd = n(dx + cy + nxy)

It is clear that n | n(dx + cy + nxy) Therefore n | ab-cd

We conclude that ab cd (mod n)

Proof

This Lecture

bull Applications Fast exponentiation and fast division test

Fast Exponentiation

1444 mod 713

= 144 144 144 144 mod 713

= 20736 144 144 mod 713

= 59 144 144 mod 713

= 8496 144 mod 713

= 653 144 mod 713

= 94032 mod 713

= 629 mod 713

20736 20736 mod 713

= 59 59 mod 713

= 3481 mod 713

= 629 mod 713

Because 20736 59 (mod 713)

Because 653 8496 (mod 713)

shortcut

Repeated Squaring

14450 mod 713

= 14432 14416 1442 mod 713

= 64848559 mod 713

= 242

1442 mod 713 = 59

1444 mod 713 = 1442 1442 mod 713= 5959 mod 713= 629

1448 mod 713= 14441444 mod 713= 629629 mod 713= 639

14416 mod 713= 14481448 mod 713= 639639 mod 713= 485

14432 mod 713= 1441614416 mod 713= 485485 mod 713= 648

Note that 50 = 32 + 16 + 2

Fast Division Test

Using the basic rules for modular addition and modular multiplication

we can derive some quick test to see if a big number is divisible by a small number

Suppose we are given the decimal representation of a big number N

To test if N is divisible by a small number n of course we can do a division to check

But can we do faster

If n = 2 we just need to check whether the last digit of N is even or not

If n = 10 we just need to check whether the last digit of N is 0 or not

If n = 5 we just need to check whether the last digit of N is either 5 or 0 or not

What about when n=3 When n=7 When n=11

Fast Division Test

A number written in decimal divisible by 9 if and only if

the sum of its digits is a multiple of 9

Example 1 9333234513171 is divisible by 9

9+3+3+3+2+3+4+5+1+3+1+7+1 = 45 is divisible by 9

Example 2 128573649683 is not divisible by 9

1+2+8+5+7+3+6+4+9+6+8+3 = 62 is not divisible by 9

Claim A number written in decimal is divisible by 9 if and

only if

the sum of its digits is a multiple of 9Hint 10 1 (mod 9)

Let the decimal representation of N be dkdk-1dk-2hellipd1d0

This means that N = dk10k + dk-110k-1 + hellip + d110 + d0

Note that di10i mod 9

= (di) (10i mod 9) mod 9

= (di) (10 mod 9) (10 mod 9) hellip (10 mod 9) mod 9

= (di) (1 mod 9) (1 mod 9) hellip (1 mod 9) mod 9

= di mod 9

i terms

Fast Division Test

Rule of modular multiplication

Let the decimal representation of n be dkdk-1dk-2hellipd1d0

This means that N = dk10k + dk-110k-1 + hellip + d110 + d0

Note that di10i mod 9 = di mod 9

Hence N mod 9 = (dk10k + dk-110k-1 + hellip + d110 + d0) mod 9

= (dk10k mod 9 + dk-110k-1 mod 9 + hellip + d110 mod 9 + d0 mod 9) mod

9

= (dk mod 9 + dk-1 mod 9 + hellip + d1 mod 9 + d0 mod 9) mod 9

= (dk + dk-1 + hellip + d1 + d0) mod 9

Hint 10 1 (mod 9)

Fast Division Test

Claim A number written in decimal is divisible by 9 if and

only if

the sum of its digits is a multiple of 9

Rule of modular addition

By previous slide

The same procedure works to test whether N is divisible by n=3

Fast Division Test

What about n=11

Hint 10 -1 (mod 11)

Let the decimal representation of N be d92d91d90hellipd1d0

Then N is divisible by 11 if and only if

d92-d91+d90hellip-d1+d0 is divisible by 11

Why Try to work it out before your TA shows you

What about n=7

Hint 1000 -1 (mod 7)

This Lecture

bull Multiplicative inverse

The multiplicative inverse of a number a is another number arsquo such that

a middot arsquo 1 (mod n)

Multiplication Inverse

Does every number has a multiplicative inverse in modular arithmetic

For real numbers every nonzero number has a multiplicative inverse

For integers only 1 has a multiplicative inverse

An interesting property of modular arithmetic is that

there are multiplicative inverse for integers

For example 2 5 = 1 mod 3 so 5 is a multiplicative inverse

for 2 under modulo 3 (and vice versa)

Multiplication Inverse

Does every number has a multiplicative inverse in modular arithmetic

Multiplication Inverse

What is the pattern

Case Study

Why 2 does not have a multiplicative inverse under modulo 6

Suppose it has a multiplicative inverse y

2y 1 (mod 6)

=gt 2y = 1 + 6x for some integer x

=gt y = frac12 + 3x

This is a contradiction since both x and y are integers

Necessary Condition

Claim An integer k does not have an multiplicative inverse under modulo n

if k and n have a common factor gt= 2 (gcd(kn) gt= 2)

Proof

This claim says that for k to have a multiplicative inverse modulo n

then a necessary condition is that k and n do not have a common factor gt= 2

Suppose by contradiction that there is an inverse krsquo for k such that

krsquok = 1 (mod n)

Then krsquok = 1 + xn for some integer x

Since both k and n have a common factor say cgt=2

then k=ck1 and n=cn1 for some integers k1 and n1

So krsquock1 = 1 + xcn1

Then krsquok1 = 1c + xn1

This is a contradiction since the LHS is an integer but the RHS is not

Sufficient Condition

What about if gcd(kn)=1

Would k always have an multiplicative inverse under modulo n

For example gcd(37) = 1 3middot5 1 (mod 7)

gcd(89) = 1

gcd(411) = 1 4middot3 1 (mod 11)

8middot8 1 (mod 9)

It seems that there is always an inverse in such a case but why

gcd(89) = 1 8s + 9t = 1 for some integers s and t

8s = 1 ndash 9t

8s 1 (mod 9)gcd(89) = spc(89)

Theorem If gcd(kn)=1 then have krsquo such that

kmiddotkrsquo 1 (mod n)

Proof Since gcd(kn)=1 there exist s and t so that sk + tn = 1

So tn = 1 - sk

This means n | 1 ndash sk

This means that 1 ndash sk 0 (mod n)

This means that 1 sk (mod n)

So krsquo = s is an multiplicative inverse for k

Sufficient Condition

gcd(kn)=spc(kn)

The multiplicative inverse can be computed by the extended Euclidean algorithm

Corollary k has a multiplicative inverse mod n if and only if gcd(kn)=1

This Lecture

bull Fermatrsquos little theorem

Cancellation

There is no general cancellation in modular arithmetic

Note that (mod n) is very similar to =

If a b (mod n) then a+c b+c (mod n)

If a b (mod n) then ac bc (mod n)

However if ac bc (mod n)

it is not necessarily true that a b (mod n)

For example 4middot2 1middot2 (mod 6) but 4 1 (mod 6)

3middot4 1middot4 (mod 8) but 3 1 (mod 8)

4middot3 1middot3 (mod 9) but 4 1 (mod 9)

Observation In all the above examples c and n have a common factor

Cancellation

Claim Assume gcd(kn) = 1 If imiddotk jmiddotk (mod n) then i j (mod n)

For example multiplicative inverse always exists if n is a prime

Proof Since gcd(kn) = 1 there exists krsquo such that kkrsquo 1 (mod n)

imiddotk jmiddotk (mod n)

=gt imiddotkmiddotkrsquo jmiddotkmiddotkrsquo (mod n)

=gt i j (mod n)

Remarks (Optional) This makes arithmetic modulo prime a field

a structure that ldquobehaves likerdquo real numbers

Arithmetic modulo prime is very useful in coding theory

In particular when p is a prime amp k not a multiple of p then gcd(kp)=1

If i j (mod p) then imiddotk jmiddotk (mod p)

Therefore

k mod p 2k mod p hellip (p-1)k mod p

are all different numbers

Fermatrsquos Little Theorem

Claim 1 Assume gcd(kn) = 1 If imiddotk jmiddotk (mod n) then i j (mod n)

Claim 2 Assume gcd(kn) = 1 If i j (mod n) then imiddotk jmiddotk (mod n)

For example when p=7 and k=3

3 mod 7 = 3 2middot3 mod 7 = 6 3middot3 mod 7 = 2 4middot3 mod 7 = 5 5middot3 mod 7 = 1 6middot3 mod 7 = 4

Notice that in the above example every number from 1 to 6 appears exactly once

In particular when p is a prime amp k not a multiple of p then gcd(kp)=1

If i j (mod p) then imiddotk jmiddotk (mod p)

Therefore

k mod p 2k mod p hellip (p-1)k mod p

are all different numbers

Fermatrsquos Little Theorem

Each of ik mod p cannot be equal to 0 because p is a prime number

Let ci = ik mod p

So 1 lt= c1 lt= p-1 1 lt= c2 lt= p-1 hellip 1lt = cp-1 lt= p-1

By the above we know that c1c2hellipcp-2cp-1 are all different

So for each i from 1 to p-1 there is exactly one cj such that cj = i

Therefore we have

(k mod p)middot(2k mod p)middothellipmiddot((p-1)k mod p) = c1middotc2middothellipmiddotcp-2middotcp-1 = 1middot2middot3hellipmiddot(p-2)middot(p-1)

Fermatrsquos Little Theorem

1 kp-1 (mod p)

Theorem If p is prime amp k not a multiple of p

For example when p=5 k=4 we have kp-1 mod p = 44 mod 5 = 1

ldquoProofrdquo

4middot3middot2middot1 [(4 mod 5) (2middot4 mod 5) (3middot4 mod 5) (4middot4 mod 5)] (mod 5)

[4 middot (2middot4) middot (3middot4) middot (4middot4)] (mod 5)

[44 middot (1middot2middot3middot4)] (mod 5)

Since gcd(1middot2middot3middot4 5)=1 we can cancel 1middot2middot3middot4 on both sides

This implies

1 44 (mod 5)

By the previous slide or direct calculation

Fermatrsquos Little Theorem

1 kp-1 (mod p)

Theorem If p is prime amp k not a multiple of p

Proof

1middot2middotmiddotmiddot(p-1) (k mod p middot 2k mod pmiddothellipmiddot(p-1)k mod p) mod p

(kmiddot2k middotmiddotmiddot (p-1)k) mod p

(kp-1)middot1middot2 middotmiddotmiddot (p-1) (mod p)

So by cancelling 1middot2 middotmiddotmiddot (p-1) on both sides applying

Claim 1

(we can cancel them because gcd(1middot2 middotmiddotmiddot (p-1) p)=1)

we have

1 kp-1 (mod p)

By 2 slides before

By the multiplication rule

Wilsonrsquos Theorem

Theorem p is a prime if and only if

(p-1) -1 (mod p)

First we consider the easy direction

If p is not a prime assume p gt= 5 (for p=4 3 2 (mod 4) )

Then p=qr for some 2 lt= q lt p and 2 lt= r lt p

If q ne r then both q and r appear in (p-1)

and so (p-1) 0 (mod p)

If q = r then p = q2 gt 2q (since we assume p gt 5 and thus q gt 2)

then both q and 2q are in (p-1)

and so again (p-1) 0 (mod p)

Wilsonrsquos Theorem

Theorem p is a prime if and only if

(p-1) -1 (mod p)

To prove the more interesting direction first we need a lemma

Lemma If p is a prime number

x2 1 (mod p) if and only if x 1 (mod p) or x -1 (mod p)

Proof x2 1 (mod p)

iff p | x2 - 1

iff p | (x ndash 1)(x + 1)

iff p | (x ndash 1) or p | (x+1)

iff x 1 (mod p) or x -1 (mod p)

Lemma p prime and p|amiddotb iff p|a or p|b

Wilsonrsquos Theorem

Theorem p is a prime if and only if

(p-1) -1 (mod p)

Letrsquos get the proof idea by considering a concrete example

10

1middot2middot3middot4middot5middot6middot7middot8middot9middot10 mod 11

1middot10middot(2middot6)middot(3middot4)middot(5middot9)middot(7middot8) mod 11

1middot-1middot(1)middot(1)middot(1)middot(1) mod 11

-1 mod 11

Besides 1 and 10 the remaining numbers are paired up into multiplicative inverse

Wilsonrsquos Theorem

Theorem p is a prime if and only if

(p-1) -1 (mod p)

ProofSince p is a prime every number from 1 to p-1 has a multiplicative inverse

By the Lemma every number 2 lt= k lt= p-2 has an inverse krsquo with knekrsquo

Since p is odd the numbers from 2 to p-2 can be grouped into pairs

(a1b1)(a2b2)hellip(a(p-3)2b(p-3)2) so that aibi 1 (mod p)

Therefore (p-1) 1middot(p-1)middot2middot3middotmiddotmiddotmiddotmiddot(p-3)middot(p-2) (mod p)

1middot(p-1)middot(a1b1)middot(a2b2)middotmiddotmiddotmiddotmiddot(a(p-3)2b(p-3)2) (mod p)

1middot(-1)middot(1)middot(1)middotmiddotmiddotmiddotmiddot(1) (mod p)

-1 (mod p)

This Lecture

bull Eulerrsquos phi function

Inclusion-Exclusion (n sets)

What is the inclusion-exclusion formula for the union of n sets

sum of sizes of all single setsndash sum of sizes of all 2-set intersections+ sum of sizes of all 3-set intersectionsndash sum of sizes of all 4-set intersectionshellip+ (ndash1)n+1 times sum of sizes of intersections of all n

sets

1 2 nA A A

1

12 1

( 1)n

ki

S nk i S

S k

A

Inclusion-Exclusion (n sets)

Inclusion-Exclusion (n sets)

sum of sizes of all single setsndash sum of sizes of all 2-set intersections+ sum of sizes of all 3-set intersectionsndash sum of sizes of all 4-set intersectionshellip+ (ndash1)n+1 times sum of sizes of intersections of all n

sets

|A1 [ A2 [ A3 [ hellip [ An|

We want to show that every element is counted exactly once

Consider an element which belongs to exactly k sets say A1 A2 A3 hellip Ak

In the formula such an element is counted the following number of times

Therefore each element is counted exactly once and thus the formula is correct

Euler Function

Given a number n how many numbers from 1 to n are relatively prime to n

When n is a prime number

then every number from 1 to n-1 is relatively prime to n

and so

When n is a prime power

then p 2p 3p 4p hellip n are not relatively prime to n

there are np = pc-1 of them

and other numbers are relatively prime to n

Therefore

Euler Function

Given a number n how many numbers from 1 to n are relatively prime to n

Suppose

Then p 2p 3p 4p hellip n are not relatively prime to n there are np of them

Also q 2q 3q 4q hellip n are not relatively prime to n and there are nq of them

Other numbers are relatively prime to n

Therefore

The numbers pq 2pq 3pq hellip n are subtracted twice and there are npq of them

So the correct answer is

Euler Function

Given a number n how many numbers from 1 to n are relatively prime to n

Let

Let S be the set of numbers from 1 to n that are not relatively prime to n

Let Ai be the set of numbers that are a multiple of piS = A1 [ A2 [ hellip [ An

For the intersection of k sets say A1 A2 A3hellip Ak

then every number in A1 Aring A2 Aring hellip Aring Ak is a multiple of p1p2hellippk

then |A1 Aring A2 Aring hellip Aring Ak| = np1p2hellippk

Euler Function

Given a number n how many numbers from 1 to n are relatively prime to n

Let

Let S be the set of numbers from 1 to n that are not relatively prime to n

Let Ai be the set of numbers that are a multiple of piS = A1 [ A2 [ hellip [ An

|A1 Aring A2 Aring hellip Aring Ak| = np1p2hellippk

When r=3 (only 3 distinct factors)

|A1 [ A2 [ A3|

= np1 + np2 + np3

- np1p2 ndash np1p3 ndash np2p3 + np1p2p3

|A1 [ A2 [ A3| = |A1| + |A2| + |A3|

ndash |A1 Aring A2| ndash |A1 Aring A3| ndash |A2 Aring A3|

+ |A1 Aring A2 Aring A3|

= n(1-p1)(1-p2)(1-p3)

Euler Function

Given a number n how many numbers from 1 to n are relatively prime to n

Let

Let S be the set of numbers from 1 to n that are not relatively prime to n

Let Ai be the set of numbers that are a multiple of piS = A1 [ A2 [ hellip [ An

|A1 Aring A2 Aring hellip Aring Ak| = np1p2hellippk

sum of sizes of all single setsndash sum of sizes of all 2-set intersections+ sum of sizes of all 3-set intersectionsndash sum of sizes of all 4-set intersectionshellip+ (ndash1)n+1 times sum of sizes of intersections of

n sets

|A1 [ A2 [ A3 [ hellip [ An|

|S| = |A1 [ A2 [ hellip [ An|

= n(1-p1)(1-p2)hellip(1-pn)

calculationshellip

  • Modular Arithmetic
  • PowerPoint Presentation
  • Slide 3
  • Slide 4
  • Slide 5
  • Slide 6
  • Slide 7
  • Slide 8
  • Slide 9
  • Slide 10
  • Slide 11
  • Slide 12
  • Slide 13
  • Slide 14
  • Slide 15
  • Slide 16
  • Slide 17
  • Slide 18
  • Slide 19
  • Slide 20
  • Slide 21
  • Slide 22
  • Slide 23
  • Slide 24
  • Slide 25
  • Slide 26
  • Slide 27
  • Slide 28
  • Slide 29
  • Slide 30
  • Slide 31
  • Slide 32
  • Slide 33
  • Slide 34
  • Slide 35
  • Slide 36
  • Slide 37
  • Slide 38
  • Slide 39
  • Slide 40
  • Slide 41
  • Slide 42
  • Slide 43
  • Slide 44
  • Slide 45
Page 4: Modular Arithmetic

Def a b (mod n) iff n|(a - b) iff a mod n = b mod n

Modular Arithmetic

eg 12 2 (mod 10)

107 207 (mod 10)

7 3 (mod 2)

7 -1 (mod 2)

13 -1 (mod 7)

-15 10 (mod 5)

Be careful a mod n means ldquothe remainder when a is divided by nrdquo

a b (mod n) means ldquoa and b have the same remainder when divided by nrdquo

12 mod 10 = 2

207 mod 10 = 7

7 mod 2 = 1

-1 mod 2 = 1

-1 mod 7 = 6

-15 mod 5 = 0

Fact a a mod n (mod n) as a and a mod n have the same remainder mod n

Fact if a b (mod n) then a = b + nx for some integer x

Lemma If a c (mod n) and b d (mod n) then

a+b c+d (mod n)

Modular Addition

When you try to understand a statement like this

first think about the familiar cases eg n=10 or n=2

When n=2 it says that if a and c have the same parity

and b and d have the same parity

then a+b and c+d have the same parity

When n=10 it says that if a and c have the same last digit

and b and d have the same last digit

then a+b and c+d have the same last digit

And the lemma says that the same principle applied for all n

Lemma If a c (mod n) and b d (mod n) then

a+b c+d (mod n)

Modular Addition

Example 1 13 1 (mod 3) 25 1 (mod 3)

=gt 12 + 25 (mod 3) 1 + 1 (mod 3) 2 (mod 3)

Example 2 87 2 (mod 17) 222 1 (mod 17)

=gt 87 + 222 (mod 17) 2 + 1 (mod 17) 3 (mod 17)

Example 3 101 2 (mod 11) 141 -2 (mod 11)

=gt 101 + 141 (mod 11) 0 (mod 11)

In particular when computing a+b mod n we can first replace

a by a mod n and b by b mod n so that the computation is faster

Lemma If a c (mod n) and b d (mod n) then

a+b c+d (mod n)

Modular Addition

a c (mod n) =gt a = c + nx for some integer x

b d (mod n) =gt b = d + ny for some integer y

To show a+b c+d (mod n) it is equivalent to showing that n | (a+b-c-d)

Consider a+b-c-d

a+b-c-d = (c+nx) + (d+ny) ndash c ndashd = nx + ny

It is clear that n | nx + ny

Therefore n | a+b-c-d

We conclude that a+b c+d (mod n)

Proof

Lemma If a c (mod n) and b d (mod n) then

ab cd (mod n)

Modular Multiplication

Example 1 9876 6 (mod 10) 17642 2 (mod 10)

=gt 9876 17642 (mod 10) 6 2 (mod 10) 2 (mod 10)

Example 2 10987 1 (mod 2) 28663 1 (mod 2)

=gt 10987 28663 (mod 2) 1 (mod 2)

Example 3 1000 -1 (mod 7) 1000000 1 (mod 7)

=gt 1000 1000000 (mod 7) -1 1 (mod 7) -1 (mod 7)

In particular when computing ab mod n we can first replace

a by a mod n and b by b mod n so that the computation is faster

Lemma If a c (mod n) and b d (mod n) then

ab cd (mod n)

Modular Multiplication

a c (mod n) =gt a = c + nx for some integer x

b d (mod n) =gt b = d + ny for some integer y

To show ab cd (mod n) it is equivalent to showing that n | (ab-cd)

Consider ab-cd

ab-cd = (c+nx) (d+ny) ndash cd

= cd + dnx + cny + n2xy ndash cd = n(dx + cy + nxy)

It is clear that n | n(dx + cy + nxy) Therefore n | ab-cd

We conclude that ab cd (mod n)

Proof

This Lecture

bull Applications Fast exponentiation and fast division test

Fast Exponentiation

1444 mod 713

= 144 144 144 144 mod 713

= 20736 144 144 mod 713

= 59 144 144 mod 713

= 8496 144 mod 713

= 653 144 mod 713

= 94032 mod 713

= 629 mod 713

20736 20736 mod 713

= 59 59 mod 713

= 3481 mod 713

= 629 mod 713

Because 20736 59 (mod 713)

Because 653 8496 (mod 713)

shortcut

Repeated Squaring

14450 mod 713

= 14432 14416 1442 mod 713

= 64848559 mod 713

= 242

1442 mod 713 = 59

1444 mod 713 = 1442 1442 mod 713= 5959 mod 713= 629

1448 mod 713= 14441444 mod 713= 629629 mod 713= 639

14416 mod 713= 14481448 mod 713= 639639 mod 713= 485

14432 mod 713= 1441614416 mod 713= 485485 mod 713= 648

Note that 50 = 32 + 16 + 2

Fast Division Test

Using the basic rules for modular addition and modular multiplication

we can derive some quick test to see if a big number is divisible by a small number

Suppose we are given the decimal representation of a big number N

To test if N is divisible by a small number n of course we can do a division to check

But can we do faster

If n = 2 we just need to check whether the last digit of N is even or not

If n = 10 we just need to check whether the last digit of N is 0 or not

If n = 5 we just need to check whether the last digit of N is either 5 or 0 or not

What about when n=3 When n=7 When n=11

Fast Division Test

A number written in decimal divisible by 9 if and only if

the sum of its digits is a multiple of 9

Example 1 9333234513171 is divisible by 9

9+3+3+3+2+3+4+5+1+3+1+7+1 = 45 is divisible by 9

Example 2 128573649683 is not divisible by 9

1+2+8+5+7+3+6+4+9+6+8+3 = 62 is not divisible by 9

Claim A number written in decimal is divisible by 9 if and

only if

the sum of its digits is a multiple of 9Hint 10 1 (mod 9)

Let the decimal representation of N be dkdk-1dk-2hellipd1d0

This means that N = dk10k + dk-110k-1 + hellip + d110 + d0

Note that di10i mod 9

= (di) (10i mod 9) mod 9

= (di) (10 mod 9) (10 mod 9) hellip (10 mod 9) mod 9

= (di) (1 mod 9) (1 mod 9) hellip (1 mod 9) mod 9

= di mod 9

i terms

Fast Division Test

Rule of modular multiplication

Let the decimal representation of n be dkdk-1dk-2hellipd1d0

This means that N = dk10k + dk-110k-1 + hellip + d110 + d0

Note that di10i mod 9 = di mod 9

Hence N mod 9 = (dk10k + dk-110k-1 + hellip + d110 + d0) mod 9

= (dk10k mod 9 + dk-110k-1 mod 9 + hellip + d110 mod 9 + d0 mod 9) mod

9

= (dk mod 9 + dk-1 mod 9 + hellip + d1 mod 9 + d0 mod 9) mod 9

= (dk + dk-1 + hellip + d1 + d0) mod 9

Hint 10 1 (mod 9)

Fast Division Test

Claim A number written in decimal is divisible by 9 if and

only if

the sum of its digits is a multiple of 9

Rule of modular addition

By previous slide

The same procedure works to test whether N is divisible by n=3

Fast Division Test

What about n=11

Hint 10 -1 (mod 11)

Let the decimal representation of N be d92d91d90hellipd1d0

Then N is divisible by 11 if and only if

d92-d91+d90hellip-d1+d0 is divisible by 11

Why Try to work it out before your TA shows you

What about n=7

Hint 1000 -1 (mod 7)

This Lecture

bull Multiplicative inverse

The multiplicative inverse of a number a is another number arsquo such that

a middot arsquo 1 (mod n)

Multiplication Inverse

Does every number has a multiplicative inverse in modular arithmetic

For real numbers every nonzero number has a multiplicative inverse

For integers only 1 has a multiplicative inverse

An interesting property of modular arithmetic is that

there are multiplicative inverse for integers

For example 2 5 = 1 mod 3 so 5 is a multiplicative inverse

for 2 under modulo 3 (and vice versa)

Multiplication Inverse

Does every number has a multiplicative inverse in modular arithmetic

Multiplication Inverse

What is the pattern

Case Study

Why 2 does not have a multiplicative inverse under modulo 6

Suppose it has a multiplicative inverse y

2y 1 (mod 6)

=gt 2y = 1 + 6x for some integer x

=gt y = frac12 + 3x

This is a contradiction since both x and y are integers

Necessary Condition

Claim An integer k does not have an multiplicative inverse under modulo n

if k and n have a common factor gt= 2 (gcd(kn) gt= 2)

Proof

This claim says that for k to have a multiplicative inverse modulo n

then a necessary condition is that k and n do not have a common factor gt= 2

Suppose by contradiction that there is an inverse krsquo for k such that

krsquok = 1 (mod n)

Then krsquok = 1 + xn for some integer x

Since both k and n have a common factor say cgt=2

then k=ck1 and n=cn1 for some integers k1 and n1

So krsquock1 = 1 + xcn1

Then krsquok1 = 1c + xn1

This is a contradiction since the LHS is an integer but the RHS is not

Sufficient Condition

What about if gcd(kn)=1

Would k always have an multiplicative inverse under modulo n

For example gcd(37) = 1 3middot5 1 (mod 7)

gcd(89) = 1

gcd(411) = 1 4middot3 1 (mod 11)

8middot8 1 (mod 9)

It seems that there is always an inverse in such a case but why

gcd(89) = 1 8s + 9t = 1 for some integers s and t

8s = 1 ndash 9t

8s 1 (mod 9)gcd(89) = spc(89)

Theorem If gcd(kn)=1 then have krsquo such that

kmiddotkrsquo 1 (mod n)

Proof Since gcd(kn)=1 there exist s and t so that sk + tn = 1

So tn = 1 - sk

This means n | 1 ndash sk

This means that 1 ndash sk 0 (mod n)

This means that 1 sk (mod n)

So krsquo = s is an multiplicative inverse for k

Sufficient Condition

gcd(kn)=spc(kn)

The multiplicative inverse can be computed by the extended Euclidean algorithm

Corollary k has a multiplicative inverse mod n if and only if gcd(kn)=1

This Lecture

bull Fermatrsquos little theorem

Cancellation

There is no general cancellation in modular arithmetic

Note that (mod n) is very similar to =

If a b (mod n) then a+c b+c (mod n)

If a b (mod n) then ac bc (mod n)

However if ac bc (mod n)

it is not necessarily true that a b (mod n)

For example 4middot2 1middot2 (mod 6) but 4 1 (mod 6)

3middot4 1middot4 (mod 8) but 3 1 (mod 8)

4middot3 1middot3 (mod 9) but 4 1 (mod 9)

Observation In all the above examples c and n have a common factor

Cancellation

Claim Assume gcd(kn) = 1 If imiddotk jmiddotk (mod n) then i j (mod n)

For example multiplicative inverse always exists if n is a prime

Proof Since gcd(kn) = 1 there exists krsquo such that kkrsquo 1 (mod n)

imiddotk jmiddotk (mod n)

=gt imiddotkmiddotkrsquo jmiddotkmiddotkrsquo (mod n)

=gt i j (mod n)

Remarks (Optional) This makes arithmetic modulo prime a field

a structure that ldquobehaves likerdquo real numbers

Arithmetic modulo prime is very useful in coding theory

In particular when p is a prime amp k not a multiple of p then gcd(kp)=1

If i j (mod p) then imiddotk jmiddotk (mod p)

Therefore

k mod p 2k mod p hellip (p-1)k mod p

are all different numbers

Fermatrsquos Little Theorem

Claim 1 Assume gcd(kn) = 1 If imiddotk jmiddotk (mod n) then i j (mod n)

Claim 2 Assume gcd(kn) = 1 If i j (mod n) then imiddotk jmiddotk (mod n)

For example when p=7 and k=3

3 mod 7 = 3 2middot3 mod 7 = 6 3middot3 mod 7 = 2 4middot3 mod 7 = 5 5middot3 mod 7 = 1 6middot3 mod 7 = 4

Notice that in the above example every number from 1 to 6 appears exactly once

In particular when p is a prime amp k not a multiple of p then gcd(kp)=1

If i j (mod p) then imiddotk jmiddotk (mod p)

Therefore

k mod p 2k mod p hellip (p-1)k mod p

are all different numbers

Fermatrsquos Little Theorem

Each of ik mod p cannot be equal to 0 because p is a prime number

Let ci = ik mod p

So 1 lt= c1 lt= p-1 1 lt= c2 lt= p-1 hellip 1lt = cp-1 lt= p-1

By the above we know that c1c2hellipcp-2cp-1 are all different

So for each i from 1 to p-1 there is exactly one cj such that cj = i

Therefore we have

(k mod p)middot(2k mod p)middothellipmiddot((p-1)k mod p) = c1middotc2middothellipmiddotcp-2middotcp-1 = 1middot2middot3hellipmiddot(p-2)middot(p-1)

Fermatrsquos Little Theorem

1 kp-1 (mod p)

Theorem If p is prime amp k not a multiple of p

For example when p=5 k=4 we have kp-1 mod p = 44 mod 5 = 1

ldquoProofrdquo

4middot3middot2middot1 [(4 mod 5) (2middot4 mod 5) (3middot4 mod 5) (4middot4 mod 5)] (mod 5)

[4 middot (2middot4) middot (3middot4) middot (4middot4)] (mod 5)

[44 middot (1middot2middot3middot4)] (mod 5)

Since gcd(1middot2middot3middot4 5)=1 we can cancel 1middot2middot3middot4 on both sides

This implies

1 44 (mod 5)

By the previous slide or direct calculation

Fermatrsquos Little Theorem

1 kp-1 (mod p)

Theorem If p is prime amp k not a multiple of p

Proof

1middot2middotmiddotmiddot(p-1) (k mod p middot 2k mod pmiddothellipmiddot(p-1)k mod p) mod p

(kmiddot2k middotmiddotmiddot (p-1)k) mod p

(kp-1)middot1middot2 middotmiddotmiddot (p-1) (mod p)

So by cancelling 1middot2 middotmiddotmiddot (p-1) on both sides applying

Claim 1

(we can cancel them because gcd(1middot2 middotmiddotmiddot (p-1) p)=1)

we have

1 kp-1 (mod p)

By 2 slides before

By the multiplication rule

Wilsonrsquos Theorem

Theorem p is a prime if and only if

(p-1) -1 (mod p)

First we consider the easy direction

If p is not a prime assume p gt= 5 (for p=4 3 2 (mod 4) )

Then p=qr for some 2 lt= q lt p and 2 lt= r lt p

If q ne r then both q and r appear in (p-1)

and so (p-1) 0 (mod p)

If q = r then p = q2 gt 2q (since we assume p gt 5 and thus q gt 2)

then both q and 2q are in (p-1)

and so again (p-1) 0 (mod p)

Wilsonrsquos Theorem

Theorem p is a prime if and only if

(p-1) -1 (mod p)

To prove the more interesting direction first we need a lemma

Lemma If p is a prime number

x2 1 (mod p) if and only if x 1 (mod p) or x -1 (mod p)

Proof x2 1 (mod p)

iff p | x2 - 1

iff p | (x ndash 1)(x + 1)

iff p | (x ndash 1) or p | (x+1)

iff x 1 (mod p) or x -1 (mod p)

Lemma p prime and p|amiddotb iff p|a or p|b

Wilsonrsquos Theorem

Theorem p is a prime if and only if

(p-1) -1 (mod p)

Letrsquos get the proof idea by considering a concrete example

10

1middot2middot3middot4middot5middot6middot7middot8middot9middot10 mod 11

1middot10middot(2middot6)middot(3middot4)middot(5middot9)middot(7middot8) mod 11

1middot-1middot(1)middot(1)middot(1)middot(1) mod 11

-1 mod 11

Besides 1 and 10 the remaining numbers are paired up into multiplicative inverse

Wilsonrsquos Theorem

Theorem p is a prime if and only if

(p-1) -1 (mod p)

ProofSince p is a prime every number from 1 to p-1 has a multiplicative inverse

By the Lemma every number 2 lt= k lt= p-2 has an inverse krsquo with knekrsquo

Since p is odd the numbers from 2 to p-2 can be grouped into pairs

(a1b1)(a2b2)hellip(a(p-3)2b(p-3)2) so that aibi 1 (mod p)

Therefore (p-1) 1middot(p-1)middot2middot3middotmiddotmiddotmiddotmiddot(p-3)middot(p-2) (mod p)

1middot(p-1)middot(a1b1)middot(a2b2)middotmiddotmiddotmiddotmiddot(a(p-3)2b(p-3)2) (mod p)

1middot(-1)middot(1)middot(1)middotmiddotmiddotmiddotmiddot(1) (mod p)

-1 (mod p)

This Lecture

bull Eulerrsquos phi function

Inclusion-Exclusion (n sets)

What is the inclusion-exclusion formula for the union of n sets

sum of sizes of all single setsndash sum of sizes of all 2-set intersections+ sum of sizes of all 3-set intersectionsndash sum of sizes of all 4-set intersectionshellip+ (ndash1)n+1 times sum of sizes of intersections of all n

sets

1 2 nA A A

1

12 1

( 1)n

ki

S nk i S

S k

A

Inclusion-Exclusion (n sets)

Inclusion-Exclusion (n sets)

sum of sizes of all single setsndash sum of sizes of all 2-set intersections+ sum of sizes of all 3-set intersectionsndash sum of sizes of all 4-set intersectionshellip+ (ndash1)n+1 times sum of sizes of intersections of all n

sets

|A1 [ A2 [ A3 [ hellip [ An|

We want to show that every element is counted exactly once

Consider an element which belongs to exactly k sets say A1 A2 A3 hellip Ak

In the formula such an element is counted the following number of times

Therefore each element is counted exactly once and thus the formula is correct

Euler Function

Given a number n how many numbers from 1 to n are relatively prime to n

When n is a prime number

then every number from 1 to n-1 is relatively prime to n

and so

When n is a prime power

then p 2p 3p 4p hellip n are not relatively prime to n

there are np = pc-1 of them

and other numbers are relatively prime to n

Therefore

Euler Function

Given a number n how many numbers from 1 to n are relatively prime to n

Suppose

Then p 2p 3p 4p hellip n are not relatively prime to n there are np of them

Also q 2q 3q 4q hellip n are not relatively prime to n and there are nq of them

Other numbers are relatively prime to n

Therefore

The numbers pq 2pq 3pq hellip n are subtracted twice and there are npq of them

So the correct answer is

Euler Function

Given a number n how many numbers from 1 to n are relatively prime to n

Let

Let S be the set of numbers from 1 to n that are not relatively prime to n

Let Ai be the set of numbers that are a multiple of piS = A1 [ A2 [ hellip [ An

For the intersection of k sets say A1 A2 A3hellip Ak

then every number in A1 Aring A2 Aring hellip Aring Ak is a multiple of p1p2hellippk

then |A1 Aring A2 Aring hellip Aring Ak| = np1p2hellippk

Euler Function

Given a number n how many numbers from 1 to n are relatively prime to n

Let

Let S be the set of numbers from 1 to n that are not relatively prime to n

Let Ai be the set of numbers that are a multiple of piS = A1 [ A2 [ hellip [ An

|A1 Aring A2 Aring hellip Aring Ak| = np1p2hellippk

When r=3 (only 3 distinct factors)

|A1 [ A2 [ A3|

= np1 + np2 + np3

- np1p2 ndash np1p3 ndash np2p3 + np1p2p3

|A1 [ A2 [ A3| = |A1| + |A2| + |A3|

ndash |A1 Aring A2| ndash |A1 Aring A3| ndash |A2 Aring A3|

+ |A1 Aring A2 Aring A3|

= n(1-p1)(1-p2)(1-p3)

Euler Function

Given a number n how many numbers from 1 to n are relatively prime to n

Let

Let S be the set of numbers from 1 to n that are not relatively prime to n

Let Ai be the set of numbers that are a multiple of piS = A1 [ A2 [ hellip [ An

|A1 Aring A2 Aring hellip Aring Ak| = np1p2hellippk

sum of sizes of all single setsndash sum of sizes of all 2-set intersections+ sum of sizes of all 3-set intersectionsndash sum of sizes of all 4-set intersectionshellip+ (ndash1)n+1 times sum of sizes of intersections of

n sets

|A1 [ A2 [ A3 [ hellip [ An|

|S| = |A1 [ A2 [ hellip [ An|

= n(1-p1)(1-p2)hellip(1-pn)

calculationshellip

  • Modular Arithmetic
  • PowerPoint Presentation
  • Slide 3
  • Slide 4
  • Slide 5
  • Slide 6
  • Slide 7
  • Slide 8
  • Slide 9
  • Slide 10
  • Slide 11
  • Slide 12
  • Slide 13
  • Slide 14
  • Slide 15
  • Slide 16
  • Slide 17
  • Slide 18
  • Slide 19
  • Slide 20
  • Slide 21
  • Slide 22
  • Slide 23
  • Slide 24
  • Slide 25
  • Slide 26
  • Slide 27
  • Slide 28
  • Slide 29
  • Slide 30
  • Slide 31
  • Slide 32
  • Slide 33
  • Slide 34
  • Slide 35
  • Slide 36
  • Slide 37
  • Slide 38
  • Slide 39
  • Slide 40
  • Slide 41
  • Slide 42
  • Slide 43
  • Slide 44
  • Slide 45
Page 5: Modular Arithmetic

Lemma If a c (mod n) and b d (mod n) then

a+b c+d (mod n)

Modular Addition

When you try to understand a statement like this

first think about the familiar cases eg n=10 or n=2

When n=2 it says that if a and c have the same parity

and b and d have the same parity

then a+b and c+d have the same parity

When n=10 it says that if a and c have the same last digit

and b and d have the same last digit

then a+b and c+d have the same last digit

And the lemma says that the same principle applied for all n

Lemma If a c (mod n) and b d (mod n) then

a+b c+d (mod n)

Modular Addition

Example 1 13 1 (mod 3) 25 1 (mod 3)

=gt 12 + 25 (mod 3) 1 + 1 (mod 3) 2 (mod 3)

Example 2 87 2 (mod 17) 222 1 (mod 17)

=gt 87 + 222 (mod 17) 2 + 1 (mod 17) 3 (mod 17)

Example 3 101 2 (mod 11) 141 -2 (mod 11)

=gt 101 + 141 (mod 11) 0 (mod 11)

In particular when computing a+b mod n we can first replace

a by a mod n and b by b mod n so that the computation is faster

Lemma If a c (mod n) and b d (mod n) then

a+b c+d (mod n)

Modular Addition

a c (mod n) =gt a = c + nx for some integer x

b d (mod n) =gt b = d + ny for some integer y

To show a+b c+d (mod n) it is equivalent to showing that n | (a+b-c-d)

Consider a+b-c-d

a+b-c-d = (c+nx) + (d+ny) ndash c ndashd = nx + ny

It is clear that n | nx + ny

Therefore n | a+b-c-d

We conclude that a+b c+d (mod n)

Proof

Lemma If a c (mod n) and b d (mod n) then

ab cd (mod n)

Modular Multiplication

Example 1 9876 6 (mod 10) 17642 2 (mod 10)

=gt 9876 17642 (mod 10) 6 2 (mod 10) 2 (mod 10)

Example 2 10987 1 (mod 2) 28663 1 (mod 2)

=gt 10987 28663 (mod 2) 1 (mod 2)

Example 3 1000 -1 (mod 7) 1000000 1 (mod 7)

=gt 1000 1000000 (mod 7) -1 1 (mod 7) -1 (mod 7)

In particular when computing ab mod n we can first replace

a by a mod n and b by b mod n so that the computation is faster

Lemma If a c (mod n) and b d (mod n) then

ab cd (mod n)

Modular Multiplication

a c (mod n) =gt a = c + nx for some integer x

b d (mod n) =gt b = d + ny for some integer y

To show ab cd (mod n) it is equivalent to showing that n | (ab-cd)

Consider ab-cd

ab-cd = (c+nx) (d+ny) ndash cd

= cd + dnx + cny + n2xy ndash cd = n(dx + cy + nxy)

It is clear that n | n(dx + cy + nxy) Therefore n | ab-cd

We conclude that ab cd (mod n)

Proof

This Lecture

bull Applications Fast exponentiation and fast division test

Fast Exponentiation

1444 mod 713

= 144 144 144 144 mod 713

= 20736 144 144 mod 713

= 59 144 144 mod 713

= 8496 144 mod 713

= 653 144 mod 713

= 94032 mod 713

= 629 mod 713

20736 20736 mod 713

= 59 59 mod 713

= 3481 mod 713

= 629 mod 713

Because 20736 59 (mod 713)

Because 653 8496 (mod 713)

shortcut

Repeated Squaring

14450 mod 713

= 14432 14416 1442 mod 713

= 64848559 mod 713

= 242

1442 mod 713 = 59

1444 mod 713 = 1442 1442 mod 713= 5959 mod 713= 629

1448 mod 713= 14441444 mod 713= 629629 mod 713= 639

14416 mod 713= 14481448 mod 713= 639639 mod 713= 485

14432 mod 713= 1441614416 mod 713= 485485 mod 713= 648

Note that 50 = 32 + 16 + 2

Fast Division Test

Using the basic rules for modular addition and modular multiplication

we can derive some quick test to see if a big number is divisible by a small number

Suppose we are given the decimal representation of a big number N

To test if N is divisible by a small number n of course we can do a division to check

But can we do faster

If n = 2 we just need to check whether the last digit of N is even or not

If n = 10 we just need to check whether the last digit of N is 0 or not

If n = 5 we just need to check whether the last digit of N is either 5 or 0 or not

What about when n=3 When n=7 When n=11

Fast Division Test

A number written in decimal divisible by 9 if and only if

the sum of its digits is a multiple of 9

Example 1 9333234513171 is divisible by 9

9+3+3+3+2+3+4+5+1+3+1+7+1 = 45 is divisible by 9

Example 2 128573649683 is not divisible by 9

1+2+8+5+7+3+6+4+9+6+8+3 = 62 is not divisible by 9

Claim A number written in decimal is divisible by 9 if and

only if

the sum of its digits is a multiple of 9Hint 10 1 (mod 9)

Let the decimal representation of N be dkdk-1dk-2hellipd1d0

This means that N = dk10k + dk-110k-1 + hellip + d110 + d0

Note that di10i mod 9

= (di) (10i mod 9) mod 9

= (di) (10 mod 9) (10 mod 9) hellip (10 mod 9) mod 9

= (di) (1 mod 9) (1 mod 9) hellip (1 mod 9) mod 9

= di mod 9

i terms

Fast Division Test

Rule of modular multiplication

Let the decimal representation of n be dkdk-1dk-2hellipd1d0

This means that N = dk10k + dk-110k-1 + hellip + d110 + d0

Note that di10i mod 9 = di mod 9

Hence N mod 9 = (dk10k + dk-110k-1 + hellip + d110 + d0) mod 9

= (dk10k mod 9 + dk-110k-1 mod 9 + hellip + d110 mod 9 + d0 mod 9) mod

9

= (dk mod 9 + dk-1 mod 9 + hellip + d1 mod 9 + d0 mod 9) mod 9

= (dk + dk-1 + hellip + d1 + d0) mod 9

Hint 10 1 (mod 9)

Fast Division Test

Claim A number written in decimal is divisible by 9 if and

only if

the sum of its digits is a multiple of 9

Rule of modular addition

By previous slide

The same procedure works to test whether N is divisible by n=3

Fast Division Test

What about n=11

Hint 10 -1 (mod 11)

Let the decimal representation of N be d92d91d90hellipd1d0

Then N is divisible by 11 if and only if

d92-d91+d90hellip-d1+d0 is divisible by 11

Why Try to work it out before your TA shows you

What about n=7

Hint 1000 -1 (mod 7)

This Lecture

bull Multiplicative inverse

The multiplicative inverse of a number a is another number arsquo such that

a middot arsquo 1 (mod n)

Multiplication Inverse

Does every number has a multiplicative inverse in modular arithmetic

For real numbers every nonzero number has a multiplicative inverse

For integers only 1 has a multiplicative inverse

An interesting property of modular arithmetic is that

there are multiplicative inverse for integers

For example 2 5 = 1 mod 3 so 5 is a multiplicative inverse

for 2 under modulo 3 (and vice versa)

Multiplication Inverse

Does every number has a multiplicative inverse in modular arithmetic

Multiplication Inverse

What is the pattern

Case Study

Why 2 does not have a multiplicative inverse under modulo 6

Suppose it has a multiplicative inverse y

2y 1 (mod 6)

=gt 2y = 1 + 6x for some integer x

=gt y = frac12 + 3x

This is a contradiction since both x and y are integers

Necessary Condition

Claim An integer k does not have an multiplicative inverse under modulo n

if k and n have a common factor gt= 2 (gcd(kn) gt= 2)

Proof

This claim says that for k to have a multiplicative inverse modulo n

then a necessary condition is that k and n do not have a common factor gt= 2

Suppose by contradiction that there is an inverse krsquo for k such that

krsquok = 1 (mod n)

Then krsquok = 1 + xn for some integer x

Since both k and n have a common factor say cgt=2

then k=ck1 and n=cn1 for some integers k1 and n1

So krsquock1 = 1 + xcn1

Then krsquok1 = 1c + xn1

This is a contradiction since the LHS is an integer but the RHS is not

Sufficient Condition

What about if gcd(kn)=1

Would k always have an multiplicative inverse under modulo n

For example gcd(37) = 1 3middot5 1 (mod 7)

gcd(89) = 1

gcd(411) = 1 4middot3 1 (mod 11)

8middot8 1 (mod 9)

It seems that there is always an inverse in such a case but why

gcd(89) = 1 8s + 9t = 1 for some integers s and t

8s = 1 ndash 9t

8s 1 (mod 9)gcd(89) = spc(89)

Theorem If gcd(kn)=1 then have krsquo such that

kmiddotkrsquo 1 (mod n)

Proof Since gcd(kn)=1 there exist s and t so that sk + tn = 1

So tn = 1 - sk

This means n | 1 ndash sk

This means that 1 ndash sk 0 (mod n)

This means that 1 sk (mod n)

So krsquo = s is an multiplicative inverse for k

Sufficient Condition

gcd(kn)=spc(kn)

The multiplicative inverse can be computed by the extended Euclidean algorithm

Corollary k has a multiplicative inverse mod n if and only if gcd(kn)=1

This Lecture

bull Fermatrsquos little theorem

Cancellation

There is no general cancellation in modular arithmetic

Note that (mod n) is very similar to =

If a b (mod n) then a+c b+c (mod n)

If a b (mod n) then ac bc (mod n)

However if ac bc (mod n)

it is not necessarily true that a b (mod n)

For example 4middot2 1middot2 (mod 6) but 4 1 (mod 6)

3middot4 1middot4 (mod 8) but 3 1 (mod 8)

4middot3 1middot3 (mod 9) but 4 1 (mod 9)

Observation In all the above examples c and n have a common factor

Cancellation

Claim Assume gcd(kn) = 1 If imiddotk jmiddotk (mod n) then i j (mod n)

For example multiplicative inverse always exists if n is a prime

Proof Since gcd(kn) = 1 there exists krsquo such that kkrsquo 1 (mod n)

imiddotk jmiddotk (mod n)

=gt imiddotkmiddotkrsquo jmiddotkmiddotkrsquo (mod n)

=gt i j (mod n)

Remarks (Optional) This makes arithmetic modulo prime a field

a structure that ldquobehaves likerdquo real numbers

Arithmetic modulo prime is very useful in coding theory

In particular when p is a prime amp k not a multiple of p then gcd(kp)=1

If i j (mod p) then imiddotk jmiddotk (mod p)

Therefore

k mod p 2k mod p hellip (p-1)k mod p

are all different numbers

Fermatrsquos Little Theorem

Claim 1 Assume gcd(kn) = 1 If imiddotk jmiddotk (mod n) then i j (mod n)

Claim 2 Assume gcd(kn) = 1 If i j (mod n) then imiddotk jmiddotk (mod n)

For example when p=7 and k=3

3 mod 7 = 3 2middot3 mod 7 = 6 3middot3 mod 7 = 2 4middot3 mod 7 = 5 5middot3 mod 7 = 1 6middot3 mod 7 = 4

Notice that in the above example every number from 1 to 6 appears exactly once

In particular when p is a prime amp k not a multiple of p then gcd(kp)=1

If i j (mod p) then imiddotk jmiddotk (mod p)

Therefore

k mod p 2k mod p hellip (p-1)k mod p

are all different numbers

Fermatrsquos Little Theorem

Each of ik mod p cannot be equal to 0 because p is a prime number

Let ci = ik mod p

So 1 lt= c1 lt= p-1 1 lt= c2 lt= p-1 hellip 1lt = cp-1 lt= p-1

By the above we know that c1c2hellipcp-2cp-1 are all different

So for each i from 1 to p-1 there is exactly one cj such that cj = i

Therefore we have

(k mod p)middot(2k mod p)middothellipmiddot((p-1)k mod p) = c1middotc2middothellipmiddotcp-2middotcp-1 = 1middot2middot3hellipmiddot(p-2)middot(p-1)

Fermatrsquos Little Theorem

1 kp-1 (mod p)

Theorem If p is prime amp k not a multiple of p

For example when p=5 k=4 we have kp-1 mod p = 44 mod 5 = 1

ldquoProofrdquo

4middot3middot2middot1 [(4 mod 5) (2middot4 mod 5) (3middot4 mod 5) (4middot4 mod 5)] (mod 5)

[4 middot (2middot4) middot (3middot4) middot (4middot4)] (mod 5)

[44 middot (1middot2middot3middot4)] (mod 5)

Since gcd(1middot2middot3middot4 5)=1 we can cancel 1middot2middot3middot4 on both sides

This implies

1 44 (mod 5)

By the previous slide or direct calculation

Fermatrsquos Little Theorem

1 kp-1 (mod p)

Theorem If p is prime amp k not a multiple of p

Proof

1middot2middotmiddotmiddot(p-1) (k mod p middot 2k mod pmiddothellipmiddot(p-1)k mod p) mod p

(kmiddot2k middotmiddotmiddot (p-1)k) mod p

(kp-1)middot1middot2 middotmiddotmiddot (p-1) (mod p)

So by cancelling 1middot2 middotmiddotmiddot (p-1) on both sides applying

Claim 1

(we can cancel them because gcd(1middot2 middotmiddotmiddot (p-1) p)=1)

we have

1 kp-1 (mod p)

By 2 slides before

By the multiplication rule

Wilsonrsquos Theorem

Theorem p is a prime if and only if

(p-1) -1 (mod p)

First we consider the easy direction

If p is not a prime assume p gt= 5 (for p=4 3 2 (mod 4) )

Then p=qr for some 2 lt= q lt p and 2 lt= r lt p

If q ne r then both q and r appear in (p-1)

and so (p-1) 0 (mod p)

If q = r then p = q2 gt 2q (since we assume p gt 5 and thus q gt 2)

then both q and 2q are in (p-1)

and so again (p-1) 0 (mod p)

Wilsonrsquos Theorem

Theorem p is a prime if and only if

(p-1) -1 (mod p)

To prove the more interesting direction first we need a lemma

Lemma If p is a prime number

x2 1 (mod p) if and only if x 1 (mod p) or x -1 (mod p)

Proof x2 1 (mod p)

iff p | x2 - 1

iff p | (x ndash 1)(x + 1)

iff p | (x ndash 1) or p | (x+1)

iff x 1 (mod p) or x -1 (mod p)

Lemma p prime and p|amiddotb iff p|a or p|b

Wilsonrsquos Theorem

Theorem p is a prime if and only if

(p-1) -1 (mod p)

Letrsquos get the proof idea by considering a concrete example

10

1middot2middot3middot4middot5middot6middot7middot8middot9middot10 mod 11

1middot10middot(2middot6)middot(3middot4)middot(5middot9)middot(7middot8) mod 11

1middot-1middot(1)middot(1)middot(1)middot(1) mod 11

-1 mod 11

Besides 1 and 10 the remaining numbers are paired up into multiplicative inverse

Wilsonrsquos Theorem

Theorem p is a prime if and only if

(p-1) -1 (mod p)

ProofSince p is a prime every number from 1 to p-1 has a multiplicative inverse

By the Lemma every number 2 lt= k lt= p-2 has an inverse krsquo with knekrsquo

Since p is odd the numbers from 2 to p-2 can be grouped into pairs

(a1b1)(a2b2)hellip(a(p-3)2b(p-3)2) so that aibi 1 (mod p)

Therefore (p-1) 1middot(p-1)middot2middot3middotmiddotmiddotmiddotmiddot(p-3)middot(p-2) (mod p)

1middot(p-1)middot(a1b1)middot(a2b2)middotmiddotmiddotmiddotmiddot(a(p-3)2b(p-3)2) (mod p)

1middot(-1)middot(1)middot(1)middotmiddotmiddotmiddotmiddot(1) (mod p)

-1 (mod p)

This Lecture

bull Eulerrsquos phi function

Inclusion-Exclusion (n sets)

What is the inclusion-exclusion formula for the union of n sets

sum of sizes of all single setsndash sum of sizes of all 2-set intersections+ sum of sizes of all 3-set intersectionsndash sum of sizes of all 4-set intersectionshellip+ (ndash1)n+1 times sum of sizes of intersections of all n

sets

1 2 nA A A

1

12 1

( 1)n

ki

S nk i S

S k

A

Inclusion-Exclusion (n sets)

Inclusion-Exclusion (n sets)

sum of sizes of all single setsndash sum of sizes of all 2-set intersections+ sum of sizes of all 3-set intersectionsndash sum of sizes of all 4-set intersectionshellip+ (ndash1)n+1 times sum of sizes of intersections of all n

sets

|A1 [ A2 [ A3 [ hellip [ An|

We want to show that every element is counted exactly once

Consider an element which belongs to exactly k sets say A1 A2 A3 hellip Ak

In the formula such an element is counted the following number of times

Therefore each element is counted exactly once and thus the formula is correct

Euler Function

Given a number n how many numbers from 1 to n are relatively prime to n

When n is a prime number

then every number from 1 to n-1 is relatively prime to n

and so

When n is a prime power

then p 2p 3p 4p hellip n are not relatively prime to n

there are np = pc-1 of them

and other numbers are relatively prime to n

Therefore

Euler Function

Given a number n how many numbers from 1 to n are relatively prime to n

Suppose

Then p 2p 3p 4p hellip n are not relatively prime to n there are np of them

Also q 2q 3q 4q hellip n are not relatively prime to n and there are nq of them

Other numbers are relatively prime to n

Therefore

The numbers pq 2pq 3pq hellip n are subtracted twice and there are npq of them

So the correct answer is

Euler Function

Given a number n how many numbers from 1 to n are relatively prime to n

Let

Let S be the set of numbers from 1 to n that are not relatively prime to n

Let Ai be the set of numbers that are a multiple of piS = A1 [ A2 [ hellip [ An

For the intersection of k sets say A1 A2 A3hellip Ak

then every number in A1 Aring A2 Aring hellip Aring Ak is a multiple of p1p2hellippk

then |A1 Aring A2 Aring hellip Aring Ak| = np1p2hellippk

Euler Function

Given a number n how many numbers from 1 to n are relatively prime to n

Let

Let S be the set of numbers from 1 to n that are not relatively prime to n

Let Ai be the set of numbers that are a multiple of piS = A1 [ A2 [ hellip [ An

|A1 Aring A2 Aring hellip Aring Ak| = np1p2hellippk

When r=3 (only 3 distinct factors)

|A1 [ A2 [ A3|

= np1 + np2 + np3

- np1p2 ndash np1p3 ndash np2p3 + np1p2p3

|A1 [ A2 [ A3| = |A1| + |A2| + |A3|

ndash |A1 Aring A2| ndash |A1 Aring A3| ndash |A2 Aring A3|

+ |A1 Aring A2 Aring A3|

= n(1-p1)(1-p2)(1-p3)

Euler Function

Given a number n how many numbers from 1 to n are relatively prime to n

Let

Let S be the set of numbers from 1 to n that are not relatively prime to n

Let Ai be the set of numbers that are a multiple of piS = A1 [ A2 [ hellip [ An

|A1 Aring A2 Aring hellip Aring Ak| = np1p2hellippk

sum of sizes of all single setsndash sum of sizes of all 2-set intersections+ sum of sizes of all 3-set intersectionsndash sum of sizes of all 4-set intersectionshellip+ (ndash1)n+1 times sum of sizes of intersections of

n sets

|A1 [ A2 [ A3 [ hellip [ An|

|S| = |A1 [ A2 [ hellip [ An|

= n(1-p1)(1-p2)hellip(1-pn)

calculationshellip

  • Modular Arithmetic
  • PowerPoint Presentation
  • Slide 3
  • Slide 4
  • Slide 5
  • Slide 6
  • Slide 7
  • Slide 8
  • Slide 9
  • Slide 10
  • Slide 11
  • Slide 12
  • Slide 13
  • Slide 14
  • Slide 15
  • Slide 16
  • Slide 17
  • Slide 18
  • Slide 19
  • Slide 20
  • Slide 21
  • Slide 22
  • Slide 23
  • Slide 24
  • Slide 25
  • Slide 26
  • Slide 27
  • Slide 28
  • Slide 29
  • Slide 30
  • Slide 31
  • Slide 32
  • Slide 33
  • Slide 34
  • Slide 35
  • Slide 36
  • Slide 37
  • Slide 38
  • Slide 39
  • Slide 40
  • Slide 41
  • Slide 42
  • Slide 43
  • Slide 44
  • Slide 45
Page 6: Modular Arithmetic

Lemma If a c (mod n) and b d (mod n) then

a+b c+d (mod n)

Modular Addition

Example 1 13 1 (mod 3) 25 1 (mod 3)

=gt 12 + 25 (mod 3) 1 + 1 (mod 3) 2 (mod 3)

Example 2 87 2 (mod 17) 222 1 (mod 17)

=gt 87 + 222 (mod 17) 2 + 1 (mod 17) 3 (mod 17)

Example 3 101 2 (mod 11) 141 -2 (mod 11)

=gt 101 + 141 (mod 11) 0 (mod 11)

In particular when computing a+b mod n we can first replace

a by a mod n and b by b mod n so that the computation is faster

Lemma If a c (mod n) and b d (mod n) then

a+b c+d (mod n)

Modular Addition

a c (mod n) =gt a = c + nx for some integer x

b d (mod n) =gt b = d + ny for some integer y

To show a+b c+d (mod n) it is equivalent to showing that n | (a+b-c-d)

Consider a+b-c-d

a+b-c-d = (c+nx) + (d+ny) ndash c ndashd = nx + ny

It is clear that n | nx + ny

Therefore n | a+b-c-d

We conclude that a+b c+d (mod n)

Proof

Lemma If a c (mod n) and b d (mod n) then

ab cd (mod n)

Modular Multiplication

Example 1 9876 6 (mod 10) 17642 2 (mod 10)

=gt 9876 17642 (mod 10) 6 2 (mod 10) 2 (mod 10)

Example 2 10987 1 (mod 2) 28663 1 (mod 2)

=gt 10987 28663 (mod 2) 1 (mod 2)

Example 3 1000 -1 (mod 7) 1000000 1 (mod 7)

=gt 1000 1000000 (mod 7) -1 1 (mod 7) -1 (mod 7)

In particular when computing ab mod n we can first replace

a by a mod n and b by b mod n so that the computation is faster

Lemma If a c (mod n) and b d (mod n) then

ab cd (mod n)

Modular Multiplication

a c (mod n) =gt a = c + nx for some integer x

b d (mod n) =gt b = d + ny for some integer y

To show ab cd (mod n) it is equivalent to showing that n | (ab-cd)

Consider ab-cd

ab-cd = (c+nx) (d+ny) ndash cd

= cd + dnx + cny + n2xy ndash cd = n(dx + cy + nxy)

It is clear that n | n(dx + cy + nxy) Therefore n | ab-cd

We conclude that ab cd (mod n)

Proof

This Lecture

bull Applications Fast exponentiation and fast division test

Fast Exponentiation

1444 mod 713

= 144 144 144 144 mod 713

= 20736 144 144 mod 713

= 59 144 144 mod 713

= 8496 144 mod 713

= 653 144 mod 713

= 94032 mod 713

= 629 mod 713

20736 20736 mod 713

= 59 59 mod 713

= 3481 mod 713

= 629 mod 713

Because 20736 59 (mod 713)

Because 653 8496 (mod 713)

shortcut

Repeated Squaring

14450 mod 713

= 14432 14416 1442 mod 713

= 64848559 mod 713

= 242

1442 mod 713 = 59

1444 mod 713 = 1442 1442 mod 713= 5959 mod 713= 629

1448 mod 713= 14441444 mod 713= 629629 mod 713= 639

14416 mod 713= 14481448 mod 713= 639639 mod 713= 485

14432 mod 713= 1441614416 mod 713= 485485 mod 713= 648

Note that 50 = 32 + 16 + 2

Fast Division Test

Using the basic rules for modular addition and modular multiplication

we can derive some quick test to see if a big number is divisible by a small number

Suppose we are given the decimal representation of a big number N

To test if N is divisible by a small number n of course we can do a division to check

But can we do faster

If n = 2 we just need to check whether the last digit of N is even or not

If n = 10 we just need to check whether the last digit of N is 0 or not

If n = 5 we just need to check whether the last digit of N is either 5 or 0 or not

What about when n=3 When n=7 When n=11

Fast Division Test

A number written in decimal divisible by 9 if and only if

the sum of its digits is a multiple of 9

Example 1 9333234513171 is divisible by 9

9+3+3+3+2+3+4+5+1+3+1+7+1 = 45 is divisible by 9

Example 2 128573649683 is not divisible by 9

1+2+8+5+7+3+6+4+9+6+8+3 = 62 is not divisible by 9

Claim A number written in decimal is divisible by 9 if and

only if

the sum of its digits is a multiple of 9Hint 10 1 (mod 9)

Let the decimal representation of N be dkdk-1dk-2hellipd1d0

This means that N = dk10k + dk-110k-1 + hellip + d110 + d0

Note that di10i mod 9

= (di) (10i mod 9) mod 9

= (di) (10 mod 9) (10 mod 9) hellip (10 mod 9) mod 9

= (di) (1 mod 9) (1 mod 9) hellip (1 mod 9) mod 9

= di mod 9

i terms

Fast Division Test

Rule of modular multiplication

Let the decimal representation of n be dkdk-1dk-2hellipd1d0

This means that N = dk10k + dk-110k-1 + hellip + d110 + d0

Note that di10i mod 9 = di mod 9

Hence N mod 9 = (dk10k + dk-110k-1 + hellip + d110 + d0) mod 9

= (dk10k mod 9 + dk-110k-1 mod 9 + hellip + d110 mod 9 + d0 mod 9) mod

9

= (dk mod 9 + dk-1 mod 9 + hellip + d1 mod 9 + d0 mod 9) mod 9

= (dk + dk-1 + hellip + d1 + d0) mod 9

Hint 10 1 (mod 9)

Fast Division Test

Claim A number written in decimal is divisible by 9 if and

only if

the sum of its digits is a multiple of 9

Rule of modular addition

By previous slide

The same procedure works to test whether N is divisible by n=3

Fast Division Test

What about n=11

Hint 10 -1 (mod 11)

Let the decimal representation of N be d92d91d90hellipd1d0

Then N is divisible by 11 if and only if

d92-d91+d90hellip-d1+d0 is divisible by 11

Why Try to work it out before your TA shows you

What about n=7

Hint 1000 -1 (mod 7)

This Lecture

bull Multiplicative inverse

The multiplicative inverse of a number a is another number arsquo such that

a middot arsquo 1 (mod n)

Multiplication Inverse

Does every number has a multiplicative inverse in modular arithmetic

For real numbers every nonzero number has a multiplicative inverse

For integers only 1 has a multiplicative inverse

An interesting property of modular arithmetic is that

there are multiplicative inverse for integers

For example 2 5 = 1 mod 3 so 5 is a multiplicative inverse

for 2 under modulo 3 (and vice versa)

Multiplication Inverse

Does every number has a multiplicative inverse in modular arithmetic

Multiplication Inverse

What is the pattern

Case Study

Why 2 does not have a multiplicative inverse under modulo 6

Suppose it has a multiplicative inverse y

2y 1 (mod 6)

=gt 2y = 1 + 6x for some integer x

=gt y = frac12 + 3x

This is a contradiction since both x and y are integers

Necessary Condition

Claim An integer k does not have an multiplicative inverse under modulo n

if k and n have a common factor gt= 2 (gcd(kn) gt= 2)

Proof

This claim says that for k to have a multiplicative inverse modulo n

then a necessary condition is that k and n do not have a common factor gt= 2

Suppose by contradiction that there is an inverse krsquo for k such that

krsquok = 1 (mod n)

Then krsquok = 1 + xn for some integer x

Since both k and n have a common factor say cgt=2

then k=ck1 and n=cn1 for some integers k1 and n1

So krsquock1 = 1 + xcn1

Then krsquok1 = 1c + xn1

This is a contradiction since the LHS is an integer but the RHS is not

Sufficient Condition

What about if gcd(kn)=1

Would k always have an multiplicative inverse under modulo n

For example gcd(37) = 1 3middot5 1 (mod 7)

gcd(89) = 1

gcd(411) = 1 4middot3 1 (mod 11)

8middot8 1 (mod 9)

It seems that there is always an inverse in such a case but why

gcd(89) = 1 8s + 9t = 1 for some integers s and t

8s = 1 ndash 9t

8s 1 (mod 9)gcd(89) = spc(89)

Theorem If gcd(kn)=1 then have krsquo such that

kmiddotkrsquo 1 (mod n)

Proof Since gcd(kn)=1 there exist s and t so that sk + tn = 1

So tn = 1 - sk

This means n | 1 ndash sk

This means that 1 ndash sk 0 (mod n)

This means that 1 sk (mod n)

So krsquo = s is an multiplicative inverse for k

Sufficient Condition

gcd(kn)=spc(kn)

The multiplicative inverse can be computed by the extended Euclidean algorithm

Corollary k has a multiplicative inverse mod n if and only if gcd(kn)=1

This Lecture

bull Fermatrsquos little theorem

Cancellation

There is no general cancellation in modular arithmetic

Note that (mod n) is very similar to =

If a b (mod n) then a+c b+c (mod n)

If a b (mod n) then ac bc (mod n)

However if ac bc (mod n)

it is not necessarily true that a b (mod n)

For example 4middot2 1middot2 (mod 6) but 4 1 (mod 6)

3middot4 1middot4 (mod 8) but 3 1 (mod 8)

4middot3 1middot3 (mod 9) but 4 1 (mod 9)

Observation In all the above examples c and n have a common factor

Cancellation

Claim Assume gcd(kn) = 1 If imiddotk jmiddotk (mod n) then i j (mod n)

For example multiplicative inverse always exists if n is a prime

Proof Since gcd(kn) = 1 there exists krsquo such that kkrsquo 1 (mod n)

imiddotk jmiddotk (mod n)

=gt imiddotkmiddotkrsquo jmiddotkmiddotkrsquo (mod n)

=gt i j (mod n)

Remarks (Optional) This makes arithmetic modulo prime a field

a structure that ldquobehaves likerdquo real numbers

Arithmetic modulo prime is very useful in coding theory

In particular when p is a prime amp k not a multiple of p then gcd(kp)=1

If i j (mod p) then imiddotk jmiddotk (mod p)

Therefore

k mod p 2k mod p hellip (p-1)k mod p

are all different numbers

Fermatrsquos Little Theorem

Claim 1 Assume gcd(kn) = 1 If imiddotk jmiddotk (mod n) then i j (mod n)

Claim 2 Assume gcd(kn) = 1 If i j (mod n) then imiddotk jmiddotk (mod n)

For example when p=7 and k=3

3 mod 7 = 3 2middot3 mod 7 = 6 3middot3 mod 7 = 2 4middot3 mod 7 = 5 5middot3 mod 7 = 1 6middot3 mod 7 = 4

Notice that in the above example every number from 1 to 6 appears exactly once

In particular when p is a prime amp k not a multiple of p then gcd(kp)=1

If i j (mod p) then imiddotk jmiddotk (mod p)

Therefore

k mod p 2k mod p hellip (p-1)k mod p

are all different numbers

Fermatrsquos Little Theorem

Each of ik mod p cannot be equal to 0 because p is a prime number

Let ci = ik mod p

So 1 lt= c1 lt= p-1 1 lt= c2 lt= p-1 hellip 1lt = cp-1 lt= p-1

By the above we know that c1c2hellipcp-2cp-1 are all different

So for each i from 1 to p-1 there is exactly one cj such that cj = i

Therefore we have

(k mod p)middot(2k mod p)middothellipmiddot((p-1)k mod p) = c1middotc2middothellipmiddotcp-2middotcp-1 = 1middot2middot3hellipmiddot(p-2)middot(p-1)

Fermatrsquos Little Theorem

1 kp-1 (mod p)

Theorem If p is prime amp k not a multiple of p

For example when p=5 k=4 we have kp-1 mod p = 44 mod 5 = 1

ldquoProofrdquo

4middot3middot2middot1 [(4 mod 5) (2middot4 mod 5) (3middot4 mod 5) (4middot4 mod 5)] (mod 5)

[4 middot (2middot4) middot (3middot4) middot (4middot4)] (mod 5)

[44 middot (1middot2middot3middot4)] (mod 5)

Since gcd(1middot2middot3middot4 5)=1 we can cancel 1middot2middot3middot4 on both sides

This implies

1 44 (mod 5)

By the previous slide or direct calculation

Fermatrsquos Little Theorem

1 kp-1 (mod p)

Theorem If p is prime amp k not a multiple of p

Proof

1middot2middotmiddotmiddot(p-1) (k mod p middot 2k mod pmiddothellipmiddot(p-1)k mod p) mod p

(kmiddot2k middotmiddotmiddot (p-1)k) mod p

(kp-1)middot1middot2 middotmiddotmiddot (p-1) (mod p)

So by cancelling 1middot2 middotmiddotmiddot (p-1) on both sides applying

Claim 1

(we can cancel them because gcd(1middot2 middotmiddotmiddot (p-1) p)=1)

we have

1 kp-1 (mod p)

By 2 slides before

By the multiplication rule

Wilsonrsquos Theorem

Theorem p is a prime if and only if

(p-1) -1 (mod p)

First we consider the easy direction

If p is not a prime assume p gt= 5 (for p=4 3 2 (mod 4) )

Then p=qr for some 2 lt= q lt p and 2 lt= r lt p

If q ne r then both q and r appear in (p-1)

and so (p-1) 0 (mod p)

If q = r then p = q2 gt 2q (since we assume p gt 5 and thus q gt 2)

then both q and 2q are in (p-1)

and so again (p-1) 0 (mod p)

Wilsonrsquos Theorem

Theorem p is a prime if and only if

(p-1) -1 (mod p)

To prove the more interesting direction first we need a lemma

Lemma If p is a prime number

x2 1 (mod p) if and only if x 1 (mod p) or x -1 (mod p)

Proof x2 1 (mod p)

iff p | x2 - 1

iff p | (x ndash 1)(x + 1)

iff p | (x ndash 1) or p | (x+1)

iff x 1 (mod p) or x -1 (mod p)

Lemma p prime and p|amiddotb iff p|a or p|b

Wilsonrsquos Theorem

Theorem p is a prime if and only if

(p-1) -1 (mod p)

Letrsquos get the proof idea by considering a concrete example

10

1middot2middot3middot4middot5middot6middot7middot8middot9middot10 mod 11

1middot10middot(2middot6)middot(3middot4)middot(5middot9)middot(7middot8) mod 11

1middot-1middot(1)middot(1)middot(1)middot(1) mod 11

-1 mod 11

Besides 1 and 10 the remaining numbers are paired up into multiplicative inverse

Wilsonrsquos Theorem

Theorem p is a prime if and only if

(p-1) -1 (mod p)

ProofSince p is a prime every number from 1 to p-1 has a multiplicative inverse

By the Lemma every number 2 lt= k lt= p-2 has an inverse krsquo with knekrsquo

Since p is odd the numbers from 2 to p-2 can be grouped into pairs

(a1b1)(a2b2)hellip(a(p-3)2b(p-3)2) so that aibi 1 (mod p)

Therefore (p-1) 1middot(p-1)middot2middot3middotmiddotmiddotmiddotmiddot(p-3)middot(p-2) (mod p)

1middot(p-1)middot(a1b1)middot(a2b2)middotmiddotmiddotmiddotmiddot(a(p-3)2b(p-3)2) (mod p)

1middot(-1)middot(1)middot(1)middotmiddotmiddotmiddotmiddot(1) (mod p)

-1 (mod p)

This Lecture

bull Eulerrsquos phi function

Inclusion-Exclusion (n sets)

What is the inclusion-exclusion formula for the union of n sets

sum of sizes of all single setsndash sum of sizes of all 2-set intersections+ sum of sizes of all 3-set intersectionsndash sum of sizes of all 4-set intersectionshellip+ (ndash1)n+1 times sum of sizes of intersections of all n

sets

1 2 nA A A

1

12 1

( 1)n

ki

S nk i S

S k

A

Inclusion-Exclusion (n sets)

Inclusion-Exclusion (n sets)

sum of sizes of all single setsndash sum of sizes of all 2-set intersections+ sum of sizes of all 3-set intersectionsndash sum of sizes of all 4-set intersectionshellip+ (ndash1)n+1 times sum of sizes of intersections of all n

sets

|A1 [ A2 [ A3 [ hellip [ An|

We want to show that every element is counted exactly once

Consider an element which belongs to exactly k sets say A1 A2 A3 hellip Ak

In the formula such an element is counted the following number of times

Therefore each element is counted exactly once and thus the formula is correct

Euler Function

Given a number n how many numbers from 1 to n are relatively prime to n

When n is a prime number

then every number from 1 to n-1 is relatively prime to n

and so

When n is a prime power

then p 2p 3p 4p hellip n are not relatively prime to n

there are np = pc-1 of them

and other numbers are relatively prime to n

Therefore

Euler Function

Given a number n how many numbers from 1 to n are relatively prime to n

Suppose

Then p 2p 3p 4p hellip n are not relatively prime to n there are np of them

Also q 2q 3q 4q hellip n are not relatively prime to n and there are nq of them

Other numbers are relatively prime to n

Therefore

The numbers pq 2pq 3pq hellip n are subtracted twice and there are npq of them

So the correct answer is

Euler Function

Given a number n how many numbers from 1 to n are relatively prime to n

Let

Let S be the set of numbers from 1 to n that are not relatively prime to n

Let Ai be the set of numbers that are a multiple of piS = A1 [ A2 [ hellip [ An

For the intersection of k sets say A1 A2 A3hellip Ak

then every number in A1 Aring A2 Aring hellip Aring Ak is a multiple of p1p2hellippk

then |A1 Aring A2 Aring hellip Aring Ak| = np1p2hellippk

Euler Function

Given a number n how many numbers from 1 to n are relatively prime to n

Let

Let S be the set of numbers from 1 to n that are not relatively prime to n

Let Ai be the set of numbers that are a multiple of piS = A1 [ A2 [ hellip [ An

|A1 Aring A2 Aring hellip Aring Ak| = np1p2hellippk

When r=3 (only 3 distinct factors)

|A1 [ A2 [ A3|

= np1 + np2 + np3

- np1p2 ndash np1p3 ndash np2p3 + np1p2p3

|A1 [ A2 [ A3| = |A1| + |A2| + |A3|

ndash |A1 Aring A2| ndash |A1 Aring A3| ndash |A2 Aring A3|

+ |A1 Aring A2 Aring A3|

= n(1-p1)(1-p2)(1-p3)

Euler Function

Given a number n how many numbers from 1 to n are relatively prime to n

Let

Let S be the set of numbers from 1 to n that are not relatively prime to n

Let Ai be the set of numbers that are a multiple of piS = A1 [ A2 [ hellip [ An

|A1 Aring A2 Aring hellip Aring Ak| = np1p2hellippk

sum of sizes of all single setsndash sum of sizes of all 2-set intersections+ sum of sizes of all 3-set intersectionsndash sum of sizes of all 4-set intersectionshellip+ (ndash1)n+1 times sum of sizes of intersections of

n sets

|A1 [ A2 [ A3 [ hellip [ An|

|S| = |A1 [ A2 [ hellip [ An|

= n(1-p1)(1-p2)hellip(1-pn)

calculationshellip

  • Modular Arithmetic
  • PowerPoint Presentation
  • Slide 3
  • Slide 4
  • Slide 5
  • Slide 6
  • Slide 7
  • Slide 8
  • Slide 9
  • Slide 10
  • Slide 11
  • Slide 12
  • Slide 13
  • Slide 14
  • Slide 15
  • Slide 16
  • Slide 17
  • Slide 18
  • Slide 19
  • Slide 20
  • Slide 21
  • Slide 22
  • Slide 23
  • Slide 24
  • Slide 25
  • Slide 26
  • Slide 27
  • Slide 28
  • Slide 29
  • Slide 30
  • Slide 31
  • Slide 32
  • Slide 33
  • Slide 34
  • Slide 35
  • Slide 36
  • Slide 37
  • Slide 38
  • Slide 39
  • Slide 40
  • Slide 41
  • Slide 42
  • Slide 43
  • Slide 44
  • Slide 45
Page 7: Modular Arithmetic

Lemma If a c (mod n) and b d (mod n) then

a+b c+d (mod n)

Modular Addition

a c (mod n) =gt a = c + nx for some integer x

b d (mod n) =gt b = d + ny for some integer y

To show a+b c+d (mod n) it is equivalent to showing that n | (a+b-c-d)

Consider a+b-c-d

a+b-c-d = (c+nx) + (d+ny) ndash c ndashd = nx + ny

It is clear that n | nx + ny

Therefore n | a+b-c-d

We conclude that a+b c+d (mod n)

Proof

Lemma If a c (mod n) and b d (mod n) then

ab cd (mod n)

Modular Multiplication

Example 1 9876 6 (mod 10) 17642 2 (mod 10)

=gt 9876 17642 (mod 10) 6 2 (mod 10) 2 (mod 10)

Example 2 10987 1 (mod 2) 28663 1 (mod 2)

=gt 10987 28663 (mod 2) 1 (mod 2)

Example 3 1000 -1 (mod 7) 1000000 1 (mod 7)

=gt 1000 1000000 (mod 7) -1 1 (mod 7) -1 (mod 7)

In particular when computing ab mod n we can first replace

a by a mod n and b by b mod n so that the computation is faster

Lemma If a c (mod n) and b d (mod n) then

ab cd (mod n)

Modular Multiplication

a c (mod n) =gt a = c + nx for some integer x

b d (mod n) =gt b = d + ny for some integer y

To show ab cd (mod n) it is equivalent to showing that n | (ab-cd)

Consider ab-cd

ab-cd = (c+nx) (d+ny) ndash cd

= cd + dnx + cny + n2xy ndash cd = n(dx + cy + nxy)

It is clear that n | n(dx + cy + nxy) Therefore n | ab-cd

We conclude that ab cd (mod n)

Proof

This Lecture

bull Applications Fast exponentiation and fast division test

Fast Exponentiation

1444 mod 713

= 144 144 144 144 mod 713

= 20736 144 144 mod 713

= 59 144 144 mod 713

= 8496 144 mod 713

= 653 144 mod 713

= 94032 mod 713

= 629 mod 713

20736 20736 mod 713

= 59 59 mod 713

= 3481 mod 713

= 629 mod 713

Because 20736 59 (mod 713)

Because 653 8496 (mod 713)

shortcut

Repeated Squaring

14450 mod 713

= 14432 14416 1442 mod 713

= 64848559 mod 713

= 242

1442 mod 713 = 59

1444 mod 713 = 1442 1442 mod 713= 5959 mod 713= 629

1448 mod 713= 14441444 mod 713= 629629 mod 713= 639

14416 mod 713= 14481448 mod 713= 639639 mod 713= 485

14432 mod 713= 1441614416 mod 713= 485485 mod 713= 648

Note that 50 = 32 + 16 + 2

Fast Division Test

Using the basic rules for modular addition and modular multiplication

we can derive some quick test to see if a big number is divisible by a small number

Suppose we are given the decimal representation of a big number N

To test if N is divisible by a small number n of course we can do a division to check

But can we do faster

If n = 2 we just need to check whether the last digit of N is even or not

If n = 10 we just need to check whether the last digit of N is 0 or not

If n = 5 we just need to check whether the last digit of N is either 5 or 0 or not

What about when n=3 When n=7 When n=11

Fast Division Test

A number written in decimal divisible by 9 if and only if

the sum of its digits is a multiple of 9

Example 1 9333234513171 is divisible by 9

9+3+3+3+2+3+4+5+1+3+1+7+1 = 45 is divisible by 9

Example 2 128573649683 is not divisible by 9

1+2+8+5+7+3+6+4+9+6+8+3 = 62 is not divisible by 9

Claim A number written in decimal is divisible by 9 if and

only if

the sum of its digits is a multiple of 9Hint 10 1 (mod 9)

Let the decimal representation of N be dkdk-1dk-2hellipd1d0

This means that N = dk10k + dk-110k-1 + hellip + d110 + d0

Note that di10i mod 9

= (di) (10i mod 9) mod 9

= (di) (10 mod 9) (10 mod 9) hellip (10 mod 9) mod 9

= (di) (1 mod 9) (1 mod 9) hellip (1 mod 9) mod 9

= di mod 9

i terms

Fast Division Test

Rule of modular multiplication

Let the decimal representation of n be dkdk-1dk-2hellipd1d0

This means that N = dk10k + dk-110k-1 + hellip + d110 + d0

Note that di10i mod 9 = di mod 9

Hence N mod 9 = (dk10k + dk-110k-1 + hellip + d110 + d0) mod 9

= (dk10k mod 9 + dk-110k-1 mod 9 + hellip + d110 mod 9 + d0 mod 9) mod

9

= (dk mod 9 + dk-1 mod 9 + hellip + d1 mod 9 + d0 mod 9) mod 9

= (dk + dk-1 + hellip + d1 + d0) mod 9

Hint 10 1 (mod 9)

Fast Division Test

Claim A number written in decimal is divisible by 9 if and

only if

the sum of its digits is a multiple of 9

Rule of modular addition

By previous slide

The same procedure works to test whether N is divisible by n=3

Fast Division Test

What about n=11

Hint 10 -1 (mod 11)

Let the decimal representation of N be d92d91d90hellipd1d0

Then N is divisible by 11 if and only if

d92-d91+d90hellip-d1+d0 is divisible by 11

Why Try to work it out before your TA shows you

What about n=7

Hint 1000 -1 (mod 7)

This Lecture

bull Multiplicative inverse

The multiplicative inverse of a number a is another number arsquo such that

a middot arsquo 1 (mod n)

Multiplication Inverse

Does every number has a multiplicative inverse in modular arithmetic

For real numbers every nonzero number has a multiplicative inverse

For integers only 1 has a multiplicative inverse

An interesting property of modular arithmetic is that

there are multiplicative inverse for integers

For example 2 5 = 1 mod 3 so 5 is a multiplicative inverse

for 2 under modulo 3 (and vice versa)

Multiplication Inverse

Does every number has a multiplicative inverse in modular arithmetic

Multiplication Inverse

What is the pattern

Case Study

Why 2 does not have a multiplicative inverse under modulo 6

Suppose it has a multiplicative inverse y

2y 1 (mod 6)

=gt 2y = 1 + 6x for some integer x

=gt y = frac12 + 3x

This is a contradiction since both x and y are integers

Necessary Condition

Claim An integer k does not have an multiplicative inverse under modulo n

if k and n have a common factor gt= 2 (gcd(kn) gt= 2)

Proof

This claim says that for k to have a multiplicative inverse modulo n

then a necessary condition is that k and n do not have a common factor gt= 2

Suppose by contradiction that there is an inverse krsquo for k such that

krsquok = 1 (mod n)

Then krsquok = 1 + xn for some integer x

Since both k and n have a common factor say cgt=2

then k=ck1 and n=cn1 for some integers k1 and n1

So krsquock1 = 1 + xcn1

Then krsquok1 = 1c + xn1

This is a contradiction since the LHS is an integer but the RHS is not

Sufficient Condition

What about if gcd(kn)=1

Would k always have an multiplicative inverse under modulo n

For example gcd(37) = 1 3middot5 1 (mod 7)

gcd(89) = 1

gcd(411) = 1 4middot3 1 (mod 11)

8middot8 1 (mod 9)

It seems that there is always an inverse in such a case but why

gcd(89) = 1 8s + 9t = 1 for some integers s and t

8s = 1 ndash 9t

8s 1 (mod 9)gcd(89) = spc(89)

Theorem If gcd(kn)=1 then have krsquo such that

kmiddotkrsquo 1 (mod n)

Proof Since gcd(kn)=1 there exist s and t so that sk + tn = 1

So tn = 1 - sk

This means n | 1 ndash sk

This means that 1 ndash sk 0 (mod n)

This means that 1 sk (mod n)

So krsquo = s is an multiplicative inverse for k

Sufficient Condition

gcd(kn)=spc(kn)

The multiplicative inverse can be computed by the extended Euclidean algorithm

Corollary k has a multiplicative inverse mod n if and only if gcd(kn)=1

This Lecture

bull Fermatrsquos little theorem

Cancellation

There is no general cancellation in modular arithmetic

Note that (mod n) is very similar to =

If a b (mod n) then a+c b+c (mod n)

If a b (mod n) then ac bc (mod n)

However if ac bc (mod n)

it is not necessarily true that a b (mod n)

For example 4middot2 1middot2 (mod 6) but 4 1 (mod 6)

3middot4 1middot4 (mod 8) but 3 1 (mod 8)

4middot3 1middot3 (mod 9) but 4 1 (mod 9)

Observation In all the above examples c and n have a common factor

Cancellation

Claim Assume gcd(kn) = 1 If imiddotk jmiddotk (mod n) then i j (mod n)

For example multiplicative inverse always exists if n is a prime

Proof Since gcd(kn) = 1 there exists krsquo such that kkrsquo 1 (mod n)

imiddotk jmiddotk (mod n)

=gt imiddotkmiddotkrsquo jmiddotkmiddotkrsquo (mod n)

=gt i j (mod n)

Remarks (Optional) This makes arithmetic modulo prime a field

a structure that ldquobehaves likerdquo real numbers

Arithmetic modulo prime is very useful in coding theory

In particular when p is a prime amp k not a multiple of p then gcd(kp)=1

If i j (mod p) then imiddotk jmiddotk (mod p)

Therefore

k mod p 2k mod p hellip (p-1)k mod p

are all different numbers

Fermatrsquos Little Theorem

Claim 1 Assume gcd(kn) = 1 If imiddotk jmiddotk (mod n) then i j (mod n)

Claim 2 Assume gcd(kn) = 1 If i j (mod n) then imiddotk jmiddotk (mod n)

For example when p=7 and k=3

3 mod 7 = 3 2middot3 mod 7 = 6 3middot3 mod 7 = 2 4middot3 mod 7 = 5 5middot3 mod 7 = 1 6middot3 mod 7 = 4

Notice that in the above example every number from 1 to 6 appears exactly once

In particular when p is a prime amp k not a multiple of p then gcd(kp)=1

If i j (mod p) then imiddotk jmiddotk (mod p)

Therefore

k mod p 2k mod p hellip (p-1)k mod p

are all different numbers

Fermatrsquos Little Theorem

Each of ik mod p cannot be equal to 0 because p is a prime number

Let ci = ik mod p

So 1 lt= c1 lt= p-1 1 lt= c2 lt= p-1 hellip 1lt = cp-1 lt= p-1

By the above we know that c1c2hellipcp-2cp-1 are all different

So for each i from 1 to p-1 there is exactly one cj such that cj = i

Therefore we have

(k mod p)middot(2k mod p)middothellipmiddot((p-1)k mod p) = c1middotc2middothellipmiddotcp-2middotcp-1 = 1middot2middot3hellipmiddot(p-2)middot(p-1)

Fermatrsquos Little Theorem

1 kp-1 (mod p)

Theorem If p is prime amp k not a multiple of p

For example when p=5 k=4 we have kp-1 mod p = 44 mod 5 = 1

ldquoProofrdquo

4middot3middot2middot1 [(4 mod 5) (2middot4 mod 5) (3middot4 mod 5) (4middot4 mod 5)] (mod 5)

[4 middot (2middot4) middot (3middot4) middot (4middot4)] (mod 5)

[44 middot (1middot2middot3middot4)] (mod 5)

Since gcd(1middot2middot3middot4 5)=1 we can cancel 1middot2middot3middot4 on both sides

This implies

1 44 (mod 5)

By the previous slide or direct calculation

Fermatrsquos Little Theorem

1 kp-1 (mod p)

Theorem If p is prime amp k not a multiple of p

Proof

1middot2middotmiddotmiddot(p-1) (k mod p middot 2k mod pmiddothellipmiddot(p-1)k mod p) mod p

(kmiddot2k middotmiddotmiddot (p-1)k) mod p

(kp-1)middot1middot2 middotmiddotmiddot (p-1) (mod p)

So by cancelling 1middot2 middotmiddotmiddot (p-1) on both sides applying

Claim 1

(we can cancel them because gcd(1middot2 middotmiddotmiddot (p-1) p)=1)

we have

1 kp-1 (mod p)

By 2 slides before

By the multiplication rule

Wilsonrsquos Theorem

Theorem p is a prime if and only if

(p-1) -1 (mod p)

First we consider the easy direction

If p is not a prime assume p gt= 5 (for p=4 3 2 (mod 4) )

Then p=qr for some 2 lt= q lt p and 2 lt= r lt p

If q ne r then both q and r appear in (p-1)

and so (p-1) 0 (mod p)

If q = r then p = q2 gt 2q (since we assume p gt 5 and thus q gt 2)

then both q and 2q are in (p-1)

and so again (p-1) 0 (mod p)

Wilsonrsquos Theorem

Theorem p is a prime if and only if

(p-1) -1 (mod p)

To prove the more interesting direction first we need a lemma

Lemma If p is a prime number

x2 1 (mod p) if and only if x 1 (mod p) or x -1 (mod p)

Proof x2 1 (mod p)

iff p | x2 - 1

iff p | (x ndash 1)(x + 1)

iff p | (x ndash 1) or p | (x+1)

iff x 1 (mod p) or x -1 (mod p)

Lemma p prime and p|amiddotb iff p|a or p|b

Wilsonrsquos Theorem

Theorem p is a prime if and only if

(p-1) -1 (mod p)

Letrsquos get the proof idea by considering a concrete example

10

1middot2middot3middot4middot5middot6middot7middot8middot9middot10 mod 11

1middot10middot(2middot6)middot(3middot4)middot(5middot9)middot(7middot8) mod 11

1middot-1middot(1)middot(1)middot(1)middot(1) mod 11

-1 mod 11

Besides 1 and 10 the remaining numbers are paired up into multiplicative inverse

Wilsonrsquos Theorem

Theorem p is a prime if and only if

(p-1) -1 (mod p)

ProofSince p is a prime every number from 1 to p-1 has a multiplicative inverse

By the Lemma every number 2 lt= k lt= p-2 has an inverse krsquo with knekrsquo

Since p is odd the numbers from 2 to p-2 can be grouped into pairs

(a1b1)(a2b2)hellip(a(p-3)2b(p-3)2) so that aibi 1 (mod p)

Therefore (p-1) 1middot(p-1)middot2middot3middotmiddotmiddotmiddotmiddot(p-3)middot(p-2) (mod p)

1middot(p-1)middot(a1b1)middot(a2b2)middotmiddotmiddotmiddotmiddot(a(p-3)2b(p-3)2) (mod p)

1middot(-1)middot(1)middot(1)middotmiddotmiddotmiddotmiddot(1) (mod p)

-1 (mod p)

This Lecture

bull Eulerrsquos phi function

Inclusion-Exclusion (n sets)

What is the inclusion-exclusion formula for the union of n sets

sum of sizes of all single setsndash sum of sizes of all 2-set intersections+ sum of sizes of all 3-set intersectionsndash sum of sizes of all 4-set intersectionshellip+ (ndash1)n+1 times sum of sizes of intersections of all n

sets

1 2 nA A A

1

12 1

( 1)n

ki

S nk i S

S k

A

Inclusion-Exclusion (n sets)

Inclusion-Exclusion (n sets)

sum of sizes of all single setsndash sum of sizes of all 2-set intersections+ sum of sizes of all 3-set intersectionsndash sum of sizes of all 4-set intersectionshellip+ (ndash1)n+1 times sum of sizes of intersections of all n

sets

|A1 [ A2 [ A3 [ hellip [ An|

We want to show that every element is counted exactly once

Consider an element which belongs to exactly k sets say A1 A2 A3 hellip Ak

In the formula such an element is counted the following number of times

Therefore each element is counted exactly once and thus the formula is correct

Euler Function

Given a number n how many numbers from 1 to n are relatively prime to n

When n is a prime number

then every number from 1 to n-1 is relatively prime to n

and so

When n is a prime power

then p 2p 3p 4p hellip n are not relatively prime to n

there are np = pc-1 of them

and other numbers are relatively prime to n

Therefore

Euler Function

Given a number n how many numbers from 1 to n are relatively prime to n

Suppose

Then p 2p 3p 4p hellip n are not relatively prime to n there are np of them

Also q 2q 3q 4q hellip n are not relatively prime to n and there are nq of them

Other numbers are relatively prime to n

Therefore

The numbers pq 2pq 3pq hellip n are subtracted twice and there are npq of them

So the correct answer is

Euler Function

Given a number n how many numbers from 1 to n are relatively prime to n

Let

Let S be the set of numbers from 1 to n that are not relatively prime to n

Let Ai be the set of numbers that are a multiple of piS = A1 [ A2 [ hellip [ An

For the intersection of k sets say A1 A2 A3hellip Ak

then every number in A1 Aring A2 Aring hellip Aring Ak is a multiple of p1p2hellippk

then |A1 Aring A2 Aring hellip Aring Ak| = np1p2hellippk

Euler Function

Given a number n how many numbers from 1 to n are relatively prime to n

Let

Let S be the set of numbers from 1 to n that are not relatively prime to n

Let Ai be the set of numbers that are a multiple of piS = A1 [ A2 [ hellip [ An

|A1 Aring A2 Aring hellip Aring Ak| = np1p2hellippk

When r=3 (only 3 distinct factors)

|A1 [ A2 [ A3|

= np1 + np2 + np3

- np1p2 ndash np1p3 ndash np2p3 + np1p2p3

|A1 [ A2 [ A3| = |A1| + |A2| + |A3|

ndash |A1 Aring A2| ndash |A1 Aring A3| ndash |A2 Aring A3|

+ |A1 Aring A2 Aring A3|

= n(1-p1)(1-p2)(1-p3)

Euler Function

Given a number n how many numbers from 1 to n are relatively prime to n

Let

Let S be the set of numbers from 1 to n that are not relatively prime to n

Let Ai be the set of numbers that are a multiple of piS = A1 [ A2 [ hellip [ An

|A1 Aring A2 Aring hellip Aring Ak| = np1p2hellippk

sum of sizes of all single setsndash sum of sizes of all 2-set intersections+ sum of sizes of all 3-set intersectionsndash sum of sizes of all 4-set intersectionshellip+ (ndash1)n+1 times sum of sizes of intersections of

n sets

|A1 [ A2 [ A3 [ hellip [ An|

|S| = |A1 [ A2 [ hellip [ An|

= n(1-p1)(1-p2)hellip(1-pn)

calculationshellip

  • Modular Arithmetic
  • PowerPoint Presentation
  • Slide 3
  • Slide 4
  • Slide 5
  • Slide 6
  • Slide 7
  • Slide 8
  • Slide 9
  • Slide 10
  • Slide 11
  • Slide 12
  • Slide 13
  • Slide 14
  • Slide 15
  • Slide 16
  • Slide 17
  • Slide 18
  • Slide 19
  • Slide 20
  • Slide 21
  • Slide 22
  • Slide 23
  • Slide 24
  • Slide 25
  • Slide 26
  • Slide 27
  • Slide 28
  • Slide 29
  • Slide 30
  • Slide 31
  • Slide 32
  • Slide 33
  • Slide 34
  • Slide 35
  • Slide 36
  • Slide 37
  • Slide 38
  • Slide 39
  • Slide 40
  • Slide 41
  • Slide 42
  • Slide 43
  • Slide 44
  • Slide 45
Page 8: Modular Arithmetic

Lemma If a c (mod n) and b d (mod n) then

ab cd (mod n)

Modular Multiplication

Example 1 9876 6 (mod 10) 17642 2 (mod 10)

=gt 9876 17642 (mod 10) 6 2 (mod 10) 2 (mod 10)

Example 2 10987 1 (mod 2) 28663 1 (mod 2)

=gt 10987 28663 (mod 2) 1 (mod 2)

Example 3 1000 -1 (mod 7) 1000000 1 (mod 7)

=gt 1000 1000000 (mod 7) -1 1 (mod 7) -1 (mod 7)

In particular when computing ab mod n we can first replace

a by a mod n and b by b mod n so that the computation is faster

Lemma If a c (mod n) and b d (mod n) then

ab cd (mod n)

Modular Multiplication

a c (mod n) =gt a = c + nx for some integer x

b d (mod n) =gt b = d + ny for some integer y

To show ab cd (mod n) it is equivalent to showing that n | (ab-cd)

Consider ab-cd

ab-cd = (c+nx) (d+ny) ndash cd

= cd + dnx + cny + n2xy ndash cd = n(dx + cy + nxy)

It is clear that n | n(dx + cy + nxy) Therefore n | ab-cd

We conclude that ab cd (mod n)

Proof

This Lecture

bull Applications Fast exponentiation and fast division test

Fast Exponentiation

1444 mod 713

= 144 144 144 144 mod 713

= 20736 144 144 mod 713

= 59 144 144 mod 713

= 8496 144 mod 713

= 653 144 mod 713

= 94032 mod 713

= 629 mod 713

20736 20736 mod 713

= 59 59 mod 713

= 3481 mod 713

= 629 mod 713

Because 20736 59 (mod 713)

Because 653 8496 (mod 713)

shortcut

Repeated Squaring

14450 mod 713

= 14432 14416 1442 mod 713

= 64848559 mod 713

= 242

1442 mod 713 = 59

1444 mod 713 = 1442 1442 mod 713= 5959 mod 713= 629

1448 mod 713= 14441444 mod 713= 629629 mod 713= 639

14416 mod 713= 14481448 mod 713= 639639 mod 713= 485

14432 mod 713= 1441614416 mod 713= 485485 mod 713= 648

Note that 50 = 32 + 16 + 2

Fast Division Test

Using the basic rules for modular addition and modular multiplication

we can derive some quick test to see if a big number is divisible by a small number

Suppose we are given the decimal representation of a big number N

To test if N is divisible by a small number n of course we can do a division to check

But can we do faster

If n = 2 we just need to check whether the last digit of N is even or not

If n = 10 we just need to check whether the last digit of N is 0 or not

If n = 5 we just need to check whether the last digit of N is either 5 or 0 or not

What about when n=3 When n=7 When n=11

Fast Division Test

A number written in decimal divisible by 9 if and only if

the sum of its digits is a multiple of 9

Example 1 9333234513171 is divisible by 9

9+3+3+3+2+3+4+5+1+3+1+7+1 = 45 is divisible by 9

Example 2 128573649683 is not divisible by 9

1+2+8+5+7+3+6+4+9+6+8+3 = 62 is not divisible by 9

Claim A number written in decimal is divisible by 9 if and

only if

the sum of its digits is a multiple of 9Hint 10 1 (mod 9)

Let the decimal representation of N be dkdk-1dk-2hellipd1d0

This means that N = dk10k + dk-110k-1 + hellip + d110 + d0

Note that di10i mod 9

= (di) (10i mod 9) mod 9

= (di) (10 mod 9) (10 mod 9) hellip (10 mod 9) mod 9

= (di) (1 mod 9) (1 mod 9) hellip (1 mod 9) mod 9

= di mod 9

i terms

Fast Division Test

Rule of modular multiplication

Let the decimal representation of n be dkdk-1dk-2hellipd1d0

This means that N = dk10k + dk-110k-1 + hellip + d110 + d0

Note that di10i mod 9 = di mod 9

Hence N mod 9 = (dk10k + dk-110k-1 + hellip + d110 + d0) mod 9

= (dk10k mod 9 + dk-110k-1 mod 9 + hellip + d110 mod 9 + d0 mod 9) mod

9

= (dk mod 9 + dk-1 mod 9 + hellip + d1 mod 9 + d0 mod 9) mod 9

= (dk + dk-1 + hellip + d1 + d0) mod 9

Hint 10 1 (mod 9)

Fast Division Test

Claim A number written in decimal is divisible by 9 if and

only if

the sum of its digits is a multiple of 9

Rule of modular addition

By previous slide

The same procedure works to test whether N is divisible by n=3

Fast Division Test

What about n=11

Hint 10 -1 (mod 11)

Let the decimal representation of N be d92d91d90hellipd1d0

Then N is divisible by 11 if and only if

d92-d91+d90hellip-d1+d0 is divisible by 11

Why Try to work it out before your TA shows you

What about n=7

Hint 1000 -1 (mod 7)

This Lecture

bull Multiplicative inverse

The multiplicative inverse of a number a is another number arsquo such that

a middot arsquo 1 (mod n)

Multiplication Inverse

Does every number has a multiplicative inverse in modular arithmetic

For real numbers every nonzero number has a multiplicative inverse

For integers only 1 has a multiplicative inverse

An interesting property of modular arithmetic is that

there are multiplicative inverse for integers

For example 2 5 = 1 mod 3 so 5 is a multiplicative inverse

for 2 under modulo 3 (and vice versa)

Multiplication Inverse

Does every number has a multiplicative inverse in modular arithmetic

Multiplication Inverse

What is the pattern

Case Study

Why 2 does not have a multiplicative inverse under modulo 6

Suppose it has a multiplicative inverse y

2y 1 (mod 6)

=gt 2y = 1 + 6x for some integer x

=gt y = frac12 + 3x

This is a contradiction since both x and y are integers

Necessary Condition

Claim An integer k does not have an multiplicative inverse under modulo n

if k and n have a common factor gt= 2 (gcd(kn) gt= 2)

Proof

This claim says that for k to have a multiplicative inverse modulo n

then a necessary condition is that k and n do not have a common factor gt= 2

Suppose by contradiction that there is an inverse krsquo for k such that

krsquok = 1 (mod n)

Then krsquok = 1 + xn for some integer x

Since both k and n have a common factor say cgt=2

then k=ck1 and n=cn1 for some integers k1 and n1

So krsquock1 = 1 + xcn1

Then krsquok1 = 1c + xn1

This is a contradiction since the LHS is an integer but the RHS is not

Sufficient Condition

What about if gcd(kn)=1

Would k always have an multiplicative inverse under modulo n

For example gcd(37) = 1 3middot5 1 (mod 7)

gcd(89) = 1

gcd(411) = 1 4middot3 1 (mod 11)

8middot8 1 (mod 9)

It seems that there is always an inverse in such a case but why

gcd(89) = 1 8s + 9t = 1 for some integers s and t

8s = 1 ndash 9t

8s 1 (mod 9)gcd(89) = spc(89)

Theorem If gcd(kn)=1 then have krsquo such that

kmiddotkrsquo 1 (mod n)

Proof Since gcd(kn)=1 there exist s and t so that sk + tn = 1

So tn = 1 - sk

This means n | 1 ndash sk

This means that 1 ndash sk 0 (mod n)

This means that 1 sk (mod n)

So krsquo = s is an multiplicative inverse for k

Sufficient Condition

gcd(kn)=spc(kn)

The multiplicative inverse can be computed by the extended Euclidean algorithm

Corollary k has a multiplicative inverse mod n if and only if gcd(kn)=1

This Lecture

bull Fermatrsquos little theorem

Cancellation

There is no general cancellation in modular arithmetic

Note that (mod n) is very similar to =

If a b (mod n) then a+c b+c (mod n)

If a b (mod n) then ac bc (mod n)

However if ac bc (mod n)

it is not necessarily true that a b (mod n)

For example 4middot2 1middot2 (mod 6) but 4 1 (mod 6)

3middot4 1middot4 (mod 8) but 3 1 (mod 8)

4middot3 1middot3 (mod 9) but 4 1 (mod 9)

Observation In all the above examples c and n have a common factor

Cancellation

Claim Assume gcd(kn) = 1 If imiddotk jmiddotk (mod n) then i j (mod n)

For example multiplicative inverse always exists if n is a prime

Proof Since gcd(kn) = 1 there exists krsquo such that kkrsquo 1 (mod n)

imiddotk jmiddotk (mod n)

=gt imiddotkmiddotkrsquo jmiddotkmiddotkrsquo (mod n)

=gt i j (mod n)

Remarks (Optional) This makes arithmetic modulo prime a field

a structure that ldquobehaves likerdquo real numbers

Arithmetic modulo prime is very useful in coding theory

In particular when p is a prime amp k not a multiple of p then gcd(kp)=1

If i j (mod p) then imiddotk jmiddotk (mod p)

Therefore

k mod p 2k mod p hellip (p-1)k mod p

are all different numbers

Fermatrsquos Little Theorem

Claim 1 Assume gcd(kn) = 1 If imiddotk jmiddotk (mod n) then i j (mod n)

Claim 2 Assume gcd(kn) = 1 If i j (mod n) then imiddotk jmiddotk (mod n)

For example when p=7 and k=3

3 mod 7 = 3 2middot3 mod 7 = 6 3middot3 mod 7 = 2 4middot3 mod 7 = 5 5middot3 mod 7 = 1 6middot3 mod 7 = 4

Notice that in the above example every number from 1 to 6 appears exactly once

In particular when p is a prime amp k not a multiple of p then gcd(kp)=1

If i j (mod p) then imiddotk jmiddotk (mod p)

Therefore

k mod p 2k mod p hellip (p-1)k mod p

are all different numbers

Fermatrsquos Little Theorem

Each of ik mod p cannot be equal to 0 because p is a prime number

Let ci = ik mod p

So 1 lt= c1 lt= p-1 1 lt= c2 lt= p-1 hellip 1lt = cp-1 lt= p-1

By the above we know that c1c2hellipcp-2cp-1 are all different

So for each i from 1 to p-1 there is exactly one cj such that cj = i

Therefore we have

(k mod p)middot(2k mod p)middothellipmiddot((p-1)k mod p) = c1middotc2middothellipmiddotcp-2middotcp-1 = 1middot2middot3hellipmiddot(p-2)middot(p-1)

Fermatrsquos Little Theorem

1 kp-1 (mod p)

Theorem If p is prime amp k not a multiple of p

For example when p=5 k=4 we have kp-1 mod p = 44 mod 5 = 1

ldquoProofrdquo

4middot3middot2middot1 [(4 mod 5) (2middot4 mod 5) (3middot4 mod 5) (4middot4 mod 5)] (mod 5)

[4 middot (2middot4) middot (3middot4) middot (4middot4)] (mod 5)

[44 middot (1middot2middot3middot4)] (mod 5)

Since gcd(1middot2middot3middot4 5)=1 we can cancel 1middot2middot3middot4 on both sides

This implies

1 44 (mod 5)

By the previous slide or direct calculation

Fermatrsquos Little Theorem

1 kp-1 (mod p)

Theorem If p is prime amp k not a multiple of p

Proof

1middot2middotmiddotmiddot(p-1) (k mod p middot 2k mod pmiddothellipmiddot(p-1)k mod p) mod p

(kmiddot2k middotmiddotmiddot (p-1)k) mod p

(kp-1)middot1middot2 middotmiddotmiddot (p-1) (mod p)

So by cancelling 1middot2 middotmiddotmiddot (p-1) on both sides applying

Claim 1

(we can cancel them because gcd(1middot2 middotmiddotmiddot (p-1) p)=1)

we have

1 kp-1 (mod p)

By 2 slides before

By the multiplication rule

Wilsonrsquos Theorem

Theorem p is a prime if and only if

(p-1) -1 (mod p)

First we consider the easy direction

If p is not a prime assume p gt= 5 (for p=4 3 2 (mod 4) )

Then p=qr for some 2 lt= q lt p and 2 lt= r lt p

If q ne r then both q and r appear in (p-1)

and so (p-1) 0 (mod p)

If q = r then p = q2 gt 2q (since we assume p gt 5 and thus q gt 2)

then both q and 2q are in (p-1)

and so again (p-1) 0 (mod p)

Wilsonrsquos Theorem

Theorem p is a prime if and only if

(p-1) -1 (mod p)

To prove the more interesting direction first we need a lemma

Lemma If p is a prime number

x2 1 (mod p) if and only if x 1 (mod p) or x -1 (mod p)

Proof x2 1 (mod p)

iff p | x2 - 1

iff p | (x ndash 1)(x + 1)

iff p | (x ndash 1) or p | (x+1)

iff x 1 (mod p) or x -1 (mod p)

Lemma p prime and p|amiddotb iff p|a or p|b

Wilsonrsquos Theorem

Theorem p is a prime if and only if

(p-1) -1 (mod p)

Letrsquos get the proof idea by considering a concrete example

10

1middot2middot3middot4middot5middot6middot7middot8middot9middot10 mod 11

1middot10middot(2middot6)middot(3middot4)middot(5middot9)middot(7middot8) mod 11

1middot-1middot(1)middot(1)middot(1)middot(1) mod 11

-1 mod 11

Besides 1 and 10 the remaining numbers are paired up into multiplicative inverse

Wilsonrsquos Theorem

Theorem p is a prime if and only if

(p-1) -1 (mod p)

ProofSince p is a prime every number from 1 to p-1 has a multiplicative inverse

By the Lemma every number 2 lt= k lt= p-2 has an inverse krsquo with knekrsquo

Since p is odd the numbers from 2 to p-2 can be grouped into pairs

(a1b1)(a2b2)hellip(a(p-3)2b(p-3)2) so that aibi 1 (mod p)

Therefore (p-1) 1middot(p-1)middot2middot3middotmiddotmiddotmiddotmiddot(p-3)middot(p-2) (mod p)

1middot(p-1)middot(a1b1)middot(a2b2)middotmiddotmiddotmiddotmiddot(a(p-3)2b(p-3)2) (mod p)

1middot(-1)middot(1)middot(1)middotmiddotmiddotmiddotmiddot(1) (mod p)

-1 (mod p)

This Lecture

bull Eulerrsquos phi function

Inclusion-Exclusion (n sets)

What is the inclusion-exclusion formula for the union of n sets

sum of sizes of all single setsndash sum of sizes of all 2-set intersections+ sum of sizes of all 3-set intersectionsndash sum of sizes of all 4-set intersectionshellip+ (ndash1)n+1 times sum of sizes of intersections of all n

sets

1 2 nA A A

1

12 1

( 1)n

ki

S nk i S

S k

A

Inclusion-Exclusion (n sets)

Inclusion-Exclusion (n sets)

sum of sizes of all single setsndash sum of sizes of all 2-set intersections+ sum of sizes of all 3-set intersectionsndash sum of sizes of all 4-set intersectionshellip+ (ndash1)n+1 times sum of sizes of intersections of all n

sets

|A1 [ A2 [ A3 [ hellip [ An|

We want to show that every element is counted exactly once

Consider an element which belongs to exactly k sets say A1 A2 A3 hellip Ak

In the formula such an element is counted the following number of times

Therefore each element is counted exactly once and thus the formula is correct

Euler Function

Given a number n how many numbers from 1 to n are relatively prime to n

When n is a prime number

then every number from 1 to n-1 is relatively prime to n

and so

When n is a prime power

then p 2p 3p 4p hellip n are not relatively prime to n

there are np = pc-1 of them

and other numbers are relatively prime to n

Therefore

Euler Function

Given a number n how many numbers from 1 to n are relatively prime to n

Suppose

Then p 2p 3p 4p hellip n are not relatively prime to n there are np of them

Also q 2q 3q 4q hellip n are not relatively prime to n and there are nq of them

Other numbers are relatively prime to n

Therefore

The numbers pq 2pq 3pq hellip n are subtracted twice and there are npq of them

So the correct answer is

Euler Function

Given a number n how many numbers from 1 to n are relatively prime to n

Let

Let S be the set of numbers from 1 to n that are not relatively prime to n

Let Ai be the set of numbers that are a multiple of piS = A1 [ A2 [ hellip [ An

For the intersection of k sets say A1 A2 A3hellip Ak

then every number in A1 Aring A2 Aring hellip Aring Ak is a multiple of p1p2hellippk

then |A1 Aring A2 Aring hellip Aring Ak| = np1p2hellippk

Euler Function

Given a number n how many numbers from 1 to n are relatively prime to n

Let

Let S be the set of numbers from 1 to n that are not relatively prime to n

Let Ai be the set of numbers that are a multiple of piS = A1 [ A2 [ hellip [ An

|A1 Aring A2 Aring hellip Aring Ak| = np1p2hellippk

When r=3 (only 3 distinct factors)

|A1 [ A2 [ A3|

= np1 + np2 + np3

- np1p2 ndash np1p3 ndash np2p3 + np1p2p3

|A1 [ A2 [ A3| = |A1| + |A2| + |A3|

ndash |A1 Aring A2| ndash |A1 Aring A3| ndash |A2 Aring A3|

+ |A1 Aring A2 Aring A3|

= n(1-p1)(1-p2)(1-p3)

Euler Function

Given a number n how many numbers from 1 to n are relatively prime to n

Let

Let S be the set of numbers from 1 to n that are not relatively prime to n

Let Ai be the set of numbers that are a multiple of piS = A1 [ A2 [ hellip [ An

|A1 Aring A2 Aring hellip Aring Ak| = np1p2hellippk

sum of sizes of all single setsndash sum of sizes of all 2-set intersections+ sum of sizes of all 3-set intersectionsndash sum of sizes of all 4-set intersectionshellip+ (ndash1)n+1 times sum of sizes of intersections of

n sets

|A1 [ A2 [ A3 [ hellip [ An|

|S| = |A1 [ A2 [ hellip [ An|

= n(1-p1)(1-p2)hellip(1-pn)

calculationshellip

  • Modular Arithmetic
  • PowerPoint Presentation
  • Slide 3
  • Slide 4
  • Slide 5
  • Slide 6
  • Slide 7
  • Slide 8
  • Slide 9
  • Slide 10
  • Slide 11
  • Slide 12
  • Slide 13
  • Slide 14
  • Slide 15
  • Slide 16
  • Slide 17
  • Slide 18
  • Slide 19
  • Slide 20
  • Slide 21
  • Slide 22
  • Slide 23
  • Slide 24
  • Slide 25
  • Slide 26
  • Slide 27
  • Slide 28
  • Slide 29
  • Slide 30
  • Slide 31
  • Slide 32
  • Slide 33
  • Slide 34
  • Slide 35
  • Slide 36
  • Slide 37
  • Slide 38
  • Slide 39
  • Slide 40
  • Slide 41
  • Slide 42
  • Slide 43
  • Slide 44
  • Slide 45
Page 9: Modular Arithmetic

Lemma If a c (mod n) and b d (mod n) then

ab cd (mod n)

Modular Multiplication

a c (mod n) =gt a = c + nx for some integer x

b d (mod n) =gt b = d + ny for some integer y

To show ab cd (mod n) it is equivalent to showing that n | (ab-cd)

Consider ab-cd

ab-cd = (c+nx) (d+ny) ndash cd

= cd + dnx + cny + n2xy ndash cd = n(dx + cy + nxy)

It is clear that n | n(dx + cy + nxy) Therefore n | ab-cd

We conclude that ab cd (mod n)

Proof

This Lecture

bull Applications Fast exponentiation and fast division test

Fast Exponentiation

1444 mod 713

= 144 144 144 144 mod 713

= 20736 144 144 mod 713

= 59 144 144 mod 713

= 8496 144 mod 713

= 653 144 mod 713

= 94032 mod 713

= 629 mod 713

20736 20736 mod 713

= 59 59 mod 713

= 3481 mod 713

= 629 mod 713

Because 20736 59 (mod 713)

Because 653 8496 (mod 713)

shortcut

Repeated Squaring

14450 mod 713

= 14432 14416 1442 mod 713

= 64848559 mod 713

= 242

1442 mod 713 = 59

1444 mod 713 = 1442 1442 mod 713= 5959 mod 713= 629

1448 mod 713= 14441444 mod 713= 629629 mod 713= 639

14416 mod 713= 14481448 mod 713= 639639 mod 713= 485

14432 mod 713= 1441614416 mod 713= 485485 mod 713= 648

Note that 50 = 32 + 16 + 2

Fast Division Test

Using the basic rules for modular addition and modular multiplication

we can derive some quick test to see if a big number is divisible by a small number

Suppose we are given the decimal representation of a big number N

To test if N is divisible by a small number n of course we can do a division to check

But can we do faster

If n = 2 we just need to check whether the last digit of N is even or not

If n = 10 we just need to check whether the last digit of N is 0 or not

If n = 5 we just need to check whether the last digit of N is either 5 or 0 or not

What about when n=3 When n=7 When n=11

Fast Division Test

A number written in decimal divisible by 9 if and only if

the sum of its digits is a multiple of 9

Example 1 9333234513171 is divisible by 9

9+3+3+3+2+3+4+5+1+3+1+7+1 = 45 is divisible by 9

Example 2 128573649683 is not divisible by 9

1+2+8+5+7+3+6+4+9+6+8+3 = 62 is not divisible by 9

Claim A number written in decimal is divisible by 9 if and

only if

the sum of its digits is a multiple of 9Hint 10 1 (mod 9)

Let the decimal representation of N be dkdk-1dk-2hellipd1d0

This means that N = dk10k + dk-110k-1 + hellip + d110 + d0

Note that di10i mod 9

= (di) (10i mod 9) mod 9

= (di) (10 mod 9) (10 mod 9) hellip (10 mod 9) mod 9

= (di) (1 mod 9) (1 mod 9) hellip (1 mod 9) mod 9

= di mod 9

i terms

Fast Division Test

Rule of modular multiplication

Let the decimal representation of n be dkdk-1dk-2hellipd1d0

This means that N = dk10k + dk-110k-1 + hellip + d110 + d0

Note that di10i mod 9 = di mod 9

Hence N mod 9 = (dk10k + dk-110k-1 + hellip + d110 + d0) mod 9

= (dk10k mod 9 + dk-110k-1 mod 9 + hellip + d110 mod 9 + d0 mod 9) mod

9

= (dk mod 9 + dk-1 mod 9 + hellip + d1 mod 9 + d0 mod 9) mod 9

= (dk + dk-1 + hellip + d1 + d0) mod 9

Hint 10 1 (mod 9)

Fast Division Test

Claim A number written in decimal is divisible by 9 if and

only if

the sum of its digits is a multiple of 9

Rule of modular addition

By previous slide

The same procedure works to test whether N is divisible by n=3

Fast Division Test

What about n=11

Hint 10 -1 (mod 11)

Let the decimal representation of N be d92d91d90hellipd1d0

Then N is divisible by 11 if and only if

d92-d91+d90hellip-d1+d0 is divisible by 11

Why Try to work it out before your TA shows you

What about n=7

Hint 1000 -1 (mod 7)

This Lecture

bull Multiplicative inverse

The multiplicative inverse of a number a is another number arsquo such that

a middot arsquo 1 (mod n)

Multiplication Inverse

Does every number has a multiplicative inverse in modular arithmetic

For real numbers every nonzero number has a multiplicative inverse

For integers only 1 has a multiplicative inverse

An interesting property of modular arithmetic is that

there are multiplicative inverse for integers

For example 2 5 = 1 mod 3 so 5 is a multiplicative inverse

for 2 under modulo 3 (and vice versa)

Multiplication Inverse

Does every number has a multiplicative inverse in modular arithmetic

Multiplication Inverse

What is the pattern

Case Study

Why 2 does not have a multiplicative inverse under modulo 6

Suppose it has a multiplicative inverse y

2y 1 (mod 6)

=gt 2y = 1 + 6x for some integer x

=gt y = frac12 + 3x

This is a contradiction since both x and y are integers

Necessary Condition

Claim An integer k does not have an multiplicative inverse under modulo n

if k and n have a common factor gt= 2 (gcd(kn) gt= 2)

Proof

This claim says that for k to have a multiplicative inverse modulo n

then a necessary condition is that k and n do not have a common factor gt= 2

Suppose by contradiction that there is an inverse krsquo for k such that

krsquok = 1 (mod n)

Then krsquok = 1 + xn for some integer x

Since both k and n have a common factor say cgt=2

then k=ck1 and n=cn1 for some integers k1 and n1

So krsquock1 = 1 + xcn1

Then krsquok1 = 1c + xn1

This is a contradiction since the LHS is an integer but the RHS is not

Sufficient Condition

What about if gcd(kn)=1

Would k always have an multiplicative inverse under modulo n

For example gcd(37) = 1 3middot5 1 (mod 7)

gcd(89) = 1

gcd(411) = 1 4middot3 1 (mod 11)

8middot8 1 (mod 9)

It seems that there is always an inverse in such a case but why

gcd(89) = 1 8s + 9t = 1 for some integers s and t

8s = 1 ndash 9t

8s 1 (mod 9)gcd(89) = spc(89)

Theorem If gcd(kn)=1 then have krsquo such that

kmiddotkrsquo 1 (mod n)

Proof Since gcd(kn)=1 there exist s and t so that sk + tn = 1

So tn = 1 - sk

This means n | 1 ndash sk

This means that 1 ndash sk 0 (mod n)

This means that 1 sk (mod n)

So krsquo = s is an multiplicative inverse for k

Sufficient Condition

gcd(kn)=spc(kn)

The multiplicative inverse can be computed by the extended Euclidean algorithm

Corollary k has a multiplicative inverse mod n if and only if gcd(kn)=1

This Lecture

bull Fermatrsquos little theorem

Cancellation

There is no general cancellation in modular arithmetic

Note that (mod n) is very similar to =

If a b (mod n) then a+c b+c (mod n)

If a b (mod n) then ac bc (mod n)

However if ac bc (mod n)

it is not necessarily true that a b (mod n)

For example 4middot2 1middot2 (mod 6) but 4 1 (mod 6)

3middot4 1middot4 (mod 8) but 3 1 (mod 8)

4middot3 1middot3 (mod 9) but 4 1 (mod 9)

Observation In all the above examples c and n have a common factor

Cancellation

Claim Assume gcd(kn) = 1 If imiddotk jmiddotk (mod n) then i j (mod n)

For example multiplicative inverse always exists if n is a prime

Proof Since gcd(kn) = 1 there exists krsquo such that kkrsquo 1 (mod n)

imiddotk jmiddotk (mod n)

=gt imiddotkmiddotkrsquo jmiddotkmiddotkrsquo (mod n)

=gt i j (mod n)

Remarks (Optional) This makes arithmetic modulo prime a field

a structure that ldquobehaves likerdquo real numbers

Arithmetic modulo prime is very useful in coding theory

In particular when p is a prime amp k not a multiple of p then gcd(kp)=1

If i j (mod p) then imiddotk jmiddotk (mod p)

Therefore

k mod p 2k mod p hellip (p-1)k mod p

are all different numbers

Fermatrsquos Little Theorem

Claim 1 Assume gcd(kn) = 1 If imiddotk jmiddotk (mod n) then i j (mod n)

Claim 2 Assume gcd(kn) = 1 If i j (mod n) then imiddotk jmiddotk (mod n)

For example when p=7 and k=3

3 mod 7 = 3 2middot3 mod 7 = 6 3middot3 mod 7 = 2 4middot3 mod 7 = 5 5middot3 mod 7 = 1 6middot3 mod 7 = 4

Notice that in the above example every number from 1 to 6 appears exactly once

In particular when p is a prime amp k not a multiple of p then gcd(kp)=1

If i j (mod p) then imiddotk jmiddotk (mod p)

Therefore

k mod p 2k mod p hellip (p-1)k mod p

are all different numbers

Fermatrsquos Little Theorem

Each of ik mod p cannot be equal to 0 because p is a prime number

Let ci = ik mod p

So 1 lt= c1 lt= p-1 1 lt= c2 lt= p-1 hellip 1lt = cp-1 lt= p-1

By the above we know that c1c2hellipcp-2cp-1 are all different

So for each i from 1 to p-1 there is exactly one cj such that cj = i

Therefore we have

(k mod p)middot(2k mod p)middothellipmiddot((p-1)k mod p) = c1middotc2middothellipmiddotcp-2middotcp-1 = 1middot2middot3hellipmiddot(p-2)middot(p-1)

Fermatrsquos Little Theorem

1 kp-1 (mod p)

Theorem If p is prime amp k not a multiple of p

For example when p=5 k=4 we have kp-1 mod p = 44 mod 5 = 1

ldquoProofrdquo

4middot3middot2middot1 [(4 mod 5) (2middot4 mod 5) (3middot4 mod 5) (4middot4 mod 5)] (mod 5)

[4 middot (2middot4) middot (3middot4) middot (4middot4)] (mod 5)

[44 middot (1middot2middot3middot4)] (mod 5)

Since gcd(1middot2middot3middot4 5)=1 we can cancel 1middot2middot3middot4 on both sides

This implies

1 44 (mod 5)

By the previous slide or direct calculation

Fermatrsquos Little Theorem

1 kp-1 (mod p)

Theorem If p is prime amp k not a multiple of p

Proof

1middot2middotmiddotmiddot(p-1) (k mod p middot 2k mod pmiddothellipmiddot(p-1)k mod p) mod p

(kmiddot2k middotmiddotmiddot (p-1)k) mod p

(kp-1)middot1middot2 middotmiddotmiddot (p-1) (mod p)

So by cancelling 1middot2 middotmiddotmiddot (p-1) on both sides applying

Claim 1

(we can cancel them because gcd(1middot2 middotmiddotmiddot (p-1) p)=1)

we have

1 kp-1 (mod p)

By 2 slides before

By the multiplication rule

Wilsonrsquos Theorem

Theorem p is a prime if and only if

(p-1) -1 (mod p)

First we consider the easy direction

If p is not a prime assume p gt= 5 (for p=4 3 2 (mod 4) )

Then p=qr for some 2 lt= q lt p and 2 lt= r lt p

If q ne r then both q and r appear in (p-1)

and so (p-1) 0 (mod p)

If q = r then p = q2 gt 2q (since we assume p gt 5 and thus q gt 2)

then both q and 2q are in (p-1)

and so again (p-1) 0 (mod p)

Wilsonrsquos Theorem

Theorem p is a prime if and only if

(p-1) -1 (mod p)

To prove the more interesting direction first we need a lemma

Lemma If p is a prime number

x2 1 (mod p) if and only if x 1 (mod p) or x -1 (mod p)

Proof x2 1 (mod p)

iff p | x2 - 1

iff p | (x ndash 1)(x + 1)

iff p | (x ndash 1) or p | (x+1)

iff x 1 (mod p) or x -1 (mod p)

Lemma p prime and p|amiddotb iff p|a or p|b

Wilsonrsquos Theorem

Theorem p is a prime if and only if

(p-1) -1 (mod p)

Letrsquos get the proof idea by considering a concrete example

10

1middot2middot3middot4middot5middot6middot7middot8middot9middot10 mod 11

1middot10middot(2middot6)middot(3middot4)middot(5middot9)middot(7middot8) mod 11

1middot-1middot(1)middot(1)middot(1)middot(1) mod 11

-1 mod 11

Besides 1 and 10 the remaining numbers are paired up into multiplicative inverse

Wilsonrsquos Theorem

Theorem p is a prime if and only if

(p-1) -1 (mod p)

ProofSince p is a prime every number from 1 to p-1 has a multiplicative inverse

By the Lemma every number 2 lt= k lt= p-2 has an inverse krsquo with knekrsquo

Since p is odd the numbers from 2 to p-2 can be grouped into pairs

(a1b1)(a2b2)hellip(a(p-3)2b(p-3)2) so that aibi 1 (mod p)

Therefore (p-1) 1middot(p-1)middot2middot3middotmiddotmiddotmiddotmiddot(p-3)middot(p-2) (mod p)

1middot(p-1)middot(a1b1)middot(a2b2)middotmiddotmiddotmiddotmiddot(a(p-3)2b(p-3)2) (mod p)

1middot(-1)middot(1)middot(1)middotmiddotmiddotmiddotmiddot(1) (mod p)

-1 (mod p)

This Lecture

bull Eulerrsquos phi function

Inclusion-Exclusion (n sets)

What is the inclusion-exclusion formula for the union of n sets

sum of sizes of all single setsndash sum of sizes of all 2-set intersections+ sum of sizes of all 3-set intersectionsndash sum of sizes of all 4-set intersectionshellip+ (ndash1)n+1 times sum of sizes of intersections of all n

sets

1 2 nA A A

1

12 1

( 1)n

ki

S nk i S

S k

A

Inclusion-Exclusion (n sets)

Inclusion-Exclusion (n sets)

sum of sizes of all single setsndash sum of sizes of all 2-set intersections+ sum of sizes of all 3-set intersectionsndash sum of sizes of all 4-set intersectionshellip+ (ndash1)n+1 times sum of sizes of intersections of all n

sets

|A1 [ A2 [ A3 [ hellip [ An|

We want to show that every element is counted exactly once

Consider an element which belongs to exactly k sets say A1 A2 A3 hellip Ak

In the formula such an element is counted the following number of times

Therefore each element is counted exactly once and thus the formula is correct

Euler Function

Given a number n how many numbers from 1 to n are relatively prime to n

When n is a prime number

then every number from 1 to n-1 is relatively prime to n

and so

When n is a prime power

then p 2p 3p 4p hellip n are not relatively prime to n

there are np = pc-1 of them

and other numbers are relatively prime to n

Therefore

Euler Function

Given a number n how many numbers from 1 to n are relatively prime to n

Suppose

Then p 2p 3p 4p hellip n are not relatively prime to n there are np of them

Also q 2q 3q 4q hellip n are not relatively prime to n and there are nq of them

Other numbers are relatively prime to n

Therefore

The numbers pq 2pq 3pq hellip n are subtracted twice and there are npq of them

So the correct answer is

Euler Function

Given a number n how many numbers from 1 to n are relatively prime to n

Let

Let S be the set of numbers from 1 to n that are not relatively prime to n

Let Ai be the set of numbers that are a multiple of piS = A1 [ A2 [ hellip [ An

For the intersection of k sets say A1 A2 A3hellip Ak

then every number in A1 Aring A2 Aring hellip Aring Ak is a multiple of p1p2hellippk

then |A1 Aring A2 Aring hellip Aring Ak| = np1p2hellippk

Euler Function

Given a number n how many numbers from 1 to n are relatively prime to n

Let

Let S be the set of numbers from 1 to n that are not relatively prime to n

Let Ai be the set of numbers that are a multiple of piS = A1 [ A2 [ hellip [ An

|A1 Aring A2 Aring hellip Aring Ak| = np1p2hellippk

When r=3 (only 3 distinct factors)

|A1 [ A2 [ A3|

= np1 + np2 + np3

- np1p2 ndash np1p3 ndash np2p3 + np1p2p3

|A1 [ A2 [ A3| = |A1| + |A2| + |A3|

ndash |A1 Aring A2| ndash |A1 Aring A3| ndash |A2 Aring A3|

+ |A1 Aring A2 Aring A3|

= n(1-p1)(1-p2)(1-p3)

Euler Function

Given a number n how many numbers from 1 to n are relatively prime to n

Let

Let S be the set of numbers from 1 to n that are not relatively prime to n

Let Ai be the set of numbers that are a multiple of piS = A1 [ A2 [ hellip [ An

|A1 Aring A2 Aring hellip Aring Ak| = np1p2hellippk

sum of sizes of all single setsndash sum of sizes of all 2-set intersections+ sum of sizes of all 3-set intersectionsndash sum of sizes of all 4-set intersectionshellip+ (ndash1)n+1 times sum of sizes of intersections of

n sets

|A1 [ A2 [ A3 [ hellip [ An|

|S| = |A1 [ A2 [ hellip [ An|

= n(1-p1)(1-p2)hellip(1-pn)

calculationshellip

  • Modular Arithmetic
  • PowerPoint Presentation
  • Slide 3
  • Slide 4
  • Slide 5
  • Slide 6
  • Slide 7
  • Slide 8
  • Slide 9
  • Slide 10
  • Slide 11
  • Slide 12
  • Slide 13
  • Slide 14
  • Slide 15
  • Slide 16
  • Slide 17
  • Slide 18
  • Slide 19
  • Slide 20
  • Slide 21
  • Slide 22
  • Slide 23
  • Slide 24
  • Slide 25
  • Slide 26
  • Slide 27
  • Slide 28
  • Slide 29
  • Slide 30
  • Slide 31
  • Slide 32
  • Slide 33
  • Slide 34
  • Slide 35
  • Slide 36
  • Slide 37
  • Slide 38
  • Slide 39
  • Slide 40
  • Slide 41
  • Slide 42
  • Slide 43
  • Slide 44
  • Slide 45
Page 10: Modular Arithmetic

This Lecture

bull Applications Fast exponentiation and fast division test

Fast Exponentiation

1444 mod 713

= 144 144 144 144 mod 713

= 20736 144 144 mod 713

= 59 144 144 mod 713

= 8496 144 mod 713

= 653 144 mod 713

= 94032 mod 713

= 629 mod 713

20736 20736 mod 713

= 59 59 mod 713

= 3481 mod 713

= 629 mod 713

Because 20736 59 (mod 713)

Because 653 8496 (mod 713)

shortcut

Repeated Squaring

14450 mod 713

= 14432 14416 1442 mod 713

= 64848559 mod 713

= 242

1442 mod 713 = 59

1444 mod 713 = 1442 1442 mod 713= 5959 mod 713= 629

1448 mod 713= 14441444 mod 713= 629629 mod 713= 639

14416 mod 713= 14481448 mod 713= 639639 mod 713= 485

14432 mod 713= 1441614416 mod 713= 485485 mod 713= 648

Note that 50 = 32 + 16 + 2

Fast Division Test

Using the basic rules for modular addition and modular multiplication

we can derive some quick test to see if a big number is divisible by a small number

Suppose we are given the decimal representation of a big number N

To test if N is divisible by a small number n of course we can do a division to check

But can we do faster

If n = 2 we just need to check whether the last digit of N is even or not

If n = 10 we just need to check whether the last digit of N is 0 or not

If n = 5 we just need to check whether the last digit of N is either 5 or 0 or not

What about when n=3 When n=7 When n=11

Fast Division Test

A number written in decimal divisible by 9 if and only if

the sum of its digits is a multiple of 9

Example 1 9333234513171 is divisible by 9

9+3+3+3+2+3+4+5+1+3+1+7+1 = 45 is divisible by 9

Example 2 128573649683 is not divisible by 9

1+2+8+5+7+3+6+4+9+6+8+3 = 62 is not divisible by 9

Claim A number written in decimal is divisible by 9 if and

only if

the sum of its digits is a multiple of 9Hint 10 1 (mod 9)

Let the decimal representation of N be dkdk-1dk-2hellipd1d0

This means that N = dk10k + dk-110k-1 + hellip + d110 + d0

Note that di10i mod 9

= (di) (10i mod 9) mod 9

= (di) (10 mod 9) (10 mod 9) hellip (10 mod 9) mod 9

= (di) (1 mod 9) (1 mod 9) hellip (1 mod 9) mod 9

= di mod 9

i terms

Fast Division Test

Rule of modular multiplication

Let the decimal representation of n be dkdk-1dk-2hellipd1d0

This means that N = dk10k + dk-110k-1 + hellip + d110 + d0

Note that di10i mod 9 = di mod 9

Hence N mod 9 = (dk10k + dk-110k-1 + hellip + d110 + d0) mod 9

= (dk10k mod 9 + dk-110k-1 mod 9 + hellip + d110 mod 9 + d0 mod 9) mod

9

= (dk mod 9 + dk-1 mod 9 + hellip + d1 mod 9 + d0 mod 9) mod 9

= (dk + dk-1 + hellip + d1 + d0) mod 9

Hint 10 1 (mod 9)

Fast Division Test

Claim A number written in decimal is divisible by 9 if and

only if

the sum of its digits is a multiple of 9

Rule of modular addition

By previous slide

The same procedure works to test whether N is divisible by n=3

Fast Division Test

What about n=11

Hint 10 -1 (mod 11)

Let the decimal representation of N be d92d91d90hellipd1d0

Then N is divisible by 11 if and only if

d92-d91+d90hellip-d1+d0 is divisible by 11

Why Try to work it out before your TA shows you

What about n=7

Hint 1000 -1 (mod 7)

This Lecture

bull Multiplicative inverse

The multiplicative inverse of a number a is another number arsquo such that

a middot arsquo 1 (mod n)

Multiplication Inverse

Does every number has a multiplicative inverse in modular arithmetic

For real numbers every nonzero number has a multiplicative inverse

For integers only 1 has a multiplicative inverse

An interesting property of modular arithmetic is that

there are multiplicative inverse for integers

For example 2 5 = 1 mod 3 so 5 is a multiplicative inverse

for 2 under modulo 3 (and vice versa)

Multiplication Inverse

Does every number has a multiplicative inverse in modular arithmetic

Multiplication Inverse

What is the pattern

Case Study

Why 2 does not have a multiplicative inverse under modulo 6

Suppose it has a multiplicative inverse y

2y 1 (mod 6)

=gt 2y = 1 + 6x for some integer x

=gt y = frac12 + 3x

This is a contradiction since both x and y are integers

Necessary Condition

Claim An integer k does not have an multiplicative inverse under modulo n

if k and n have a common factor gt= 2 (gcd(kn) gt= 2)

Proof

This claim says that for k to have a multiplicative inverse modulo n

then a necessary condition is that k and n do not have a common factor gt= 2

Suppose by contradiction that there is an inverse krsquo for k such that

krsquok = 1 (mod n)

Then krsquok = 1 + xn for some integer x

Since both k and n have a common factor say cgt=2

then k=ck1 and n=cn1 for some integers k1 and n1

So krsquock1 = 1 + xcn1

Then krsquok1 = 1c + xn1

This is a contradiction since the LHS is an integer but the RHS is not

Sufficient Condition

What about if gcd(kn)=1

Would k always have an multiplicative inverse under modulo n

For example gcd(37) = 1 3middot5 1 (mod 7)

gcd(89) = 1

gcd(411) = 1 4middot3 1 (mod 11)

8middot8 1 (mod 9)

It seems that there is always an inverse in such a case but why

gcd(89) = 1 8s + 9t = 1 for some integers s and t

8s = 1 ndash 9t

8s 1 (mod 9)gcd(89) = spc(89)

Theorem If gcd(kn)=1 then have krsquo such that

kmiddotkrsquo 1 (mod n)

Proof Since gcd(kn)=1 there exist s and t so that sk + tn = 1

So tn = 1 - sk

This means n | 1 ndash sk

This means that 1 ndash sk 0 (mod n)

This means that 1 sk (mod n)

So krsquo = s is an multiplicative inverse for k

Sufficient Condition

gcd(kn)=spc(kn)

The multiplicative inverse can be computed by the extended Euclidean algorithm

Corollary k has a multiplicative inverse mod n if and only if gcd(kn)=1

This Lecture

bull Fermatrsquos little theorem

Cancellation

There is no general cancellation in modular arithmetic

Note that (mod n) is very similar to =

If a b (mod n) then a+c b+c (mod n)

If a b (mod n) then ac bc (mod n)

However if ac bc (mod n)

it is not necessarily true that a b (mod n)

For example 4middot2 1middot2 (mod 6) but 4 1 (mod 6)

3middot4 1middot4 (mod 8) but 3 1 (mod 8)

4middot3 1middot3 (mod 9) but 4 1 (mod 9)

Observation In all the above examples c and n have a common factor

Cancellation

Claim Assume gcd(kn) = 1 If imiddotk jmiddotk (mod n) then i j (mod n)

For example multiplicative inverse always exists if n is a prime

Proof Since gcd(kn) = 1 there exists krsquo such that kkrsquo 1 (mod n)

imiddotk jmiddotk (mod n)

=gt imiddotkmiddotkrsquo jmiddotkmiddotkrsquo (mod n)

=gt i j (mod n)

Remarks (Optional) This makes arithmetic modulo prime a field

a structure that ldquobehaves likerdquo real numbers

Arithmetic modulo prime is very useful in coding theory

In particular when p is a prime amp k not a multiple of p then gcd(kp)=1

If i j (mod p) then imiddotk jmiddotk (mod p)

Therefore

k mod p 2k mod p hellip (p-1)k mod p

are all different numbers

Fermatrsquos Little Theorem

Claim 1 Assume gcd(kn) = 1 If imiddotk jmiddotk (mod n) then i j (mod n)

Claim 2 Assume gcd(kn) = 1 If i j (mod n) then imiddotk jmiddotk (mod n)

For example when p=7 and k=3

3 mod 7 = 3 2middot3 mod 7 = 6 3middot3 mod 7 = 2 4middot3 mod 7 = 5 5middot3 mod 7 = 1 6middot3 mod 7 = 4

Notice that in the above example every number from 1 to 6 appears exactly once

In particular when p is a prime amp k not a multiple of p then gcd(kp)=1

If i j (mod p) then imiddotk jmiddotk (mod p)

Therefore

k mod p 2k mod p hellip (p-1)k mod p

are all different numbers

Fermatrsquos Little Theorem

Each of ik mod p cannot be equal to 0 because p is a prime number

Let ci = ik mod p

So 1 lt= c1 lt= p-1 1 lt= c2 lt= p-1 hellip 1lt = cp-1 lt= p-1

By the above we know that c1c2hellipcp-2cp-1 are all different

So for each i from 1 to p-1 there is exactly one cj such that cj = i

Therefore we have

(k mod p)middot(2k mod p)middothellipmiddot((p-1)k mod p) = c1middotc2middothellipmiddotcp-2middotcp-1 = 1middot2middot3hellipmiddot(p-2)middot(p-1)

Fermatrsquos Little Theorem

1 kp-1 (mod p)

Theorem If p is prime amp k not a multiple of p

For example when p=5 k=4 we have kp-1 mod p = 44 mod 5 = 1

ldquoProofrdquo

4middot3middot2middot1 [(4 mod 5) (2middot4 mod 5) (3middot4 mod 5) (4middot4 mod 5)] (mod 5)

[4 middot (2middot4) middot (3middot4) middot (4middot4)] (mod 5)

[44 middot (1middot2middot3middot4)] (mod 5)

Since gcd(1middot2middot3middot4 5)=1 we can cancel 1middot2middot3middot4 on both sides

This implies

1 44 (mod 5)

By the previous slide or direct calculation

Fermatrsquos Little Theorem

1 kp-1 (mod p)

Theorem If p is prime amp k not a multiple of p

Proof

1middot2middotmiddotmiddot(p-1) (k mod p middot 2k mod pmiddothellipmiddot(p-1)k mod p) mod p

(kmiddot2k middotmiddotmiddot (p-1)k) mod p

(kp-1)middot1middot2 middotmiddotmiddot (p-1) (mod p)

So by cancelling 1middot2 middotmiddotmiddot (p-1) on both sides applying

Claim 1

(we can cancel them because gcd(1middot2 middotmiddotmiddot (p-1) p)=1)

we have

1 kp-1 (mod p)

By 2 slides before

By the multiplication rule

Wilsonrsquos Theorem

Theorem p is a prime if and only if

(p-1) -1 (mod p)

First we consider the easy direction

If p is not a prime assume p gt= 5 (for p=4 3 2 (mod 4) )

Then p=qr for some 2 lt= q lt p and 2 lt= r lt p

If q ne r then both q and r appear in (p-1)

and so (p-1) 0 (mod p)

If q = r then p = q2 gt 2q (since we assume p gt 5 and thus q gt 2)

then both q and 2q are in (p-1)

and so again (p-1) 0 (mod p)

Wilsonrsquos Theorem

Theorem p is a prime if and only if

(p-1) -1 (mod p)

To prove the more interesting direction first we need a lemma

Lemma If p is a prime number

x2 1 (mod p) if and only if x 1 (mod p) or x -1 (mod p)

Proof x2 1 (mod p)

iff p | x2 - 1

iff p | (x ndash 1)(x + 1)

iff p | (x ndash 1) or p | (x+1)

iff x 1 (mod p) or x -1 (mod p)

Lemma p prime and p|amiddotb iff p|a or p|b

Wilsonrsquos Theorem

Theorem p is a prime if and only if

(p-1) -1 (mod p)

Letrsquos get the proof idea by considering a concrete example

10

1middot2middot3middot4middot5middot6middot7middot8middot9middot10 mod 11

1middot10middot(2middot6)middot(3middot4)middot(5middot9)middot(7middot8) mod 11

1middot-1middot(1)middot(1)middot(1)middot(1) mod 11

-1 mod 11

Besides 1 and 10 the remaining numbers are paired up into multiplicative inverse

Wilsonrsquos Theorem

Theorem p is a prime if and only if

(p-1) -1 (mod p)

ProofSince p is a prime every number from 1 to p-1 has a multiplicative inverse

By the Lemma every number 2 lt= k lt= p-2 has an inverse krsquo with knekrsquo

Since p is odd the numbers from 2 to p-2 can be grouped into pairs

(a1b1)(a2b2)hellip(a(p-3)2b(p-3)2) so that aibi 1 (mod p)

Therefore (p-1) 1middot(p-1)middot2middot3middotmiddotmiddotmiddotmiddot(p-3)middot(p-2) (mod p)

1middot(p-1)middot(a1b1)middot(a2b2)middotmiddotmiddotmiddotmiddot(a(p-3)2b(p-3)2) (mod p)

1middot(-1)middot(1)middot(1)middotmiddotmiddotmiddotmiddot(1) (mod p)

-1 (mod p)

This Lecture

bull Eulerrsquos phi function

Inclusion-Exclusion (n sets)

What is the inclusion-exclusion formula for the union of n sets

sum of sizes of all single setsndash sum of sizes of all 2-set intersections+ sum of sizes of all 3-set intersectionsndash sum of sizes of all 4-set intersectionshellip+ (ndash1)n+1 times sum of sizes of intersections of all n

sets

1 2 nA A A

1

12 1

( 1)n

ki

S nk i S

S k

A

Inclusion-Exclusion (n sets)

Inclusion-Exclusion (n sets)

sum of sizes of all single setsndash sum of sizes of all 2-set intersections+ sum of sizes of all 3-set intersectionsndash sum of sizes of all 4-set intersectionshellip+ (ndash1)n+1 times sum of sizes of intersections of all n

sets

|A1 [ A2 [ A3 [ hellip [ An|

We want to show that every element is counted exactly once

Consider an element which belongs to exactly k sets say A1 A2 A3 hellip Ak

In the formula such an element is counted the following number of times

Therefore each element is counted exactly once and thus the formula is correct

Euler Function

Given a number n how many numbers from 1 to n are relatively prime to n

When n is a prime number

then every number from 1 to n-1 is relatively prime to n

and so

When n is a prime power

then p 2p 3p 4p hellip n are not relatively prime to n

there are np = pc-1 of them

and other numbers are relatively prime to n

Therefore

Euler Function

Given a number n how many numbers from 1 to n are relatively prime to n

Suppose

Then p 2p 3p 4p hellip n are not relatively prime to n there are np of them

Also q 2q 3q 4q hellip n are not relatively prime to n and there are nq of them

Other numbers are relatively prime to n

Therefore

The numbers pq 2pq 3pq hellip n are subtracted twice and there are npq of them

So the correct answer is

Euler Function

Given a number n how many numbers from 1 to n are relatively prime to n

Let

Let S be the set of numbers from 1 to n that are not relatively prime to n

Let Ai be the set of numbers that are a multiple of piS = A1 [ A2 [ hellip [ An

For the intersection of k sets say A1 A2 A3hellip Ak

then every number in A1 Aring A2 Aring hellip Aring Ak is a multiple of p1p2hellippk

then |A1 Aring A2 Aring hellip Aring Ak| = np1p2hellippk

Euler Function

Given a number n how many numbers from 1 to n are relatively prime to n

Let

Let S be the set of numbers from 1 to n that are not relatively prime to n

Let Ai be the set of numbers that are a multiple of piS = A1 [ A2 [ hellip [ An

|A1 Aring A2 Aring hellip Aring Ak| = np1p2hellippk

When r=3 (only 3 distinct factors)

|A1 [ A2 [ A3|

= np1 + np2 + np3

- np1p2 ndash np1p3 ndash np2p3 + np1p2p3

|A1 [ A2 [ A3| = |A1| + |A2| + |A3|

ndash |A1 Aring A2| ndash |A1 Aring A3| ndash |A2 Aring A3|

+ |A1 Aring A2 Aring A3|

= n(1-p1)(1-p2)(1-p3)

Euler Function

Given a number n how many numbers from 1 to n are relatively prime to n

Let

Let S be the set of numbers from 1 to n that are not relatively prime to n

Let Ai be the set of numbers that are a multiple of piS = A1 [ A2 [ hellip [ An

|A1 Aring A2 Aring hellip Aring Ak| = np1p2hellippk

sum of sizes of all single setsndash sum of sizes of all 2-set intersections+ sum of sizes of all 3-set intersectionsndash sum of sizes of all 4-set intersectionshellip+ (ndash1)n+1 times sum of sizes of intersections of

n sets

|A1 [ A2 [ A3 [ hellip [ An|

|S| = |A1 [ A2 [ hellip [ An|

= n(1-p1)(1-p2)hellip(1-pn)

calculationshellip

  • Modular Arithmetic
  • PowerPoint Presentation
  • Slide 3
  • Slide 4
  • Slide 5
  • Slide 6
  • Slide 7
  • Slide 8
  • Slide 9
  • Slide 10
  • Slide 11
  • Slide 12
  • Slide 13
  • Slide 14
  • Slide 15
  • Slide 16
  • Slide 17
  • Slide 18
  • Slide 19
  • Slide 20
  • Slide 21
  • Slide 22
  • Slide 23
  • Slide 24
  • Slide 25
  • Slide 26
  • Slide 27
  • Slide 28
  • Slide 29
  • Slide 30
  • Slide 31
  • Slide 32
  • Slide 33
  • Slide 34
  • Slide 35
  • Slide 36
  • Slide 37
  • Slide 38
  • Slide 39
  • Slide 40
  • Slide 41
  • Slide 42
  • Slide 43
  • Slide 44
  • Slide 45
Page 11: Modular Arithmetic

Fast Exponentiation

1444 mod 713

= 144 144 144 144 mod 713

= 20736 144 144 mod 713

= 59 144 144 mod 713

= 8496 144 mod 713

= 653 144 mod 713

= 94032 mod 713

= 629 mod 713

20736 20736 mod 713

= 59 59 mod 713

= 3481 mod 713

= 629 mod 713

Because 20736 59 (mod 713)

Because 653 8496 (mod 713)

shortcut

Repeated Squaring

14450 mod 713

= 14432 14416 1442 mod 713

= 64848559 mod 713

= 242

1442 mod 713 = 59

1444 mod 713 = 1442 1442 mod 713= 5959 mod 713= 629

1448 mod 713= 14441444 mod 713= 629629 mod 713= 639

14416 mod 713= 14481448 mod 713= 639639 mod 713= 485

14432 mod 713= 1441614416 mod 713= 485485 mod 713= 648

Note that 50 = 32 + 16 + 2

Fast Division Test

Using the basic rules for modular addition and modular multiplication

we can derive some quick test to see if a big number is divisible by a small number

Suppose we are given the decimal representation of a big number N

To test if N is divisible by a small number n of course we can do a division to check

But can we do faster

If n = 2 we just need to check whether the last digit of N is even or not

If n = 10 we just need to check whether the last digit of N is 0 or not

If n = 5 we just need to check whether the last digit of N is either 5 or 0 or not

What about when n=3 When n=7 When n=11

Fast Division Test

A number written in decimal divisible by 9 if and only if

the sum of its digits is a multiple of 9

Example 1 9333234513171 is divisible by 9

9+3+3+3+2+3+4+5+1+3+1+7+1 = 45 is divisible by 9

Example 2 128573649683 is not divisible by 9

1+2+8+5+7+3+6+4+9+6+8+3 = 62 is not divisible by 9

Claim A number written in decimal is divisible by 9 if and

only if

the sum of its digits is a multiple of 9Hint 10 1 (mod 9)

Let the decimal representation of N be dkdk-1dk-2hellipd1d0

This means that N = dk10k + dk-110k-1 + hellip + d110 + d0

Note that di10i mod 9

= (di) (10i mod 9) mod 9

= (di) (10 mod 9) (10 mod 9) hellip (10 mod 9) mod 9

= (di) (1 mod 9) (1 mod 9) hellip (1 mod 9) mod 9

= di mod 9

i terms

Fast Division Test

Rule of modular multiplication

Let the decimal representation of n be dkdk-1dk-2hellipd1d0

This means that N = dk10k + dk-110k-1 + hellip + d110 + d0

Note that di10i mod 9 = di mod 9

Hence N mod 9 = (dk10k + dk-110k-1 + hellip + d110 + d0) mod 9

= (dk10k mod 9 + dk-110k-1 mod 9 + hellip + d110 mod 9 + d0 mod 9) mod

9

= (dk mod 9 + dk-1 mod 9 + hellip + d1 mod 9 + d0 mod 9) mod 9

= (dk + dk-1 + hellip + d1 + d0) mod 9

Hint 10 1 (mod 9)

Fast Division Test

Claim A number written in decimal is divisible by 9 if and

only if

the sum of its digits is a multiple of 9

Rule of modular addition

By previous slide

The same procedure works to test whether N is divisible by n=3

Fast Division Test

What about n=11

Hint 10 -1 (mod 11)

Let the decimal representation of N be d92d91d90hellipd1d0

Then N is divisible by 11 if and only if

d92-d91+d90hellip-d1+d0 is divisible by 11

Why Try to work it out before your TA shows you

What about n=7

Hint 1000 -1 (mod 7)

This Lecture

bull Multiplicative inverse

The multiplicative inverse of a number a is another number arsquo such that

a middot arsquo 1 (mod n)

Multiplication Inverse

Does every number has a multiplicative inverse in modular arithmetic

For real numbers every nonzero number has a multiplicative inverse

For integers only 1 has a multiplicative inverse

An interesting property of modular arithmetic is that

there are multiplicative inverse for integers

For example 2 5 = 1 mod 3 so 5 is a multiplicative inverse

for 2 under modulo 3 (and vice versa)

Multiplication Inverse

Does every number has a multiplicative inverse in modular arithmetic

Multiplication Inverse

What is the pattern

Case Study

Why 2 does not have a multiplicative inverse under modulo 6

Suppose it has a multiplicative inverse y

2y 1 (mod 6)

=gt 2y = 1 + 6x for some integer x

=gt y = frac12 + 3x

This is a contradiction since both x and y are integers

Necessary Condition

Claim An integer k does not have an multiplicative inverse under modulo n

if k and n have a common factor gt= 2 (gcd(kn) gt= 2)

Proof

This claim says that for k to have a multiplicative inverse modulo n

then a necessary condition is that k and n do not have a common factor gt= 2

Suppose by contradiction that there is an inverse krsquo for k such that

krsquok = 1 (mod n)

Then krsquok = 1 + xn for some integer x

Since both k and n have a common factor say cgt=2

then k=ck1 and n=cn1 for some integers k1 and n1

So krsquock1 = 1 + xcn1

Then krsquok1 = 1c + xn1

This is a contradiction since the LHS is an integer but the RHS is not

Sufficient Condition

What about if gcd(kn)=1

Would k always have an multiplicative inverse under modulo n

For example gcd(37) = 1 3middot5 1 (mod 7)

gcd(89) = 1

gcd(411) = 1 4middot3 1 (mod 11)

8middot8 1 (mod 9)

It seems that there is always an inverse in such a case but why

gcd(89) = 1 8s + 9t = 1 for some integers s and t

8s = 1 ndash 9t

8s 1 (mod 9)gcd(89) = spc(89)

Theorem If gcd(kn)=1 then have krsquo such that

kmiddotkrsquo 1 (mod n)

Proof Since gcd(kn)=1 there exist s and t so that sk + tn = 1

So tn = 1 - sk

This means n | 1 ndash sk

This means that 1 ndash sk 0 (mod n)

This means that 1 sk (mod n)

So krsquo = s is an multiplicative inverse for k

Sufficient Condition

gcd(kn)=spc(kn)

The multiplicative inverse can be computed by the extended Euclidean algorithm

Corollary k has a multiplicative inverse mod n if and only if gcd(kn)=1

This Lecture

bull Fermatrsquos little theorem

Cancellation

There is no general cancellation in modular arithmetic

Note that (mod n) is very similar to =

If a b (mod n) then a+c b+c (mod n)

If a b (mod n) then ac bc (mod n)

However if ac bc (mod n)

it is not necessarily true that a b (mod n)

For example 4middot2 1middot2 (mod 6) but 4 1 (mod 6)

3middot4 1middot4 (mod 8) but 3 1 (mod 8)

4middot3 1middot3 (mod 9) but 4 1 (mod 9)

Observation In all the above examples c and n have a common factor

Cancellation

Claim Assume gcd(kn) = 1 If imiddotk jmiddotk (mod n) then i j (mod n)

For example multiplicative inverse always exists if n is a prime

Proof Since gcd(kn) = 1 there exists krsquo such that kkrsquo 1 (mod n)

imiddotk jmiddotk (mod n)

=gt imiddotkmiddotkrsquo jmiddotkmiddotkrsquo (mod n)

=gt i j (mod n)

Remarks (Optional) This makes arithmetic modulo prime a field

a structure that ldquobehaves likerdquo real numbers

Arithmetic modulo prime is very useful in coding theory

In particular when p is a prime amp k not a multiple of p then gcd(kp)=1

If i j (mod p) then imiddotk jmiddotk (mod p)

Therefore

k mod p 2k mod p hellip (p-1)k mod p

are all different numbers

Fermatrsquos Little Theorem

Claim 1 Assume gcd(kn) = 1 If imiddotk jmiddotk (mod n) then i j (mod n)

Claim 2 Assume gcd(kn) = 1 If i j (mod n) then imiddotk jmiddotk (mod n)

For example when p=7 and k=3

3 mod 7 = 3 2middot3 mod 7 = 6 3middot3 mod 7 = 2 4middot3 mod 7 = 5 5middot3 mod 7 = 1 6middot3 mod 7 = 4

Notice that in the above example every number from 1 to 6 appears exactly once

In particular when p is a prime amp k not a multiple of p then gcd(kp)=1

If i j (mod p) then imiddotk jmiddotk (mod p)

Therefore

k mod p 2k mod p hellip (p-1)k mod p

are all different numbers

Fermatrsquos Little Theorem

Each of ik mod p cannot be equal to 0 because p is a prime number

Let ci = ik mod p

So 1 lt= c1 lt= p-1 1 lt= c2 lt= p-1 hellip 1lt = cp-1 lt= p-1

By the above we know that c1c2hellipcp-2cp-1 are all different

So for each i from 1 to p-1 there is exactly one cj such that cj = i

Therefore we have

(k mod p)middot(2k mod p)middothellipmiddot((p-1)k mod p) = c1middotc2middothellipmiddotcp-2middotcp-1 = 1middot2middot3hellipmiddot(p-2)middot(p-1)

Fermatrsquos Little Theorem

1 kp-1 (mod p)

Theorem If p is prime amp k not a multiple of p

For example when p=5 k=4 we have kp-1 mod p = 44 mod 5 = 1

ldquoProofrdquo

4middot3middot2middot1 [(4 mod 5) (2middot4 mod 5) (3middot4 mod 5) (4middot4 mod 5)] (mod 5)

[4 middot (2middot4) middot (3middot4) middot (4middot4)] (mod 5)

[44 middot (1middot2middot3middot4)] (mod 5)

Since gcd(1middot2middot3middot4 5)=1 we can cancel 1middot2middot3middot4 on both sides

This implies

1 44 (mod 5)

By the previous slide or direct calculation

Fermatrsquos Little Theorem

1 kp-1 (mod p)

Theorem If p is prime amp k not a multiple of p

Proof

1middot2middotmiddotmiddot(p-1) (k mod p middot 2k mod pmiddothellipmiddot(p-1)k mod p) mod p

(kmiddot2k middotmiddotmiddot (p-1)k) mod p

(kp-1)middot1middot2 middotmiddotmiddot (p-1) (mod p)

So by cancelling 1middot2 middotmiddotmiddot (p-1) on both sides applying

Claim 1

(we can cancel them because gcd(1middot2 middotmiddotmiddot (p-1) p)=1)

we have

1 kp-1 (mod p)

By 2 slides before

By the multiplication rule

Wilsonrsquos Theorem

Theorem p is a prime if and only if

(p-1) -1 (mod p)

First we consider the easy direction

If p is not a prime assume p gt= 5 (for p=4 3 2 (mod 4) )

Then p=qr for some 2 lt= q lt p and 2 lt= r lt p

If q ne r then both q and r appear in (p-1)

and so (p-1) 0 (mod p)

If q = r then p = q2 gt 2q (since we assume p gt 5 and thus q gt 2)

then both q and 2q are in (p-1)

and so again (p-1) 0 (mod p)

Wilsonrsquos Theorem

Theorem p is a prime if and only if

(p-1) -1 (mod p)

To prove the more interesting direction first we need a lemma

Lemma If p is a prime number

x2 1 (mod p) if and only if x 1 (mod p) or x -1 (mod p)

Proof x2 1 (mod p)

iff p | x2 - 1

iff p | (x ndash 1)(x + 1)

iff p | (x ndash 1) or p | (x+1)

iff x 1 (mod p) or x -1 (mod p)

Lemma p prime and p|amiddotb iff p|a or p|b

Wilsonrsquos Theorem

Theorem p is a prime if and only if

(p-1) -1 (mod p)

Letrsquos get the proof idea by considering a concrete example

10

1middot2middot3middot4middot5middot6middot7middot8middot9middot10 mod 11

1middot10middot(2middot6)middot(3middot4)middot(5middot9)middot(7middot8) mod 11

1middot-1middot(1)middot(1)middot(1)middot(1) mod 11

-1 mod 11

Besides 1 and 10 the remaining numbers are paired up into multiplicative inverse

Wilsonrsquos Theorem

Theorem p is a prime if and only if

(p-1) -1 (mod p)

ProofSince p is a prime every number from 1 to p-1 has a multiplicative inverse

By the Lemma every number 2 lt= k lt= p-2 has an inverse krsquo with knekrsquo

Since p is odd the numbers from 2 to p-2 can be grouped into pairs

(a1b1)(a2b2)hellip(a(p-3)2b(p-3)2) so that aibi 1 (mod p)

Therefore (p-1) 1middot(p-1)middot2middot3middotmiddotmiddotmiddotmiddot(p-3)middot(p-2) (mod p)

1middot(p-1)middot(a1b1)middot(a2b2)middotmiddotmiddotmiddotmiddot(a(p-3)2b(p-3)2) (mod p)

1middot(-1)middot(1)middot(1)middotmiddotmiddotmiddotmiddot(1) (mod p)

-1 (mod p)

This Lecture

bull Eulerrsquos phi function

Inclusion-Exclusion (n sets)

What is the inclusion-exclusion formula for the union of n sets

sum of sizes of all single setsndash sum of sizes of all 2-set intersections+ sum of sizes of all 3-set intersectionsndash sum of sizes of all 4-set intersectionshellip+ (ndash1)n+1 times sum of sizes of intersections of all n

sets

1 2 nA A A

1

12 1

( 1)n

ki

S nk i S

S k

A

Inclusion-Exclusion (n sets)

Inclusion-Exclusion (n sets)

sum of sizes of all single setsndash sum of sizes of all 2-set intersections+ sum of sizes of all 3-set intersectionsndash sum of sizes of all 4-set intersectionshellip+ (ndash1)n+1 times sum of sizes of intersections of all n

sets

|A1 [ A2 [ A3 [ hellip [ An|

We want to show that every element is counted exactly once

Consider an element which belongs to exactly k sets say A1 A2 A3 hellip Ak

In the formula such an element is counted the following number of times

Therefore each element is counted exactly once and thus the formula is correct

Euler Function

Given a number n how many numbers from 1 to n are relatively prime to n

When n is a prime number

then every number from 1 to n-1 is relatively prime to n

and so

When n is a prime power

then p 2p 3p 4p hellip n are not relatively prime to n

there are np = pc-1 of them

and other numbers are relatively prime to n

Therefore

Euler Function

Given a number n how many numbers from 1 to n are relatively prime to n

Suppose

Then p 2p 3p 4p hellip n are not relatively prime to n there are np of them

Also q 2q 3q 4q hellip n are not relatively prime to n and there are nq of them

Other numbers are relatively prime to n

Therefore

The numbers pq 2pq 3pq hellip n are subtracted twice and there are npq of them

So the correct answer is

Euler Function

Given a number n how many numbers from 1 to n are relatively prime to n

Let

Let S be the set of numbers from 1 to n that are not relatively prime to n

Let Ai be the set of numbers that are a multiple of piS = A1 [ A2 [ hellip [ An

For the intersection of k sets say A1 A2 A3hellip Ak

then every number in A1 Aring A2 Aring hellip Aring Ak is a multiple of p1p2hellippk

then |A1 Aring A2 Aring hellip Aring Ak| = np1p2hellippk

Euler Function

Given a number n how many numbers from 1 to n are relatively prime to n

Let

Let S be the set of numbers from 1 to n that are not relatively prime to n

Let Ai be the set of numbers that are a multiple of piS = A1 [ A2 [ hellip [ An

|A1 Aring A2 Aring hellip Aring Ak| = np1p2hellippk

When r=3 (only 3 distinct factors)

|A1 [ A2 [ A3|

= np1 + np2 + np3

- np1p2 ndash np1p3 ndash np2p3 + np1p2p3

|A1 [ A2 [ A3| = |A1| + |A2| + |A3|

ndash |A1 Aring A2| ndash |A1 Aring A3| ndash |A2 Aring A3|

+ |A1 Aring A2 Aring A3|

= n(1-p1)(1-p2)(1-p3)

Euler Function

Given a number n how many numbers from 1 to n are relatively prime to n

Let

Let S be the set of numbers from 1 to n that are not relatively prime to n

Let Ai be the set of numbers that are a multiple of piS = A1 [ A2 [ hellip [ An

|A1 Aring A2 Aring hellip Aring Ak| = np1p2hellippk

sum of sizes of all single setsndash sum of sizes of all 2-set intersections+ sum of sizes of all 3-set intersectionsndash sum of sizes of all 4-set intersectionshellip+ (ndash1)n+1 times sum of sizes of intersections of

n sets

|A1 [ A2 [ A3 [ hellip [ An|

|S| = |A1 [ A2 [ hellip [ An|

= n(1-p1)(1-p2)hellip(1-pn)

calculationshellip

  • Modular Arithmetic
  • PowerPoint Presentation
  • Slide 3
  • Slide 4
  • Slide 5
  • Slide 6
  • Slide 7
  • Slide 8
  • Slide 9
  • Slide 10
  • Slide 11
  • Slide 12
  • Slide 13
  • Slide 14
  • Slide 15
  • Slide 16
  • Slide 17
  • Slide 18
  • Slide 19
  • Slide 20
  • Slide 21
  • Slide 22
  • Slide 23
  • Slide 24
  • Slide 25
  • Slide 26
  • Slide 27
  • Slide 28
  • Slide 29
  • Slide 30
  • Slide 31
  • Slide 32
  • Slide 33
  • Slide 34
  • Slide 35
  • Slide 36
  • Slide 37
  • Slide 38
  • Slide 39
  • Slide 40
  • Slide 41
  • Slide 42
  • Slide 43
  • Slide 44
  • Slide 45
Page 12: Modular Arithmetic

Repeated Squaring

14450 mod 713

= 14432 14416 1442 mod 713

= 64848559 mod 713

= 242

1442 mod 713 = 59

1444 mod 713 = 1442 1442 mod 713= 5959 mod 713= 629

1448 mod 713= 14441444 mod 713= 629629 mod 713= 639

14416 mod 713= 14481448 mod 713= 639639 mod 713= 485

14432 mod 713= 1441614416 mod 713= 485485 mod 713= 648

Note that 50 = 32 + 16 + 2

Fast Division Test

Using the basic rules for modular addition and modular multiplication

we can derive some quick test to see if a big number is divisible by a small number

Suppose we are given the decimal representation of a big number N

To test if N is divisible by a small number n of course we can do a division to check

But can we do faster

If n = 2 we just need to check whether the last digit of N is even or not

If n = 10 we just need to check whether the last digit of N is 0 or not

If n = 5 we just need to check whether the last digit of N is either 5 or 0 or not

What about when n=3 When n=7 When n=11

Fast Division Test

A number written in decimal divisible by 9 if and only if

the sum of its digits is a multiple of 9

Example 1 9333234513171 is divisible by 9

9+3+3+3+2+3+4+5+1+3+1+7+1 = 45 is divisible by 9

Example 2 128573649683 is not divisible by 9

1+2+8+5+7+3+6+4+9+6+8+3 = 62 is not divisible by 9

Claim A number written in decimal is divisible by 9 if and

only if

the sum of its digits is a multiple of 9Hint 10 1 (mod 9)

Let the decimal representation of N be dkdk-1dk-2hellipd1d0

This means that N = dk10k + dk-110k-1 + hellip + d110 + d0

Note that di10i mod 9

= (di) (10i mod 9) mod 9

= (di) (10 mod 9) (10 mod 9) hellip (10 mod 9) mod 9

= (di) (1 mod 9) (1 mod 9) hellip (1 mod 9) mod 9

= di mod 9

i terms

Fast Division Test

Rule of modular multiplication

Let the decimal representation of n be dkdk-1dk-2hellipd1d0

This means that N = dk10k + dk-110k-1 + hellip + d110 + d0

Note that di10i mod 9 = di mod 9

Hence N mod 9 = (dk10k + dk-110k-1 + hellip + d110 + d0) mod 9

= (dk10k mod 9 + dk-110k-1 mod 9 + hellip + d110 mod 9 + d0 mod 9) mod

9

= (dk mod 9 + dk-1 mod 9 + hellip + d1 mod 9 + d0 mod 9) mod 9

= (dk + dk-1 + hellip + d1 + d0) mod 9

Hint 10 1 (mod 9)

Fast Division Test

Claim A number written in decimal is divisible by 9 if and

only if

the sum of its digits is a multiple of 9

Rule of modular addition

By previous slide

The same procedure works to test whether N is divisible by n=3

Fast Division Test

What about n=11

Hint 10 -1 (mod 11)

Let the decimal representation of N be d92d91d90hellipd1d0

Then N is divisible by 11 if and only if

d92-d91+d90hellip-d1+d0 is divisible by 11

Why Try to work it out before your TA shows you

What about n=7

Hint 1000 -1 (mod 7)

This Lecture

bull Multiplicative inverse

The multiplicative inverse of a number a is another number arsquo such that

a middot arsquo 1 (mod n)

Multiplication Inverse

Does every number has a multiplicative inverse in modular arithmetic

For real numbers every nonzero number has a multiplicative inverse

For integers only 1 has a multiplicative inverse

An interesting property of modular arithmetic is that

there are multiplicative inverse for integers

For example 2 5 = 1 mod 3 so 5 is a multiplicative inverse

for 2 under modulo 3 (and vice versa)

Multiplication Inverse

Does every number has a multiplicative inverse in modular arithmetic

Multiplication Inverse

What is the pattern

Case Study

Why 2 does not have a multiplicative inverse under modulo 6

Suppose it has a multiplicative inverse y

2y 1 (mod 6)

=gt 2y = 1 + 6x for some integer x

=gt y = frac12 + 3x

This is a contradiction since both x and y are integers

Necessary Condition

Claim An integer k does not have an multiplicative inverse under modulo n

if k and n have a common factor gt= 2 (gcd(kn) gt= 2)

Proof

This claim says that for k to have a multiplicative inverse modulo n

then a necessary condition is that k and n do not have a common factor gt= 2

Suppose by contradiction that there is an inverse krsquo for k such that

krsquok = 1 (mod n)

Then krsquok = 1 + xn for some integer x

Since both k and n have a common factor say cgt=2

then k=ck1 and n=cn1 for some integers k1 and n1

So krsquock1 = 1 + xcn1

Then krsquok1 = 1c + xn1

This is a contradiction since the LHS is an integer but the RHS is not

Sufficient Condition

What about if gcd(kn)=1

Would k always have an multiplicative inverse under modulo n

For example gcd(37) = 1 3middot5 1 (mod 7)

gcd(89) = 1

gcd(411) = 1 4middot3 1 (mod 11)

8middot8 1 (mod 9)

It seems that there is always an inverse in such a case but why

gcd(89) = 1 8s + 9t = 1 for some integers s and t

8s = 1 ndash 9t

8s 1 (mod 9)gcd(89) = spc(89)

Theorem If gcd(kn)=1 then have krsquo such that

kmiddotkrsquo 1 (mod n)

Proof Since gcd(kn)=1 there exist s and t so that sk + tn = 1

So tn = 1 - sk

This means n | 1 ndash sk

This means that 1 ndash sk 0 (mod n)

This means that 1 sk (mod n)

So krsquo = s is an multiplicative inverse for k

Sufficient Condition

gcd(kn)=spc(kn)

The multiplicative inverse can be computed by the extended Euclidean algorithm

Corollary k has a multiplicative inverse mod n if and only if gcd(kn)=1

This Lecture

bull Fermatrsquos little theorem

Cancellation

There is no general cancellation in modular arithmetic

Note that (mod n) is very similar to =

If a b (mod n) then a+c b+c (mod n)

If a b (mod n) then ac bc (mod n)

However if ac bc (mod n)

it is not necessarily true that a b (mod n)

For example 4middot2 1middot2 (mod 6) but 4 1 (mod 6)

3middot4 1middot4 (mod 8) but 3 1 (mod 8)

4middot3 1middot3 (mod 9) but 4 1 (mod 9)

Observation In all the above examples c and n have a common factor

Cancellation

Claim Assume gcd(kn) = 1 If imiddotk jmiddotk (mod n) then i j (mod n)

For example multiplicative inverse always exists if n is a prime

Proof Since gcd(kn) = 1 there exists krsquo such that kkrsquo 1 (mod n)

imiddotk jmiddotk (mod n)

=gt imiddotkmiddotkrsquo jmiddotkmiddotkrsquo (mod n)

=gt i j (mod n)

Remarks (Optional) This makes arithmetic modulo prime a field

a structure that ldquobehaves likerdquo real numbers

Arithmetic modulo prime is very useful in coding theory

In particular when p is a prime amp k not a multiple of p then gcd(kp)=1

If i j (mod p) then imiddotk jmiddotk (mod p)

Therefore

k mod p 2k mod p hellip (p-1)k mod p

are all different numbers

Fermatrsquos Little Theorem

Claim 1 Assume gcd(kn) = 1 If imiddotk jmiddotk (mod n) then i j (mod n)

Claim 2 Assume gcd(kn) = 1 If i j (mod n) then imiddotk jmiddotk (mod n)

For example when p=7 and k=3

3 mod 7 = 3 2middot3 mod 7 = 6 3middot3 mod 7 = 2 4middot3 mod 7 = 5 5middot3 mod 7 = 1 6middot3 mod 7 = 4

Notice that in the above example every number from 1 to 6 appears exactly once

In particular when p is a prime amp k not a multiple of p then gcd(kp)=1

If i j (mod p) then imiddotk jmiddotk (mod p)

Therefore

k mod p 2k mod p hellip (p-1)k mod p

are all different numbers

Fermatrsquos Little Theorem

Each of ik mod p cannot be equal to 0 because p is a prime number

Let ci = ik mod p

So 1 lt= c1 lt= p-1 1 lt= c2 lt= p-1 hellip 1lt = cp-1 lt= p-1

By the above we know that c1c2hellipcp-2cp-1 are all different

So for each i from 1 to p-1 there is exactly one cj such that cj = i

Therefore we have

(k mod p)middot(2k mod p)middothellipmiddot((p-1)k mod p) = c1middotc2middothellipmiddotcp-2middotcp-1 = 1middot2middot3hellipmiddot(p-2)middot(p-1)

Fermatrsquos Little Theorem

1 kp-1 (mod p)

Theorem If p is prime amp k not a multiple of p

For example when p=5 k=4 we have kp-1 mod p = 44 mod 5 = 1

ldquoProofrdquo

4middot3middot2middot1 [(4 mod 5) (2middot4 mod 5) (3middot4 mod 5) (4middot4 mod 5)] (mod 5)

[4 middot (2middot4) middot (3middot4) middot (4middot4)] (mod 5)

[44 middot (1middot2middot3middot4)] (mod 5)

Since gcd(1middot2middot3middot4 5)=1 we can cancel 1middot2middot3middot4 on both sides

This implies

1 44 (mod 5)

By the previous slide or direct calculation

Fermatrsquos Little Theorem

1 kp-1 (mod p)

Theorem If p is prime amp k not a multiple of p

Proof

1middot2middotmiddotmiddot(p-1) (k mod p middot 2k mod pmiddothellipmiddot(p-1)k mod p) mod p

(kmiddot2k middotmiddotmiddot (p-1)k) mod p

(kp-1)middot1middot2 middotmiddotmiddot (p-1) (mod p)

So by cancelling 1middot2 middotmiddotmiddot (p-1) on both sides applying

Claim 1

(we can cancel them because gcd(1middot2 middotmiddotmiddot (p-1) p)=1)

we have

1 kp-1 (mod p)

By 2 slides before

By the multiplication rule

Wilsonrsquos Theorem

Theorem p is a prime if and only if

(p-1) -1 (mod p)

First we consider the easy direction

If p is not a prime assume p gt= 5 (for p=4 3 2 (mod 4) )

Then p=qr for some 2 lt= q lt p and 2 lt= r lt p

If q ne r then both q and r appear in (p-1)

and so (p-1) 0 (mod p)

If q = r then p = q2 gt 2q (since we assume p gt 5 and thus q gt 2)

then both q and 2q are in (p-1)

and so again (p-1) 0 (mod p)

Wilsonrsquos Theorem

Theorem p is a prime if and only if

(p-1) -1 (mod p)

To prove the more interesting direction first we need a lemma

Lemma If p is a prime number

x2 1 (mod p) if and only if x 1 (mod p) or x -1 (mod p)

Proof x2 1 (mod p)

iff p | x2 - 1

iff p | (x ndash 1)(x + 1)

iff p | (x ndash 1) or p | (x+1)

iff x 1 (mod p) or x -1 (mod p)

Lemma p prime and p|amiddotb iff p|a or p|b

Wilsonrsquos Theorem

Theorem p is a prime if and only if

(p-1) -1 (mod p)

Letrsquos get the proof idea by considering a concrete example

10

1middot2middot3middot4middot5middot6middot7middot8middot9middot10 mod 11

1middot10middot(2middot6)middot(3middot4)middot(5middot9)middot(7middot8) mod 11

1middot-1middot(1)middot(1)middot(1)middot(1) mod 11

-1 mod 11

Besides 1 and 10 the remaining numbers are paired up into multiplicative inverse

Wilsonrsquos Theorem

Theorem p is a prime if and only if

(p-1) -1 (mod p)

ProofSince p is a prime every number from 1 to p-1 has a multiplicative inverse

By the Lemma every number 2 lt= k lt= p-2 has an inverse krsquo with knekrsquo

Since p is odd the numbers from 2 to p-2 can be grouped into pairs

(a1b1)(a2b2)hellip(a(p-3)2b(p-3)2) so that aibi 1 (mod p)

Therefore (p-1) 1middot(p-1)middot2middot3middotmiddotmiddotmiddotmiddot(p-3)middot(p-2) (mod p)

1middot(p-1)middot(a1b1)middot(a2b2)middotmiddotmiddotmiddotmiddot(a(p-3)2b(p-3)2) (mod p)

1middot(-1)middot(1)middot(1)middotmiddotmiddotmiddotmiddot(1) (mod p)

-1 (mod p)

This Lecture

bull Eulerrsquos phi function

Inclusion-Exclusion (n sets)

What is the inclusion-exclusion formula for the union of n sets

sum of sizes of all single setsndash sum of sizes of all 2-set intersections+ sum of sizes of all 3-set intersectionsndash sum of sizes of all 4-set intersectionshellip+ (ndash1)n+1 times sum of sizes of intersections of all n

sets

1 2 nA A A

1

12 1

( 1)n

ki

S nk i S

S k

A

Inclusion-Exclusion (n sets)

Inclusion-Exclusion (n sets)

sum of sizes of all single setsndash sum of sizes of all 2-set intersections+ sum of sizes of all 3-set intersectionsndash sum of sizes of all 4-set intersectionshellip+ (ndash1)n+1 times sum of sizes of intersections of all n

sets

|A1 [ A2 [ A3 [ hellip [ An|

We want to show that every element is counted exactly once

Consider an element which belongs to exactly k sets say A1 A2 A3 hellip Ak

In the formula such an element is counted the following number of times

Therefore each element is counted exactly once and thus the formula is correct

Euler Function

Given a number n how many numbers from 1 to n are relatively prime to n

When n is a prime number

then every number from 1 to n-1 is relatively prime to n

and so

When n is a prime power

then p 2p 3p 4p hellip n are not relatively prime to n

there are np = pc-1 of them

and other numbers are relatively prime to n

Therefore

Euler Function

Given a number n how many numbers from 1 to n are relatively prime to n

Suppose

Then p 2p 3p 4p hellip n are not relatively prime to n there are np of them

Also q 2q 3q 4q hellip n are not relatively prime to n and there are nq of them

Other numbers are relatively prime to n

Therefore

The numbers pq 2pq 3pq hellip n are subtracted twice and there are npq of them

So the correct answer is

Euler Function

Given a number n how many numbers from 1 to n are relatively prime to n

Let

Let S be the set of numbers from 1 to n that are not relatively prime to n

Let Ai be the set of numbers that are a multiple of piS = A1 [ A2 [ hellip [ An

For the intersection of k sets say A1 A2 A3hellip Ak

then every number in A1 Aring A2 Aring hellip Aring Ak is a multiple of p1p2hellippk

then |A1 Aring A2 Aring hellip Aring Ak| = np1p2hellippk

Euler Function

Given a number n how many numbers from 1 to n are relatively prime to n

Let

Let S be the set of numbers from 1 to n that are not relatively prime to n

Let Ai be the set of numbers that are a multiple of piS = A1 [ A2 [ hellip [ An

|A1 Aring A2 Aring hellip Aring Ak| = np1p2hellippk

When r=3 (only 3 distinct factors)

|A1 [ A2 [ A3|

= np1 + np2 + np3

- np1p2 ndash np1p3 ndash np2p3 + np1p2p3

|A1 [ A2 [ A3| = |A1| + |A2| + |A3|

ndash |A1 Aring A2| ndash |A1 Aring A3| ndash |A2 Aring A3|

+ |A1 Aring A2 Aring A3|

= n(1-p1)(1-p2)(1-p3)

Euler Function

Given a number n how many numbers from 1 to n are relatively prime to n

Let

Let S be the set of numbers from 1 to n that are not relatively prime to n

Let Ai be the set of numbers that are a multiple of piS = A1 [ A2 [ hellip [ An

|A1 Aring A2 Aring hellip Aring Ak| = np1p2hellippk

sum of sizes of all single setsndash sum of sizes of all 2-set intersections+ sum of sizes of all 3-set intersectionsndash sum of sizes of all 4-set intersectionshellip+ (ndash1)n+1 times sum of sizes of intersections of

n sets

|A1 [ A2 [ A3 [ hellip [ An|

|S| = |A1 [ A2 [ hellip [ An|

= n(1-p1)(1-p2)hellip(1-pn)

calculationshellip

  • Modular Arithmetic
  • PowerPoint Presentation
  • Slide 3
  • Slide 4
  • Slide 5
  • Slide 6
  • Slide 7
  • Slide 8
  • Slide 9
  • Slide 10
  • Slide 11
  • Slide 12
  • Slide 13
  • Slide 14
  • Slide 15
  • Slide 16
  • Slide 17
  • Slide 18
  • Slide 19
  • Slide 20
  • Slide 21
  • Slide 22
  • Slide 23
  • Slide 24
  • Slide 25
  • Slide 26
  • Slide 27
  • Slide 28
  • Slide 29
  • Slide 30
  • Slide 31
  • Slide 32
  • Slide 33
  • Slide 34
  • Slide 35
  • Slide 36
  • Slide 37
  • Slide 38
  • Slide 39
  • Slide 40
  • Slide 41
  • Slide 42
  • Slide 43
  • Slide 44
  • Slide 45
Page 13: Modular Arithmetic

Fast Division Test

Using the basic rules for modular addition and modular multiplication

we can derive some quick test to see if a big number is divisible by a small number

Suppose we are given the decimal representation of a big number N

To test if N is divisible by a small number n of course we can do a division to check

But can we do faster

If n = 2 we just need to check whether the last digit of N is even or not

If n = 10 we just need to check whether the last digit of N is 0 or not

If n = 5 we just need to check whether the last digit of N is either 5 or 0 or not

What about when n=3 When n=7 When n=11

Fast Division Test

A number written in decimal divisible by 9 if and only if

the sum of its digits is a multiple of 9

Example 1 9333234513171 is divisible by 9

9+3+3+3+2+3+4+5+1+3+1+7+1 = 45 is divisible by 9

Example 2 128573649683 is not divisible by 9

1+2+8+5+7+3+6+4+9+6+8+3 = 62 is not divisible by 9

Claim A number written in decimal is divisible by 9 if and

only if

the sum of its digits is a multiple of 9Hint 10 1 (mod 9)

Let the decimal representation of N be dkdk-1dk-2hellipd1d0

This means that N = dk10k + dk-110k-1 + hellip + d110 + d0

Note that di10i mod 9

= (di) (10i mod 9) mod 9

= (di) (10 mod 9) (10 mod 9) hellip (10 mod 9) mod 9

= (di) (1 mod 9) (1 mod 9) hellip (1 mod 9) mod 9

= di mod 9

i terms

Fast Division Test

Rule of modular multiplication

Let the decimal representation of n be dkdk-1dk-2hellipd1d0

This means that N = dk10k + dk-110k-1 + hellip + d110 + d0

Note that di10i mod 9 = di mod 9

Hence N mod 9 = (dk10k + dk-110k-1 + hellip + d110 + d0) mod 9

= (dk10k mod 9 + dk-110k-1 mod 9 + hellip + d110 mod 9 + d0 mod 9) mod

9

= (dk mod 9 + dk-1 mod 9 + hellip + d1 mod 9 + d0 mod 9) mod 9

= (dk + dk-1 + hellip + d1 + d0) mod 9

Hint 10 1 (mod 9)

Fast Division Test

Claim A number written in decimal is divisible by 9 if and

only if

the sum of its digits is a multiple of 9

Rule of modular addition

By previous slide

The same procedure works to test whether N is divisible by n=3

Fast Division Test

What about n=11

Hint 10 -1 (mod 11)

Let the decimal representation of N be d92d91d90hellipd1d0

Then N is divisible by 11 if and only if

d92-d91+d90hellip-d1+d0 is divisible by 11

Why Try to work it out before your TA shows you

What about n=7

Hint 1000 -1 (mod 7)

This Lecture

bull Multiplicative inverse

The multiplicative inverse of a number a is another number arsquo such that

a middot arsquo 1 (mod n)

Multiplication Inverse

Does every number has a multiplicative inverse in modular arithmetic

For real numbers every nonzero number has a multiplicative inverse

For integers only 1 has a multiplicative inverse

An interesting property of modular arithmetic is that

there are multiplicative inverse for integers

For example 2 5 = 1 mod 3 so 5 is a multiplicative inverse

for 2 under modulo 3 (and vice versa)

Multiplication Inverse

Does every number has a multiplicative inverse in modular arithmetic

Multiplication Inverse

What is the pattern

Case Study

Why 2 does not have a multiplicative inverse under modulo 6

Suppose it has a multiplicative inverse y

2y 1 (mod 6)

=gt 2y = 1 + 6x for some integer x

=gt y = frac12 + 3x

This is a contradiction since both x and y are integers

Necessary Condition

Claim An integer k does not have an multiplicative inverse under modulo n

if k and n have a common factor gt= 2 (gcd(kn) gt= 2)

Proof

This claim says that for k to have a multiplicative inverse modulo n

then a necessary condition is that k and n do not have a common factor gt= 2

Suppose by contradiction that there is an inverse krsquo for k such that

krsquok = 1 (mod n)

Then krsquok = 1 + xn for some integer x

Since both k and n have a common factor say cgt=2

then k=ck1 and n=cn1 for some integers k1 and n1

So krsquock1 = 1 + xcn1

Then krsquok1 = 1c + xn1

This is a contradiction since the LHS is an integer but the RHS is not

Sufficient Condition

What about if gcd(kn)=1

Would k always have an multiplicative inverse under modulo n

For example gcd(37) = 1 3middot5 1 (mod 7)

gcd(89) = 1

gcd(411) = 1 4middot3 1 (mod 11)

8middot8 1 (mod 9)

It seems that there is always an inverse in such a case but why

gcd(89) = 1 8s + 9t = 1 for some integers s and t

8s = 1 ndash 9t

8s 1 (mod 9)gcd(89) = spc(89)

Theorem If gcd(kn)=1 then have krsquo such that

kmiddotkrsquo 1 (mod n)

Proof Since gcd(kn)=1 there exist s and t so that sk + tn = 1

So tn = 1 - sk

This means n | 1 ndash sk

This means that 1 ndash sk 0 (mod n)

This means that 1 sk (mod n)

So krsquo = s is an multiplicative inverse for k

Sufficient Condition

gcd(kn)=spc(kn)

The multiplicative inverse can be computed by the extended Euclidean algorithm

Corollary k has a multiplicative inverse mod n if and only if gcd(kn)=1

This Lecture

bull Fermatrsquos little theorem

Cancellation

There is no general cancellation in modular arithmetic

Note that (mod n) is very similar to =

If a b (mod n) then a+c b+c (mod n)

If a b (mod n) then ac bc (mod n)

However if ac bc (mod n)

it is not necessarily true that a b (mod n)

For example 4middot2 1middot2 (mod 6) but 4 1 (mod 6)

3middot4 1middot4 (mod 8) but 3 1 (mod 8)

4middot3 1middot3 (mod 9) but 4 1 (mod 9)

Observation In all the above examples c and n have a common factor

Cancellation

Claim Assume gcd(kn) = 1 If imiddotk jmiddotk (mod n) then i j (mod n)

For example multiplicative inverse always exists if n is a prime

Proof Since gcd(kn) = 1 there exists krsquo such that kkrsquo 1 (mod n)

imiddotk jmiddotk (mod n)

=gt imiddotkmiddotkrsquo jmiddotkmiddotkrsquo (mod n)

=gt i j (mod n)

Remarks (Optional) This makes arithmetic modulo prime a field

a structure that ldquobehaves likerdquo real numbers

Arithmetic modulo prime is very useful in coding theory

In particular when p is a prime amp k not a multiple of p then gcd(kp)=1

If i j (mod p) then imiddotk jmiddotk (mod p)

Therefore

k mod p 2k mod p hellip (p-1)k mod p

are all different numbers

Fermatrsquos Little Theorem

Claim 1 Assume gcd(kn) = 1 If imiddotk jmiddotk (mod n) then i j (mod n)

Claim 2 Assume gcd(kn) = 1 If i j (mod n) then imiddotk jmiddotk (mod n)

For example when p=7 and k=3

3 mod 7 = 3 2middot3 mod 7 = 6 3middot3 mod 7 = 2 4middot3 mod 7 = 5 5middot3 mod 7 = 1 6middot3 mod 7 = 4

Notice that in the above example every number from 1 to 6 appears exactly once

In particular when p is a prime amp k not a multiple of p then gcd(kp)=1

If i j (mod p) then imiddotk jmiddotk (mod p)

Therefore

k mod p 2k mod p hellip (p-1)k mod p

are all different numbers

Fermatrsquos Little Theorem

Each of ik mod p cannot be equal to 0 because p is a prime number

Let ci = ik mod p

So 1 lt= c1 lt= p-1 1 lt= c2 lt= p-1 hellip 1lt = cp-1 lt= p-1

By the above we know that c1c2hellipcp-2cp-1 are all different

So for each i from 1 to p-1 there is exactly one cj such that cj = i

Therefore we have

(k mod p)middot(2k mod p)middothellipmiddot((p-1)k mod p) = c1middotc2middothellipmiddotcp-2middotcp-1 = 1middot2middot3hellipmiddot(p-2)middot(p-1)

Fermatrsquos Little Theorem

1 kp-1 (mod p)

Theorem If p is prime amp k not a multiple of p

For example when p=5 k=4 we have kp-1 mod p = 44 mod 5 = 1

ldquoProofrdquo

4middot3middot2middot1 [(4 mod 5) (2middot4 mod 5) (3middot4 mod 5) (4middot4 mod 5)] (mod 5)

[4 middot (2middot4) middot (3middot4) middot (4middot4)] (mod 5)

[44 middot (1middot2middot3middot4)] (mod 5)

Since gcd(1middot2middot3middot4 5)=1 we can cancel 1middot2middot3middot4 on both sides

This implies

1 44 (mod 5)

By the previous slide or direct calculation

Fermatrsquos Little Theorem

1 kp-1 (mod p)

Theorem If p is prime amp k not a multiple of p

Proof

1middot2middotmiddotmiddot(p-1) (k mod p middot 2k mod pmiddothellipmiddot(p-1)k mod p) mod p

(kmiddot2k middotmiddotmiddot (p-1)k) mod p

(kp-1)middot1middot2 middotmiddotmiddot (p-1) (mod p)

So by cancelling 1middot2 middotmiddotmiddot (p-1) on both sides applying

Claim 1

(we can cancel them because gcd(1middot2 middotmiddotmiddot (p-1) p)=1)

we have

1 kp-1 (mod p)

By 2 slides before

By the multiplication rule

Wilsonrsquos Theorem

Theorem p is a prime if and only if

(p-1) -1 (mod p)

First we consider the easy direction

If p is not a prime assume p gt= 5 (for p=4 3 2 (mod 4) )

Then p=qr for some 2 lt= q lt p and 2 lt= r lt p

If q ne r then both q and r appear in (p-1)

and so (p-1) 0 (mod p)

If q = r then p = q2 gt 2q (since we assume p gt 5 and thus q gt 2)

then both q and 2q are in (p-1)

and so again (p-1) 0 (mod p)

Wilsonrsquos Theorem

Theorem p is a prime if and only if

(p-1) -1 (mod p)

To prove the more interesting direction first we need a lemma

Lemma If p is a prime number

x2 1 (mod p) if and only if x 1 (mod p) or x -1 (mod p)

Proof x2 1 (mod p)

iff p | x2 - 1

iff p | (x ndash 1)(x + 1)

iff p | (x ndash 1) or p | (x+1)

iff x 1 (mod p) or x -1 (mod p)

Lemma p prime and p|amiddotb iff p|a or p|b

Wilsonrsquos Theorem

Theorem p is a prime if and only if

(p-1) -1 (mod p)

Letrsquos get the proof idea by considering a concrete example

10

1middot2middot3middot4middot5middot6middot7middot8middot9middot10 mod 11

1middot10middot(2middot6)middot(3middot4)middot(5middot9)middot(7middot8) mod 11

1middot-1middot(1)middot(1)middot(1)middot(1) mod 11

-1 mod 11

Besides 1 and 10 the remaining numbers are paired up into multiplicative inverse

Wilsonrsquos Theorem

Theorem p is a prime if and only if

(p-1) -1 (mod p)

ProofSince p is a prime every number from 1 to p-1 has a multiplicative inverse

By the Lemma every number 2 lt= k lt= p-2 has an inverse krsquo with knekrsquo

Since p is odd the numbers from 2 to p-2 can be grouped into pairs

(a1b1)(a2b2)hellip(a(p-3)2b(p-3)2) so that aibi 1 (mod p)

Therefore (p-1) 1middot(p-1)middot2middot3middotmiddotmiddotmiddotmiddot(p-3)middot(p-2) (mod p)

1middot(p-1)middot(a1b1)middot(a2b2)middotmiddotmiddotmiddotmiddot(a(p-3)2b(p-3)2) (mod p)

1middot(-1)middot(1)middot(1)middotmiddotmiddotmiddotmiddot(1) (mod p)

-1 (mod p)

This Lecture

bull Eulerrsquos phi function

Inclusion-Exclusion (n sets)

What is the inclusion-exclusion formula for the union of n sets

sum of sizes of all single setsndash sum of sizes of all 2-set intersections+ sum of sizes of all 3-set intersectionsndash sum of sizes of all 4-set intersectionshellip+ (ndash1)n+1 times sum of sizes of intersections of all n

sets

1 2 nA A A

1

12 1

( 1)n

ki

S nk i S

S k

A

Inclusion-Exclusion (n sets)

Inclusion-Exclusion (n sets)

sum of sizes of all single setsndash sum of sizes of all 2-set intersections+ sum of sizes of all 3-set intersectionsndash sum of sizes of all 4-set intersectionshellip+ (ndash1)n+1 times sum of sizes of intersections of all n

sets

|A1 [ A2 [ A3 [ hellip [ An|

We want to show that every element is counted exactly once

Consider an element which belongs to exactly k sets say A1 A2 A3 hellip Ak

In the formula such an element is counted the following number of times

Therefore each element is counted exactly once and thus the formula is correct

Euler Function

Given a number n how many numbers from 1 to n are relatively prime to n

When n is a prime number

then every number from 1 to n-1 is relatively prime to n

and so

When n is a prime power

then p 2p 3p 4p hellip n are not relatively prime to n

there are np = pc-1 of them

and other numbers are relatively prime to n

Therefore

Euler Function

Given a number n how many numbers from 1 to n are relatively prime to n

Suppose

Then p 2p 3p 4p hellip n are not relatively prime to n there are np of them

Also q 2q 3q 4q hellip n are not relatively prime to n and there are nq of them

Other numbers are relatively prime to n

Therefore

The numbers pq 2pq 3pq hellip n are subtracted twice and there are npq of them

So the correct answer is

Euler Function

Given a number n how many numbers from 1 to n are relatively prime to n

Let

Let S be the set of numbers from 1 to n that are not relatively prime to n

Let Ai be the set of numbers that are a multiple of piS = A1 [ A2 [ hellip [ An

For the intersection of k sets say A1 A2 A3hellip Ak

then every number in A1 Aring A2 Aring hellip Aring Ak is a multiple of p1p2hellippk

then |A1 Aring A2 Aring hellip Aring Ak| = np1p2hellippk

Euler Function

Given a number n how many numbers from 1 to n are relatively prime to n

Let

Let S be the set of numbers from 1 to n that are not relatively prime to n

Let Ai be the set of numbers that are a multiple of piS = A1 [ A2 [ hellip [ An

|A1 Aring A2 Aring hellip Aring Ak| = np1p2hellippk

When r=3 (only 3 distinct factors)

|A1 [ A2 [ A3|

= np1 + np2 + np3

- np1p2 ndash np1p3 ndash np2p3 + np1p2p3

|A1 [ A2 [ A3| = |A1| + |A2| + |A3|

ndash |A1 Aring A2| ndash |A1 Aring A3| ndash |A2 Aring A3|

+ |A1 Aring A2 Aring A3|

= n(1-p1)(1-p2)(1-p3)

Euler Function

Given a number n how many numbers from 1 to n are relatively prime to n

Let

Let S be the set of numbers from 1 to n that are not relatively prime to n

Let Ai be the set of numbers that are a multiple of piS = A1 [ A2 [ hellip [ An

|A1 Aring A2 Aring hellip Aring Ak| = np1p2hellippk

sum of sizes of all single setsndash sum of sizes of all 2-set intersections+ sum of sizes of all 3-set intersectionsndash sum of sizes of all 4-set intersectionshellip+ (ndash1)n+1 times sum of sizes of intersections of

n sets

|A1 [ A2 [ A3 [ hellip [ An|

|S| = |A1 [ A2 [ hellip [ An|

= n(1-p1)(1-p2)hellip(1-pn)

calculationshellip

  • Modular Arithmetic
  • PowerPoint Presentation
  • Slide 3
  • Slide 4
  • Slide 5
  • Slide 6
  • Slide 7
  • Slide 8
  • Slide 9
  • Slide 10
  • Slide 11
  • Slide 12
  • Slide 13
  • Slide 14
  • Slide 15
  • Slide 16
  • Slide 17
  • Slide 18
  • Slide 19
  • Slide 20
  • Slide 21
  • Slide 22
  • Slide 23
  • Slide 24
  • Slide 25
  • Slide 26
  • Slide 27
  • Slide 28
  • Slide 29
  • Slide 30
  • Slide 31
  • Slide 32
  • Slide 33
  • Slide 34
  • Slide 35
  • Slide 36
  • Slide 37
  • Slide 38
  • Slide 39
  • Slide 40
  • Slide 41
  • Slide 42
  • Slide 43
  • Slide 44
  • Slide 45
Page 14: Modular Arithmetic

Fast Division Test

A number written in decimal divisible by 9 if and only if

the sum of its digits is a multiple of 9

Example 1 9333234513171 is divisible by 9

9+3+3+3+2+3+4+5+1+3+1+7+1 = 45 is divisible by 9

Example 2 128573649683 is not divisible by 9

1+2+8+5+7+3+6+4+9+6+8+3 = 62 is not divisible by 9

Claim A number written in decimal is divisible by 9 if and

only if

the sum of its digits is a multiple of 9Hint 10 1 (mod 9)

Let the decimal representation of N be dkdk-1dk-2hellipd1d0

This means that N = dk10k + dk-110k-1 + hellip + d110 + d0

Note that di10i mod 9

= (di) (10i mod 9) mod 9

= (di) (10 mod 9) (10 mod 9) hellip (10 mod 9) mod 9

= (di) (1 mod 9) (1 mod 9) hellip (1 mod 9) mod 9

= di mod 9

i terms

Fast Division Test

Rule of modular multiplication

Let the decimal representation of n be dkdk-1dk-2hellipd1d0

This means that N = dk10k + dk-110k-1 + hellip + d110 + d0

Note that di10i mod 9 = di mod 9

Hence N mod 9 = (dk10k + dk-110k-1 + hellip + d110 + d0) mod 9

= (dk10k mod 9 + dk-110k-1 mod 9 + hellip + d110 mod 9 + d0 mod 9) mod

9

= (dk mod 9 + dk-1 mod 9 + hellip + d1 mod 9 + d0 mod 9) mod 9

= (dk + dk-1 + hellip + d1 + d0) mod 9

Hint 10 1 (mod 9)

Fast Division Test

Claim A number written in decimal is divisible by 9 if and

only if

the sum of its digits is a multiple of 9

Rule of modular addition

By previous slide

The same procedure works to test whether N is divisible by n=3

Fast Division Test

What about n=11

Hint 10 -1 (mod 11)

Let the decimal representation of N be d92d91d90hellipd1d0

Then N is divisible by 11 if and only if

d92-d91+d90hellip-d1+d0 is divisible by 11

Why Try to work it out before your TA shows you

What about n=7

Hint 1000 -1 (mod 7)

This Lecture

bull Multiplicative inverse

The multiplicative inverse of a number a is another number arsquo such that

a middot arsquo 1 (mod n)

Multiplication Inverse

Does every number has a multiplicative inverse in modular arithmetic

For real numbers every nonzero number has a multiplicative inverse

For integers only 1 has a multiplicative inverse

An interesting property of modular arithmetic is that

there are multiplicative inverse for integers

For example 2 5 = 1 mod 3 so 5 is a multiplicative inverse

for 2 under modulo 3 (and vice versa)

Multiplication Inverse

Does every number has a multiplicative inverse in modular arithmetic

Multiplication Inverse

What is the pattern

Case Study

Why 2 does not have a multiplicative inverse under modulo 6

Suppose it has a multiplicative inverse y

2y 1 (mod 6)

=gt 2y = 1 + 6x for some integer x

=gt y = frac12 + 3x

This is a contradiction since both x and y are integers

Necessary Condition

Claim An integer k does not have an multiplicative inverse under modulo n

if k and n have a common factor gt= 2 (gcd(kn) gt= 2)

Proof

This claim says that for k to have a multiplicative inverse modulo n

then a necessary condition is that k and n do not have a common factor gt= 2

Suppose by contradiction that there is an inverse krsquo for k such that

krsquok = 1 (mod n)

Then krsquok = 1 + xn for some integer x

Since both k and n have a common factor say cgt=2

then k=ck1 and n=cn1 for some integers k1 and n1

So krsquock1 = 1 + xcn1

Then krsquok1 = 1c + xn1

This is a contradiction since the LHS is an integer but the RHS is not

Sufficient Condition

What about if gcd(kn)=1

Would k always have an multiplicative inverse under modulo n

For example gcd(37) = 1 3middot5 1 (mod 7)

gcd(89) = 1

gcd(411) = 1 4middot3 1 (mod 11)

8middot8 1 (mod 9)

It seems that there is always an inverse in such a case but why

gcd(89) = 1 8s + 9t = 1 for some integers s and t

8s = 1 ndash 9t

8s 1 (mod 9)gcd(89) = spc(89)

Theorem If gcd(kn)=1 then have krsquo such that

kmiddotkrsquo 1 (mod n)

Proof Since gcd(kn)=1 there exist s and t so that sk + tn = 1

So tn = 1 - sk

This means n | 1 ndash sk

This means that 1 ndash sk 0 (mod n)

This means that 1 sk (mod n)

So krsquo = s is an multiplicative inverse for k

Sufficient Condition

gcd(kn)=spc(kn)

The multiplicative inverse can be computed by the extended Euclidean algorithm

Corollary k has a multiplicative inverse mod n if and only if gcd(kn)=1

This Lecture

bull Fermatrsquos little theorem

Cancellation

There is no general cancellation in modular arithmetic

Note that (mod n) is very similar to =

If a b (mod n) then a+c b+c (mod n)

If a b (mod n) then ac bc (mod n)

However if ac bc (mod n)

it is not necessarily true that a b (mod n)

For example 4middot2 1middot2 (mod 6) but 4 1 (mod 6)

3middot4 1middot4 (mod 8) but 3 1 (mod 8)

4middot3 1middot3 (mod 9) but 4 1 (mod 9)

Observation In all the above examples c and n have a common factor

Cancellation

Claim Assume gcd(kn) = 1 If imiddotk jmiddotk (mod n) then i j (mod n)

For example multiplicative inverse always exists if n is a prime

Proof Since gcd(kn) = 1 there exists krsquo such that kkrsquo 1 (mod n)

imiddotk jmiddotk (mod n)

=gt imiddotkmiddotkrsquo jmiddotkmiddotkrsquo (mod n)

=gt i j (mod n)

Remarks (Optional) This makes arithmetic modulo prime a field

a structure that ldquobehaves likerdquo real numbers

Arithmetic modulo prime is very useful in coding theory

In particular when p is a prime amp k not a multiple of p then gcd(kp)=1

If i j (mod p) then imiddotk jmiddotk (mod p)

Therefore

k mod p 2k mod p hellip (p-1)k mod p

are all different numbers

Fermatrsquos Little Theorem

Claim 1 Assume gcd(kn) = 1 If imiddotk jmiddotk (mod n) then i j (mod n)

Claim 2 Assume gcd(kn) = 1 If i j (mod n) then imiddotk jmiddotk (mod n)

For example when p=7 and k=3

3 mod 7 = 3 2middot3 mod 7 = 6 3middot3 mod 7 = 2 4middot3 mod 7 = 5 5middot3 mod 7 = 1 6middot3 mod 7 = 4

Notice that in the above example every number from 1 to 6 appears exactly once

In particular when p is a prime amp k not a multiple of p then gcd(kp)=1

If i j (mod p) then imiddotk jmiddotk (mod p)

Therefore

k mod p 2k mod p hellip (p-1)k mod p

are all different numbers

Fermatrsquos Little Theorem

Each of ik mod p cannot be equal to 0 because p is a prime number

Let ci = ik mod p

So 1 lt= c1 lt= p-1 1 lt= c2 lt= p-1 hellip 1lt = cp-1 lt= p-1

By the above we know that c1c2hellipcp-2cp-1 are all different

So for each i from 1 to p-1 there is exactly one cj such that cj = i

Therefore we have

(k mod p)middot(2k mod p)middothellipmiddot((p-1)k mod p) = c1middotc2middothellipmiddotcp-2middotcp-1 = 1middot2middot3hellipmiddot(p-2)middot(p-1)

Fermatrsquos Little Theorem

1 kp-1 (mod p)

Theorem If p is prime amp k not a multiple of p

For example when p=5 k=4 we have kp-1 mod p = 44 mod 5 = 1

ldquoProofrdquo

4middot3middot2middot1 [(4 mod 5) (2middot4 mod 5) (3middot4 mod 5) (4middot4 mod 5)] (mod 5)

[4 middot (2middot4) middot (3middot4) middot (4middot4)] (mod 5)

[44 middot (1middot2middot3middot4)] (mod 5)

Since gcd(1middot2middot3middot4 5)=1 we can cancel 1middot2middot3middot4 on both sides

This implies

1 44 (mod 5)

By the previous slide or direct calculation

Fermatrsquos Little Theorem

1 kp-1 (mod p)

Theorem If p is prime amp k not a multiple of p

Proof

1middot2middotmiddotmiddot(p-1) (k mod p middot 2k mod pmiddothellipmiddot(p-1)k mod p) mod p

(kmiddot2k middotmiddotmiddot (p-1)k) mod p

(kp-1)middot1middot2 middotmiddotmiddot (p-1) (mod p)

So by cancelling 1middot2 middotmiddotmiddot (p-1) on both sides applying

Claim 1

(we can cancel them because gcd(1middot2 middotmiddotmiddot (p-1) p)=1)

we have

1 kp-1 (mod p)

By 2 slides before

By the multiplication rule

Wilsonrsquos Theorem

Theorem p is a prime if and only if

(p-1) -1 (mod p)

First we consider the easy direction

If p is not a prime assume p gt= 5 (for p=4 3 2 (mod 4) )

Then p=qr for some 2 lt= q lt p and 2 lt= r lt p

If q ne r then both q and r appear in (p-1)

and so (p-1) 0 (mod p)

If q = r then p = q2 gt 2q (since we assume p gt 5 and thus q gt 2)

then both q and 2q are in (p-1)

and so again (p-1) 0 (mod p)

Wilsonrsquos Theorem

Theorem p is a prime if and only if

(p-1) -1 (mod p)

To prove the more interesting direction first we need a lemma

Lemma If p is a prime number

x2 1 (mod p) if and only if x 1 (mod p) or x -1 (mod p)

Proof x2 1 (mod p)

iff p | x2 - 1

iff p | (x ndash 1)(x + 1)

iff p | (x ndash 1) or p | (x+1)

iff x 1 (mod p) or x -1 (mod p)

Lemma p prime and p|amiddotb iff p|a or p|b

Wilsonrsquos Theorem

Theorem p is a prime if and only if

(p-1) -1 (mod p)

Letrsquos get the proof idea by considering a concrete example

10

1middot2middot3middot4middot5middot6middot7middot8middot9middot10 mod 11

1middot10middot(2middot6)middot(3middot4)middot(5middot9)middot(7middot8) mod 11

1middot-1middot(1)middot(1)middot(1)middot(1) mod 11

-1 mod 11

Besides 1 and 10 the remaining numbers are paired up into multiplicative inverse

Wilsonrsquos Theorem

Theorem p is a prime if and only if

(p-1) -1 (mod p)

ProofSince p is a prime every number from 1 to p-1 has a multiplicative inverse

By the Lemma every number 2 lt= k lt= p-2 has an inverse krsquo with knekrsquo

Since p is odd the numbers from 2 to p-2 can be grouped into pairs

(a1b1)(a2b2)hellip(a(p-3)2b(p-3)2) so that aibi 1 (mod p)

Therefore (p-1) 1middot(p-1)middot2middot3middotmiddotmiddotmiddotmiddot(p-3)middot(p-2) (mod p)

1middot(p-1)middot(a1b1)middot(a2b2)middotmiddotmiddotmiddotmiddot(a(p-3)2b(p-3)2) (mod p)

1middot(-1)middot(1)middot(1)middotmiddotmiddotmiddotmiddot(1) (mod p)

-1 (mod p)

This Lecture

bull Eulerrsquos phi function

Inclusion-Exclusion (n sets)

What is the inclusion-exclusion formula for the union of n sets

sum of sizes of all single setsndash sum of sizes of all 2-set intersections+ sum of sizes of all 3-set intersectionsndash sum of sizes of all 4-set intersectionshellip+ (ndash1)n+1 times sum of sizes of intersections of all n

sets

1 2 nA A A

1

12 1

( 1)n

ki

S nk i S

S k

A

Inclusion-Exclusion (n sets)

Inclusion-Exclusion (n sets)

sum of sizes of all single setsndash sum of sizes of all 2-set intersections+ sum of sizes of all 3-set intersectionsndash sum of sizes of all 4-set intersectionshellip+ (ndash1)n+1 times sum of sizes of intersections of all n

sets

|A1 [ A2 [ A3 [ hellip [ An|

We want to show that every element is counted exactly once

Consider an element which belongs to exactly k sets say A1 A2 A3 hellip Ak

In the formula such an element is counted the following number of times

Therefore each element is counted exactly once and thus the formula is correct

Euler Function

Given a number n how many numbers from 1 to n are relatively prime to n

When n is a prime number

then every number from 1 to n-1 is relatively prime to n

and so

When n is a prime power

then p 2p 3p 4p hellip n are not relatively prime to n

there are np = pc-1 of them

and other numbers are relatively prime to n

Therefore

Euler Function

Given a number n how many numbers from 1 to n are relatively prime to n

Suppose

Then p 2p 3p 4p hellip n are not relatively prime to n there are np of them

Also q 2q 3q 4q hellip n are not relatively prime to n and there are nq of them

Other numbers are relatively prime to n

Therefore

The numbers pq 2pq 3pq hellip n are subtracted twice and there are npq of them

So the correct answer is

Euler Function

Given a number n how many numbers from 1 to n are relatively prime to n

Let

Let S be the set of numbers from 1 to n that are not relatively prime to n

Let Ai be the set of numbers that are a multiple of piS = A1 [ A2 [ hellip [ An

For the intersection of k sets say A1 A2 A3hellip Ak

then every number in A1 Aring A2 Aring hellip Aring Ak is a multiple of p1p2hellippk

then |A1 Aring A2 Aring hellip Aring Ak| = np1p2hellippk

Euler Function

Given a number n how many numbers from 1 to n are relatively prime to n

Let

Let S be the set of numbers from 1 to n that are not relatively prime to n

Let Ai be the set of numbers that are a multiple of piS = A1 [ A2 [ hellip [ An

|A1 Aring A2 Aring hellip Aring Ak| = np1p2hellippk

When r=3 (only 3 distinct factors)

|A1 [ A2 [ A3|

= np1 + np2 + np3

- np1p2 ndash np1p3 ndash np2p3 + np1p2p3

|A1 [ A2 [ A3| = |A1| + |A2| + |A3|

ndash |A1 Aring A2| ndash |A1 Aring A3| ndash |A2 Aring A3|

+ |A1 Aring A2 Aring A3|

= n(1-p1)(1-p2)(1-p3)

Euler Function

Given a number n how many numbers from 1 to n are relatively prime to n

Let

Let S be the set of numbers from 1 to n that are not relatively prime to n

Let Ai be the set of numbers that are a multiple of piS = A1 [ A2 [ hellip [ An

|A1 Aring A2 Aring hellip Aring Ak| = np1p2hellippk

sum of sizes of all single setsndash sum of sizes of all 2-set intersections+ sum of sizes of all 3-set intersectionsndash sum of sizes of all 4-set intersectionshellip+ (ndash1)n+1 times sum of sizes of intersections of

n sets

|A1 [ A2 [ A3 [ hellip [ An|

|S| = |A1 [ A2 [ hellip [ An|

= n(1-p1)(1-p2)hellip(1-pn)

calculationshellip

  • Modular Arithmetic
  • PowerPoint Presentation
  • Slide 3
  • Slide 4
  • Slide 5
  • Slide 6
  • Slide 7
  • Slide 8
  • Slide 9
  • Slide 10
  • Slide 11
  • Slide 12
  • Slide 13
  • Slide 14
  • Slide 15
  • Slide 16
  • Slide 17
  • Slide 18
  • Slide 19
  • Slide 20
  • Slide 21
  • Slide 22
  • Slide 23
  • Slide 24
  • Slide 25
  • Slide 26
  • Slide 27
  • Slide 28
  • Slide 29
  • Slide 30
  • Slide 31
  • Slide 32
  • Slide 33
  • Slide 34
  • Slide 35
  • Slide 36
  • Slide 37
  • Slide 38
  • Slide 39
  • Slide 40
  • Slide 41
  • Slide 42
  • Slide 43
  • Slide 44
  • Slide 45
Page 15: Modular Arithmetic

Claim A number written in decimal is divisible by 9 if and

only if

the sum of its digits is a multiple of 9Hint 10 1 (mod 9)

Let the decimal representation of N be dkdk-1dk-2hellipd1d0

This means that N = dk10k + dk-110k-1 + hellip + d110 + d0

Note that di10i mod 9

= (di) (10i mod 9) mod 9

= (di) (10 mod 9) (10 mod 9) hellip (10 mod 9) mod 9

= (di) (1 mod 9) (1 mod 9) hellip (1 mod 9) mod 9

= di mod 9

i terms

Fast Division Test

Rule of modular multiplication

Let the decimal representation of n be dkdk-1dk-2hellipd1d0

This means that N = dk10k + dk-110k-1 + hellip + d110 + d0

Note that di10i mod 9 = di mod 9

Hence N mod 9 = (dk10k + dk-110k-1 + hellip + d110 + d0) mod 9

= (dk10k mod 9 + dk-110k-1 mod 9 + hellip + d110 mod 9 + d0 mod 9) mod

9

= (dk mod 9 + dk-1 mod 9 + hellip + d1 mod 9 + d0 mod 9) mod 9

= (dk + dk-1 + hellip + d1 + d0) mod 9

Hint 10 1 (mod 9)

Fast Division Test

Claim A number written in decimal is divisible by 9 if and

only if

the sum of its digits is a multiple of 9

Rule of modular addition

By previous slide

The same procedure works to test whether N is divisible by n=3

Fast Division Test

What about n=11

Hint 10 -1 (mod 11)

Let the decimal representation of N be d92d91d90hellipd1d0

Then N is divisible by 11 if and only if

d92-d91+d90hellip-d1+d0 is divisible by 11

Why Try to work it out before your TA shows you

What about n=7

Hint 1000 -1 (mod 7)

This Lecture

bull Multiplicative inverse

The multiplicative inverse of a number a is another number arsquo such that

a middot arsquo 1 (mod n)

Multiplication Inverse

Does every number has a multiplicative inverse in modular arithmetic

For real numbers every nonzero number has a multiplicative inverse

For integers only 1 has a multiplicative inverse

An interesting property of modular arithmetic is that

there are multiplicative inverse for integers

For example 2 5 = 1 mod 3 so 5 is a multiplicative inverse

for 2 under modulo 3 (and vice versa)

Multiplication Inverse

Does every number has a multiplicative inverse in modular arithmetic

Multiplication Inverse

What is the pattern

Case Study

Why 2 does not have a multiplicative inverse under modulo 6

Suppose it has a multiplicative inverse y

2y 1 (mod 6)

=gt 2y = 1 + 6x for some integer x

=gt y = frac12 + 3x

This is a contradiction since both x and y are integers

Necessary Condition

Claim An integer k does not have an multiplicative inverse under modulo n

if k and n have a common factor gt= 2 (gcd(kn) gt= 2)

Proof

This claim says that for k to have a multiplicative inverse modulo n

then a necessary condition is that k and n do not have a common factor gt= 2

Suppose by contradiction that there is an inverse krsquo for k such that

krsquok = 1 (mod n)

Then krsquok = 1 + xn for some integer x

Since both k and n have a common factor say cgt=2

then k=ck1 and n=cn1 for some integers k1 and n1

So krsquock1 = 1 + xcn1

Then krsquok1 = 1c + xn1

This is a contradiction since the LHS is an integer but the RHS is not

Sufficient Condition

What about if gcd(kn)=1

Would k always have an multiplicative inverse under modulo n

For example gcd(37) = 1 3middot5 1 (mod 7)

gcd(89) = 1

gcd(411) = 1 4middot3 1 (mod 11)

8middot8 1 (mod 9)

It seems that there is always an inverse in such a case but why

gcd(89) = 1 8s + 9t = 1 for some integers s and t

8s = 1 ndash 9t

8s 1 (mod 9)gcd(89) = spc(89)

Theorem If gcd(kn)=1 then have krsquo such that

kmiddotkrsquo 1 (mod n)

Proof Since gcd(kn)=1 there exist s and t so that sk + tn = 1

So tn = 1 - sk

This means n | 1 ndash sk

This means that 1 ndash sk 0 (mod n)

This means that 1 sk (mod n)

So krsquo = s is an multiplicative inverse for k

Sufficient Condition

gcd(kn)=spc(kn)

The multiplicative inverse can be computed by the extended Euclidean algorithm

Corollary k has a multiplicative inverse mod n if and only if gcd(kn)=1

This Lecture

bull Fermatrsquos little theorem

Cancellation

There is no general cancellation in modular arithmetic

Note that (mod n) is very similar to =

If a b (mod n) then a+c b+c (mod n)

If a b (mod n) then ac bc (mod n)

However if ac bc (mod n)

it is not necessarily true that a b (mod n)

For example 4middot2 1middot2 (mod 6) but 4 1 (mod 6)

3middot4 1middot4 (mod 8) but 3 1 (mod 8)

4middot3 1middot3 (mod 9) but 4 1 (mod 9)

Observation In all the above examples c and n have a common factor

Cancellation

Claim Assume gcd(kn) = 1 If imiddotk jmiddotk (mod n) then i j (mod n)

For example multiplicative inverse always exists if n is a prime

Proof Since gcd(kn) = 1 there exists krsquo such that kkrsquo 1 (mod n)

imiddotk jmiddotk (mod n)

=gt imiddotkmiddotkrsquo jmiddotkmiddotkrsquo (mod n)

=gt i j (mod n)

Remarks (Optional) This makes arithmetic modulo prime a field

a structure that ldquobehaves likerdquo real numbers

Arithmetic modulo prime is very useful in coding theory

In particular when p is a prime amp k not a multiple of p then gcd(kp)=1

If i j (mod p) then imiddotk jmiddotk (mod p)

Therefore

k mod p 2k mod p hellip (p-1)k mod p

are all different numbers

Fermatrsquos Little Theorem

Claim 1 Assume gcd(kn) = 1 If imiddotk jmiddotk (mod n) then i j (mod n)

Claim 2 Assume gcd(kn) = 1 If i j (mod n) then imiddotk jmiddotk (mod n)

For example when p=7 and k=3

3 mod 7 = 3 2middot3 mod 7 = 6 3middot3 mod 7 = 2 4middot3 mod 7 = 5 5middot3 mod 7 = 1 6middot3 mod 7 = 4

Notice that in the above example every number from 1 to 6 appears exactly once

In particular when p is a prime amp k not a multiple of p then gcd(kp)=1

If i j (mod p) then imiddotk jmiddotk (mod p)

Therefore

k mod p 2k mod p hellip (p-1)k mod p

are all different numbers

Fermatrsquos Little Theorem

Each of ik mod p cannot be equal to 0 because p is a prime number

Let ci = ik mod p

So 1 lt= c1 lt= p-1 1 lt= c2 lt= p-1 hellip 1lt = cp-1 lt= p-1

By the above we know that c1c2hellipcp-2cp-1 are all different

So for each i from 1 to p-1 there is exactly one cj such that cj = i

Therefore we have

(k mod p)middot(2k mod p)middothellipmiddot((p-1)k mod p) = c1middotc2middothellipmiddotcp-2middotcp-1 = 1middot2middot3hellipmiddot(p-2)middot(p-1)

Fermatrsquos Little Theorem

1 kp-1 (mod p)

Theorem If p is prime amp k not a multiple of p

For example when p=5 k=4 we have kp-1 mod p = 44 mod 5 = 1

ldquoProofrdquo

4middot3middot2middot1 [(4 mod 5) (2middot4 mod 5) (3middot4 mod 5) (4middot4 mod 5)] (mod 5)

[4 middot (2middot4) middot (3middot4) middot (4middot4)] (mod 5)

[44 middot (1middot2middot3middot4)] (mod 5)

Since gcd(1middot2middot3middot4 5)=1 we can cancel 1middot2middot3middot4 on both sides

This implies

1 44 (mod 5)

By the previous slide or direct calculation

Fermatrsquos Little Theorem

1 kp-1 (mod p)

Theorem If p is prime amp k not a multiple of p

Proof

1middot2middotmiddotmiddot(p-1) (k mod p middot 2k mod pmiddothellipmiddot(p-1)k mod p) mod p

(kmiddot2k middotmiddotmiddot (p-1)k) mod p

(kp-1)middot1middot2 middotmiddotmiddot (p-1) (mod p)

So by cancelling 1middot2 middotmiddotmiddot (p-1) on both sides applying

Claim 1

(we can cancel them because gcd(1middot2 middotmiddotmiddot (p-1) p)=1)

we have

1 kp-1 (mod p)

By 2 slides before

By the multiplication rule

Wilsonrsquos Theorem

Theorem p is a prime if and only if

(p-1) -1 (mod p)

First we consider the easy direction

If p is not a prime assume p gt= 5 (for p=4 3 2 (mod 4) )

Then p=qr for some 2 lt= q lt p and 2 lt= r lt p

If q ne r then both q and r appear in (p-1)

and so (p-1) 0 (mod p)

If q = r then p = q2 gt 2q (since we assume p gt 5 and thus q gt 2)

then both q and 2q are in (p-1)

and so again (p-1) 0 (mod p)

Wilsonrsquos Theorem

Theorem p is a prime if and only if

(p-1) -1 (mod p)

To prove the more interesting direction first we need a lemma

Lemma If p is a prime number

x2 1 (mod p) if and only if x 1 (mod p) or x -1 (mod p)

Proof x2 1 (mod p)

iff p | x2 - 1

iff p | (x ndash 1)(x + 1)

iff p | (x ndash 1) or p | (x+1)

iff x 1 (mod p) or x -1 (mod p)

Lemma p prime and p|amiddotb iff p|a or p|b

Wilsonrsquos Theorem

Theorem p is a prime if and only if

(p-1) -1 (mod p)

Letrsquos get the proof idea by considering a concrete example

10

1middot2middot3middot4middot5middot6middot7middot8middot9middot10 mod 11

1middot10middot(2middot6)middot(3middot4)middot(5middot9)middot(7middot8) mod 11

1middot-1middot(1)middot(1)middot(1)middot(1) mod 11

-1 mod 11

Besides 1 and 10 the remaining numbers are paired up into multiplicative inverse

Wilsonrsquos Theorem

Theorem p is a prime if and only if

(p-1) -1 (mod p)

ProofSince p is a prime every number from 1 to p-1 has a multiplicative inverse

By the Lemma every number 2 lt= k lt= p-2 has an inverse krsquo with knekrsquo

Since p is odd the numbers from 2 to p-2 can be grouped into pairs

(a1b1)(a2b2)hellip(a(p-3)2b(p-3)2) so that aibi 1 (mod p)

Therefore (p-1) 1middot(p-1)middot2middot3middotmiddotmiddotmiddotmiddot(p-3)middot(p-2) (mod p)

1middot(p-1)middot(a1b1)middot(a2b2)middotmiddotmiddotmiddotmiddot(a(p-3)2b(p-3)2) (mod p)

1middot(-1)middot(1)middot(1)middotmiddotmiddotmiddotmiddot(1) (mod p)

-1 (mod p)

This Lecture

bull Eulerrsquos phi function

Inclusion-Exclusion (n sets)

What is the inclusion-exclusion formula for the union of n sets

sum of sizes of all single setsndash sum of sizes of all 2-set intersections+ sum of sizes of all 3-set intersectionsndash sum of sizes of all 4-set intersectionshellip+ (ndash1)n+1 times sum of sizes of intersections of all n

sets

1 2 nA A A

1

12 1

( 1)n

ki

S nk i S

S k

A

Inclusion-Exclusion (n sets)

Inclusion-Exclusion (n sets)

sum of sizes of all single setsndash sum of sizes of all 2-set intersections+ sum of sizes of all 3-set intersectionsndash sum of sizes of all 4-set intersectionshellip+ (ndash1)n+1 times sum of sizes of intersections of all n

sets

|A1 [ A2 [ A3 [ hellip [ An|

We want to show that every element is counted exactly once

Consider an element which belongs to exactly k sets say A1 A2 A3 hellip Ak

In the formula such an element is counted the following number of times

Therefore each element is counted exactly once and thus the formula is correct

Euler Function

Given a number n how many numbers from 1 to n are relatively prime to n

When n is a prime number

then every number from 1 to n-1 is relatively prime to n

and so

When n is a prime power

then p 2p 3p 4p hellip n are not relatively prime to n

there are np = pc-1 of them

and other numbers are relatively prime to n

Therefore

Euler Function

Given a number n how many numbers from 1 to n are relatively prime to n

Suppose

Then p 2p 3p 4p hellip n are not relatively prime to n there are np of them

Also q 2q 3q 4q hellip n are not relatively prime to n and there are nq of them

Other numbers are relatively prime to n

Therefore

The numbers pq 2pq 3pq hellip n are subtracted twice and there are npq of them

So the correct answer is

Euler Function

Given a number n how many numbers from 1 to n are relatively prime to n

Let

Let S be the set of numbers from 1 to n that are not relatively prime to n

Let Ai be the set of numbers that are a multiple of piS = A1 [ A2 [ hellip [ An

For the intersection of k sets say A1 A2 A3hellip Ak

then every number in A1 Aring A2 Aring hellip Aring Ak is a multiple of p1p2hellippk

then |A1 Aring A2 Aring hellip Aring Ak| = np1p2hellippk

Euler Function

Given a number n how many numbers from 1 to n are relatively prime to n

Let

Let S be the set of numbers from 1 to n that are not relatively prime to n

Let Ai be the set of numbers that are a multiple of piS = A1 [ A2 [ hellip [ An

|A1 Aring A2 Aring hellip Aring Ak| = np1p2hellippk

When r=3 (only 3 distinct factors)

|A1 [ A2 [ A3|

= np1 + np2 + np3

- np1p2 ndash np1p3 ndash np2p3 + np1p2p3

|A1 [ A2 [ A3| = |A1| + |A2| + |A3|

ndash |A1 Aring A2| ndash |A1 Aring A3| ndash |A2 Aring A3|

+ |A1 Aring A2 Aring A3|

= n(1-p1)(1-p2)(1-p3)

Euler Function

Given a number n how many numbers from 1 to n are relatively prime to n

Let

Let S be the set of numbers from 1 to n that are not relatively prime to n

Let Ai be the set of numbers that are a multiple of piS = A1 [ A2 [ hellip [ An

|A1 Aring A2 Aring hellip Aring Ak| = np1p2hellippk

sum of sizes of all single setsndash sum of sizes of all 2-set intersections+ sum of sizes of all 3-set intersectionsndash sum of sizes of all 4-set intersectionshellip+ (ndash1)n+1 times sum of sizes of intersections of

n sets

|A1 [ A2 [ A3 [ hellip [ An|

|S| = |A1 [ A2 [ hellip [ An|

= n(1-p1)(1-p2)hellip(1-pn)

calculationshellip

  • Modular Arithmetic
  • PowerPoint Presentation
  • Slide 3
  • Slide 4
  • Slide 5
  • Slide 6
  • Slide 7
  • Slide 8
  • Slide 9
  • Slide 10
  • Slide 11
  • Slide 12
  • Slide 13
  • Slide 14
  • Slide 15
  • Slide 16
  • Slide 17
  • Slide 18
  • Slide 19
  • Slide 20
  • Slide 21
  • Slide 22
  • Slide 23
  • Slide 24
  • Slide 25
  • Slide 26
  • Slide 27
  • Slide 28
  • Slide 29
  • Slide 30
  • Slide 31
  • Slide 32
  • Slide 33
  • Slide 34
  • Slide 35
  • Slide 36
  • Slide 37
  • Slide 38
  • Slide 39
  • Slide 40
  • Slide 41
  • Slide 42
  • Slide 43
  • Slide 44
  • Slide 45
Page 16: Modular Arithmetic

Let the decimal representation of n be dkdk-1dk-2hellipd1d0

This means that N = dk10k + dk-110k-1 + hellip + d110 + d0

Note that di10i mod 9 = di mod 9

Hence N mod 9 = (dk10k + dk-110k-1 + hellip + d110 + d0) mod 9

= (dk10k mod 9 + dk-110k-1 mod 9 + hellip + d110 mod 9 + d0 mod 9) mod

9

= (dk mod 9 + dk-1 mod 9 + hellip + d1 mod 9 + d0 mod 9) mod 9

= (dk + dk-1 + hellip + d1 + d0) mod 9

Hint 10 1 (mod 9)

Fast Division Test

Claim A number written in decimal is divisible by 9 if and

only if

the sum of its digits is a multiple of 9

Rule of modular addition

By previous slide

The same procedure works to test whether N is divisible by n=3

Fast Division Test

What about n=11

Hint 10 -1 (mod 11)

Let the decimal representation of N be d92d91d90hellipd1d0

Then N is divisible by 11 if and only if

d92-d91+d90hellip-d1+d0 is divisible by 11

Why Try to work it out before your TA shows you

What about n=7

Hint 1000 -1 (mod 7)

This Lecture

bull Multiplicative inverse

The multiplicative inverse of a number a is another number arsquo such that

a middot arsquo 1 (mod n)

Multiplication Inverse

Does every number has a multiplicative inverse in modular arithmetic

For real numbers every nonzero number has a multiplicative inverse

For integers only 1 has a multiplicative inverse

An interesting property of modular arithmetic is that

there are multiplicative inverse for integers

For example 2 5 = 1 mod 3 so 5 is a multiplicative inverse

for 2 under modulo 3 (and vice versa)

Multiplication Inverse

Does every number has a multiplicative inverse in modular arithmetic

Multiplication Inverse

What is the pattern

Case Study

Why 2 does not have a multiplicative inverse under modulo 6

Suppose it has a multiplicative inverse y

2y 1 (mod 6)

=gt 2y = 1 + 6x for some integer x

=gt y = frac12 + 3x

This is a contradiction since both x and y are integers

Necessary Condition

Claim An integer k does not have an multiplicative inverse under modulo n

if k and n have a common factor gt= 2 (gcd(kn) gt= 2)

Proof

This claim says that for k to have a multiplicative inverse modulo n

then a necessary condition is that k and n do not have a common factor gt= 2

Suppose by contradiction that there is an inverse krsquo for k such that

krsquok = 1 (mod n)

Then krsquok = 1 + xn for some integer x

Since both k and n have a common factor say cgt=2

then k=ck1 and n=cn1 for some integers k1 and n1

So krsquock1 = 1 + xcn1

Then krsquok1 = 1c + xn1

This is a contradiction since the LHS is an integer but the RHS is not

Sufficient Condition

What about if gcd(kn)=1

Would k always have an multiplicative inverse under modulo n

For example gcd(37) = 1 3middot5 1 (mod 7)

gcd(89) = 1

gcd(411) = 1 4middot3 1 (mod 11)

8middot8 1 (mod 9)

It seems that there is always an inverse in such a case but why

gcd(89) = 1 8s + 9t = 1 for some integers s and t

8s = 1 ndash 9t

8s 1 (mod 9)gcd(89) = spc(89)

Theorem If gcd(kn)=1 then have krsquo such that

kmiddotkrsquo 1 (mod n)

Proof Since gcd(kn)=1 there exist s and t so that sk + tn = 1

So tn = 1 - sk

This means n | 1 ndash sk

This means that 1 ndash sk 0 (mod n)

This means that 1 sk (mod n)

So krsquo = s is an multiplicative inverse for k

Sufficient Condition

gcd(kn)=spc(kn)

The multiplicative inverse can be computed by the extended Euclidean algorithm

Corollary k has a multiplicative inverse mod n if and only if gcd(kn)=1

This Lecture

bull Fermatrsquos little theorem

Cancellation

There is no general cancellation in modular arithmetic

Note that (mod n) is very similar to =

If a b (mod n) then a+c b+c (mod n)

If a b (mod n) then ac bc (mod n)

However if ac bc (mod n)

it is not necessarily true that a b (mod n)

For example 4middot2 1middot2 (mod 6) but 4 1 (mod 6)

3middot4 1middot4 (mod 8) but 3 1 (mod 8)

4middot3 1middot3 (mod 9) but 4 1 (mod 9)

Observation In all the above examples c and n have a common factor

Cancellation

Claim Assume gcd(kn) = 1 If imiddotk jmiddotk (mod n) then i j (mod n)

For example multiplicative inverse always exists if n is a prime

Proof Since gcd(kn) = 1 there exists krsquo such that kkrsquo 1 (mod n)

imiddotk jmiddotk (mod n)

=gt imiddotkmiddotkrsquo jmiddotkmiddotkrsquo (mod n)

=gt i j (mod n)

Remarks (Optional) This makes arithmetic modulo prime a field

a structure that ldquobehaves likerdquo real numbers

Arithmetic modulo prime is very useful in coding theory

In particular when p is a prime amp k not a multiple of p then gcd(kp)=1

If i j (mod p) then imiddotk jmiddotk (mod p)

Therefore

k mod p 2k mod p hellip (p-1)k mod p

are all different numbers

Fermatrsquos Little Theorem

Claim 1 Assume gcd(kn) = 1 If imiddotk jmiddotk (mod n) then i j (mod n)

Claim 2 Assume gcd(kn) = 1 If i j (mod n) then imiddotk jmiddotk (mod n)

For example when p=7 and k=3

3 mod 7 = 3 2middot3 mod 7 = 6 3middot3 mod 7 = 2 4middot3 mod 7 = 5 5middot3 mod 7 = 1 6middot3 mod 7 = 4

Notice that in the above example every number from 1 to 6 appears exactly once

In particular when p is a prime amp k not a multiple of p then gcd(kp)=1

If i j (mod p) then imiddotk jmiddotk (mod p)

Therefore

k mod p 2k mod p hellip (p-1)k mod p

are all different numbers

Fermatrsquos Little Theorem

Each of ik mod p cannot be equal to 0 because p is a prime number

Let ci = ik mod p

So 1 lt= c1 lt= p-1 1 lt= c2 lt= p-1 hellip 1lt = cp-1 lt= p-1

By the above we know that c1c2hellipcp-2cp-1 are all different

So for each i from 1 to p-1 there is exactly one cj such that cj = i

Therefore we have

(k mod p)middot(2k mod p)middothellipmiddot((p-1)k mod p) = c1middotc2middothellipmiddotcp-2middotcp-1 = 1middot2middot3hellipmiddot(p-2)middot(p-1)

Fermatrsquos Little Theorem

1 kp-1 (mod p)

Theorem If p is prime amp k not a multiple of p

For example when p=5 k=4 we have kp-1 mod p = 44 mod 5 = 1

ldquoProofrdquo

4middot3middot2middot1 [(4 mod 5) (2middot4 mod 5) (3middot4 mod 5) (4middot4 mod 5)] (mod 5)

[4 middot (2middot4) middot (3middot4) middot (4middot4)] (mod 5)

[44 middot (1middot2middot3middot4)] (mod 5)

Since gcd(1middot2middot3middot4 5)=1 we can cancel 1middot2middot3middot4 on both sides

This implies

1 44 (mod 5)

By the previous slide or direct calculation

Fermatrsquos Little Theorem

1 kp-1 (mod p)

Theorem If p is prime amp k not a multiple of p

Proof

1middot2middotmiddotmiddot(p-1) (k mod p middot 2k mod pmiddothellipmiddot(p-1)k mod p) mod p

(kmiddot2k middotmiddotmiddot (p-1)k) mod p

(kp-1)middot1middot2 middotmiddotmiddot (p-1) (mod p)

So by cancelling 1middot2 middotmiddotmiddot (p-1) on both sides applying

Claim 1

(we can cancel them because gcd(1middot2 middotmiddotmiddot (p-1) p)=1)

we have

1 kp-1 (mod p)

By 2 slides before

By the multiplication rule

Wilsonrsquos Theorem

Theorem p is a prime if and only if

(p-1) -1 (mod p)

First we consider the easy direction

If p is not a prime assume p gt= 5 (for p=4 3 2 (mod 4) )

Then p=qr for some 2 lt= q lt p and 2 lt= r lt p

If q ne r then both q and r appear in (p-1)

and so (p-1) 0 (mod p)

If q = r then p = q2 gt 2q (since we assume p gt 5 and thus q gt 2)

then both q and 2q are in (p-1)

and so again (p-1) 0 (mod p)

Wilsonrsquos Theorem

Theorem p is a prime if and only if

(p-1) -1 (mod p)

To prove the more interesting direction first we need a lemma

Lemma If p is a prime number

x2 1 (mod p) if and only if x 1 (mod p) or x -1 (mod p)

Proof x2 1 (mod p)

iff p | x2 - 1

iff p | (x ndash 1)(x + 1)

iff p | (x ndash 1) or p | (x+1)

iff x 1 (mod p) or x -1 (mod p)

Lemma p prime and p|amiddotb iff p|a or p|b

Wilsonrsquos Theorem

Theorem p is a prime if and only if

(p-1) -1 (mod p)

Letrsquos get the proof idea by considering a concrete example

10

1middot2middot3middot4middot5middot6middot7middot8middot9middot10 mod 11

1middot10middot(2middot6)middot(3middot4)middot(5middot9)middot(7middot8) mod 11

1middot-1middot(1)middot(1)middot(1)middot(1) mod 11

-1 mod 11

Besides 1 and 10 the remaining numbers are paired up into multiplicative inverse

Wilsonrsquos Theorem

Theorem p is a prime if and only if

(p-1) -1 (mod p)

ProofSince p is a prime every number from 1 to p-1 has a multiplicative inverse

By the Lemma every number 2 lt= k lt= p-2 has an inverse krsquo with knekrsquo

Since p is odd the numbers from 2 to p-2 can be grouped into pairs

(a1b1)(a2b2)hellip(a(p-3)2b(p-3)2) so that aibi 1 (mod p)

Therefore (p-1) 1middot(p-1)middot2middot3middotmiddotmiddotmiddotmiddot(p-3)middot(p-2) (mod p)

1middot(p-1)middot(a1b1)middot(a2b2)middotmiddotmiddotmiddotmiddot(a(p-3)2b(p-3)2) (mod p)

1middot(-1)middot(1)middot(1)middotmiddotmiddotmiddotmiddot(1) (mod p)

-1 (mod p)

This Lecture

bull Eulerrsquos phi function

Inclusion-Exclusion (n sets)

What is the inclusion-exclusion formula for the union of n sets

sum of sizes of all single setsndash sum of sizes of all 2-set intersections+ sum of sizes of all 3-set intersectionsndash sum of sizes of all 4-set intersectionshellip+ (ndash1)n+1 times sum of sizes of intersections of all n

sets

1 2 nA A A

1

12 1

( 1)n

ki

S nk i S

S k

A

Inclusion-Exclusion (n sets)

Inclusion-Exclusion (n sets)

sum of sizes of all single setsndash sum of sizes of all 2-set intersections+ sum of sizes of all 3-set intersectionsndash sum of sizes of all 4-set intersectionshellip+ (ndash1)n+1 times sum of sizes of intersections of all n

sets

|A1 [ A2 [ A3 [ hellip [ An|

We want to show that every element is counted exactly once

Consider an element which belongs to exactly k sets say A1 A2 A3 hellip Ak

In the formula such an element is counted the following number of times

Therefore each element is counted exactly once and thus the formula is correct

Euler Function

Given a number n how many numbers from 1 to n are relatively prime to n

When n is a prime number

then every number from 1 to n-1 is relatively prime to n

and so

When n is a prime power

then p 2p 3p 4p hellip n are not relatively prime to n

there are np = pc-1 of them

and other numbers are relatively prime to n

Therefore

Euler Function

Given a number n how many numbers from 1 to n are relatively prime to n

Suppose

Then p 2p 3p 4p hellip n are not relatively prime to n there are np of them

Also q 2q 3q 4q hellip n are not relatively prime to n and there are nq of them

Other numbers are relatively prime to n

Therefore

The numbers pq 2pq 3pq hellip n are subtracted twice and there are npq of them

So the correct answer is

Euler Function

Given a number n how many numbers from 1 to n are relatively prime to n

Let

Let S be the set of numbers from 1 to n that are not relatively prime to n

Let Ai be the set of numbers that are a multiple of piS = A1 [ A2 [ hellip [ An

For the intersection of k sets say A1 A2 A3hellip Ak

then every number in A1 Aring A2 Aring hellip Aring Ak is a multiple of p1p2hellippk

then |A1 Aring A2 Aring hellip Aring Ak| = np1p2hellippk

Euler Function

Given a number n how many numbers from 1 to n are relatively prime to n

Let

Let S be the set of numbers from 1 to n that are not relatively prime to n

Let Ai be the set of numbers that are a multiple of piS = A1 [ A2 [ hellip [ An

|A1 Aring A2 Aring hellip Aring Ak| = np1p2hellippk

When r=3 (only 3 distinct factors)

|A1 [ A2 [ A3|

= np1 + np2 + np3

- np1p2 ndash np1p3 ndash np2p3 + np1p2p3

|A1 [ A2 [ A3| = |A1| + |A2| + |A3|

ndash |A1 Aring A2| ndash |A1 Aring A3| ndash |A2 Aring A3|

+ |A1 Aring A2 Aring A3|

= n(1-p1)(1-p2)(1-p3)

Euler Function

Given a number n how many numbers from 1 to n are relatively prime to n

Let

Let S be the set of numbers from 1 to n that are not relatively prime to n

Let Ai be the set of numbers that are a multiple of piS = A1 [ A2 [ hellip [ An

|A1 Aring A2 Aring hellip Aring Ak| = np1p2hellippk

sum of sizes of all single setsndash sum of sizes of all 2-set intersections+ sum of sizes of all 3-set intersectionsndash sum of sizes of all 4-set intersectionshellip+ (ndash1)n+1 times sum of sizes of intersections of

n sets

|A1 [ A2 [ A3 [ hellip [ An|

|S| = |A1 [ A2 [ hellip [ An|

= n(1-p1)(1-p2)hellip(1-pn)

calculationshellip

  • Modular Arithmetic
  • PowerPoint Presentation
  • Slide 3
  • Slide 4
  • Slide 5
  • Slide 6
  • Slide 7
  • Slide 8
  • Slide 9
  • Slide 10
  • Slide 11
  • Slide 12
  • Slide 13
  • Slide 14
  • Slide 15
  • Slide 16
  • Slide 17
  • Slide 18
  • Slide 19
  • Slide 20
  • Slide 21
  • Slide 22
  • Slide 23
  • Slide 24
  • Slide 25
  • Slide 26
  • Slide 27
  • Slide 28
  • Slide 29
  • Slide 30
  • Slide 31
  • Slide 32
  • Slide 33
  • Slide 34
  • Slide 35
  • Slide 36
  • Slide 37
  • Slide 38
  • Slide 39
  • Slide 40
  • Slide 41
  • Slide 42
  • Slide 43
  • Slide 44
  • Slide 45
Page 17: Modular Arithmetic

The same procedure works to test whether N is divisible by n=3

Fast Division Test

What about n=11

Hint 10 -1 (mod 11)

Let the decimal representation of N be d92d91d90hellipd1d0

Then N is divisible by 11 if and only if

d92-d91+d90hellip-d1+d0 is divisible by 11

Why Try to work it out before your TA shows you

What about n=7

Hint 1000 -1 (mod 7)

This Lecture

bull Multiplicative inverse

The multiplicative inverse of a number a is another number arsquo such that

a middot arsquo 1 (mod n)

Multiplication Inverse

Does every number has a multiplicative inverse in modular arithmetic

For real numbers every nonzero number has a multiplicative inverse

For integers only 1 has a multiplicative inverse

An interesting property of modular arithmetic is that

there are multiplicative inverse for integers

For example 2 5 = 1 mod 3 so 5 is a multiplicative inverse

for 2 under modulo 3 (and vice versa)

Multiplication Inverse

Does every number has a multiplicative inverse in modular arithmetic

Multiplication Inverse

What is the pattern

Case Study

Why 2 does not have a multiplicative inverse under modulo 6

Suppose it has a multiplicative inverse y

2y 1 (mod 6)

=gt 2y = 1 + 6x for some integer x

=gt y = frac12 + 3x

This is a contradiction since both x and y are integers

Necessary Condition

Claim An integer k does not have an multiplicative inverse under modulo n

if k and n have a common factor gt= 2 (gcd(kn) gt= 2)

Proof

This claim says that for k to have a multiplicative inverse modulo n

then a necessary condition is that k and n do not have a common factor gt= 2

Suppose by contradiction that there is an inverse krsquo for k such that

krsquok = 1 (mod n)

Then krsquok = 1 + xn for some integer x

Since both k and n have a common factor say cgt=2

then k=ck1 and n=cn1 for some integers k1 and n1

So krsquock1 = 1 + xcn1

Then krsquok1 = 1c + xn1

This is a contradiction since the LHS is an integer but the RHS is not

Sufficient Condition

What about if gcd(kn)=1

Would k always have an multiplicative inverse under modulo n

For example gcd(37) = 1 3middot5 1 (mod 7)

gcd(89) = 1

gcd(411) = 1 4middot3 1 (mod 11)

8middot8 1 (mod 9)

It seems that there is always an inverse in such a case but why

gcd(89) = 1 8s + 9t = 1 for some integers s and t

8s = 1 ndash 9t

8s 1 (mod 9)gcd(89) = spc(89)

Theorem If gcd(kn)=1 then have krsquo such that

kmiddotkrsquo 1 (mod n)

Proof Since gcd(kn)=1 there exist s and t so that sk + tn = 1

So tn = 1 - sk

This means n | 1 ndash sk

This means that 1 ndash sk 0 (mod n)

This means that 1 sk (mod n)

So krsquo = s is an multiplicative inverse for k

Sufficient Condition

gcd(kn)=spc(kn)

The multiplicative inverse can be computed by the extended Euclidean algorithm

Corollary k has a multiplicative inverse mod n if and only if gcd(kn)=1

This Lecture

bull Fermatrsquos little theorem

Cancellation

There is no general cancellation in modular arithmetic

Note that (mod n) is very similar to =

If a b (mod n) then a+c b+c (mod n)

If a b (mod n) then ac bc (mod n)

However if ac bc (mod n)

it is not necessarily true that a b (mod n)

For example 4middot2 1middot2 (mod 6) but 4 1 (mod 6)

3middot4 1middot4 (mod 8) but 3 1 (mod 8)

4middot3 1middot3 (mod 9) but 4 1 (mod 9)

Observation In all the above examples c and n have a common factor

Cancellation

Claim Assume gcd(kn) = 1 If imiddotk jmiddotk (mod n) then i j (mod n)

For example multiplicative inverse always exists if n is a prime

Proof Since gcd(kn) = 1 there exists krsquo such that kkrsquo 1 (mod n)

imiddotk jmiddotk (mod n)

=gt imiddotkmiddotkrsquo jmiddotkmiddotkrsquo (mod n)

=gt i j (mod n)

Remarks (Optional) This makes arithmetic modulo prime a field

a structure that ldquobehaves likerdquo real numbers

Arithmetic modulo prime is very useful in coding theory

In particular when p is a prime amp k not a multiple of p then gcd(kp)=1

If i j (mod p) then imiddotk jmiddotk (mod p)

Therefore

k mod p 2k mod p hellip (p-1)k mod p

are all different numbers

Fermatrsquos Little Theorem

Claim 1 Assume gcd(kn) = 1 If imiddotk jmiddotk (mod n) then i j (mod n)

Claim 2 Assume gcd(kn) = 1 If i j (mod n) then imiddotk jmiddotk (mod n)

For example when p=7 and k=3

3 mod 7 = 3 2middot3 mod 7 = 6 3middot3 mod 7 = 2 4middot3 mod 7 = 5 5middot3 mod 7 = 1 6middot3 mod 7 = 4

Notice that in the above example every number from 1 to 6 appears exactly once

In particular when p is a prime amp k not a multiple of p then gcd(kp)=1

If i j (mod p) then imiddotk jmiddotk (mod p)

Therefore

k mod p 2k mod p hellip (p-1)k mod p

are all different numbers

Fermatrsquos Little Theorem

Each of ik mod p cannot be equal to 0 because p is a prime number

Let ci = ik mod p

So 1 lt= c1 lt= p-1 1 lt= c2 lt= p-1 hellip 1lt = cp-1 lt= p-1

By the above we know that c1c2hellipcp-2cp-1 are all different

So for each i from 1 to p-1 there is exactly one cj such that cj = i

Therefore we have

(k mod p)middot(2k mod p)middothellipmiddot((p-1)k mod p) = c1middotc2middothellipmiddotcp-2middotcp-1 = 1middot2middot3hellipmiddot(p-2)middot(p-1)

Fermatrsquos Little Theorem

1 kp-1 (mod p)

Theorem If p is prime amp k not a multiple of p

For example when p=5 k=4 we have kp-1 mod p = 44 mod 5 = 1

ldquoProofrdquo

4middot3middot2middot1 [(4 mod 5) (2middot4 mod 5) (3middot4 mod 5) (4middot4 mod 5)] (mod 5)

[4 middot (2middot4) middot (3middot4) middot (4middot4)] (mod 5)

[44 middot (1middot2middot3middot4)] (mod 5)

Since gcd(1middot2middot3middot4 5)=1 we can cancel 1middot2middot3middot4 on both sides

This implies

1 44 (mod 5)

By the previous slide or direct calculation

Fermatrsquos Little Theorem

1 kp-1 (mod p)

Theorem If p is prime amp k not a multiple of p

Proof

1middot2middotmiddotmiddot(p-1) (k mod p middot 2k mod pmiddothellipmiddot(p-1)k mod p) mod p

(kmiddot2k middotmiddotmiddot (p-1)k) mod p

(kp-1)middot1middot2 middotmiddotmiddot (p-1) (mod p)

So by cancelling 1middot2 middotmiddotmiddot (p-1) on both sides applying

Claim 1

(we can cancel them because gcd(1middot2 middotmiddotmiddot (p-1) p)=1)

we have

1 kp-1 (mod p)

By 2 slides before

By the multiplication rule

Wilsonrsquos Theorem

Theorem p is a prime if and only if

(p-1) -1 (mod p)

First we consider the easy direction

If p is not a prime assume p gt= 5 (for p=4 3 2 (mod 4) )

Then p=qr for some 2 lt= q lt p and 2 lt= r lt p

If q ne r then both q and r appear in (p-1)

and so (p-1) 0 (mod p)

If q = r then p = q2 gt 2q (since we assume p gt 5 and thus q gt 2)

then both q and 2q are in (p-1)

and so again (p-1) 0 (mod p)

Wilsonrsquos Theorem

Theorem p is a prime if and only if

(p-1) -1 (mod p)

To prove the more interesting direction first we need a lemma

Lemma If p is a prime number

x2 1 (mod p) if and only if x 1 (mod p) or x -1 (mod p)

Proof x2 1 (mod p)

iff p | x2 - 1

iff p | (x ndash 1)(x + 1)

iff p | (x ndash 1) or p | (x+1)

iff x 1 (mod p) or x -1 (mod p)

Lemma p prime and p|amiddotb iff p|a or p|b

Wilsonrsquos Theorem

Theorem p is a prime if and only if

(p-1) -1 (mod p)

Letrsquos get the proof idea by considering a concrete example

10

1middot2middot3middot4middot5middot6middot7middot8middot9middot10 mod 11

1middot10middot(2middot6)middot(3middot4)middot(5middot9)middot(7middot8) mod 11

1middot-1middot(1)middot(1)middot(1)middot(1) mod 11

-1 mod 11

Besides 1 and 10 the remaining numbers are paired up into multiplicative inverse

Wilsonrsquos Theorem

Theorem p is a prime if and only if

(p-1) -1 (mod p)

ProofSince p is a prime every number from 1 to p-1 has a multiplicative inverse

By the Lemma every number 2 lt= k lt= p-2 has an inverse krsquo with knekrsquo

Since p is odd the numbers from 2 to p-2 can be grouped into pairs

(a1b1)(a2b2)hellip(a(p-3)2b(p-3)2) so that aibi 1 (mod p)

Therefore (p-1) 1middot(p-1)middot2middot3middotmiddotmiddotmiddotmiddot(p-3)middot(p-2) (mod p)

1middot(p-1)middot(a1b1)middot(a2b2)middotmiddotmiddotmiddotmiddot(a(p-3)2b(p-3)2) (mod p)

1middot(-1)middot(1)middot(1)middotmiddotmiddotmiddotmiddot(1) (mod p)

-1 (mod p)

This Lecture

bull Eulerrsquos phi function

Inclusion-Exclusion (n sets)

What is the inclusion-exclusion formula for the union of n sets

sum of sizes of all single setsndash sum of sizes of all 2-set intersections+ sum of sizes of all 3-set intersectionsndash sum of sizes of all 4-set intersectionshellip+ (ndash1)n+1 times sum of sizes of intersections of all n

sets

1 2 nA A A

1

12 1

( 1)n

ki

S nk i S

S k

A

Inclusion-Exclusion (n sets)

Inclusion-Exclusion (n sets)

sum of sizes of all single setsndash sum of sizes of all 2-set intersections+ sum of sizes of all 3-set intersectionsndash sum of sizes of all 4-set intersectionshellip+ (ndash1)n+1 times sum of sizes of intersections of all n

sets

|A1 [ A2 [ A3 [ hellip [ An|

We want to show that every element is counted exactly once

Consider an element which belongs to exactly k sets say A1 A2 A3 hellip Ak

In the formula such an element is counted the following number of times

Therefore each element is counted exactly once and thus the formula is correct

Euler Function

Given a number n how many numbers from 1 to n are relatively prime to n

When n is a prime number

then every number from 1 to n-1 is relatively prime to n

and so

When n is a prime power

then p 2p 3p 4p hellip n are not relatively prime to n

there are np = pc-1 of them

and other numbers are relatively prime to n

Therefore

Euler Function

Given a number n how many numbers from 1 to n are relatively prime to n

Suppose

Then p 2p 3p 4p hellip n are not relatively prime to n there are np of them

Also q 2q 3q 4q hellip n are not relatively prime to n and there are nq of them

Other numbers are relatively prime to n

Therefore

The numbers pq 2pq 3pq hellip n are subtracted twice and there are npq of them

So the correct answer is

Euler Function

Given a number n how many numbers from 1 to n are relatively prime to n

Let

Let S be the set of numbers from 1 to n that are not relatively prime to n

Let Ai be the set of numbers that are a multiple of piS = A1 [ A2 [ hellip [ An

For the intersection of k sets say A1 A2 A3hellip Ak

then every number in A1 Aring A2 Aring hellip Aring Ak is a multiple of p1p2hellippk

then |A1 Aring A2 Aring hellip Aring Ak| = np1p2hellippk

Euler Function

Given a number n how many numbers from 1 to n are relatively prime to n

Let

Let S be the set of numbers from 1 to n that are not relatively prime to n

Let Ai be the set of numbers that are a multiple of piS = A1 [ A2 [ hellip [ An

|A1 Aring A2 Aring hellip Aring Ak| = np1p2hellippk

When r=3 (only 3 distinct factors)

|A1 [ A2 [ A3|

= np1 + np2 + np3

- np1p2 ndash np1p3 ndash np2p3 + np1p2p3

|A1 [ A2 [ A3| = |A1| + |A2| + |A3|

ndash |A1 Aring A2| ndash |A1 Aring A3| ndash |A2 Aring A3|

+ |A1 Aring A2 Aring A3|

= n(1-p1)(1-p2)(1-p3)

Euler Function

Given a number n how many numbers from 1 to n are relatively prime to n

Let

Let S be the set of numbers from 1 to n that are not relatively prime to n

Let Ai be the set of numbers that are a multiple of piS = A1 [ A2 [ hellip [ An

|A1 Aring A2 Aring hellip Aring Ak| = np1p2hellippk

sum of sizes of all single setsndash sum of sizes of all 2-set intersections+ sum of sizes of all 3-set intersectionsndash sum of sizes of all 4-set intersectionshellip+ (ndash1)n+1 times sum of sizes of intersections of

n sets

|A1 [ A2 [ A3 [ hellip [ An|

|S| = |A1 [ A2 [ hellip [ An|

= n(1-p1)(1-p2)hellip(1-pn)

calculationshellip

  • Modular Arithmetic
  • PowerPoint Presentation
  • Slide 3
  • Slide 4
  • Slide 5
  • Slide 6
  • Slide 7
  • Slide 8
  • Slide 9
  • Slide 10
  • Slide 11
  • Slide 12
  • Slide 13
  • Slide 14
  • Slide 15
  • Slide 16
  • Slide 17
  • Slide 18
  • Slide 19
  • Slide 20
  • Slide 21
  • Slide 22
  • Slide 23
  • Slide 24
  • Slide 25
  • Slide 26
  • Slide 27
  • Slide 28
  • Slide 29
  • Slide 30
  • Slide 31
  • Slide 32
  • Slide 33
  • Slide 34
  • Slide 35
  • Slide 36
  • Slide 37
  • Slide 38
  • Slide 39
  • Slide 40
  • Slide 41
  • Slide 42
  • Slide 43
  • Slide 44
  • Slide 45
Page 18: Modular Arithmetic

This Lecture

bull Multiplicative inverse

The multiplicative inverse of a number a is another number arsquo such that

a middot arsquo 1 (mod n)

Multiplication Inverse

Does every number has a multiplicative inverse in modular arithmetic

For real numbers every nonzero number has a multiplicative inverse

For integers only 1 has a multiplicative inverse

An interesting property of modular arithmetic is that

there are multiplicative inverse for integers

For example 2 5 = 1 mod 3 so 5 is a multiplicative inverse

for 2 under modulo 3 (and vice versa)

Multiplication Inverse

Does every number has a multiplicative inverse in modular arithmetic

Multiplication Inverse

What is the pattern

Case Study

Why 2 does not have a multiplicative inverse under modulo 6

Suppose it has a multiplicative inverse y

2y 1 (mod 6)

=gt 2y = 1 + 6x for some integer x

=gt y = frac12 + 3x

This is a contradiction since both x and y are integers

Necessary Condition

Claim An integer k does not have an multiplicative inverse under modulo n

if k and n have a common factor gt= 2 (gcd(kn) gt= 2)

Proof

This claim says that for k to have a multiplicative inverse modulo n

then a necessary condition is that k and n do not have a common factor gt= 2

Suppose by contradiction that there is an inverse krsquo for k such that

krsquok = 1 (mod n)

Then krsquok = 1 + xn for some integer x

Since both k and n have a common factor say cgt=2

then k=ck1 and n=cn1 for some integers k1 and n1

So krsquock1 = 1 + xcn1

Then krsquok1 = 1c + xn1

This is a contradiction since the LHS is an integer but the RHS is not

Sufficient Condition

What about if gcd(kn)=1

Would k always have an multiplicative inverse under modulo n

For example gcd(37) = 1 3middot5 1 (mod 7)

gcd(89) = 1

gcd(411) = 1 4middot3 1 (mod 11)

8middot8 1 (mod 9)

It seems that there is always an inverse in such a case but why

gcd(89) = 1 8s + 9t = 1 for some integers s and t

8s = 1 ndash 9t

8s 1 (mod 9)gcd(89) = spc(89)

Theorem If gcd(kn)=1 then have krsquo such that

kmiddotkrsquo 1 (mod n)

Proof Since gcd(kn)=1 there exist s and t so that sk + tn = 1

So tn = 1 - sk

This means n | 1 ndash sk

This means that 1 ndash sk 0 (mod n)

This means that 1 sk (mod n)

So krsquo = s is an multiplicative inverse for k

Sufficient Condition

gcd(kn)=spc(kn)

The multiplicative inverse can be computed by the extended Euclidean algorithm

Corollary k has a multiplicative inverse mod n if and only if gcd(kn)=1

This Lecture

bull Fermatrsquos little theorem

Cancellation

There is no general cancellation in modular arithmetic

Note that (mod n) is very similar to =

If a b (mod n) then a+c b+c (mod n)

If a b (mod n) then ac bc (mod n)

However if ac bc (mod n)

it is not necessarily true that a b (mod n)

For example 4middot2 1middot2 (mod 6) but 4 1 (mod 6)

3middot4 1middot4 (mod 8) but 3 1 (mod 8)

4middot3 1middot3 (mod 9) but 4 1 (mod 9)

Observation In all the above examples c and n have a common factor

Cancellation

Claim Assume gcd(kn) = 1 If imiddotk jmiddotk (mod n) then i j (mod n)

For example multiplicative inverse always exists if n is a prime

Proof Since gcd(kn) = 1 there exists krsquo such that kkrsquo 1 (mod n)

imiddotk jmiddotk (mod n)

=gt imiddotkmiddotkrsquo jmiddotkmiddotkrsquo (mod n)

=gt i j (mod n)

Remarks (Optional) This makes arithmetic modulo prime a field

a structure that ldquobehaves likerdquo real numbers

Arithmetic modulo prime is very useful in coding theory

In particular when p is a prime amp k not a multiple of p then gcd(kp)=1

If i j (mod p) then imiddotk jmiddotk (mod p)

Therefore

k mod p 2k mod p hellip (p-1)k mod p

are all different numbers

Fermatrsquos Little Theorem

Claim 1 Assume gcd(kn) = 1 If imiddotk jmiddotk (mod n) then i j (mod n)

Claim 2 Assume gcd(kn) = 1 If i j (mod n) then imiddotk jmiddotk (mod n)

For example when p=7 and k=3

3 mod 7 = 3 2middot3 mod 7 = 6 3middot3 mod 7 = 2 4middot3 mod 7 = 5 5middot3 mod 7 = 1 6middot3 mod 7 = 4

Notice that in the above example every number from 1 to 6 appears exactly once

In particular when p is a prime amp k not a multiple of p then gcd(kp)=1

If i j (mod p) then imiddotk jmiddotk (mod p)

Therefore

k mod p 2k mod p hellip (p-1)k mod p

are all different numbers

Fermatrsquos Little Theorem

Each of ik mod p cannot be equal to 0 because p is a prime number

Let ci = ik mod p

So 1 lt= c1 lt= p-1 1 lt= c2 lt= p-1 hellip 1lt = cp-1 lt= p-1

By the above we know that c1c2hellipcp-2cp-1 are all different

So for each i from 1 to p-1 there is exactly one cj such that cj = i

Therefore we have

(k mod p)middot(2k mod p)middothellipmiddot((p-1)k mod p) = c1middotc2middothellipmiddotcp-2middotcp-1 = 1middot2middot3hellipmiddot(p-2)middot(p-1)

Fermatrsquos Little Theorem

1 kp-1 (mod p)

Theorem If p is prime amp k not a multiple of p

For example when p=5 k=4 we have kp-1 mod p = 44 mod 5 = 1

ldquoProofrdquo

4middot3middot2middot1 [(4 mod 5) (2middot4 mod 5) (3middot4 mod 5) (4middot4 mod 5)] (mod 5)

[4 middot (2middot4) middot (3middot4) middot (4middot4)] (mod 5)

[44 middot (1middot2middot3middot4)] (mod 5)

Since gcd(1middot2middot3middot4 5)=1 we can cancel 1middot2middot3middot4 on both sides

This implies

1 44 (mod 5)

By the previous slide or direct calculation

Fermatrsquos Little Theorem

1 kp-1 (mod p)

Theorem If p is prime amp k not a multiple of p

Proof

1middot2middotmiddotmiddot(p-1) (k mod p middot 2k mod pmiddothellipmiddot(p-1)k mod p) mod p

(kmiddot2k middotmiddotmiddot (p-1)k) mod p

(kp-1)middot1middot2 middotmiddotmiddot (p-1) (mod p)

So by cancelling 1middot2 middotmiddotmiddot (p-1) on both sides applying

Claim 1

(we can cancel them because gcd(1middot2 middotmiddotmiddot (p-1) p)=1)

we have

1 kp-1 (mod p)

By 2 slides before

By the multiplication rule

Wilsonrsquos Theorem

Theorem p is a prime if and only if

(p-1) -1 (mod p)

First we consider the easy direction

If p is not a prime assume p gt= 5 (for p=4 3 2 (mod 4) )

Then p=qr for some 2 lt= q lt p and 2 lt= r lt p

If q ne r then both q and r appear in (p-1)

and so (p-1) 0 (mod p)

If q = r then p = q2 gt 2q (since we assume p gt 5 and thus q gt 2)

then both q and 2q are in (p-1)

and so again (p-1) 0 (mod p)

Wilsonrsquos Theorem

Theorem p is a prime if and only if

(p-1) -1 (mod p)

To prove the more interesting direction first we need a lemma

Lemma If p is a prime number

x2 1 (mod p) if and only if x 1 (mod p) or x -1 (mod p)

Proof x2 1 (mod p)

iff p | x2 - 1

iff p | (x ndash 1)(x + 1)

iff p | (x ndash 1) or p | (x+1)

iff x 1 (mod p) or x -1 (mod p)

Lemma p prime and p|amiddotb iff p|a or p|b

Wilsonrsquos Theorem

Theorem p is a prime if and only if

(p-1) -1 (mod p)

Letrsquos get the proof idea by considering a concrete example

10

1middot2middot3middot4middot5middot6middot7middot8middot9middot10 mod 11

1middot10middot(2middot6)middot(3middot4)middot(5middot9)middot(7middot8) mod 11

1middot-1middot(1)middot(1)middot(1)middot(1) mod 11

-1 mod 11

Besides 1 and 10 the remaining numbers are paired up into multiplicative inverse

Wilsonrsquos Theorem

Theorem p is a prime if and only if

(p-1) -1 (mod p)

ProofSince p is a prime every number from 1 to p-1 has a multiplicative inverse

By the Lemma every number 2 lt= k lt= p-2 has an inverse krsquo with knekrsquo

Since p is odd the numbers from 2 to p-2 can be grouped into pairs

(a1b1)(a2b2)hellip(a(p-3)2b(p-3)2) so that aibi 1 (mod p)

Therefore (p-1) 1middot(p-1)middot2middot3middotmiddotmiddotmiddotmiddot(p-3)middot(p-2) (mod p)

1middot(p-1)middot(a1b1)middot(a2b2)middotmiddotmiddotmiddotmiddot(a(p-3)2b(p-3)2) (mod p)

1middot(-1)middot(1)middot(1)middotmiddotmiddotmiddotmiddot(1) (mod p)

-1 (mod p)

This Lecture

bull Eulerrsquos phi function

Inclusion-Exclusion (n sets)

What is the inclusion-exclusion formula for the union of n sets

sum of sizes of all single setsndash sum of sizes of all 2-set intersections+ sum of sizes of all 3-set intersectionsndash sum of sizes of all 4-set intersectionshellip+ (ndash1)n+1 times sum of sizes of intersections of all n

sets

1 2 nA A A

1

12 1

( 1)n

ki

S nk i S

S k

A

Inclusion-Exclusion (n sets)

Inclusion-Exclusion (n sets)

sum of sizes of all single setsndash sum of sizes of all 2-set intersections+ sum of sizes of all 3-set intersectionsndash sum of sizes of all 4-set intersectionshellip+ (ndash1)n+1 times sum of sizes of intersections of all n

sets

|A1 [ A2 [ A3 [ hellip [ An|

We want to show that every element is counted exactly once

Consider an element which belongs to exactly k sets say A1 A2 A3 hellip Ak

In the formula such an element is counted the following number of times

Therefore each element is counted exactly once and thus the formula is correct

Euler Function

Given a number n how many numbers from 1 to n are relatively prime to n

When n is a prime number

then every number from 1 to n-1 is relatively prime to n

and so

When n is a prime power

then p 2p 3p 4p hellip n are not relatively prime to n

there are np = pc-1 of them

and other numbers are relatively prime to n

Therefore

Euler Function

Given a number n how many numbers from 1 to n are relatively prime to n

Suppose

Then p 2p 3p 4p hellip n are not relatively prime to n there are np of them

Also q 2q 3q 4q hellip n are not relatively prime to n and there are nq of them

Other numbers are relatively prime to n

Therefore

The numbers pq 2pq 3pq hellip n are subtracted twice and there are npq of them

So the correct answer is

Euler Function

Given a number n how many numbers from 1 to n are relatively prime to n

Let

Let S be the set of numbers from 1 to n that are not relatively prime to n

Let Ai be the set of numbers that are a multiple of piS = A1 [ A2 [ hellip [ An

For the intersection of k sets say A1 A2 A3hellip Ak

then every number in A1 Aring A2 Aring hellip Aring Ak is a multiple of p1p2hellippk

then |A1 Aring A2 Aring hellip Aring Ak| = np1p2hellippk

Euler Function

Given a number n how many numbers from 1 to n are relatively prime to n

Let

Let S be the set of numbers from 1 to n that are not relatively prime to n

Let Ai be the set of numbers that are a multiple of piS = A1 [ A2 [ hellip [ An

|A1 Aring A2 Aring hellip Aring Ak| = np1p2hellippk

When r=3 (only 3 distinct factors)

|A1 [ A2 [ A3|

= np1 + np2 + np3

- np1p2 ndash np1p3 ndash np2p3 + np1p2p3

|A1 [ A2 [ A3| = |A1| + |A2| + |A3|

ndash |A1 Aring A2| ndash |A1 Aring A3| ndash |A2 Aring A3|

+ |A1 Aring A2 Aring A3|

= n(1-p1)(1-p2)(1-p3)

Euler Function

Given a number n how many numbers from 1 to n are relatively prime to n

Let

Let S be the set of numbers from 1 to n that are not relatively prime to n

Let Ai be the set of numbers that are a multiple of piS = A1 [ A2 [ hellip [ An

|A1 Aring A2 Aring hellip Aring Ak| = np1p2hellippk

sum of sizes of all single setsndash sum of sizes of all 2-set intersections+ sum of sizes of all 3-set intersectionsndash sum of sizes of all 4-set intersectionshellip+ (ndash1)n+1 times sum of sizes of intersections of

n sets

|A1 [ A2 [ A3 [ hellip [ An|

|S| = |A1 [ A2 [ hellip [ An|

= n(1-p1)(1-p2)hellip(1-pn)

calculationshellip

  • Modular Arithmetic
  • PowerPoint Presentation
  • Slide 3
  • Slide 4
  • Slide 5
  • Slide 6
  • Slide 7
  • Slide 8
  • Slide 9
  • Slide 10
  • Slide 11
  • Slide 12
  • Slide 13
  • Slide 14
  • Slide 15
  • Slide 16
  • Slide 17
  • Slide 18
  • Slide 19
  • Slide 20
  • Slide 21
  • Slide 22
  • Slide 23
  • Slide 24
  • Slide 25
  • Slide 26
  • Slide 27
  • Slide 28
  • Slide 29
  • Slide 30
  • Slide 31
  • Slide 32
  • Slide 33
  • Slide 34
  • Slide 35
  • Slide 36
  • Slide 37
  • Slide 38
  • Slide 39
  • Slide 40
  • Slide 41
  • Slide 42
  • Slide 43
  • Slide 44
  • Slide 45
Page 19: Modular Arithmetic

The multiplicative inverse of a number a is another number arsquo such that

a middot arsquo 1 (mod n)

Multiplication Inverse

Does every number has a multiplicative inverse in modular arithmetic

For real numbers every nonzero number has a multiplicative inverse

For integers only 1 has a multiplicative inverse

An interesting property of modular arithmetic is that

there are multiplicative inverse for integers

For example 2 5 = 1 mod 3 so 5 is a multiplicative inverse

for 2 under modulo 3 (and vice versa)

Multiplication Inverse

Does every number has a multiplicative inverse in modular arithmetic

Multiplication Inverse

What is the pattern

Case Study

Why 2 does not have a multiplicative inverse under modulo 6

Suppose it has a multiplicative inverse y

2y 1 (mod 6)

=gt 2y = 1 + 6x for some integer x

=gt y = frac12 + 3x

This is a contradiction since both x and y are integers

Necessary Condition

Claim An integer k does not have an multiplicative inverse under modulo n

if k and n have a common factor gt= 2 (gcd(kn) gt= 2)

Proof

This claim says that for k to have a multiplicative inverse modulo n

then a necessary condition is that k and n do not have a common factor gt= 2

Suppose by contradiction that there is an inverse krsquo for k such that

krsquok = 1 (mod n)

Then krsquok = 1 + xn for some integer x

Since both k and n have a common factor say cgt=2

then k=ck1 and n=cn1 for some integers k1 and n1

So krsquock1 = 1 + xcn1

Then krsquok1 = 1c + xn1

This is a contradiction since the LHS is an integer but the RHS is not

Sufficient Condition

What about if gcd(kn)=1

Would k always have an multiplicative inverse under modulo n

For example gcd(37) = 1 3middot5 1 (mod 7)

gcd(89) = 1

gcd(411) = 1 4middot3 1 (mod 11)

8middot8 1 (mod 9)

It seems that there is always an inverse in such a case but why

gcd(89) = 1 8s + 9t = 1 for some integers s and t

8s = 1 ndash 9t

8s 1 (mod 9)gcd(89) = spc(89)

Theorem If gcd(kn)=1 then have krsquo such that

kmiddotkrsquo 1 (mod n)

Proof Since gcd(kn)=1 there exist s and t so that sk + tn = 1

So tn = 1 - sk

This means n | 1 ndash sk

This means that 1 ndash sk 0 (mod n)

This means that 1 sk (mod n)

So krsquo = s is an multiplicative inverse for k

Sufficient Condition

gcd(kn)=spc(kn)

The multiplicative inverse can be computed by the extended Euclidean algorithm

Corollary k has a multiplicative inverse mod n if and only if gcd(kn)=1

This Lecture

bull Fermatrsquos little theorem

Cancellation

There is no general cancellation in modular arithmetic

Note that (mod n) is very similar to =

If a b (mod n) then a+c b+c (mod n)

If a b (mod n) then ac bc (mod n)

However if ac bc (mod n)

it is not necessarily true that a b (mod n)

For example 4middot2 1middot2 (mod 6) but 4 1 (mod 6)

3middot4 1middot4 (mod 8) but 3 1 (mod 8)

4middot3 1middot3 (mod 9) but 4 1 (mod 9)

Observation In all the above examples c and n have a common factor

Cancellation

Claim Assume gcd(kn) = 1 If imiddotk jmiddotk (mod n) then i j (mod n)

For example multiplicative inverse always exists if n is a prime

Proof Since gcd(kn) = 1 there exists krsquo such that kkrsquo 1 (mod n)

imiddotk jmiddotk (mod n)

=gt imiddotkmiddotkrsquo jmiddotkmiddotkrsquo (mod n)

=gt i j (mod n)

Remarks (Optional) This makes arithmetic modulo prime a field

a structure that ldquobehaves likerdquo real numbers

Arithmetic modulo prime is very useful in coding theory

In particular when p is a prime amp k not a multiple of p then gcd(kp)=1

If i j (mod p) then imiddotk jmiddotk (mod p)

Therefore

k mod p 2k mod p hellip (p-1)k mod p

are all different numbers

Fermatrsquos Little Theorem

Claim 1 Assume gcd(kn) = 1 If imiddotk jmiddotk (mod n) then i j (mod n)

Claim 2 Assume gcd(kn) = 1 If i j (mod n) then imiddotk jmiddotk (mod n)

For example when p=7 and k=3

3 mod 7 = 3 2middot3 mod 7 = 6 3middot3 mod 7 = 2 4middot3 mod 7 = 5 5middot3 mod 7 = 1 6middot3 mod 7 = 4

Notice that in the above example every number from 1 to 6 appears exactly once

In particular when p is a prime amp k not a multiple of p then gcd(kp)=1

If i j (mod p) then imiddotk jmiddotk (mod p)

Therefore

k mod p 2k mod p hellip (p-1)k mod p

are all different numbers

Fermatrsquos Little Theorem

Each of ik mod p cannot be equal to 0 because p is a prime number

Let ci = ik mod p

So 1 lt= c1 lt= p-1 1 lt= c2 lt= p-1 hellip 1lt = cp-1 lt= p-1

By the above we know that c1c2hellipcp-2cp-1 are all different

So for each i from 1 to p-1 there is exactly one cj such that cj = i

Therefore we have

(k mod p)middot(2k mod p)middothellipmiddot((p-1)k mod p) = c1middotc2middothellipmiddotcp-2middotcp-1 = 1middot2middot3hellipmiddot(p-2)middot(p-1)

Fermatrsquos Little Theorem

1 kp-1 (mod p)

Theorem If p is prime amp k not a multiple of p

For example when p=5 k=4 we have kp-1 mod p = 44 mod 5 = 1

ldquoProofrdquo

4middot3middot2middot1 [(4 mod 5) (2middot4 mod 5) (3middot4 mod 5) (4middot4 mod 5)] (mod 5)

[4 middot (2middot4) middot (3middot4) middot (4middot4)] (mod 5)

[44 middot (1middot2middot3middot4)] (mod 5)

Since gcd(1middot2middot3middot4 5)=1 we can cancel 1middot2middot3middot4 on both sides

This implies

1 44 (mod 5)

By the previous slide or direct calculation

Fermatrsquos Little Theorem

1 kp-1 (mod p)

Theorem If p is prime amp k not a multiple of p

Proof

1middot2middotmiddotmiddot(p-1) (k mod p middot 2k mod pmiddothellipmiddot(p-1)k mod p) mod p

(kmiddot2k middotmiddotmiddot (p-1)k) mod p

(kp-1)middot1middot2 middotmiddotmiddot (p-1) (mod p)

So by cancelling 1middot2 middotmiddotmiddot (p-1) on both sides applying

Claim 1

(we can cancel them because gcd(1middot2 middotmiddotmiddot (p-1) p)=1)

we have

1 kp-1 (mod p)

By 2 slides before

By the multiplication rule

Wilsonrsquos Theorem

Theorem p is a prime if and only if

(p-1) -1 (mod p)

First we consider the easy direction

If p is not a prime assume p gt= 5 (for p=4 3 2 (mod 4) )

Then p=qr for some 2 lt= q lt p and 2 lt= r lt p

If q ne r then both q and r appear in (p-1)

and so (p-1) 0 (mod p)

If q = r then p = q2 gt 2q (since we assume p gt 5 and thus q gt 2)

then both q and 2q are in (p-1)

and so again (p-1) 0 (mod p)

Wilsonrsquos Theorem

Theorem p is a prime if and only if

(p-1) -1 (mod p)

To prove the more interesting direction first we need a lemma

Lemma If p is a prime number

x2 1 (mod p) if and only if x 1 (mod p) or x -1 (mod p)

Proof x2 1 (mod p)

iff p | x2 - 1

iff p | (x ndash 1)(x + 1)

iff p | (x ndash 1) or p | (x+1)

iff x 1 (mod p) or x -1 (mod p)

Lemma p prime and p|amiddotb iff p|a or p|b

Wilsonrsquos Theorem

Theorem p is a prime if and only if

(p-1) -1 (mod p)

Letrsquos get the proof idea by considering a concrete example

10

1middot2middot3middot4middot5middot6middot7middot8middot9middot10 mod 11

1middot10middot(2middot6)middot(3middot4)middot(5middot9)middot(7middot8) mod 11

1middot-1middot(1)middot(1)middot(1)middot(1) mod 11

-1 mod 11

Besides 1 and 10 the remaining numbers are paired up into multiplicative inverse

Wilsonrsquos Theorem

Theorem p is a prime if and only if

(p-1) -1 (mod p)

ProofSince p is a prime every number from 1 to p-1 has a multiplicative inverse

By the Lemma every number 2 lt= k lt= p-2 has an inverse krsquo with knekrsquo

Since p is odd the numbers from 2 to p-2 can be grouped into pairs

(a1b1)(a2b2)hellip(a(p-3)2b(p-3)2) so that aibi 1 (mod p)

Therefore (p-1) 1middot(p-1)middot2middot3middotmiddotmiddotmiddotmiddot(p-3)middot(p-2) (mod p)

1middot(p-1)middot(a1b1)middot(a2b2)middotmiddotmiddotmiddotmiddot(a(p-3)2b(p-3)2) (mod p)

1middot(-1)middot(1)middot(1)middotmiddotmiddotmiddotmiddot(1) (mod p)

-1 (mod p)

This Lecture

bull Eulerrsquos phi function

Inclusion-Exclusion (n sets)

What is the inclusion-exclusion formula for the union of n sets

sum of sizes of all single setsndash sum of sizes of all 2-set intersections+ sum of sizes of all 3-set intersectionsndash sum of sizes of all 4-set intersectionshellip+ (ndash1)n+1 times sum of sizes of intersections of all n

sets

1 2 nA A A

1

12 1

( 1)n

ki

S nk i S

S k

A

Inclusion-Exclusion (n sets)

Inclusion-Exclusion (n sets)

sum of sizes of all single setsndash sum of sizes of all 2-set intersections+ sum of sizes of all 3-set intersectionsndash sum of sizes of all 4-set intersectionshellip+ (ndash1)n+1 times sum of sizes of intersections of all n

sets

|A1 [ A2 [ A3 [ hellip [ An|

We want to show that every element is counted exactly once

Consider an element which belongs to exactly k sets say A1 A2 A3 hellip Ak

In the formula such an element is counted the following number of times

Therefore each element is counted exactly once and thus the formula is correct

Euler Function

Given a number n how many numbers from 1 to n are relatively prime to n

When n is a prime number

then every number from 1 to n-1 is relatively prime to n

and so

When n is a prime power

then p 2p 3p 4p hellip n are not relatively prime to n

there are np = pc-1 of them

and other numbers are relatively prime to n

Therefore

Euler Function

Given a number n how many numbers from 1 to n are relatively prime to n

Suppose

Then p 2p 3p 4p hellip n are not relatively prime to n there are np of them

Also q 2q 3q 4q hellip n are not relatively prime to n and there are nq of them

Other numbers are relatively prime to n

Therefore

The numbers pq 2pq 3pq hellip n are subtracted twice and there are npq of them

So the correct answer is

Euler Function

Given a number n how many numbers from 1 to n are relatively prime to n

Let

Let S be the set of numbers from 1 to n that are not relatively prime to n

Let Ai be the set of numbers that are a multiple of piS = A1 [ A2 [ hellip [ An

For the intersection of k sets say A1 A2 A3hellip Ak

then every number in A1 Aring A2 Aring hellip Aring Ak is a multiple of p1p2hellippk

then |A1 Aring A2 Aring hellip Aring Ak| = np1p2hellippk

Euler Function

Given a number n how many numbers from 1 to n are relatively prime to n

Let

Let S be the set of numbers from 1 to n that are not relatively prime to n

Let Ai be the set of numbers that are a multiple of piS = A1 [ A2 [ hellip [ An

|A1 Aring A2 Aring hellip Aring Ak| = np1p2hellippk

When r=3 (only 3 distinct factors)

|A1 [ A2 [ A3|

= np1 + np2 + np3

- np1p2 ndash np1p3 ndash np2p3 + np1p2p3

|A1 [ A2 [ A3| = |A1| + |A2| + |A3|

ndash |A1 Aring A2| ndash |A1 Aring A3| ndash |A2 Aring A3|

+ |A1 Aring A2 Aring A3|

= n(1-p1)(1-p2)(1-p3)

Euler Function

Given a number n how many numbers from 1 to n are relatively prime to n

Let

Let S be the set of numbers from 1 to n that are not relatively prime to n

Let Ai be the set of numbers that are a multiple of piS = A1 [ A2 [ hellip [ An

|A1 Aring A2 Aring hellip Aring Ak| = np1p2hellippk

sum of sizes of all single setsndash sum of sizes of all 2-set intersections+ sum of sizes of all 3-set intersectionsndash sum of sizes of all 4-set intersectionshellip+ (ndash1)n+1 times sum of sizes of intersections of

n sets

|A1 [ A2 [ A3 [ hellip [ An|

|S| = |A1 [ A2 [ hellip [ An|

= n(1-p1)(1-p2)hellip(1-pn)

calculationshellip

  • Modular Arithmetic
  • PowerPoint Presentation
  • Slide 3
  • Slide 4
  • Slide 5
  • Slide 6
  • Slide 7
  • Slide 8
  • Slide 9
  • Slide 10
  • Slide 11
  • Slide 12
  • Slide 13
  • Slide 14
  • Slide 15
  • Slide 16
  • Slide 17
  • Slide 18
  • Slide 19
  • Slide 20
  • Slide 21
  • Slide 22
  • Slide 23
  • Slide 24
  • Slide 25
  • Slide 26
  • Slide 27
  • Slide 28
  • Slide 29
  • Slide 30
  • Slide 31
  • Slide 32
  • Slide 33
  • Slide 34
  • Slide 35
  • Slide 36
  • Slide 37
  • Slide 38
  • Slide 39
  • Slide 40
  • Slide 41
  • Slide 42
  • Slide 43
  • Slide 44
  • Slide 45
Page 20: Modular Arithmetic

Multiplication Inverse

Does every number has a multiplicative inverse in modular arithmetic

Multiplication Inverse

What is the pattern

Case Study

Why 2 does not have a multiplicative inverse under modulo 6

Suppose it has a multiplicative inverse y

2y 1 (mod 6)

=gt 2y = 1 + 6x for some integer x

=gt y = frac12 + 3x

This is a contradiction since both x and y are integers

Necessary Condition

Claim An integer k does not have an multiplicative inverse under modulo n

if k and n have a common factor gt= 2 (gcd(kn) gt= 2)

Proof

This claim says that for k to have a multiplicative inverse modulo n

then a necessary condition is that k and n do not have a common factor gt= 2

Suppose by contradiction that there is an inverse krsquo for k such that

krsquok = 1 (mod n)

Then krsquok = 1 + xn for some integer x

Since both k and n have a common factor say cgt=2

then k=ck1 and n=cn1 for some integers k1 and n1

So krsquock1 = 1 + xcn1

Then krsquok1 = 1c + xn1

This is a contradiction since the LHS is an integer but the RHS is not

Sufficient Condition

What about if gcd(kn)=1

Would k always have an multiplicative inverse under modulo n

For example gcd(37) = 1 3middot5 1 (mod 7)

gcd(89) = 1

gcd(411) = 1 4middot3 1 (mod 11)

8middot8 1 (mod 9)

It seems that there is always an inverse in such a case but why

gcd(89) = 1 8s + 9t = 1 for some integers s and t

8s = 1 ndash 9t

8s 1 (mod 9)gcd(89) = spc(89)

Theorem If gcd(kn)=1 then have krsquo such that

kmiddotkrsquo 1 (mod n)

Proof Since gcd(kn)=1 there exist s and t so that sk + tn = 1

So tn = 1 - sk

This means n | 1 ndash sk

This means that 1 ndash sk 0 (mod n)

This means that 1 sk (mod n)

So krsquo = s is an multiplicative inverse for k

Sufficient Condition

gcd(kn)=spc(kn)

The multiplicative inverse can be computed by the extended Euclidean algorithm

Corollary k has a multiplicative inverse mod n if and only if gcd(kn)=1

This Lecture

bull Fermatrsquos little theorem

Cancellation

There is no general cancellation in modular arithmetic

Note that (mod n) is very similar to =

If a b (mod n) then a+c b+c (mod n)

If a b (mod n) then ac bc (mod n)

However if ac bc (mod n)

it is not necessarily true that a b (mod n)

For example 4middot2 1middot2 (mod 6) but 4 1 (mod 6)

3middot4 1middot4 (mod 8) but 3 1 (mod 8)

4middot3 1middot3 (mod 9) but 4 1 (mod 9)

Observation In all the above examples c and n have a common factor

Cancellation

Claim Assume gcd(kn) = 1 If imiddotk jmiddotk (mod n) then i j (mod n)

For example multiplicative inverse always exists if n is a prime

Proof Since gcd(kn) = 1 there exists krsquo such that kkrsquo 1 (mod n)

imiddotk jmiddotk (mod n)

=gt imiddotkmiddotkrsquo jmiddotkmiddotkrsquo (mod n)

=gt i j (mod n)

Remarks (Optional) This makes arithmetic modulo prime a field

a structure that ldquobehaves likerdquo real numbers

Arithmetic modulo prime is very useful in coding theory

In particular when p is a prime amp k not a multiple of p then gcd(kp)=1

If i j (mod p) then imiddotk jmiddotk (mod p)

Therefore

k mod p 2k mod p hellip (p-1)k mod p

are all different numbers

Fermatrsquos Little Theorem

Claim 1 Assume gcd(kn) = 1 If imiddotk jmiddotk (mod n) then i j (mod n)

Claim 2 Assume gcd(kn) = 1 If i j (mod n) then imiddotk jmiddotk (mod n)

For example when p=7 and k=3

3 mod 7 = 3 2middot3 mod 7 = 6 3middot3 mod 7 = 2 4middot3 mod 7 = 5 5middot3 mod 7 = 1 6middot3 mod 7 = 4

Notice that in the above example every number from 1 to 6 appears exactly once

In particular when p is a prime amp k not a multiple of p then gcd(kp)=1

If i j (mod p) then imiddotk jmiddotk (mod p)

Therefore

k mod p 2k mod p hellip (p-1)k mod p

are all different numbers

Fermatrsquos Little Theorem

Each of ik mod p cannot be equal to 0 because p is a prime number

Let ci = ik mod p

So 1 lt= c1 lt= p-1 1 lt= c2 lt= p-1 hellip 1lt = cp-1 lt= p-1

By the above we know that c1c2hellipcp-2cp-1 are all different

So for each i from 1 to p-1 there is exactly one cj such that cj = i

Therefore we have

(k mod p)middot(2k mod p)middothellipmiddot((p-1)k mod p) = c1middotc2middothellipmiddotcp-2middotcp-1 = 1middot2middot3hellipmiddot(p-2)middot(p-1)

Fermatrsquos Little Theorem

1 kp-1 (mod p)

Theorem If p is prime amp k not a multiple of p

For example when p=5 k=4 we have kp-1 mod p = 44 mod 5 = 1

ldquoProofrdquo

4middot3middot2middot1 [(4 mod 5) (2middot4 mod 5) (3middot4 mod 5) (4middot4 mod 5)] (mod 5)

[4 middot (2middot4) middot (3middot4) middot (4middot4)] (mod 5)

[44 middot (1middot2middot3middot4)] (mod 5)

Since gcd(1middot2middot3middot4 5)=1 we can cancel 1middot2middot3middot4 on both sides

This implies

1 44 (mod 5)

By the previous slide or direct calculation

Fermatrsquos Little Theorem

1 kp-1 (mod p)

Theorem If p is prime amp k not a multiple of p

Proof

1middot2middotmiddotmiddot(p-1) (k mod p middot 2k mod pmiddothellipmiddot(p-1)k mod p) mod p

(kmiddot2k middotmiddotmiddot (p-1)k) mod p

(kp-1)middot1middot2 middotmiddotmiddot (p-1) (mod p)

So by cancelling 1middot2 middotmiddotmiddot (p-1) on both sides applying

Claim 1

(we can cancel them because gcd(1middot2 middotmiddotmiddot (p-1) p)=1)

we have

1 kp-1 (mod p)

By 2 slides before

By the multiplication rule

Wilsonrsquos Theorem

Theorem p is a prime if and only if

(p-1) -1 (mod p)

First we consider the easy direction

If p is not a prime assume p gt= 5 (for p=4 3 2 (mod 4) )

Then p=qr for some 2 lt= q lt p and 2 lt= r lt p

If q ne r then both q and r appear in (p-1)

and so (p-1) 0 (mod p)

If q = r then p = q2 gt 2q (since we assume p gt 5 and thus q gt 2)

then both q and 2q are in (p-1)

and so again (p-1) 0 (mod p)

Wilsonrsquos Theorem

Theorem p is a prime if and only if

(p-1) -1 (mod p)

To prove the more interesting direction first we need a lemma

Lemma If p is a prime number

x2 1 (mod p) if and only if x 1 (mod p) or x -1 (mod p)

Proof x2 1 (mod p)

iff p | x2 - 1

iff p | (x ndash 1)(x + 1)

iff p | (x ndash 1) or p | (x+1)

iff x 1 (mod p) or x -1 (mod p)

Lemma p prime and p|amiddotb iff p|a or p|b

Wilsonrsquos Theorem

Theorem p is a prime if and only if

(p-1) -1 (mod p)

Letrsquos get the proof idea by considering a concrete example

10

1middot2middot3middot4middot5middot6middot7middot8middot9middot10 mod 11

1middot10middot(2middot6)middot(3middot4)middot(5middot9)middot(7middot8) mod 11

1middot-1middot(1)middot(1)middot(1)middot(1) mod 11

-1 mod 11

Besides 1 and 10 the remaining numbers are paired up into multiplicative inverse

Wilsonrsquos Theorem

Theorem p is a prime if and only if

(p-1) -1 (mod p)

ProofSince p is a prime every number from 1 to p-1 has a multiplicative inverse

By the Lemma every number 2 lt= k lt= p-2 has an inverse krsquo with knekrsquo

Since p is odd the numbers from 2 to p-2 can be grouped into pairs

(a1b1)(a2b2)hellip(a(p-3)2b(p-3)2) so that aibi 1 (mod p)

Therefore (p-1) 1middot(p-1)middot2middot3middotmiddotmiddotmiddotmiddot(p-3)middot(p-2) (mod p)

1middot(p-1)middot(a1b1)middot(a2b2)middotmiddotmiddotmiddotmiddot(a(p-3)2b(p-3)2) (mod p)

1middot(-1)middot(1)middot(1)middotmiddotmiddotmiddotmiddot(1) (mod p)

-1 (mod p)

This Lecture

bull Eulerrsquos phi function

Inclusion-Exclusion (n sets)

What is the inclusion-exclusion formula for the union of n sets

sum of sizes of all single setsndash sum of sizes of all 2-set intersections+ sum of sizes of all 3-set intersectionsndash sum of sizes of all 4-set intersectionshellip+ (ndash1)n+1 times sum of sizes of intersections of all n

sets

1 2 nA A A

1

12 1

( 1)n

ki

S nk i S

S k

A

Inclusion-Exclusion (n sets)

Inclusion-Exclusion (n sets)

sum of sizes of all single setsndash sum of sizes of all 2-set intersections+ sum of sizes of all 3-set intersectionsndash sum of sizes of all 4-set intersectionshellip+ (ndash1)n+1 times sum of sizes of intersections of all n

sets

|A1 [ A2 [ A3 [ hellip [ An|

We want to show that every element is counted exactly once

Consider an element which belongs to exactly k sets say A1 A2 A3 hellip Ak

In the formula such an element is counted the following number of times

Therefore each element is counted exactly once and thus the formula is correct

Euler Function

Given a number n how many numbers from 1 to n are relatively prime to n

When n is a prime number

then every number from 1 to n-1 is relatively prime to n

and so

When n is a prime power

then p 2p 3p 4p hellip n are not relatively prime to n

there are np = pc-1 of them

and other numbers are relatively prime to n

Therefore

Euler Function

Given a number n how many numbers from 1 to n are relatively prime to n

Suppose

Then p 2p 3p 4p hellip n are not relatively prime to n there are np of them

Also q 2q 3q 4q hellip n are not relatively prime to n and there are nq of them

Other numbers are relatively prime to n

Therefore

The numbers pq 2pq 3pq hellip n are subtracted twice and there are npq of them

So the correct answer is

Euler Function

Given a number n how many numbers from 1 to n are relatively prime to n

Let

Let S be the set of numbers from 1 to n that are not relatively prime to n

Let Ai be the set of numbers that are a multiple of piS = A1 [ A2 [ hellip [ An

For the intersection of k sets say A1 A2 A3hellip Ak

then every number in A1 Aring A2 Aring hellip Aring Ak is a multiple of p1p2hellippk

then |A1 Aring A2 Aring hellip Aring Ak| = np1p2hellippk

Euler Function

Given a number n how many numbers from 1 to n are relatively prime to n

Let

Let S be the set of numbers from 1 to n that are not relatively prime to n

Let Ai be the set of numbers that are a multiple of piS = A1 [ A2 [ hellip [ An

|A1 Aring A2 Aring hellip Aring Ak| = np1p2hellippk

When r=3 (only 3 distinct factors)

|A1 [ A2 [ A3|

= np1 + np2 + np3

- np1p2 ndash np1p3 ndash np2p3 + np1p2p3

|A1 [ A2 [ A3| = |A1| + |A2| + |A3|

ndash |A1 Aring A2| ndash |A1 Aring A3| ndash |A2 Aring A3|

+ |A1 Aring A2 Aring A3|

= n(1-p1)(1-p2)(1-p3)

Euler Function

Given a number n how many numbers from 1 to n are relatively prime to n

Let

Let S be the set of numbers from 1 to n that are not relatively prime to n

Let Ai be the set of numbers that are a multiple of piS = A1 [ A2 [ hellip [ An

|A1 Aring A2 Aring hellip Aring Ak| = np1p2hellippk

sum of sizes of all single setsndash sum of sizes of all 2-set intersections+ sum of sizes of all 3-set intersectionsndash sum of sizes of all 4-set intersectionshellip+ (ndash1)n+1 times sum of sizes of intersections of

n sets

|A1 [ A2 [ A3 [ hellip [ An|

|S| = |A1 [ A2 [ hellip [ An|

= n(1-p1)(1-p2)hellip(1-pn)

calculationshellip

  • Modular Arithmetic
  • PowerPoint Presentation
  • Slide 3
  • Slide 4
  • Slide 5
  • Slide 6
  • Slide 7
  • Slide 8
  • Slide 9
  • Slide 10
  • Slide 11
  • Slide 12
  • Slide 13
  • Slide 14
  • Slide 15
  • Slide 16
  • Slide 17
  • Slide 18
  • Slide 19
  • Slide 20
  • Slide 21
  • Slide 22
  • Slide 23
  • Slide 24
  • Slide 25
  • Slide 26
  • Slide 27
  • Slide 28
  • Slide 29
  • Slide 30
  • Slide 31
  • Slide 32
  • Slide 33
  • Slide 34
  • Slide 35
  • Slide 36
  • Slide 37
  • Slide 38
  • Slide 39
  • Slide 40
  • Slide 41
  • Slide 42
  • Slide 43
  • Slide 44
  • Slide 45
Page 21: Modular Arithmetic

Multiplication Inverse

What is the pattern

Case Study

Why 2 does not have a multiplicative inverse under modulo 6

Suppose it has a multiplicative inverse y

2y 1 (mod 6)

=gt 2y = 1 + 6x for some integer x

=gt y = frac12 + 3x

This is a contradiction since both x and y are integers

Necessary Condition

Claim An integer k does not have an multiplicative inverse under modulo n

if k and n have a common factor gt= 2 (gcd(kn) gt= 2)

Proof

This claim says that for k to have a multiplicative inverse modulo n

then a necessary condition is that k and n do not have a common factor gt= 2

Suppose by contradiction that there is an inverse krsquo for k such that

krsquok = 1 (mod n)

Then krsquok = 1 + xn for some integer x

Since both k and n have a common factor say cgt=2

then k=ck1 and n=cn1 for some integers k1 and n1

So krsquock1 = 1 + xcn1

Then krsquok1 = 1c + xn1

This is a contradiction since the LHS is an integer but the RHS is not

Sufficient Condition

What about if gcd(kn)=1

Would k always have an multiplicative inverse under modulo n

For example gcd(37) = 1 3middot5 1 (mod 7)

gcd(89) = 1

gcd(411) = 1 4middot3 1 (mod 11)

8middot8 1 (mod 9)

It seems that there is always an inverse in such a case but why

gcd(89) = 1 8s + 9t = 1 for some integers s and t

8s = 1 ndash 9t

8s 1 (mod 9)gcd(89) = spc(89)

Theorem If gcd(kn)=1 then have krsquo such that

kmiddotkrsquo 1 (mod n)

Proof Since gcd(kn)=1 there exist s and t so that sk + tn = 1

So tn = 1 - sk

This means n | 1 ndash sk

This means that 1 ndash sk 0 (mod n)

This means that 1 sk (mod n)

So krsquo = s is an multiplicative inverse for k

Sufficient Condition

gcd(kn)=spc(kn)

The multiplicative inverse can be computed by the extended Euclidean algorithm

Corollary k has a multiplicative inverse mod n if and only if gcd(kn)=1

This Lecture

bull Fermatrsquos little theorem

Cancellation

There is no general cancellation in modular arithmetic

Note that (mod n) is very similar to =

If a b (mod n) then a+c b+c (mod n)

If a b (mod n) then ac bc (mod n)

However if ac bc (mod n)

it is not necessarily true that a b (mod n)

For example 4middot2 1middot2 (mod 6) but 4 1 (mod 6)

3middot4 1middot4 (mod 8) but 3 1 (mod 8)

4middot3 1middot3 (mod 9) but 4 1 (mod 9)

Observation In all the above examples c and n have a common factor

Cancellation

Claim Assume gcd(kn) = 1 If imiddotk jmiddotk (mod n) then i j (mod n)

For example multiplicative inverse always exists if n is a prime

Proof Since gcd(kn) = 1 there exists krsquo such that kkrsquo 1 (mod n)

imiddotk jmiddotk (mod n)

=gt imiddotkmiddotkrsquo jmiddotkmiddotkrsquo (mod n)

=gt i j (mod n)

Remarks (Optional) This makes arithmetic modulo prime a field

a structure that ldquobehaves likerdquo real numbers

Arithmetic modulo prime is very useful in coding theory

In particular when p is a prime amp k not a multiple of p then gcd(kp)=1

If i j (mod p) then imiddotk jmiddotk (mod p)

Therefore

k mod p 2k mod p hellip (p-1)k mod p

are all different numbers

Fermatrsquos Little Theorem

Claim 1 Assume gcd(kn) = 1 If imiddotk jmiddotk (mod n) then i j (mod n)

Claim 2 Assume gcd(kn) = 1 If i j (mod n) then imiddotk jmiddotk (mod n)

For example when p=7 and k=3

3 mod 7 = 3 2middot3 mod 7 = 6 3middot3 mod 7 = 2 4middot3 mod 7 = 5 5middot3 mod 7 = 1 6middot3 mod 7 = 4

Notice that in the above example every number from 1 to 6 appears exactly once

In particular when p is a prime amp k not a multiple of p then gcd(kp)=1

If i j (mod p) then imiddotk jmiddotk (mod p)

Therefore

k mod p 2k mod p hellip (p-1)k mod p

are all different numbers

Fermatrsquos Little Theorem

Each of ik mod p cannot be equal to 0 because p is a prime number

Let ci = ik mod p

So 1 lt= c1 lt= p-1 1 lt= c2 lt= p-1 hellip 1lt = cp-1 lt= p-1

By the above we know that c1c2hellipcp-2cp-1 are all different

So for each i from 1 to p-1 there is exactly one cj such that cj = i

Therefore we have

(k mod p)middot(2k mod p)middothellipmiddot((p-1)k mod p) = c1middotc2middothellipmiddotcp-2middotcp-1 = 1middot2middot3hellipmiddot(p-2)middot(p-1)

Fermatrsquos Little Theorem

1 kp-1 (mod p)

Theorem If p is prime amp k not a multiple of p

For example when p=5 k=4 we have kp-1 mod p = 44 mod 5 = 1

ldquoProofrdquo

4middot3middot2middot1 [(4 mod 5) (2middot4 mod 5) (3middot4 mod 5) (4middot4 mod 5)] (mod 5)

[4 middot (2middot4) middot (3middot4) middot (4middot4)] (mod 5)

[44 middot (1middot2middot3middot4)] (mod 5)

Since gcd(1middot2middot3middot4 5)=1 we can cancel 1middot2middot3middot4 on both sides

This implies

1 44 (mod 5)

By the previous slide or direct calculation

Fermatrsquos Little Theorem

1 kp-1 (mod p)

Theorem If p is prime amp k not a multiple of p

Proof

1middot2middotmiddotmiddot(p-1) (k mod p middot 2k mod pmiddothellipmiddot(p-1)k mod p) mod p

(kmiddot2k middotmiddotmiddot (p-1)k) mod p

(kp-1)middot1middot2 middotmiddotmiddot (p-1) (mod p)

So by cancelling 1middot2 middotmiddotmiddot (p-1) on both sides applying

Claim 1

(we can cancel them because gcd(1middot2 middotmiddotmiddot (p-1) p)=1)

we have

1 kp-1 (mod p)

By 2 slides before

By the multiplication rule

Wilsonrsquos Theorem

Theorem p is a prime if and only if

(p-1) -1 (mod p)

First we consider the easy direction

If p is not a prime assume p gt= 5 (for p=4 3 2 (mod 4) )

Then p=qr for some 2 lt= q lt p and 2 lt= r lt p

If q ne r then both q and r appear in (p-1)

and so (p-1) 0 (mod p)

If q = r then p = q2 gt 2q (since we assume p gt 5 and thus q gt 2)

then both q and 2q are in (p-1)

and so again (p-1) 0 (mod p)

Wilsonrsquos Theorem

Theorem p is a prime if and only if

(p-1) -1 (mod p)

To prove the more interesting direction first we need a lemma

Lemma If p is a prime number

x2 1 (mod p) if and only if x 1 (mod p) or x -1 (mod p)

Proof x2 1 (mod p)

iff p | x2 - 1

iff p | (x ndash 1)(x + 1)

iff p | (x ndash 1) or p | (x+1)

iff x 1 (mod p) or x -1 (mod p)

Lemma p prime and p|amiddotb iff p|a or p|b

Wilsonrsquos Theorem

Theorem p is a prime if and only if

(p-1) -1 (mod p)

Letrsquos get the proof idea by considering a concrete example

10

1middot2middot3middot4middot5middot6middot7middot8middot9middot10 mod 11

1middot10middot(2middot6)middot(3middot4)middot(5middot9)middot(7middot8) mod 11

1middot-1middot(1)middot(1)middot(1)middot(1) mod 11

-1 mod 11

Besides 1 and 10 the remaining numbers are paired up into multiplicative inverse

Wilsonrsquos Theorem

Theorem p is a prime if and only if

(p-1) -1 (mod p)

ProofSince p is a prime every number from 1 to p-1 has a multiplicative inverse

By the Lemma every number 2 lt= k lt= p-2 has an inverse krsquo with knekrsquo

Since p is odd the numbers from 2 to p-2 can be grouped into pairs

(a1b1)(a2b2)hellip(a(p-3)2b(p-3)2) so that aibi 1 (mod p)

Therefore (p-1) 1middot(p-1)middot2middot3middotmiddotmiddotmiddotmiddot(p-3)middot(p-2) (mod p)

1middot(p-1)middot(a1b1)middot(a2b2)middotmiddotmiddotmiddotmiddot(a(p-3)2b(p-3)2) (mod p)

1middot(-1)middot(1)middot(1)middotmiddotmiddotmiddotmiddot(1) (mod p)

-1 (mod p)

This Lecture

bull Eulerrsquos phi function

Inclusion-Exclusion (n sets)

What is the inclusion-exclusion formula for the union of n sets

sum of sizes of all single setsndash sum of sizes of all 2-set intersections+ sum of sizes of all 3-set intersectionsndash sum of sizes of all 4-set intersectionshellip+ (ndash1)n+1 times sum of sizes of intersections of all n

sets

1 2 nA A A

1

12 1

( 1)n

ki

S nk i S

S k

A

Inclusion-Exclusion (n sets)

Inclusion-Exclusion (n sets)

sum of sizes of all single setsndash sum of sizes of all 2-set intersections+ sum of sizes of all 3-set intersectionsndash sum of sizes of all 4-set intersectionshellip+ (ndash1)n+1 times sum of sizes of intersections of all n

sets

|A1 [ A2 [ A3 [ hellip [ An|

We want to show that every element is counted exactly once

Consider an element which belongs to exactly k sets say A1 A2 A3 hellip Ak

In the formula such an element is counted the following number of times

Therefore each element is counted exactly once and thus the formula is correct

Euler Function

Given a number n how many numbers from 1 to n are relatively prime to n

When n is a prime number

then every number from 1 to n-1 is relatively prime to n

and so

When n is a prime power

then p 2p 3p 4p hellip n are not relatively prime to n

there are np = pc-1 of them

and other numbers are relatively prime to n

Therefore

Euler Function

Given a number n how many numbers from 1 to n are relatively prime to n

Suppose

Then p 2p 3p 4p hellip n are not relatively prime to n there are np of them

Also q 2q 3q 4q hellip n are not relatively prime to n and there are nq of them

Other numbers are relatively prime to n

Therefore

The numbers pq 2pq 3pq hellip n are subtracted twice and there are npq of them

So the correct answer is

Euler Function

Given a number n how many numbers from 1 to n are relatively prime to n

Let

Let S be the set of numbers from 1 to n that are not relatively prime to n

Let Ai be the set of numbers that are a multiple of piS = A1 [ A2 [ hellip [ An

For the intersection of k sets say A1 A2 A3hellip Ak

then every number in A1 Aring A2 Aring hellip Aring Ak is a multiple of p1p2hellippk

then |A1 Aring A2 Aring hellip Aring Ak| = np1p2hellippk

Euler Function

Given a number n how many numbers from 1 to n are relatively prime to n

Let

Let S be the set of numbers from 1 to n that are not relatively prime to n

Let Ai be the set of numbers that are a multiple of piS = A1 [ A2 [ hellip [ An

|A1 Aring A2 Aring hellip Aring Ak| = np1p2hellippk

When r=3 (only 3 distinct factors)

|A1 [ A2 [ A3|

= np1 + np2 + np3

- np1p2 ndash np1p3 ndash np2p3 + np1p2p3

|A1 [ A2 [ A3| = |A1| + |A2| + |A3|

ndash |A1 Aring A2| ndash |A1 Aring A3| ndash |A2 Aring A3|

+ |A1 Aring A2 Aring A3|

= n(1-p1)(1-p2)(1-p3)

Euler Function

Given a number n how many numbers from 1 to n are relatively prime to n

Let

Let S be the set of numbers from 1 to n that are not relatively prime to n

Let Ai be the set of numbers that are a multiple of piS = A1 [ A2 [ hellip [ An

|A1 Aring A2 Aring hellip Aring Ak| = np1p2hellippk

sum of sizes of all single setsndash sum of sizes of all 2-set intersections+ sum of sizes of all 3-set intersectionsndash sum of sizes of all 4-set intersectionshellip+ (ndash1)n+1 times sum of sizes of intersections of

n sets

|A1 [ A2 [ A3 [ hellip [ An|

|S| = |A1 [ A2 [ hellip [ An|

= n(1-p1)(1-p2)hellip(1-pn)

calculationshellip

  • Modular Arithmetic
  • PowerPoint Presentation
  • Slide 3
  • Slide 4
  • Slide 5
  • Slide 6
  • Slide 7
  • Slide 8
  • Slide 9
  • Slide 10
  • Slide 11
  • Slide 12
  • Slide 13
  • Slide 14
  • Slide 15
  • Slide 16
  • Slide 17
  • Slide 18
  • Slide 19
  • Slide 20
  • Slide 21
  • Slide 22
  • Slide 23
  • Slide 24
  • Slide 25
  • Slide 26
  • Slide 27
  • Slide 28
  • Slide 29
  • Slide 30
  • Slide 31
  • Slide 32
  • Slide 33
  • Slide 34
  • Slide 35
  • Slide 36
  • Slide 37
  • Slide 38
  • Slide 39
  • Slide 40
  • Slide 41
  • Slide 42
  • Slide 43
  • Slide 44
  • Slide 45
Page 22: Modular Arithmetic

Case Study

Why 2 does not have a multiplicative inverse under modulo 6

Suppose it has a multiplicative inverse y

2y 1 (mod 6)

=gt 2y = 1 + 6x for some integer x

=gt y = frac12 + 3x

This is a contradiction since both x and y are integers

Necessary Condition

Claim An integer k does not have an multiplicative inverse under modulo n

if k and n have a common factor gt= 2 (gcd(kn) gt= 2)

Proof

This claim says that for k to have a multiplicative inverse modulo n

then a necessary condition is that k and n do not have a common factor gt= 2

Suppose by contradiction that there is an inverse krsquo for k such that

krsquok = 1 (mod n)

Then krsquok = 1 + xn for some integer x

Since both k and n have a common factor say cgt=2

then k=ck1 and n=cn1 for some integers k1 and n1

So krsquock1 = 1 + xcn1

Then krsquok1 = 1c + xn1

This is a contradiction since the LHS is an integer but the RHS is not

Sufficient Condition

What about if gcd(kn)=1

Would k always have an multiplicative inverse under modulo n

For example gcd(37) = 1 3middot5 1 (mod 7)

gcd(89) = 1

gcd(411) = 1 4middot3 1 (mod 11)

8middot8 1 (mod 9)

It seems that there is always an inverse in such a case but why

gcd(89) = 1 8s + 9t = 1 for some integers s and t

8s = 1 ndash 9t

8s 1 (mod 9)gcd(89) = spc(89)

Theorem If gcd(kn)=1 then have krsquo such that

kmiddotkrsquo 1 (mod n)

Proof Since gcd(kn)=1 there exist s and t so that sk + tn = 1

So tn = 1 - sk

This means n | 1 ndash sk

This means that 1 ndash sk 0 (mod n)

This means that 1 sk (mod n)

So krsquo = s is an multiplicative inverse for k

Sufficient Condition

gcd(kn)=spc(kn)

The multiplicative inverse can be computed by the extended Euclidean algorithm

Corollary k has a multiplicative inverse mod n if and only if gcd(kn)=1

This Lecture

bull Fermatrsquos little theorem

Cancellation

There is no general cancellation in modular arithmetic

Note that (mod n) is very similar to =

If a b (mod n) then a+c b+c (mod n)

If a b (mod n) then ac bc (mod n)

However if ac bc (mod n)

it is not necessarily true that a b (mod n)

For example 4middot2 1middot2 (mod 6) but 4 1 (mod 6)

3middot4 1middot4 (mod 8) but 3 1 (mod 8)

4middot3 1middot3 (mod 9) but 4 1 (mod 9)

Observation In all the above examples c and n have a common factor

Cancellation

Claim Assume gcd(kn) = 1 If imiddotk jmiddotk (mod n) then i j (mod n)

For example multiplicative inverse always exists if n is a prime

Proof Since gcd(kn) = 1 there exists krsquo such that kkrsquo 1 (mod n)

imiddotk jmiddotk (mod n)

=gt imiddotkmiddotkrsquo jmiddotkmiddotkrsquo (mod n)

=gt i j (mod n)

Remarks (Optional) This makes arithmetic modulo prime a field

a structure that ldquobehaves likerdquo real numbers

Arithmetic modulo prime is very useful in coding theory

In particular when p is a prime amp k not a multiple of p then gcd(kp)=1

If i j (mod p) then imiddotk jmiddotk (mod p)

Therefore

k mod p 2k mod p hellip (p-1)k mod p

are all different numbers

Fermatrsquos Little Theorem

Claim 1 Assume gcd(kn) = 1 If imiddotk jmiddotk (mod n) then i j (mod n)

Claim 2 Assume gcd(kn) = 1 If i j (mod n) then imiddotk jmiddotk (mod n)

For example when p=7 and k=3

3 mod 7 = 3 2middot3 mod 7 = 6 3middot3 mod 7 = 2 4middot3 mod 7 = 5 5middot3 mod 7 = 1 6middot3 mod 7 = 4

Notice that in the above example every number from 1 to 6 appears exactly once

In particular when p is a prime amp k not a multiple of p then gcd(kp)=1

If i j (mod p) then imiddotk jmiddotk (mod p)

Therefore

k mod p 2k mod p hellip (p-1)k mod p

are all different numbers

Fermatrsquos Little Theorem

Each of ik mod p cannot be equal to 0 because p is a prime number

Let ci = ik mod p

So 1 lt= c1 lt= p-1 1 lt= c2 lt= p-1 hellip 1lt = cp-1 lt= p-1

By the above we know that c1c2hellipcp-2cp-1 are all different

So for each i from 1 to p-1 there is exactly one cj such that cj = i

Therefore we have

(k mod p)middot(2k mod p)middothellipmiddot((p-1)k mod p) = c1middotc2middothellipmiddotcp-2middotcp-1 = 1middot2middot3hellipmiddot(p-2)middot(p-1)

Fermatrsquos Little Theorem

1 kp-1 (mod p)

Theorem If p is prime amp k not a multiple of p

For example when p=5 k=4 we have kp-1 mod p = 44 mod 5 = 1

ldquoProofrdquo

4middot3middot2middot1 [(4 mod 5) (2middot4 mod 5) (3middot4 mod 5) (4middot4 mod 5)] (mod 5)

[4 middot (2middot4) middot (3middot4) middot (4middot4)] (mod 5)

[44 middot (1middot2middot3middot4)] (mod 5)

Since gcd(1middot2middot3middot4 5)=1 we can cancel 1middot2middot3middot4 on both sides

This implies

1 44 (mod 5)

By the previous slide or direct calculation

Fermatrsquos Little Theorem

1 kp-1 (mod p)

Theorem If p is prime amp k not a multiple of p

Proof

1middot2middotmiddotmiddot(p-1) (k mod p middot 2k mod pmiddothellipmiddot(p-1)k mod p) mod p

(kmiddot2k middotmiddotmiddot (p-1)k) mod p

(kp-1)middot1middot2 middotmiddotmiddot (p-1) (mod p)

So by cancelling 1middot2 middotmiddotmiddot (p-1) on both sides applying

Claim 1

(we can cancel them because gcd(1middot2 middotmiddotmiddot (p-1) p)=1)

we have

1 kp-1 (mod p)

By 2 slides before

By the multiplication rule

Wilsonrsquos Theorem

Theorem p is a prime if and only if

(p-1) -1 (mod p)

First we consider the easy direction

If p is not a prime assume p gt= 5 (for p=4 3 2 (mod 4) )

Then p=qr for some 2 lt= q lt p and 2 lt= r lt p

If q ne r then both q and r appear in (p-1)

and so (p-1) 0 (mod p)

If q = r then p = q2 gt 2q (since we assume p gt 5 and thus q gt 2)

then both q and 2q are in (p-1)

and so again (p-1) 0 (mod p)

Wilsonrsquos Theorem

Theorem p is a prime if and only if

(p-1) -1 (mod p)

To prove the more interesting direction first we need a lemma

Lemma If p is a prime number

x2 1 (mod p) if and only if x 1 (mod p) or x -1 (mod p)

Proof x2 1 (mod p)

iff p | x2 - 1

iff p | (x ndash 1)(x + 1)

iff p | (x ndash 1) or p | (x+1)

iff x 1 (mod p) or x -1 (mod p)

Lemma p prime and p|amiddotb iff p|a or p|b

Wilsonrsquos Theorem

Theorem p is a prime if and only if

(p-1) -1 (mod p)

Letrsquos get the proof idea by considering a concrete example

10

1middot2middot3middot4middot5middot6middot7middot8middot9middot10 mod 11

1middot10middot(2middot6)middot(3middot4)middot(5middot9)middot(7middot8) mod 11

1middot-1middot(1)middot(1)middot(1)middot(1) mod 11

-1 mod 11

Besides 1 and 10 the remaining numbers are paired up into multiplicative inverse

Wilsonrsquos Theorem

Theorem p is a prime if and only if

(p-1) -1 (mod p)

ProofSince p is a prime every number from 1 to p-1 has a multiplicative inverse

By the Lemma every number 2 lt= k lt= p-2 has an inverse krsquo with knekrsquo

Since p is odd the numbers from 2 to p-2 can be grouped into pairs

(a1b1)(a2b2)hellip(a(p-3)2b(p-3)2) so that aibi 1 (mod p)

Therefore (p-1) 1middot(p-1)middot2middot3middotmiddotmiddotmiddotmiddot(p-3)middot(p-2) (mod p)

1middot(p-1)middot(a1b1)middot(a2b2)middotmiddotmiddotmiddotmiddot(a(p-3)2b(p-3)2) (mod p)

1middot(-1)middot(1)middot(1)middotmiddotmiddotmiddotmiddot(1) (mod p)

-1 (mod p)

This Lecture

bull Eulerrsquos phi function

Inclusion-Exclusion (n sets)

What is the inclusion-exclusion formula for the union of n sets

sum of sizes of all single setsndash sum of sizes of all 2-set intersections+ sum of sizes of all 3-set intersectionsndash sum of sizes of all 4-set intersectionshellip+ (ndash1)n+1 times sum of sizes of intersections of all n

sets

1 2 nA A A

1

12 1

( 1)n

ki

S nk i S

S k

A

Inclusion-Exclusion (n sets)

Inclusion-Exclusion (n sets)

sum of sizes of all single setsndash sum of sizes of all 2-set intersections+ sum of sizes of all 3-set intersectionsndash sum of sizes of all 4-set intersectionshellip+ (ndash1)n+1 times sum of sizes of intersections of all n

sets

|A1 [ A2 [ A3 [ hellip [ An|

We want to show that every element is counted exactly once

Consider an element which belongs to exactly k sets say A1 A2 A3 hellip Ak

In the formula such an element is counted the following number of times

Therefore each element is counted exactly once and thus the formula is correct

Euler Function

Given a number n how many numbers from 1 to n are relatively prime to n

When n is a prime number

then every number from 1 to n-1 is relatively prime to n

and so

When n is a prime power

then p 2p 3p 4p hellip n are not relatively prime to n

there are np = pc-1 of them

and other numbers are relatively prime to n

Therefore

Euler Function

Given a number n how many numbers from 1 to n are relatively prime to n

Suppose

Then p 2p 3p 4p hellip n are not relatively prime to n there are np of them

Also q 2q 3q 4q hellip n are not relatively prime to n and there are nq of them

Other numbers are relatively prime to n

Therefore

The numbers pq 2pq 3pq hellip n are subtracted twice and there are npq of them

So the correct answer is

Euler Function

Given a number n how many numbers from 1 to n are relatively prime to n

Let

Let S be the set of numbers from 1 to n that are not relatively prime to n

Let Ai be the set of numbers that are a multiple of piS = A1 [ A2 [ hellip [ An

For the intersection of k sets say A1 A2 A3hellip Ak

then every number in A1 Aring A2 Aring hellip Aring Ak is a multiple of p1p2hellippk

then |A1 Aring A2 Aring hellip Aring Ak| = np1p2hellippk

Euler Function

Given a number n how many numbers from 1 to n are relatively prime to n

Let

Let S be the set of numbers from 1 to n that are not relatively prime to n

Let Ai be the set of numbers that are a multiple of piS = A1 [ A2 [ hellip [ An

|A1 Aring A2 Aring hellip Aring Ak| = np1p2hellippk

When r=3 (only 3 distinct factors)

|A1 [ A2 [ A3|

= np1 + np2 + np3

- np1p2 ndash np1p3 ndash np2p3 + np1p2p3

|A1 [ A2 [ A3| = |A1| + |A2| + |A3|

ndash |A1 Aring A2| ndash |A1 Aring A3| ndash |A2 Aring A3|

+ |A1 Aring A2 Aring A3|

= n(1-p1)(1-p2)(1-p3)

Euler Function

Given a number n how many numbers from 1 to n are relatively prime to n

Let

Let S be the set of numbers from 1 to n that are not relatively prime to n

Let Ai be the set of numbers that are a multiple of piS = A1 [ A2 [ hellip [ An

|A1 Aring A2 Aring hellip Aring Ak| = np1p2hellippk

sum of sizes of all single setsndash sum of sizes of all 2-set intersections+ sum of sizes of all 3-set intersectionsndash sum of sizes of all 4-set intersectionshellip+ (ndash1)n+1 times sum of sizes of intersections of

n sets

|A1 [ A2 [ A3 [ hellip [ An|

|S| = |A1 [ A2 [ hellip [ An|

= n(1-p1)(1-p2)hellip(1-pn)

calculationshellip

  • Modular Arithmetic
  • PowerPoint Presentation
  • Slide 3
  • Slide 4
  • Slide 5
  • Slide 6
  • Slide 7
  • Slide 8
  • Slide 9
  • Slide 10
  • Slide 11
  • Slide 12
  • Slide 13
  • Slide 14
  • Slide 15
  • Slide 16
  • Slide 17
  • Slide 18
  • Slide 19
  • Slide 20
  • Slide 21
  • Slide 22
  • Slide 23
  • Slide 24
  • Slide 25
  • Slide 26
  • Slide 27
  • Slide 28
  • Slide 29
  • Slide 30
  • Slide 31
  • Slide 32
  • Slide 33
  • Slide 34
  • Slide 35
  • Slide 36
  • Slide 37
  • Slide 38
  • Slide 39
  • Slide 40
  • Slide 41
  • Slide 42
  • Slide 43
  • Slide 44
  • Slide 45
Page 23: Modular Arithmetic

Necessary Condition

Claim An integer k does not have an multiplicative inverse under modulo n

if k and n have a common factor gt= 2 (gcd(kn) gt= 2)

Proof

This claim says that for k to have a multiplicative inverse modulo n

then a necessary condition is that k and n do not have a common factor gt= 2

Suppose by contradiction that there is an inverse krsquo for k such that

krsquok = 1 (mod n)

Then krsquok = 1 + xn for some integer x

Since both k and n have a common factor say cgt=2

then k=ck1 and n=cn1 for some integers k1 and n1

So krsquock1 = 1 + xcn1

Then krsquok1 = 1c + xn1

This is a contradiction since the LHS is an integer but the RHS is not

Sufficient Condition

What about if gcd(kn)=1

Would k always have an multiplicative inverse under modulo n

For example gcd(37) = 1 3middot5 1 (mod 7)

gcd(89) = 1

gcd(411) = 1 4middot3 1 (mod 11)

8middot8 1 (mod 9)

It seems that there is always an inverse in such a case but why

gcd(89) = 1 8s + 9t = 1 for some integers s and t

8s = 1 ndash 9t

8s 1 (mod 9)gcd(89) = spc(89)

Theorem If gcd(kn)=1 then have krsquo such that

kmiddotkrsquo 1 (mod n)

Proof Since gcd(kn)=1 there exist s and t so that sk + tn = 1

So tn = 1 - sk

This means n | 1 ndash sk

This means that 1 ndash sk 0 (mod n)

This means that 1 sk (mod n)

So krsquo = s is an multiplicative inverse for k

Sufficient Condition

gcd(kn)=spc(kn)

The multiplicative inverse can be computed by the extended Euclidean algorithm

Corollary k has a multiplicative inverse mod n if and only if gcd(kn)=1

This Lecture

bull Fermatrsquos little theorem

Cancellation

There is no general cancellation in modular arithmetic

Note that (mod n) is very similar to =

If a b (mod n) then a+c b+c (mod n)

If a b (mod n) then ac bc (mod n)

However if ac bc (mod n)

it is not necessarily true that a b (mod n)

For example 4middot2 1middot2 (mod 6) but 4 1 (mod 6)

3middot4 1middot4 (mod 8) but 3 1 (mod 8)

4middot3 1middot3 (mod 9) but 4 1 (mod 9)

Observation In all the above examples c and n have a common factor

Cancellation

Claim Assume gcd(kn) = 1 If imiddotk jmiddotk (mod n) then i j (mod n)

For example multiplicative inverse always exists if n is a prime

Proof Since gcd(kn) = 1 there exists krsquo such that kkrsquo 1 (mod n)

imiddotk jmiddotk (mod n)

=gt imiddotkmiddotkrsquo jmiddotkmiddotkrsquo (mod n)

=gt i j (mod n)

Remarks (Optional) This makes arithmetic modulo prime a field

a structure that ldquobehaves likerdquo real numbers

Arithmetic modulo prime is very useful in coding theory

In particular when p is a prime amp k not a multiple of p then gcd(kp)=1

If i j (mod p) then imiddotk jmiddotk (mod p)

Therefore

k mod p 2k mod p hellip (p-1)k mod p

are all different numbers

Fermatrsquos Little Theorem

Claim 1 Assume gcd(kn) = 1 If imiddotk jmiddotk (mod n) then i j (mod n)

Claim 2 Assume gcd(kn) = 1 If i j (mod n) then imiddotk jmiddotk (mod n)

For example when p=7 and k=3

3 mod 7 = 3 2middot3 mod 7 = 6 3middot3 mod 7 = 2 4middot3 mod 7 = 5 5middot3 mod 7 = 1 6middot3 mod 7 = 4

Notice that in the above example every number from 1 to 6 appears exactly once

In particular when p is a prime amp k not a multiple of p then gcd(kp)=1

If i j (mod p) then imiddotk jmiddotk (mod p)

Therefore

k mod p 2k mod p hellip (p-1)k mod p

are all different numbers

Fermatrsquos Little Theorem

Each of ik mod p cannot be equal to 0 because p is a prime number

Let ci = ik mod p

So 1 lt= c1 lt= p-1 1 lt= c2 lt= p-1 hellip 1lt = cp-1 lt= p-1

By the above we know that c1c2hellipcp-2cp-1 are all different

So for each i from 1 to p-1 there is exactly one cj such that cj = i

Therefore we have

(k mod p)middot(2k mod p)middothellipmiddot((p-1)k mod p) = c1middotc2middothellipmiddotcp-2middotcp-1 = 1middot2middot3hellipmiddot(p-2)middot(p-1)

Fermatrsquos Little Theorem

1 kp-1 (mod p)

Theorem If p is prime amp k not a multiple of p

For example when p=5 k=4 we have kp-1 mod p = 44 mod 5 = 1

ldquoProofrdquo

4middot3middot2middot1 [(4 mod 5) (2middot4 mod 5) (3middot4 mod 5) (4middot4 mod 5)] (mod 5)

[4 middot (2middot4) middot (3middot4) middot (4middot4)] (mod 5)

[44 middot (1middot2middot3middot4)] (mod 5)

Since gcd(1middot2middot3middot4 5)=1 we can cancel 1middot2middot3middot4 on both sides

This implies

1 44 (mod 5)

By the previous slide or direct calculation

Fermatrsquos Little Theorem

1 kp-1 (mod p)

Theorem If p is prime amp k not a multiple of p

Proof

1middot2middotmiddotmiddot(p-1) (k mod p middot 2k mod pmiddothellipmiddot(p-1)k mod p) mod p

(kmiddot2k middotmiddotmiddot (p-1)k) mod p

(kp-1)middot1middot2 middotmiddotmiddot (p-1) (mod p)

So by cancelling 1middot2 middotmiddotmiddot (p-1) on both sides applying

Claim 1

(we can cancel them because gcd(1middot2 middotmiddotmiddot (p-1) p)=1)

we have

1 kp-1 (mod p)

By 2 slides before

By the multiplication rule

Wilsonrsquos Theorem

Theorem p is a prime if and only if

(p-1) -1 (mod p)

First we consider the easy direction

If p is not a prime assume p gt= 5 (for p=4 3 2 (mod 4) )

Then p=qr for some 2 lt= q lt p and 2 lt= r lt p

If q ne r then both q and r appear in (p-1)

and so (p-1) 0 (mod p)

If q = r then p = q2 gt 2q (since we assume p gt 5 and thus q gt 2)

then both q and 2q are in (p-1)

and so again (p-1) 0 (mod p)

Wilsonrsquos Theorem

Theorem p is a prime if and only if

(p-1) -1 (mod p)

To prove the more interesting direction first we need a lemma

Lemma If p is a prime number

x2 1 (mod p) if and only if x 1 (mod p) or x -1 (mod p)

Proof x2 1 (mod p)

iff p | x2 - 1

iff p | (x ndash 1)(x + 1)

iff p | (x ndash 1) or p | (x+1)

iff x 1 (mod p) or x -1 (mod p)

Lemma p prime and p|amiddotb iff p|a or p|b

Wilsonrsquos Theorem

Theorem p is a prime if and only if

(p-1) -1 (mod p)

Letrsquos get the proof idea by considering a concrete example

10

1middot2middot3middot4middot5middot6middot7middot8middot9middot10 mod 11

1middot10middot(2middot6)middot(3middot4)middot(5middot9)middot(7middot8) mod 11

1middot-1middot(1)middot(1)middot(1)middot(1) mod 11

-1 mod 11

Besides 1 and 10 the remaining numbers are paired up into multiplicative inverse

Wilsonrsquos Theorem

Theorem p is a prime if and only if

(p-1) -1 (mod p)

ProofSince p is a prime every number from 1 to p-1 has a multiplicative inverse

By the Lemma every number 2 lt= k lt= p-2 has an inverse krsquo with knekrsquo

Since p is odd the numbers from 2 to p-2 can be grouped into pairs

(a1b1)(a2b2)hellip(a(p-3)2b(p-3)2) so that aibi 1 (mod p)

Therefore (p-1) 1middot(p-1)middot2middot3middotmiddotmiddotmiddotmiddot(p-3)middot(p-2) (mod p)

1middot(p-1)middot(a1b1)middot(a2b2)middotmiddotmiddotmiddotmiddot(a(p-3)2b(p-3)2) (mod p)

1middot(-1)middot(1)middot(1)middotmiddotmiddotmiddotmiddot(1) (mod p)

-1 (mod p)

This Lecture

bull Eulerrsquos phi function

Inclusion-Exclusion (n sets)

What is the inclusion-exclusion formula for the union of n sets

sum of sizes of all single setsndash sum of sizes of all 2-set intersections+ sum of sizes of all 3-set intersectionsndash sum of sizes of all 4-set intersectionshellip+ (ndash1)n+1 times sum of sizes of intersections of all n

sets

1 2 nA A A

1

12 1

( 1)n

ki

S nk i S

S k

A

Inclusion-Exclusion (n sets)

Inclusion-Exclusion (n sets)

sum of sizes of all single setsndash sum of sizes of all 2-set intersections+ sum of sizes of all 3-set intersectionsndash sum of sizes of all 4-set intersectionshellip+ (ndash1)n+1 times sum of sizes of intersections of all n

sets

|A1 [ A2 [ A3 [ hellip [ An|

We want to show that every element is counted exactly once

Consider an element which belongs to exactly k sets say A1 A2 A3 hellip Ak

In the formula such an element is counted the following number of times

Therefore each element is counted exactly once and thus the formula is correct

Euler Function

Given a number n how many numbers from 1 to n are relatively prime to n

When n is a prime number

then every number from 1 to n-1 is relatively prime to n

and so

When n is a prime power

then p 2p 3p 4p hellip n are not relatively prime to n

there are np = pc-1 of them

and other numbers are relatively prime to n

Therefore

Euler Function

Given a number n how many numbers from 1 to n are relatively prime to n

Suppose

Then p 2p 3p 4p hellip n are not relatively prime to n there are np of them

Also q 2q 3q 4q hellip n are not relatively prime to n and there are nq of them

Other numbers are relatively prime to n

Therefore

The numbers pq 2pq 3pq hellip n are subtracted twice and there are npq of them

So the correct answer is

Euler Function

Given a number n how many numbers from 1 to n are relatively prime to n

Let

Let S be the set of numbers from 1 to n that are not relatively prime to n

Let Ai be the set of numbers that are a multiple of piS = A1 [ A2 [ hellip [ An

For the intersection of k sets say A1 A2 A3hellip Ak

then every number in A1 Aring A2 Aring hellip Aring Ak is a multiple of p1p2hellippk

then |A1 Aring A2 Aring hellip Aring Ak| = np1p2hellippk

Euler Function

Given a number n how many numbers from 1 to n are relatively prime to n

Let

Let S be the set of numbers from 1 to n that are not relatively prime to n

Let Ai be the set of numbers that are a multiple of piS = A1 [ A2 [ hellip [ An

|A1 Aring A2 Aring hellip Aring Ak| = np1p2hellippk

When r=3 (only 3 distinct factors)

|A1 [ A2 [ A3|

= np1 + np2 + np3

- np1p2 ndash np1p3 ndash np2p3 + np1p2p3

|A1 [ A2 [ A3| = |A1| + |A2| + |A3|

ndash |A1 Aring A2| ndash |A1 Aring A3| ndash |A2 Aring A3|

+ |A1 Aring A2 Aring A3|

= n(1-p1)(1-p2)(1-p3)

Euler Function

Given a number n how many numbers from 1 to n are relatively prime to n

Let

Let S be the set of numbers from 1 to n that are not relatively prime to n

Let Ai be the set of numbers that are a multiple of piS = A1 [ A2 [ hellip [ An

|A1 Aring A2 Aring hellip Aring Ak| = np1p2hellippk

sum of sizes of all single setsndash sum of sizes of all 2-set intersections+ sum of sizes of all 3-set intersectionsndash sum of sizes of all 4-set intersectionshellip+ (ndash1)n+1 times sum of sizes of intersections of

n sets

|A1 [ A2 [ A3 [ hellip [ An|

|S| = |A1 [ A2 [ hellip [ An|

= n(1-p1)(1-p2)hellip(1-pn)

calculationshellip

  • Modular Arithmetic
  • PowerPoint Presentation
  • Slide 3
  • Slide 4
  • Slide 5
  • Slide 6
  • Slide 7
  • Slide 8
  • Slide 9
  • Slide 10
  • Slide 11
  • Slide 12
  • Slide 13
  • Slide 14
  • Slide 15
  • Slide 16
  • Slide 17
  • Slide 18
  • Slide 19
  • Slide 20
  • Slide 21
  • Slide 22
  • Slide 23
  • Slide 24
  • Slide 25
  • Slide 26
  • Slide 27
  • Slide 28
  • Slide 29
  • Slide 30
  • Slide 31
  • Slide 32
  • Slide 33
  • Slide 34
  • Slide 35
  • Slide 36
  • Slide 37
  • Slide 38
  • Slide 39
  • Slide 40
  • Slide 41
  • Slide 42
  • Slide 43
  • Slide 44
  • Slide 45
Page 24: Modular Arithmetic

Sufficient Condition

What about if gcd(kn)=1

Would k always have an multiplicative inverse under modulo n

For example gcd(37) = 1 3middot5 1 (mod 7)

gcd(89) = 1

gcd(411) = 1 4middot3 1 (mod 11)

8middot8 1 (mod 9)

It seems that there is always an inverse in such a case but why

gcd(89) = 1 8s + 9t = 1 for some integers s and t

8s = 1 ndash 9t

8s 1 (mod 9)gcd(89) = spc(89)

Theorem If gcd(kn)=1 then have krsquo such that

kmiddotkrsquo 1 (mod n)

Proof Since gcd(kn)=1 there exist s and t so that sk + tn = 1

So tn = 1 - sk

This means n | 1 ndash sk

This means that 1 ndash sk 0 (mod n)

This means that 1 sk (mod n)

So krsquo = s is an multiplicative inverse for k

Sufficient Condition

gcd(kn)=spc(kn)

The multiplicative inverse can be computed by the extended Euclidean algorithm

Corollary k has a multiplicative inverse mod n if and only if gcd(kn)=1

This Lecture

bull Fermatrsquos little theorem

Cancellation

There is no general cancellation in modular arithmetic

Note that (mod n) is very similar to =

If a b (mod n) then a+c b+c (mod n)

If a b (mod n) then ac bc (mod n)

However if ac bc (mod n)

it is not necessarily true that a b (mod n)

For example 4middot2 1middot2 (mod 6) but 4 1 (mod 6)

3middot4 1middot4 (mod 8) but 3 1 (mod 8)

4middot3 1middot3 (mod 9) but 4 1 (mod 9)

Observation In all the above examples c and n have a common factor

Cancellation

Claim Assume gcd(kn) = 1 If imiddotk jmiddotk (mod n) then i j (mod n)

For example multiplicative inverse always exists if n is a prime

Proof Since gcd(kn) = 1 there exists krsquo such that kkrsquo 1 (mod n)

imiddotk jmiddotk (mod n)

=gt imiddotkmiddotkrsquo jmiddotkmiddotkrsquo (mod n)

=gt i j (mod n)

Remarks (Optional) This makes arithmetic modulo prime a field

a structure that ldquobehaves likerdquo real numbers

Arithmetic modulo prime is very useful in coding theory

In particular when p is a prime amp k not a multiple of p then gcd(kp)=1

If i j (mod p) then imiddotk jmiddotk (mod p)

Therefore

k mod p 2k mod p hellip (p-1)k mod p

are all different numbers

Fermatrsquos Little Theorem

Claim 1 Assume gcd(kn) = 1 If imiddotk jmiddotk (mod n) then i j (mod n)

Claim 2 Assume gcd(kn) = 1 If i j (mod n) then imiddotk jmiddotk (mod n)

For example when p=7 and k=3

3 mod 7 = 3 2middot3 mod 7 = 6 3middot3 mod 7 = 2 4middot3 mod 7 = 5 5middot3 mod 7 = 1 6middot3 mod 7 = 4

Notice that in the above example every number from 1 to 6 appears exactly once

In particular when p is a prime amp k not a multiple of p then gcd(kp)=1

If i j (mod p) then imiddotk jmiddotk (mod p)

Therefore

k mod p 2k mod p hellip (p-1)k mod p

are all different numbers

Fermatrsquos Little Theorem

Each of ik mod p cannot be equal to 0 because p is a prime number

Let ci = ik mod p

So 1 lt= c1 lt= p-1 1 lt= c2 lt= p-1 hellip 1lt = cp-1 lt= p-1

By the above we know that c1c2hellipcp-2cp-1 are all different

So for each i from 1 to p-1 there is exactly one cj such that cj = i

Therefore we have

(k mod p)middot(2k mod p)middothellipmiddot((p-1)k mod p) = c1middotc2middothellipmiddotcp-2middotcp-1 = 1middot2middot3hellipmiddot(p-2)middot(p-1)

Fermatrsquos Little Theorem

1 kp-1 (mod p)

Theorem If p is prime amp k not a multiple of p

For example when p=5 k=4 we have kp-1 mod p = 44 mod 5 = 1

ldquoProofrdquo

4middot3middot2middot1 [(4 mod 5) (2middot4 mod 5) (3middot4 mod 5) (4middot4 mod 5)] (mod 5)

[4 middot (2middot4) middot (3middot4) middot (4middot4)] (mod 5)

[44 middot (1middot2middot3middot4)] (mod 5)

Since gcd(1middot2middot3middot4 5)=1 we can cancel 1middot2middot3middot4 on both sides

This implies

1 44 (mod 5)

By the previous slide or direct calculation

Fermatrsquos Little Theorem

1 kp-1 (mod p)

Theorem If p is prime amp k not a multiple of p

Proof

1middot2middotmiddotmiddot(p-1) (k mod p middot 2k mod pmiddothellipmiddot(p-1)k mod p) mod p

(kmiddot2k middotmiddotmiddot (p-1)k) mod p

(kp-1)middot1middot2 middotmiddotmiddot (p-1) (mod p)

So by cancelling 1middot2 middotmiddotmiddot (p-1) on both sides applying

Claim 1

(we can cancel them because gcd(1middot2 middotmiddotmiddot (p-1) p)=1)

we have

1 kp-1 (mod p)

By 2 slides before

By the multiplication rule

Wilsonrsquos Theorem

Theorem p is a prime if and only if

(p-1) -1 (mod p)

First we consider the easy direction

If p is not a prime assume p gt= 5 (for p=4 3 2 (mod 4) )

Then p=qr for some 2 lt= q lt p and 2 lt= r lt p

If q ne r then both q and r appear in (p-1)

and so (p-1) 0 (mod p)

If q = r then p = q2 gt 2q (since we assume p gt 5 and thus q gt 2)

then both q and 2q are in (p-1)

and so again (p-1) 0 (mod p)

Wilsonrsquos Theorem

Theorem p is a prime if and only if

(p-1) -1 (mod p)

To prove the more interesting direction first we need a lemma

Lemma If p is a prime number

x2 1 (mod p) if and only if x 1 (mod p) or x -1 (mod p)

Proof x2 1 (mod p)

iff p | x2 - 1

iff p | (x ndash 1)(x + 1)

iff p | (x ndash 1) or p | (x+1)

iff x 1 (mod p) or x -1 (mod p)

Lemma p prime and p|amiddotb iff p|a or p|b

Wilsonrsquos Theorem

Theorem p is a prime if and only if

(p-1) -1 (mod p)

Letrsquos get the proof idea by considering a concrete example

10

1middot2middot3middot4middot5middot6middot7middot8middot9middot10 mod 11

1middot10middot(2middot6)middot(3middot4)middot(5middot9)middot(7middot8) mod 11

1middot-1middot(1)middot(1)middot(1)middot(1) mod 11

-1 mod 11

Besides 1 and 10 the remaining numbers are paired up into multiplicative inverse

Wilsonrsquos Theorem

Theorem p is a prime if and only if

(p-1) -1 (mod p)

ProofSince p is a prime every number from 1 to p-1 has a multiplicative inverse

By the Lemma every number 2 lt= k lt= p-2 has an inverse krsquo with knekrsquo

Since p is odd the numbers from 2 to p-2 can be grouped into pairs

(a1b1)(a2b2)hellip(a(p-3)2b(p-3)2) so that aibi 1 (mod p)

Therefore (p-1) 1middot(p-1)middot2middot3middotmiddotmiddotmiddotmiddot(p-3)middot(p-2) (mod p)

1middot(p-1)middot(a1b1)middot(a2b2)middotmiddotmiddotmiddotmiddot(a(p-3)2b(p-3)2) (mod p)

1middot(-1)middot(1)middot(1)middotmiddotmiddotmiddotmiddot(1) (mod p)

-1 (mod p)

This Lecture

bull Eulerrsquos phi function

Inclusion-Exclusion (n sets)

What is the inclusion-exclusion formula for the union of n sets

sum of sizes of all single setsndash sum of sizes of all 2-set intersections+ sum of sizes of all 3-set intersectionsndash sum of sizes of all 4-set intersectionshellip+ (ndash1)n+1 times sum of sizes of intersections of all n

sets

1 2 nA A A

1

12 1

( 1)n

ki

S nk i S

S k

A

Inclusion-Exclusion (n sets)

Inclusion-Exclusion (n sets)

sum of sizes of all single setsndash sum of sizes of all 2-set intersections+ sum of sizes of all 3-set intersectionsndash sum of sizes of all 4-set intersectionshellip+ (ndash1)n+1 times sum of sizes of intersections of all n

sets

|A1 [ A2 [ A3 [ hellip [ An|

We want to show that every element is counted exactly once

Consider an element which belongs to exactly k sets say A1 A2 A3 hellip Ak

In the formula such an element is counted the following number of times

Therefore each element is counted exactly once and thus the formula is correct

Euler Function

Given a number n how many numbers from 1 to n are relatively prime to n

When n is a prime number

then every number from 1 to n-1 is relatively prime to n

and so

When n is a prime power

then p 2p 3p 4p hellip n are not relatively prime to n

there are np = pc-1 of them

and other numbers are relatively prime to n

Therefore

Euler Function

Given a number n how many numbers from 1 to n are relatively prime to n

Suppose

Then p 2p 3p 4p hellip n are not relatively prime to n there are np of them

Also q 2q 3q 4q hellip n are not relatively prime to n and there are nq of them

Other numbers are relatively prime to n

Therefore

The numbers pq 2pq 3pq hellip n are subtracted twice and there are npq of them

So the correct answer is

Euler Function

Given a number n how many numbers from 1 to n are relatively prime to n

Let

Let S be the set of numbers from 1 to n that are not relatively prime to n

Let Ai be the set of numbers that are a multiple of piS = A1 [ A2 [ hellip [ An

For the intersection of k sets say A1 A2 A3hellip Ak

then every number in A1 Aring A2 Aring hellip Aring Ak is a multiple of p1p2hellippk

then |A1 Aring A2 Aring hellip Aring Ak| = np1p2hellippk

Euler Function

Given a number n how many numbers from 1 to n are relatively prime to n

Let

Let S be the set of numbers from 1 to n that are not relatively prime to n

Let Ai be the set of numbers that are a multiple of piS = A1 [ A2 [ hellip [ An

|A1 Aring A2 Aring hellip Aring Ak| = np1p2hellippk

When r=3 (only 3 distinct factors)

|A1 [ A2 [ A3|

= np1 + np2 + np3

- np1p2 ndash np1p3 ndash np2p3 + np1p2p3

|A1 [ A2 [ A3| = |A1| + |A2| + |A3|

ndash |A1 Aring A2| ndash |A1 Aring A3| ndash |A2 Aring A3|

+ |A1 Aring A2 Aring A3|

= n(1-p1)(1-p2)(1-p3)

Euler Function

Given a number n how many numbers from 1 to n are relatively prime to n

Let

Let S be the set of numbers from 1 to n that are not relatively prime to n

Let Ai be the set of numbers that are a multiple of piS = A1 [ A2 [ hellip [ An

|A1 Aring A2 Aring hellip Aring Ak| = np1p2hellippk

sum of sizes of all single setsndash sum of sizes of all 2-set intersections+ sum of sizes of all 3-set intersectionsndash sum of sizes of all 4-set intersectionshellip+ (ndash1)n+1 times sum of sizes of intersections of

n sets

|A1 [ A2 [ A3 [ hellip [ An|

|S| = |A1 [ A2 [ hellip [ An|

= n(1-p1)(1-p2)hellip(1-pn)

calculationshellip

  • Modular Arithmetic
  • PowerPoint Presentation
  • Slide 3
  • Slide 4
  • Slide 5
  • Slide 6
  • Slide 7
  • Slide 8
  • Slide 9
  • Slide 10
  • Slide 11
  • Slide 12
  • Slide 13
  • Slide 14
  • Slide 15
  • Slide 16
  • Slide 17
  • Slide 18
  • Slide 19
  • Slide 20
  • Slide 21
  • Slide 22
  • Slide 23
  • Slide 24
  • Slide 25
  • Slide 26
  • Slide 27
  • Slide 28
  • Slide 29
  • Slide 30
  • Slide 31
  • Slide 32
  • Slide 33
  • Slide 34
  • Slide 35
  • Slide 36
  • Slide 37
  • Slide 38
  • Slide 39
  • Slide 40
  • Slide 41
  • Slide 42
  • Slide 43
  • Slide 44
  • Slide 45
Page 25: Modular Arithmetic

Theorem If gcd(kn)=1 then have krsquo such that

kmiddotkrsquo 1 (mod n)

Proof Since gcd(kn)=1 there exist s and t so that sk + tn = 1

So tn = 1 - sk

This means n | 1 ndash sk

This means that 1 ndash sk 0 (mod n)

This means that 1 sk (mod n)

So krsquo = s is an multiplicative inverse for k

Sufficient Condition

gcd(kn)=spc(kn)

The multiplicative inverse can be computed by the extended Euclidean algorithm

Corollary k has a multiplicative inverse mod n if and only if gcd(kn)=1

This Lecture

bull Fermatrsquos little theorem

Cancellation

There is no general cancellation in modular arithmetic

Note that (mod n) is very similar to =

If a b (mod n) then a+c b+c (mod n)

If a b (mod n) then ac bc (mod n)

However if ac bc (mod n)

it is not necessarily true that a b (mod n)

For example 4middot2 1middot2 (mod 6) but 4 1 (mod 6)

3middot4 1middot4 (mod 8) but 3 1 (mod 8)

4middot3 1middot3 (mod 9) but 4 1 (mod 9)

Observation In all the above examples c and n have a common factor

Cancellation

Claim Assume gcd(kn) = 1 If imiddotk jmiddotk (mod n) then i j (mod n)

For example multiplicative inverse always exists if n is a prime

Proof Since gcd(kn) = 1 there exists krsquo such that kkrsquo 1 (mod n)

imiddotk jmiddotk (mod n)

=gt imiddotkmiddotkrsquo jmiddotkmiddotkrsquo (mod n)

=gt i j (mod n)

Remarks (Optional) This makes arithmetic modulo prime a field

a structure that ldquobehaves likerdquo real numbers

Arithmetic modulo prime is very useful in coding theory

In particular when p is a prime amp k not a multiple of p then gcd(kp)=1

If i j (mod p) then imiddotk jmiddotk (mod p)

Therefore

k mod p 2k mod p hellip (p-1)k mod p

are all different numbers

Fermatrsquos Little Theorem

Claim 1 Assume gcd(kn) = 1 If imiddotk jmiddotk (mod n) then i j (mod n)

Claim 2 Assume gcd(kn) = 1 If i j (mod n) then imiddotk jmiddotk (mod n)

For example when p=7 and k=3

3 mod 7 = 3 2middot3 mod 7 = 6 3middot3 mod 7 = 2 4middot3 mod 7 = 5 5middot3 mod 7 = 1 6middot3 mod 7 = 4

Notice that in the above example every number from 1 to 6 appears exactly once

In particular when p is a prime amp k not a multiple of p then gcd(kp)=1

If i j (mod p) then imiddotk jmiddotk (mod p)

Therefore

k mod p 2k mod p hellip (p-1)k mod p

are all different numbers

Fermatrsquos Little Theorem

Each of ik mod p cannot be equal to 0 because p is a prime number

Let ci = ik mod p

So 1 lt= c1 lt= p-1 1 lt= c2 lt= p-1 hellip 1lt = cp-1 lt= p-1

By the above we know that c1c2hellipcp-2cp-1 are all different

So for each i from 1 to p-1 there is exactly one cj such that cj = i

Therefore we have

(k mod p)middot(2k mod p)middothellipmiddot((p-1)k mod p) = c1middotc2middothellipmiddotcp-2middotcp-1 = 1middot2middot3hellipmiddot(p-2)middot(p-1)

Fermatrsquos Little Theorem

1 kp-1 (mod p)

Theorem If p is prime amp k not a multiple of p

For example when p=5 k=4 we have kp-1 mod p = 44 mod 5 = 1

ldquoProofrdquo

4middot3middot2middot1 [(4 mod 5) (2middot4 mod 5) (3middot4 mod 5) (4middot4 mod 5)] (mod 5)

[4 middot (2middot4) middot (3middot4) middot (4middot4)] (mod 5)

[44 middot (1middot2middot3middot4)] (mod 5)

Since gcd(1middot2middot3middot4 5)=1 we can cancel 1middot2middot3middot4 on both sides

This implies

1 44 (mod 5)

By the previous slide or direct calculation

Fermatrsquos Little Theorem

1 kp-1 (mod p)

Theorem If p is prime amp k not a multiple of p

Proof

1middot2middotmiddotmiddot(p-1) (k mod p middot 2k mod pmiddothellipmiddot(p-1)k mod p) mod p

(kmiddot2k middotmiddotmiddot (p-1)k) mod p

(kp-1)middot1middot2 middotmiddotmiddot (p-1) (mod p)

So by cancelling 1middot2 middotmiddotmiddot (p-1) on both sides applying

Claim 1

(we can cancel them because gcd(1middot2 middotmiddotmiddot (p-1) p)=1)

we have

1 kp-1 (mod p)

By 2 slides before

By the multiplication rule

Wilsonrsquos Theorem

Theorem p is a prime if and only if

(p-1) -1 (mod p)

First we consider the easy direction

If p is not a prime assume p gt= 5 (for p=4 3 2 (mod 4) )

Then p=qr for some 2 lt= q lt p and 2 lt= r lt p

If q ne r then both q and r appear in (p-1)

and so (p-1) 0 (mod p)

If q = r then p = q2 gt 2q (since we assume p gt 5 and thus q gt 2)

then both q and 2q are in (p-1)

and so again (p-1) 0 (mod p)

Wilsonrsquos Theorem

Theorem p is a prime if and only if

(p-1) -1 (mod p)

To prove the more interesting direction first we need a lemma

Lemma If p is a prime number

x2 1 (mod p) if and only if x 1 (mod p) or x -1 (mod p)

Proof x2 1 (mod p)

iff p | x2 - 1

iff p | (x ndash 1)(x + 1)

iff p | (x ndash 1) or p | (x+1)

iff x 1 (mod p) or x -1 (mod p)

Lemma p prime and p|amiddotb iff p|a or p|b

Wilsonrsquos Theorem

Theorem p is a prime if and only if

(p-1) -1 (mod p)

Letrsquos get the proof idea by considering a concrete example

10

1middot2middot3middot4middot5middot6middot7middot8middot9middot10 mod 11

1middot10middot(2middot6)middot(3middot4)middot(5middot9)middot(7middot8) mod 11

1middot-1middot(1)middot(1)middot(1)middot(1) mod 11

-1 mod 11

Besides 1 and 10 the remaining numbers are paired up into multiplicative inverse

Wilsonrsquos Theorem

Theorem p is a prime if and only if

(p-1) -1 (mod p)

ProofSince p is a prime every number from 1 to p-1 has a multiplicative inverse

By the Lemma every number 2 lt= k lt= p-2 has an inverse krsquo with knekrsquo

Since p is odd the numbers from 2 to p-2 can be grouped into pairs

(a1b1)(a2b2)hellip(a(p-3)2b(p-3)2) so that aibi 1 (mod p)

Therefore (p-1) 1middot(p-1)middot2middot3middotmiddotmiddotmiddotmiddot(p-3)middot(p-2) (mod p)

1middot(p-1)middot(a1b1)middot(a2b2)middotmiddotmiddotmiddotmiddot(a(p-3)2b(p-3)2) (mod p)

1middot(-1)middot(1)middot(1)middotmiddotmiddotmiddotmiddot(1) (mod p)

-1 (mod p)

This Lecture

bull Eulerrsquos phi function

Inclusion-Exclusion (n sets)

What is the inclusion-exclusion formula for the union of n sets

sum of sizes of all single setsndash sum of sizes of all 2-set intersections+ sum of sizes of all 3-set intersectionsndash sum of sizes of all 4-set intersectionshellip+ (ndash1)n+1 times sum of sizes of intersections of all n

sets

1 2 nA A A

1

12 1

( 1)n

ki

S nk i S

S k

A

Inclusion-Exclusion (n sets)

Inclusion-Exclusion (n sets)

sum of sizes of all single setsndash sum of sizes of all 2-set intersections+ sum of sizes of all 3-set intersectionsndash sum of sizes of all 4-set intersectionshellip+ (ndash1)n+1 times sum of sizes of intersections of all n

sets

|A1 [ A2 [ A3 [ hellip [ An|

We want to show that every element is counted exactly once

Consider an element which belongs to exactly k sets say A1 A2 A3 hellip Ak

In the formula such an element is counted the following number of times

Therefore each element is counted exactly once and thus the formula is correct

Euler Function

Given a number n how many numbers from 1 to n are relatively prime to n

When n is a prime number

then every number from 1 to n-1 is relatively prime to n

and so

When n is a prime power

then p 2p 3p 4p hellip n are not relatively prime to n

there are np = pc-1 of them

and other numbers are relatively prime to n

Therefore

Euler Function

Given a number n how many numbers from 1 to n are relatively prime to n

Suppose

Then p 2p 3p 4p hellip n are not relatively prime to n there are np of them

Also q 2q 3q 4q hellip n are not relatively prime to n and there are nq of them

Other numbers are relatively prime to n

Therefore

The numbers pq 2pq 3pq hellip n are subtracted twice and there are npq of them

So the correct answer is

Euler Function

Given a number n how many numbers from 1 to n are relatively prime to n

Let

Let S be the set of numbers from 1 to n that are not relatively prime to n

Let Ai be the set of numbers that are a multiple of piS = A1 [ A2 [ hellip [ An

For the intersection of k sets say A1 A2 A3hellip Ak

then every number in A1 Aring A2 Aring hellip Aring Ak is a multiple of p1p2hellippk

then |A1 Aring A2 Aring hellip Aring Ak| = np1p2hellippk

Euler Function

Given a number n how many numbers from 1 to n are relatively prime to n

Let

Let S be the set of numbers from 1 to n that are not relatively prime to n

Let Ai be the set of numbers that are a multiple of piS = A1 [ A2 [ hellip [ An

|A1 Aring A2 Aring hellip Aring Ak| = np1p2hellippk

When r=3 (only 3 distinct factors)

|A1 [ A2 [ A3|

= np1 + np2 + np3

- np1p2 ndash np1p3 ndash np2p3 + np1p2p3

|A1 [ A2 [ A3| = |A1| + |A2| + |A3|

ndash |A1 Aring A2| ndash |A1 Aring A3| ndash |A2 Aring A3|

+ |A1 Aring A2 Aring A3|

= n(1-p1)(1-p2)(1-p3)

Euler Function

Given a number n how many numbers from 1 to n are relatively prime to n

Let

Let S be the set of numbers from 1 to n that are not relatively prime to n

Let Ai be the set of numbers that are a multiple of piS = A1 [ A2 [ hellip [ An

|A1 Aring A2 Aring hellip Aring Ak| = np1p2hellippk

sum of sizes of all single setsndash sum of sizes of all 2-set intersections+ sum of sizes of all 3-set intersectionsndash sum of sizes of all 4-set intersectionshellip+ (ndash1)n+1 times sum of sizes of intersections of

n sets

|A1 [ A2 [ A3 [ hellip [ An|

|S| = |A1 [ A2 [ hellip [ An|

= n(1-p1)(1-p2)hellip(1-pn)

calculationshellip

  • Modular Arithmetic
  • PowerPoint Presentation
  • Slide 3
  • Slide 4
  • Slide 5
  • Slide 6
  • Slide 7
  • Slide 8
  • Slide 9
  • Slide 10
  • Slide 11
  • Slide 12
  • Slide 13
  • Slide 14
  • Slide 15
  • Slide 16
  • Slide 17
  • Slide 18
  • Slide 19
  • Slide 20
  • Slide 21
  • Slide 22
  • Slide 23
  • Slide 24
  • Slide 25
  • Slide 26
  • Slide 27
  • Slide 28
  • Slide 29
  • Slide 30
  • Slide 31
  • Slide 32
  • Slide 33
  • Slide 34
  • Slide 35
  • Slide 36
  • Slide 37
  • Slide 38
  • Slide 39
  • Slide 40
  • Slide 41
  • Slide 42
  • Slide 43
  • Slide 44
  • Slide 45
Page 26: Modular Arithmetic

This Lecture

bull Fermatrsquos little theorem

Cancellation

There is no general cancellation in modular arithmetic

Note that (mod n) is very similar to =

If a b (mod n) then a+c b+c (mod n)

If a b (mod n) then ac bc (mod n)

However if ac bc (mod n)

it is not necessarily true that a b (mod n)

For example 4middot2 1middot2 (mod 6) but 4 1 (mod 6)

3middot4 1middot4 (mod 8) but 3 1 (mod 8)

4middot3 1middot3 (mod 9) but 4 1 (mod 9)

Observation In all the above examples c and n have a common factor

Cancellation

Claim Assume gcd(kn) = 1 If imiddotk jmiddotk (mod n) then i j (mod n)

For example multiplicative inverse always exists if n is a prime

Proof Since gcd(kn) = 1 there exists krsquo such that kkrsquo 1 (mod n)

imiddotk jmiddotk (mod n)

=gt imiddotkmiddotkrsquo jmiddotkmiddotkrsquo (mod n)

=gt i j (mod n)

Remarks (Optional) This makes arithmetic modulo prime a field

a structure that ldquobehaves likerdquo real numbers

Arithmetic modulo prime is very useful in coding theory

In particular when p is a prime amp k not a multiple of p then gcd(kp)=1

If i j (mod p) then imiddotk jmiddotk (mod p)

Therefore

k mod p 2k mod p hellip (p-1)k mod p

are all different numbers

Fermatrsquos Little Theorem

Claim 1 Assume gcd(kn) = 1 If imiddotk jmiddotk (mod n) then i j (mod n)

Claim 2 Assume gcd(kn) = 1 If i j (mod n) then imiddotk jmiddotk (mod n)

For example when p=7 and k=3

3 mod 7 = 3 2middot3 mod 7 = 6 3middot3 mod 7 = 2 4middot3 mod 7 = 5 5middot3 mod 7 = 1 6middot3 mod 7 = 4

Notice that in the above example every number from 1 to 6 appears exactly once

In particular when p is a prime amp k not a multiple of p then gcd(kp)=1

If i j (mod p) then imiddotk jmiddotk (mod p)

Therefore

k mod p 2k mod p hellip (p-1)k mod p

are all different numbers

Fermatrsquos Little Theorem

Each of ik mod p cannot be equal to 0 because p is a prime number

Let ci = ik mod p

So 1 lt= c1 lt= p-1 1 lt= c2 lt= p-1 hellip 1lt = cp-1 lt= p-1

By the above we know that c1c2hellipcp-2cp-1 are all different

So for each i from 1 to p-1 there is exactly one cj such that cj = i

Therefore we have

(k mod p)middot(2k mod p)middothellipmiddot((p-1)k mod p) = c1middotc2middothellipmiddotcp-2middotcp-1 = 1middot2middot3hellipmiddot(p-2)middot(p-1)

Fermatrsquos Little Theorem

1 kp-1 (mod p)

Theorem If p is prime amp k not a multiple of p

For example when p=5 k=4 we have kp-1 mod p = 44 mod 5 = 1

ldquoProofrdquo

4middot3middot2middot1 [(4 mod 5) (2middot4 mod 5) (3middot4 mod 5) (4middot4 mod 5)] (mod 5)

[4 middot (2middot4) middot (3middot4) middot (4middot4)] (mod 5)

[44 middot (1middot2middot3middot4)] (mod 5)

Since gcd(1middot2middot3middot4 5)=1 we can cancel 1middot2middot3middot4 on both sides

This implies

1 44 (mod 5)

By the previous slide or direct calculation

Fermatrsquos Little Theorem

1 kp-1 (mod p)

Theorem If p is prime amp k not a multiple of p

Proof

1middot2middotmiddotmiddot(p-1) (k mod p middot 2k mod pmiddothellipmiddot(p-1)k mod p) mod p

(kmiddot2k middotmiddotmiddot (p-1)k) mod p

(kp-1)middot1middot2 middotmiddotmiddot (p-1) (mod p)

So by cancelling 1middot2 middotmiddotmiddot (p-1) on both sides applying

Claim 1

(we can cancel them because gcd(1middot2 middotmiddotmiddot (p-1) p)=1)

we have

1 kp-1 (mod p)

By 2 slides before

By the multiplication rule

Wilsonrsquos Theorem

Theorem p is a prime if and only if

(p-1) -1 (mod p)

First we consider the easy direction

If p is not a prime assume p gt= 5 (for p=4 3 2 (mod 4) )

Then p=qr for some 2 lt= q lt p and 2 lt= r lt p

If q ne r then both q and r appear in (p-1)

and so (p-1) 0 (mod p)

If q = r then p = q2 gt 2q (since we assume p gt 5 and thus q gt 2)

then both q and 2q are in (p-1)

and so again (p-1) 0 (mod p)

Wilsonrsquos Theorem

Theorem p is a prime if and only if

(p-1) -1 (mod p)

To prove the more interesting direction first we need a lemma

Lemma If p is a prime number

x2 1 (mod p) if and only if x 1 (mod p) or x -1 (mod p)

Proof x2 1 (mod p)

iff p | x2 - 1

iff p | (x ndash 1)(x + 1)

iff p | (x ndash 1) or p | (x+1)

iff x 1 (mod p) or x -1 (mod p)

Lemma p prime and p|amiddotb iff p|a or p|b

Wilsonrsquos Theorem

Theorem p is a prime if and only if

(p-1) -1 (mod p)

Letrsquos get the proof idea by considering a concrete example

10

1middot2middot3middot4middot5middot6middot7middot8middot9middot10 mod 11

1middot10middot(2middot6)middot(3middot4)middot(5middot9)middot(7middot8) mod 11

1middot-1middot(1)middot(1)middot(1)middot(1) mod 11

-1 mod 11

Besides 1 and 10 the remaining numbers are paired up into multiplicative inverse

Wilsonrsquos Theorem

Theorem p is a prime if and only if

(p-1) -1 (mod p)

ProofSince p is a prime every number from 1 to p-1 has a multiplicative inverse

By the Lemma every number 2 lt= k lt= p-2 has an inverse krsquo with knekrsquo

Since p is odd the numbers from 2 to p-2 can be grouped into pairs

(a1b1)(a2b2)hellip(a(p-3)2b(p-3)2) so that aibi 1 (mod p)

Therefore (p-1) 1middot(p-1)middot2middot3middotmiddotmiddotmiddotmiddot(p-3)middot(p-2) (mod p)

1middot(p-1)middot(a1b1)middot(a2b2)middotmiddotmiddotmiddotmiddot(a(p-3)2b(p-3)2) (mod p)

1middot(-1)middot(1)middot(1)middotmiddotmiddotmiddotmiddot(1) (mod p)

-1 (mod p)

This Lecture

bull Eulerrsquos phi function

Inclusion-Exclusion (n sets)

What is the inclusion-exclusion formula for the union of n sets

sum of sizes of all single setsndash sum of sizes of all 2-set intersections+ sum of sizes of all 3-set intersectionsndash sum of sizes of all 4-set intersectionshellip+ (ndash1)n+1 times sum of sizes of intersections of all n

sets

1 2 nA A A

1

12 1

( 1)n

ki

S nk i S

S k

A

Inclusion-Exclusion (n sets)

Inclusion-Exclusion (n sets)

sum of sizes of all single setsndash sum of sizes of all 2-set intersections+ sum of sizes of all 3-set intersectionsndash sum of sizes of all 4-set intersectionshellip+ (ndash1)n+1 times sum of sizes of intersections of all n

sets

|A1 [ A2 [ A3 [ hellip [ An|

We want to show that every element is counted exactly once

Consider an element which belongs to exactly k sets say A1 A2 A3 hellip Ak

In the formula such an element is counted the following number of times

Therefore each element is counted exactly once and thus the formula is correct

Euler Function

Given a number n how many numbers from 1 to n are relatively prime to n

When n is a prime number

then every number from 1 to n-1 is relatively prime to n

and so

When n is a prime power

then p 2p 3p 4p hellip n are not relatively prime to n

there are np = pc-1 of them

and other numbers are relatively prime to n

Therefore

Euler Function

Given a number n how many numbers from 1 to n are relatively prime to n

Suppose

Then p 2p 3p 4p hellip n are not relatively prime to n there are np of them

Also q 2q 3q 4q hellip n are not relatively prime to n and there are nq of them

Other numbers are relatively prime to n

Therefore

The numbers pq 2pq 3pq hellip n are subtracted twice and there are npq of them

So the correct answer is

Euler Function

Given a number n how many numbers from 1 to n are relatively prime to n

Let

Let S be the set of numbers from 1 to n that are not relatively prime to n

Let Ai be the set of numbers that are a multiple of piS = A1 [ A2 [ hellip [ An

For the intersection of k sets say A1 A2 A3hellip Ak

then every number in A1 Aring A2 Aring hellip Aring Ak is a multiple of p1p2hellippk

then |A1 Aring A2 Aring hellip Aring Ak| = np1p2hellippk

Euler Function

Given a number n how many numbers from 1 to n are relatively prime to n

Let

Let S be the set of numbers from 1 to n that are not relatively prime to n

Let Ai be the set of numbers that are a multiple of piS = A1 [ A2 [ hellip [ An

|A1 Aring A2 Aring hellip Aring Ak| = np1p2hellippk

When r=3 (only 3 distinct factors)

|A1 [ A2 [ A3|

= np1 + np2 + np3

- np1p2 ndash np1p3 ndash np2p3 + np1p2p3

|A1 [ A2 [ A3| = |A1| + |A2| + |A3|

ndash |A1 Aring A2| ndash |A1 Aring A3| ndash |A2 Aring A3|

+ |A1 Aring A2 Aring A3|

= n(1-p1)(1-p2)(1-p3)

Euler Function

Given a number n how many numbers from 1 to n are relatively prime to n

Let

Let S be the set of numbers from 1 to n that are not relatively prime to n

Let Ai be the set of numbers that are a multiple of piS = A1 [ A2 [ hellip [ An

|A1 Aring A2 Aring hellip Aring Ak| = np1p2hellippk

sum of sizes of all single setsndash sum of sizes of all 2-set intersections+ sum of sizes of all 3-set intersectionsndash sum of sizes of all 4-set intersectionshellip+ (ndash1)n+1 times sum of sizes of intersections of

n sets

|A1 [ A2 [ A3 [ hellip [ An|

|S| = |A1 [ A2 [ hellip [ An|

= n(1-p1)(1-p2)hellip(1-pn)

calculationshellip

  • Modular Arithmetic
  • PowerPoint Presentation
  • Slide 3
  • Slide 4
  • Slide 5
  • Slide 6
  • Slide 7
  • Slide 8
  • Slide 9
  • Slide 10
  • Slide 11
  • Slide 12
  • Slide 13
  • Slide 14
  • Slide 15
  • Slide 16
  • Slide 17
  • Slide 18
  • Slide 19
  • Slide 20
  • Slide 21
  • Slide 22
  • Slide 23
  • Slide 24
  • Slide 25
  • Slide 26
  • Slide 27
  • Slide 28
  • Slide 29
  • Slide 30
  • Slide 31
  • Slide 32
  • Slide 33
  • Slide 34
  • Slide 35
  • Slide 36
  • Slide 37
  • Slide 38
  • Slide 39
  • Slide 40
  • Slide 41
  • Slide 42
  • Slide 43
  • Slide 44
  • Slide 45
Page 27: Modular Arithmetic

Cancellation

There is no general cancellation in modular arithmetic

Note that (mod n) is very similar to =

If a b (mod n) then a+c b+c (mod n)

If a b (mod n) then ac bc (mod n)

However if ac bc (mod n)

it is not necessarily true that a b (mod n)

For example 4middot2 1middot2 (mod 6) but 4 1 (mod 6)

3middot4 1middot4 (mod 8) but 3 1 (mod 8)

4middot3 1middot3 (mod 9) but 4 1 (mod 9)

Observation In all the above examples c and n have a common factor

Cancellation

Claim Assume gcd(kn) = 1 If imiddotk jmiddotk (mod n) then i j (mod n)

For example multiplicative inverse always exists if n is a prime

Proof Since gcd(kn) = 1 there exists krsquo such that kkrsquo 1 (mod n)

imiddotk jmiddotk (mod n)

=gt imiddotkmiddotkrsquo jmiddotkmiddotkrsquo (mod n)

=gt i j (mod n)

Remarks (Optional) This makes arithmetic modulo prime a field

a structure that ldquobehaves likerdquo real numbers

Arithmetic modulo prime is very useful in coding theory

In particular when p is a prime amp k not a multiple of p then gcd(kp)=1

If i j (mod p) then imiddotk jmiddotk (mod p)

Therefore

k mod p 2k mod p hellip (p-1)k mod p

are all different numbers

Fermatrsquos Little Theorem

Claim 1 Assume gcd(kn) = 1 If imiddotk jmiddotk (mod n) then i j (mod n)

Claim 2 Assume gcd(kn) = 1 If i j (mod n) then imiddotk jmiddotk (mod n)

For example when p=7 and k=3

3 mod 7 = 3 2middot3 mod 7 = 6 3middot3 mod 7 = 2 4middot3 mod 7 = 5 5middot3 mod 7 = 1 6middot3 mod 7 = 4

Notice that in the above example every number from 1 to 6 appears exactly once

In particular when p is a prime amp k not a multiple of p then gcd(kp)=1

If i j (mod p) then imiddotk jmiddotk (mod p)

Therefore

k mod p 2k mod p hellip (p-1)k mod p

are all different numbers

Fermatrsquos Little Theorem

Each of ik mod p cannot be equal to 0 because p is a prime number

Let ci = ik mod p

So 1 lt= c1 lt= p-1 1 lt= c2 lt= p-1 hellip 1lt = cp-1 lt= p-1

By the above we know that c1c2hellipcp-2cp-1 are all different

So for each i from 1 to p-1 there is exactly one cj such that cj = i

Therefore we have

(k mod p)middot(2k mod p)middothellipmiddot((p-1)k mod p) = c1middotc2middothellipmiddotcp-2middotcp-1 = 1middot2middot3hellipmiddot(p-2)middot(p-1)

Fermatrsquos Little Theorem

1 kp-1 (mod p)

Theorem If p is prime amp k not a multiple of p

For example when p=5 k=4 we have kp-1 mod p = 44 mod 5 = 1

ldquoProofrdquo

4middot3middot2middot1 [(4 mod 5) (2middot4 mod 5) (3middot4 mod 5) (4middot4 mod 5)] (mod 5)

[4 middot (2middot4) middot (3middot4) middot (4middot4)] (mod 5)

[44 middot (1middot2middot3middot4)] (mod 5)

Since gcd(1middot2middot3middot4 5)=1 we can cancel 1middot2middot3middot4 on both sides

This implies

1 44 (mod 5)

By the previous slide or direct calculation

Fermatrsquos Little Theorem

1 kp-1 (mod p)

Theorem If p is prime amp k not a multiple of p

Proof

1middot2middotmiddotmiddot(p-1) (k mod p middot 2k mod pmiddothellipmiddot(p-1)k mod p) mod p

(kmiddot2k middotmiddotmiddot (p-1)k) mod p

(kp-1)middot1middot2 middotmiddotmiddot (p-1) (mod p)

So by cancelling 1middot2 middotmiddotmiddot (p-1) on both sides applying

Claim 1

(we can cancel them because gcd(1middot2 middotmiddotmiddot (p-1) p)=1)

we have

1 kp-1 (mod p)

By 2 slides before

By the multiplication rule

Wilsonrsquos Theorem

Theorem p is a prime if and only if

(p-1) -1 (mod p)

First we consider the easy direction

If p is not a prime assume p gt= 5 (for p=4 3 2 (mod 4) )

Then p=qr for some 2 lt= q lt p and 2 lt= r lt p

If q ne r then both q and r appear in (p-1)

and so (p-1) 0 (mod p)

If q = r then p = q2 gt 2q (since we assume p gt 5 and thus q gt 2)

then both q and 2q are in (p-1)

and so again (p-1) 0 (mod p)

Wilsonrsquos Theorem

Theorem p is a prime if and only if

(p-1) -1 (mod p)

To prove the more interesting direction first we need a lemma

Lemma If p is a prime number

x2 1 (mod p) if and only if x 1 (mod p) or x -1 (mod p)

Proof x2 1 (mod p)

iff p | x2 - 1

iff p | (x ndash 1)(x + 1)

iff p | (x ndash 1) or p | (x+1)

iff x 1 (mod p) or x -1 (mod p)

Lemma p prime and p|amiddotb iff p|a or p|b

Wilsonrsquos Theorem

Theorem p is a prime if and only if

(p-1) -1 (mod p)

Letrsquos get the proof idea by considering a concrete example

10

1middot2middot3middot4middot5middot6middot7middot8middot9middot10 mod 11

1middot10middot(2middot6)middot(3middot4)middot(5middot9)middot(7middot8) mod 11

1middot-1middot(1)middot(1)middot(1)middot(1) mod 11

-1 mod 11

Besides 1 and 10 the remaining numbers are paired up into multiplicative inverse

Wilsonrsquos Theorem

Theorem p is a prime if and only if

(p-1) -1 (mod p)

ProofSince p is a prime every number from 1 to p-1 has a multiplicative inverse

By the Lemma every number 2 lt= k lt= p-2 has an inverse krsquo with knekrsquo

Since p is odd the numbers from 2 to p-2 can be grouped into pairs

(a1b1)(a2b2)hellip(a(p-3)2b(p-3)2) so that aibi 1 (mod p)

Therefore (p-1) 1middot(p-1)middot2middot3middotmiddotmiddotmiddotmiddot(p-3)middot(p-2) (mod p)

1middot(p-1)middot(a1b1)middot(a2b2)middotmiddotmiddotmiddotmiddot(a(p-3)2b(p-3)2) (mod p)

1middot(-1)middot(1)middot(1)middotmiddotmiddotmiddotmiddot(1) (mod p)

-1 (mod p)

This Lecture

bull Eulerrsquos phi function

Inclusion-Exclusion (n sets)

What is the inclusion-exclusion formula for the union of n sets

sum of sizes of all single setsndash sum of sizes of all 2-set intersections+ sum of sizes of all 3-set intersectionsndash sum of sizes of all 4-set intersectionshellip+ (ndash1)n+1 times sum of sizes of intersections of all n

sets

1 2 nA A A

1

12 1

( 1)n

ki

S nk i S

S k

A

Inclusion-Exclusion (n sets)

Inclusion-Exclusion (n sets)

sum of sizes of all single setsndash sum of sizes of all 2-set intersections+ sum of sizes of all 3-set intersectionsndash sum of sizes of all 4-set intersectionshellip+ (ndash1)n+1 times sum of sizes of intersections of all n

sets

|A1 [ A2 [ A3 [ hellip [ An|

We want to show that every element is counted exactly once

Consider an element which belongs to exactly k sets say A1 A2 A3 hellip Ak

In the formula such an element is counted the following number of times

Therefore each element is counted exactly once and thus the formula is correct

Euler Function

Given a number n how many numbers from 1 to n are relatively prime to n

When n is a prime number

then every number from 1 to n-1 is relatively prime to n

and so

When n is a prime power

then p 2p 3p 4p hellip n are not relatively prime to n

there are np = pc-1 of them

and other numbers are relatively prime to n

Therefore

Euler Function

Given a number n how many numbers from 1 to n are relatively prime to n

Suppose

Then p 2p 3p 4p hellip n are not relatively prime to n there are np of them

Also q 2q 3q 4q hellip n are not relatively prime to n and there are nq of them

Other numbers are relatively prime to n

Therefore

The numbers pq 2pq 3pq hellip n are subtracted twice and there are npq of them

So the correct answer is

Euler Function

Given a number n how many numbers from 1 to n are relatively prime to n

Let

Let S be the set of numbers from 1 to n that are not relatively prime to n

Let Ai be the set of numbers that are a multiple of piS = A1 [ A2 [ hellip [ An

For the intersection of k sets say A1 A2 A3hellip Ak

then every number in A1 Aring A2 Aring hellip Aring Ak is a multiple of p1p2hellippk

then |A1 Aring A2 Aring hellip Aring Ak| = np1p2hellippk

Euler Function

Given a number n how many numbers from 1 to n are relatively prime to n

Let

Let S be the set of numbers from 1 to n that are not relatively prime to n

Let Ai be the set of numbers that are a multiple of piS = A1 [ A2 [ hellip [ An

|A1 Aring A2 Aring hellip Aring Ak| = np1p2hellippk

When r=3 (only 3 distinct factors)

|A1 [ A2 [ A3|

= np1 + np2 + np3

- np1p2 ndash np1p3 ndash np2p3 + np1p2p3

|A1 [ A2 [ A3| = |A1| + |A2| + |A3|

ndash |A1 Aring A2| ndash |A1 Aring A3| ndash |A2 Aring A3|

+ |A1 Aring A2 Aring A3|

= n(1-p1)(1-p2)(1-p3)

Euler Function

Given a number n how many numbers from 1 to n are relatively prime to n

Let

Let S be the set of numbers from 1 to n that are not relatively prime to n

Let Ai be the set of numbers that are a multiple of piS = A1 [ A2 [ hellip [ An

|A1 Aring A2 Aring hellip Aring Ak| = np1p2hellippk

sum of sizes of all single setsndash sum of sizes of all 2-set intersections+ sum of sizes of all 3-set intersectionsndash sum of sizes of all 4-set intersectionshellip+ (ndash1)n+1 times sum of sizes of intersections of

n sets

|A1 [ A2 [ A3 [ hellip [ An|

|S| = |A1 [ A2 [ hellip [ An|

= n(1-p1)(1-p2)hellip(1-pn)

calculationshellip

  • Modular Arithmetic
  • PowerPoint Presentation
  • Slide 3
  • Slide 4
  • Slide 5
  • Slide 6
  • Slide 7
  • Slide 8
  • Slide 9
  • Slide 10
  • Slide 11
  • Slide 12
  • Slide 13
  • Slide 14
  • Slide 15
  • Slide 16
  • Slide 17
  • Slide 18
  • Slide 19
  • Slide 20
  • Slide 21
  • Slide 22
  • Slide 23
  • Slide 24
  • Slide 25
  • Slide 26
  • Slide 27
  • Slide 28
  • Slide 29
  • Slide 30
  • Slide 31
  • Slide 32
  • Slide 33
  • Slide 34
  • Slide 35
  • Slide 36
  • Slide 37
  • Slide 38
  • Slide 39
  • Slide 40
  • Slide 41
  • Slide 42
  • Slide 43
  • Slide 44
  • Slide 45
Page 28: Modular Arithmetic

Cancellation

Claim Assume gcd(kn) = 1 If imiddotk jmiddotk (mod n) then i j (mod n)

For example multiplicative inverse always exists if n is a prime

Proof Since gcd(kn) = 1 there exists krsquo such that kkrsquo 1 (mod n)

imiddotk jmiddotk (mod n)

=gt imiddotkmiddotkrsquo jmiddotkmiddotkrsquo (mod n)

=gt i j (mod n)

Remarks (Optional) This makes arithmetic modulo prime a field

a structure that ldquobehaves likerdquo real numbers

Arithmetic modulo prime is very useful in coding theory

In particular when p is a prime amp k not a multiple of p then gcd(kp)=1

If i j (mod p) then imiddotk jmiddotk (mod p)

Therefore

k mod p 2k mod p hellip (p-1)k mod p

are all different numbers

Fermatrsquos Little Theorem

Claim 1 Assume gcd(kn) = 1 If imiddotk jmiddotk (mod n) then i j (mod n)

Claim 2 Assume gcd(kn) = 1 If i j (mod n) then imiddotk jmiddotk (mod n)

For example when p=7 and k=3

3 mod 7 = 3 2middot3 mod 7 = 6 3middot3 mod 7 = 2 4middot3 mod 7 = 5 5middot3 mod 7 = 1 6middot3 mod 7 = 4

Notice that in the above example every number from 1 to 6 appears exactly once

In particular when p is a prime amp k not a multiple of p then gcd(kp)=1

If i j (mod p) then imiddotk jmiddotk (mod p)

Therefore

k mod p 2k mod p hellip (p-1)k mod p

are all different numbers

Fermatrsquos Little Theorem

Each of ik mod p cannot be equal to 0 because p is a prime number

Let ci = ik mod p

So 1 lt= c1 lt= p-1 1 lt= c2 lt= p-1 hellip 1lt = cp-1 lt= p-1

By the above we know that c1c2hellipcp-2cp-1 are all different

So for each i from 1 to p-1 there is exactly one cj such that cj = i

Therefore we have

(k mod p)middot(2k mod p)middothellipmiddot((p-1)k mod p) = c1middotc2middothellipmiddotcp-2middotcp-1 = 1middot2middot3hellipmiddot(p-2)middot(p-1)

Fermatrsquos Little Theorem

1 kp-1 (mod p)

Theorem If p is prime amp k not a multiple of p

For example when p=5 k=4 we have kp-1 mod p = 44 mod 5 = 1

ldquoProofrdquo

4middot3middot2middot1 [(4 mod 5) (2middot4 mod 5) (3middot4 mod 5) (4middot4 mod 5)] (mod 5)

[4 middot (2middot4) middot (3middot4) middot (4middot4)] (mod 5)

[44 middot (1middot2middot3middot4)] (mod 5)

Since gcd(1middot2middot3middot4 5)=1 we can cancel 1middot2middot3middot4 on both sides

This implies

1 44 (mod 5)

By the previous slide or direct calculation

Fermatrsquos Little Theorem

1 kp-1 (mod p)

Theorem If p is prime amp k not a multiple of p

Proof

1middot2middotmiddotmiddot(p-1) (k mod p middot 2k mod pmiddothellipmiddot(p-1)k mod p) mod p

(kmiddot2k middotmiddotmiddot (p-1)k) mod p

(kp-1)middot1middot2 middotmiddotmiddot (p-1) (mod p)

So by cancelling 1middot2 middotmiddotmiddot (p-1) on both sides applying

Claim 1

(we can cancel them because gcd(1middot2 middotmiddotmiddot (p-1) p)=1)

we have

1 kp-1 (mod p)

By 2 slides before

By the multiplication rule

Wilsonrsquos Theorem

Theorem p is a prime if and only if

(p-1) -1 (mod p)

First we consider the easy direction

If p is not a prime assume p gt= 5 (for p=4 3 2 (mod 4) )

Then p=qr for some 2 lt= q lt p and 2 lt= r lt p

If q ne r then both q and r appear in (p-1)

and so (p-1) 0 (mod p)

If q = r then p = q2 gt 2q (since we assume p gt 5 and thus q gt 2)

then both q and 2q are in (p-1)

and so again (p-1) 0 (mod p)

Wilsonrsquos Theorem

Theorem p is a prime if and only if

(p-1) -1 (mod p)

To prove the more interesting direction first we need a lemma

Lemma If p is a prime number

x2 1 (mod p) if and only if x 1 (mod p) or x -1 (mod p)

Proof x2 1 (mod p)

iff p | x2 - 1

iff p | (x ndash 1)(x + 1)

iff p | (x ndash 1) or p | (x+1)

iff x 1 (mod p) or x -1 (mod p)

Lemma p prime and p|amiddotb iff p|a or p|b

Wilsonrsquos Theorem

Theorem p is a prime if and only if

(p-1) -1 (mod p)

Letrsquos get the proof idea by considering a concrete example

10

1middot2middot3middot4middot5middot6middot7middot8middot9middot10 mod 11

1middot10middot(2middot6)middot(3middot4)middot(5middot9)middot(7middot8) mod 11

1middot-1middot(1)middot(1)middot(1)middot(1) mod 11

-1 mod 11

Besides 1 and 10 the remaining numbers are paired up into multiplicative inverse

Wilsonrsquos Theorem

Theorem p is a prime if and only if

(p-1) -1 (mod p)

ProofSince p is a prime every number from 1 to p-1 has a multiplicative inverse

By the Lemma every number 2 lt= k lt= p-2 has an inverse krsquo with knekrsquo

Since p is odd the numbers from 2 to p-2 can be grouped into pairs

(a1b1)(a2b2)hellip(a(p-3)2b(p-3)2) so that aibi 1 (mod p)

Therefore (p-1) 1middot(p-1)middot2middot3middotmiddotmiddotmiddotmiddot(p-3)middot(p-2) (mod p)

1middot(p-1)middot(a1b1)middot(a2b2)middotmiddotmiddotmiddotmiddot(a(p-3)2b(p-3)2) (mod p)

1middot(-1)middot(1)middot(1)middotmiddotmiddotmiddotmiddot(1) (mod p)

-1 (mod p)

This Lecture

bull Eulerrsquos phi function

Inclusion-Exclusion (n sets)

What is the inclusion-exclusion formula for the union of n sets

sum of sizes of all single setsndash sum of sizes of all 2-set intersections+ sum of sizes of all 3-set intersectionsndash sum of sizes of all 4-set intersectionshellip+ (ndash1)n+1 times sum of sizes of intersections of all n

sets

1 2 nA A A

1

12 1

( 1)n

ki

S nk i S

S k

A

Inclusion-Exclusion (n sets)

Inclusion-Exclusion (n sets)

sum of sizes of all single setsndash sum of sizes of all 2-set intersections+ sum of sizes of all 3-set intersectionsndash sum of sizes of all 4-set intersectionshellip+ (ndash1)n+1 times sum of sizes of intersections of all n

sets

|A1 [ A2 [ A3 [ hellip [ An|

We want to show that every element is counted exactly once

Consider an element which belongs to exactly k sets say A1 A2 A3 hellip Ak

In the formula such an element is counted the following number of times

Therefore each element is counted exactly once and thus the formula is correct

Euler Function

Given a number n how many numbers from 1 to n are relatively prime to n

When n is a prime number

then every number from 1 to n-1 is relatively prime to n

and so

When n is a prime power

then p 2p 3p 4p hellip n are not relatively prime to n

there are np = pc-1 of them

and other numbers are relatively prime to n

Therefore

Euler Function

Given a number n how many numbers from 1 to n are relatively prime to n

Suppose

Then p 2p 3p 4p hellip n are not relatively prime to n there are np of them

Also q 2q 3q 4q hellip n are not relatively prime to n and there are nq of them

Other numbers are relatively prime to n

Therefore

The numbers pq 2pq 3pq hellip n are subtracted twice and there are npq of them

So the correct answer is

Euler Function

Given a number n how many numbers from 1 to n are relatively prime to n

Let

Let S be the set of numbers from 1 to n that are not relatively prime to n

Let Ai be the set of numbers that are a multiple of piS = A1 [ A2 [ hellip [ An

For the intersection of k sets say A1 A2 A3hellip Ak

then every number in A1 Aring A2 Aring hellip Aring Ak is a multiple of p1p2hellippk

then |A1 Aring A2 Aring hellip Aring Ak| = np1p2hellippk

Euler Function

Given a number n how many numbers from 1 to n are relatively prime to n

Let

Let S be the set of numbers from 1 to n that are not relatively prime to n

Let Ai be the set of numbers that are a multiple of piS = A1 [ A2 [ hellip [ An

|A1 Aring A2 Aring hellip Aring Ak| = np1p2hellippk

When r=3 (only 3 distinct factors)

|A1 [ A2 [ A3|

= np1 + np2 + np3

- np1p2 ndash np1p3 ndash np2p3 + np1p2p3

|A1 [ A2 [ A3| = |A1| + |A2| + |A3|

ndash |A1 Aring A2| ndash |A1 Aring A3| ndash |A2 Aring A3|

+ |A1 Aring A2 Aring A3|

= n(1-p1)(1-p2)(1-p3)

Euler Function

Given a number n how many numbers from 1 to n are relatively prime to n

Let

Let S be the set of numbers from 1 to n that are not relatively prime to n

Let Ai be the set of numbers that are a multiple of piS = A1 [ A2 [ hellip [ An

|A1 Aring A2 Aring hellip Aring Ak| = np1p2hellippk

sum of sizes of all single setsndash sum of sizes of all 2-set intersections+ sum of sizes of all 3-set intersectionsndash sum of sizes of all 4-set intersectionshellip+ (ndash1)n+1 times sum of sizes of intersections of

n sets

|A1 [ A2 [ A3 [ hellip [ An|

|S| = |A1 [ A2 [ hellip [ An|

= n(1-p1)(1-p2)hellip(1-pn)

calculationshellip

  • Modular Arithmetic
  • PowerPoint Presentation
  • Slide 3
  • Slide 4
  • Slide 5
  • Slide 6
  • Slide 7
  • Slide 8
  • Slide 9
  • Slide 10
  • Slide 11
  • Slide 12
  • Slide 13
  • Slide 14
  • Slide 15
  • Slide 16
  • Slide 17
  • Slide 18
  • Slide 19
  • Slide 20
  • Slide 21
  • Slide 22
  • Slide 23
  • Slide 24
  • Slide 25
  • Slide 26
  • Slide 27
  • Slide 28
  • Slide 29
  • Slide 30
  • Slide 31
  • Slide 32
  • Slide 33
  • Slide 34
  • Slide 35
  • Slide 36
  • Slide 37
  • Slide 38
  • Slide 39
  • Slide 40
  • Slide 41
  • Slide 42
  • Slide 43
  • Slide 44
  • Slide 45
Page 29: Modular Arithmetic

In particular when p is a prime amp k not a multiple of p then gcd(kp)=1

If i j (mod p) then imiddotk jmiddotk (mod p)

Therefore

k mod p 2k mod p hellip (p-1)k mod p

are all different numbers

Fermatrsquos Little Theorem

Claim 1 Assume gcd(kn) = 1 If imiddotk jmiddotk (mod n) then i j (mod n)

Claim 2 Assume gcd(kn) = 1 If i j (mod n) then imiddotk jmiddotk (mod n)

For example when p=7 and k=3

3 mod 7 = 3 2middot3 mod 7 = 6 3middot3 mod 7 = 2 4middot3 mod 7 = 5 5middot3 mod 7 = 1 6middot3 mod 7 = 4

Notice that in the above example every number from 1 to 6 appears exactly once

In particular when p is a prime amp k not a multiple of p then gcd(kp)=1

If i j (mod p) then imiddotk jmiddotk (mod p)

Therefore

k mod p 2k mod p hellip (p-1)k mod p

are all different numbers

Fermatrsquos Little Theorem

Each of ik mod p cannot be equal to 0 because p is a prime number

Let ci = ik mod p

So 1 lt= c1 lt= p-1 1 lt= c2 lt= p-1 hellip 1lt = cp-1 lt= p-1

By the above we know that c1c2hellipcp-2cp-1 are all different

So for each i from 1 to p-1 there is exactly one cj such that cj = i

Therefore we have

(k mod p)middot(2k mod p)middothellipmiddot((p-1)k mod p) = c1middotc2middothellipmiddotcp-2middotcp-1 = 1middot2middot3hellipmiddot(p-2)middot(p-1)

Fermatrsquos Little Theorem

1 kp-1 (mod p)

Theorem If p is prime amp k not a multiple of p

For example when p=5 k=4 we have kp-1 mod p = 44 mod 5 = 1

ldquoProofrdquo

4middot3middot2middot1 [(4 mod 5) (2middot4 mod 5) (3middot4 mod 5) (4middot4 mod 5)] (mod 5)

[4 middot (2middot4) middot (3middot4) middot (4middot4)] (mod 5)

[44 middot (1middot2middot3middot4)] (mod 5)

Since gcd(1middot2middot3middot4 5)=1 we can cancel 1middot2middot3middot4 on both sides

This implies

1 44 (mod 5)

By the previous slide or direct calculation

Fermatrsquos Little Theorem

1 kp-1 (mod p)

Theorem If p is prime amp k not a multiple of p

Proof

1middot2middotmiddotmiddot(p-1) (k mod p middot 2k mod pmiddothellipmiddot(p-1)k mod p) mod p

(kmiddot2k middotmiddotmiddot (p-1)k) mod p

(kp-1)middot1middot2 middotmiddotmiddot (p-1) (mod p)

So by cancelling 1middot2 middotmiddotmiddot (p-1) on both sides applying

Claim 1

(we can cancel them because gcd(1middot2 middotmiddotmiddot (p-1) p)=1)

we have

1 kp-1 (mod p)

By 2 slides before

By the multiplication rule

Wilsonrsquos Theorem

Theorem p is a prime if and only if

(p-1) -1 (mod p)

First we consider the easy direction

If p is not a prime assume p gt= 5 (for p=4 3 2 (mod 4) )

Then p=qr for some 2 lt= q lt p and 2 lt= r lt p

If q ne r then both q and r appear in (p-1)

and so (p-1) 0 (mod p)

If q = r then p = q2 gt 2q (since we assume p gt 5 and thus q gt 2)

then both q and 2q are in (p-1)

and so again (p-1) 0 (mod p)

Wilsonrsquos Theorem

Theorem p is a prime if and only if

(p-1) -1 (mod p)

To prove the more interesting direction first we need a lemma

Lemma If p is a prime number

x2 1 (mod p) if and only if x 1 (mod p) or x -1 (mod p)

Proof x2 1 (mod p)

iff p | x2 - 1

iff p | (x ndash 1)(x + 1)

iff p | (x ndash 1) or p | (x+1)

iff x 1 (mod p) or x -1 (mod p)

Lemma p prime and p|amiddotb iff p|a or p|b

Wilsonrsquos Theorem

Theorem p is a prime if and only if

(p-1) -1 (mod p)

Letrsquos get the proof idea by considering a concrete example

10

1middot2middot3middot4middot5middot6middot7middot8middot9middot10 mod 11

1middot10middot(2middot6)middot(3middot4)middot(5middot9)middot(7middot8) mod 11

1middot-1middot(1)middot(1)middot(1)middot(1) mod 11

-1 mod 11

Besides 1 and 10 the remaining numbers are paired up into multiplicative inverse

Wilsonrsquos Theorem

Theorem p is a prime if and only if

(p-1) -1 (mod p)

ProofSince p is a prime every number from 1 to p-1 has a multiplicative inverse

By the Lemma every number 2 lt= k lt= p-2 has an inverse krsquo with knekrsquo

Since p is odd the numbers from 2 to p-2 can be grouped into pairs

(a1b1)(a2b2)hellip(a(p-3)2b(p-3)2) so that aibi 1 (mod p)

Therefore (p-1) 1middot(p-1)middot2middot3middotmiddotmiddotmiddotmiddot(p-3)middot(p-2) (mod p)

1middot(p-1)middot(a1b1)middot(a2b2)middotmiddotmiddotmiddotmiddot(a(p-3)2b(p-3)2) (mod p)

1middot(-1)middot(1)middot(1)middotmiddotmiddotmiddotmiddot(1) (mod p)

-1 (mod p)

This Lecture

bull Eulerrsquos phi function

Inclusion-Exclusion (n sets)

What is the inclusion-exclusion formula for the union of n sets

sum of sizes of all single setsndash sum of sizes of all 2-set intersections+ sum of sizes of all 3-set intersectionsndash sum of sizes of all 4-set intersectionshellip+ (ndash1)n+1 times sum of sizes of intersections of all n

sets

1 2 nA A A

1

12 1

( 1)n

ki

S nk i S

S k

A

Inclusion-Exclusion (n sets)

Inclusion-Exclusion (n sets)

sum of sizes of all single setsndash sum of sizes of all 2-set intersections+ sum of sizes of all 3-set intersectionsndash sum of sizes of all 4-set intersectionshellip+ (ndash1)n+1 times sum of sizes of intersections of all n

sets

|A1 [ A2 [ A3 [ hellip [ An|

We want to show that every element is counted exactly once

Consider an element which belongs to exactly k sets say A1 A2 A3 hellip Ak

In the formula such an element is counted the following number of times

Therefore each element is counted exactly once and thus the formula is correct

Euler Function

Given a number n how many numbers from 1 to n are relatively prime to n

When n is a prime number

then every number from 1 to n-1 is relatively prime to n

and so

When n is a prime power

then p 2p 3p 4p hellip n are not relatively prime to n

there are np = pc-1 of them

and other numbers are relatively prime to n

Therefore

Euler Function

Given a number n how many numbers from 1 to n are relatively prime to n

Suppose

Then p 2p 3p 4p hellip n are not relatively prime to n there are np of them

Also q 2q 3q 4q hellip n are not relatively prime to n and there are nq of them

Other numbers are relatively prime to n

Therefore

The numbers pq 2pq 3pq hellip n are subtracted twice and there are npq of them

So the correct answer is

Euler Function

Given a number n how many numbers from 1 to n are relatively prime to n

Let

Let S be the set of numbers from 1 to n that are not relatively prime to n

Let Ai be the set of numbers that are a multiple of piS = A1 [ A2 [ hellip [ An

For the intersection of k sets say A1 A2 A3hellip Ak

then every number in A1 Aring A2 Aring hellip Aring Ak is a multiple of p1p2hellippk

then |A1 Aring A2 Aring hellip Aring Ak| = np1p2hellippk

Euler Function

Given a number n how many numbers from 1 to n are relatively prime to n

Let

Let S be the set of numbers from 1 to n that are not relatively prime to n

Let Ai be the set of numbers that are a multiple of piS = A1 [ A2 [ hellip [ An

|A1 Aring A2 Aring hellip Aring Ak| = np1p2hellippk

When r=3 (only 3 distinct factors)

|A1 [ A2 [ A3|

= np1 + np2 + np3

- np1p2 ndash np1p3 ndash np2p3 + np1p2p3

|A1 [ A2 [ A3| = |A1| + |A2| + |A3|

ndash |A1 Aring A2| ndash |A1 Aring A3| ndash |A2 Aring A3|

+ |A1 Aring A2 Aring A3|

= n(1-p1)(1-p2)(1-p3)

Euler Function

Given a number n how many numbers from 1 to n are relatively prime to n

Let

Let S be the set of numbers from 1 to n that are not relatively prime to n

Let Ai be the set of numbers that are a multiple of piS = A1 [ A2 [ hellip [ An

|A1 Aring A2 Aring hellip Aring Ak| = np1p2hellippk

sum of sizes of all single setsndash sum of sizes of all 2-set intersections+ sum of sizes of all 3-set intersectionsndash sum of sizes of all 4-set intersectionshellip+ (ndash1)n+1 times sum of sizes of intersections of

n sets

|A1 [ A2 [ A3 [ hellip [ An|

|S| = |A1 [ A2 [ hellip [ An|

= n(1-p1)(1-p2)hellip(1-pn)

calculationshellip

  • Modular Arithmetic
  • PowerPoint Presentation
  • Slide 3
  • Slide 4
  • Slide 5
  • Slide 6
  • Slide 7
  • Slide 8
  • Slide 9
  • Slide 10
  • Slide 11
  • Slide 12
  • Slide 13
  • Slide 14
  • Slide 15
  • Slide 16
  • Slide 17
  • Slide 18
  • Slide 19
  • Slide 20
  • Slide 21
  • Slide 22
  • Slide 23
  • Slide 24
  • Slide 25
  • Slide 26
  • Slide 27
  • Slide 28
  • Slide 29
  • Slide 30
  • Slide 31
  • Slide 32
  • Slide 33
  • Slide 34
  • Slide 35
  • Slide 36
  • Slide 37
  • Slide 38
  • Slide 39
  • Slide 40
  • Slide 41
  • Slide 42
  • Slide 43
  • Slide 44
  • Slide 45
Page 30: Modular Arithmetic

In particular when p is a prime amp k not a multiple of p then gcd(kp)=1

If i j (mod p) then imiddotk jmiddotk (mod p)

Therefore

k mod p 2k mod p hellip (p-1)k mod p

are all different numbers

Fermatrsquos Little Theorem

Each of ik mod p cannot be equal to 0 because p is a prime number

Let ci = ik mod p

So 1 lt= c1 lt= p-1 1 lt= c2 lt= p-1 hellip 1lt = cp-1 lt= p-1

By the above we know that c1c2hellipcp-2cp-1 are all different

So for each i from 1 to p-1 there is exactly one cj such that cj = i

Therefore we have

(k mod p)middot(2k mod p)middothellipmiddot((p-1)k mod p) = c1middotc2middothellipmiddotcp-2middotcp-1 = 1middot2middot3hellipmiddot(p-2)middot(p-1)

Fermatrsquos Little Theorem

1 kp-1 (mod p)

Theorem If p is prime amp k not a multiple of p

For example when p=5 k=4 we have kp-1 mod p = 44 mod 5 = 1

ldquoProofrdquo

4middot3middot2middot1 [(4 mod 5) (2middot4 mod 5) (3middot4 mod 5) (4middot4 mod 5)] (mod 5)

[4 middot (2middot4) middot (3middot4) middot (4middot4)] (mod 5)

[44 middot (1middot2middot3middot4)] (mod 5)

Since gcd(1middot2middot3middot4 5)=1 we can cancel 1middot2middot3middot4 on both sides

This implies

1 44 (mod 5)

By the previous slide or direct calculation

Fermatrsquos Little Theorem

1 kp-1 (mod p)

Theorem If p is prime amp k not a multiple of p

Proof

1middot2middotmiddotmiddot(p-1) (k mod p middot 2k mod pmiddothellipmiddot(p-1)k mod p) mod p

(kmiddot2k middotmiddotmiddot (p-1)k) mod p

(kp-1)middot1middot2 middotmiddotmiddot (p-1) (mod p)

So by cancelling 1middot2 middotmiddotmiddot (p-1) on both sides applying

Claim 1

(we can cancel them because gcd(1middot2 middotmiddotmiddot (p-1) p)=1)

we have

1 kp-1 (mod p)

By 2 slides before

By the multiplication rule

Wilsonrsquos Theorem

Theorem p is a prime if and only if

(p-1) -1 (mod p)

First we consider the easy direction

If p is not a prime assume p gt= 5 (for p=4 3 2 (mod 4) )

Then p=qr for some 2 lt= q lt p and 2 lt= r lt p

If q ne r then both q and r appear in (p-1)

and so (p-1) 0 (mod p)

If q = r then p = q2 gt 2q (since we assume p gt 5 and thus q gt 2)

then both q and 2q are in (p-1)

and so again (p-1) 0 (mod p)

Wilsonrsquos Theorem

Theorem p is a prime if and only if

(p-1) -1 (mod p)

To prove the more interesting direction first we need a lemma

Lemma If p is a prime number

x2 1 (mod p) if and only if x 1 (mod p) or x -1 (mod p)

Proof x2 1 (mod p)

iff p | x2 - 1

iff p | (x ndash 1)(x + 1)

iff p | (x ndash 1) or p | (x+1)

iff x 1 (mod p) or x -1 (mod p)

Lemma p prime and p|amiddotb iff p|a or p|b

Wilsonrsquos Theorem

Theorem p is a prime if and only if

(p-1) -1 (mod p)

Letrsquos get the proof idea by considering a concrete example

10

1middot2middot3middot4middot5middot6middot7middot8middot9middot10 mod 11

1middot10middot(2middot6)middot(3middot4)middot(5middot9)middot(7middot8) mod 11

1middot-1middot(1)middot(1)middot(1)middot(1) mod 11

-1 mod 11

Besides 1 and 10 the remaining numbers are paired up into multiplicative inverse

Wilsonrsquos Theorem

Theorem p is a prime if and only if

(p-1) -1 (mod p)

ProofSince p is a prime every number from 1 to p-1 has a multiplicative inverse

By the Lemma every number 2 lt= k lt= p-2 has an inverse krsquo with knekrsquo

Since p is odd the numbers from 2 to p-2 can be grouped into pairs

(a1b1)(a2b2)hellip(a(p-3)2b(p-3)2) so that aibi 1 (mod p)

Therefore (p-1) 1middot(p-1)middot2middot3middotmiddotmiddotmiddotmiddot(p-3)middot(p-2) (mod p)

1middot(p-1)middot(a1b1)middot(a2b2)middotmiddotmiddotmiddotmiddot(a(p-3)2b(p-3)2) (mod p)

1middot(-1)middot(1)middot(1)middotmiddotmiddotmiddotmiddot(1) (mod p)

-1 (mod p)

This Lecture

bull Eulerrsquos phi function

Inclusion-Exclusion (n sets)

What is the inclusion-exclusion formula for the union of n sets

sum of sizes of all single setsndash sum of sizes of all 2-set intersections+ sum of sizes of all 3-set intersectionsndash sum of sizes of all 4-set intersectionshellip+ (ndash1)n+1 times sum of sizes of intersections of all n

sets

1 2 nA A A

1

12 1

( 1)n

ki

S nk i S

S k

A

Inclusion-Exclusion (n sets)

Inclusion-Exclusion (n sets)

sum of sizes of all single setsndash sum of sizes of all 2-set intersections+ sum of sizes of all 3-set intersectionsndash sum of sizes of all 4-set intersectionshellip+ (ndash1)n+1 times sum of sizes of intersections of all n

sets

|A1 [ A2 [ A3 [ hellip [ An|

We want to show that every element is counted exactly once

Consider an element which belongs to exactly k sets say A1 A2 A3 hellip Ak

In the formula such an element is counted the following number of times

Therefore each element is counted exactly once and thus the formula is correct

Euler Function

Given a number n how many numbers from 1 to n are relatively prime to n

When n is a prime number

then every number from 1 to n-1 is relatively prime to n

and so

When n is a prime power

then p 2p 3p 4p hellip n are not relatively prime to n

there are np = pc-1 of them

and other numbers are relatively prime to n

Therefore

Euler Function

Given a number n how many numbers from 1 to n are relatively prime to n

Suppose

Then p 2p 3p 4p hellip n are not relatively prime to n there are np of them

Also q 2q 3q 4q hellip n are not relatively prime to n and there are nq of them

Other numbers are relatively prime to n

Therefore

The numbers pq 2pq 3pq hellip n are subtracted twice and there are npq of them

So the correct answer is

Euler Function

Given a number n how many numbers from 1 to n are relatively prime to n

Let

Let S be the set of numbers from 1 to n that are not relatively prime to n

Let Ai be the set of numbers that are a multiple of piS = A1 [ A2 [ hellip [ An

For the intersection of k sets say A1 A2 A3hellip Ak

then every number in A1 Aring A2 Aring hellip Aring Ak is a multiple of p1p2hellippk

then |A1 Aring A2 Aring hellip Aring Ak| = np1p2hellippk

Euler Function

Given a number n how many numbers from 1 to n are relatively prime to n

Let

Let S be the set of numbers from 1 to n that are not relatively prime to n

Let Ai be the set of numbers that are a multiple of piS = A1 [ A2 [ hellip [ An

|A1 Aring A2 Aring hellip Aring Ak| = np1p2hellippk

When r=3 (only 3 distinct factors)

|A1 [ A2 [ A3|

= np1 + np2 + np3

- np1p2 ndash np1p3 ndash np2p3 + np1p2p3

|A1 [ A2 [ A3| = |A1| + |A2| + |A3|

ndash |A1 Aring A2| ndash |A1 Aring A3| ndash |A2 Aring A3|

+ |A1 Aring A2 Aring A3|

= n(1-p1)(1-p2)(1-p3)

Euler Function

Given a number n how many numbers from 1 to n are relatively prime to n

Let

Let S be the set of numbers from 1 to n that are not relatively prime to n

Let Ai be the set of numbers that are a multiple of piS = A1 [ A2 [ hellip [ An

|A1 Aring A2 Aring hellip Aring Ak| = np1p2hellippk

sum of sizes of all single setsndash sum of sizes of all 2-set intersections+ sum of sizes of all 3-set intersectionsndash sum of sizes of all 4-set intersectionshellip+ (ndash1)n+1 times sum of sizes of intersections of

n sets

|A1 [ A2 [ A3 [ hellip [ An|

|S| = |A1 [ A2 [ hellip [ An|

= n(1-p1)(1-p2)hellip(1-pn)

calculationshellip

  • Modular Arithmetic
  • PowerPoint Presentation
  • Slide 3
  • Slide 4
  • Slide 5
  • Slide 6
  • Slide 7
  • Slide 8
  • Slide 9
  • Slide 10
  • Slide 11
  • Slide 12
  • Slide 13
  • Slide 14
  • Slide 15
  • Slide 16
  • Slide 17
  • Slide 18
  • Slide 19
  • Slide 20
  • Slide 21
  • Slide 22
  • Slide 23
  • Slide 24
  • Slide 25
  • Slide 26
  • Slide 27
  • Slide 28
  • Slide 29
  • Slide 30
  • Slide 31
  • Slide 32
  • Slide 33
  • Slide 34
  • Slide 35
  • Slide 36
  • Slide 37
  • Slide 38
  • Slide 39
  • Slide 40
  • Slide 41
  • Slide 42
  • Slide 43
  • Slide 44
  • Slide 45
Page 31: Modular Arithmetic

Fermatrsquos Little Theorem

1 kp-1 (mod p)

Theorem If p is prime amp k not a multiple of p

For example when p=5 k=4 we have kp-1 mod p = 44 mod 5 = 1

ldquoProofrdquo

4middot3middot2middot1 [(4 mod 5) (2middot4 mod 5) (3middot4 mod 5) (4middot4 mod 5)] (mod 5)

[4 middot (2middot4) middot (3middot4) middot (4middot4)] (mod 5)

[44 middot (1middot2middot3middot4)] (mod 5)

Since gcd(1middot2middot3middot4 5)=1 we can cancel 1middot2middot3middot4 on both sides

This implies

1 44 (mod 5)

By the previous slide or direct calculation

Fermatrsquos Little Theorem

1 kp-1 (mod p)

Theorem If p is prime amp k not a multiple of p

Proof

1middot2middotmiddotmiddot(p-1) (k mod p middot 2k mod pmiddothellipmiddot(p-1)k mod p) mod p

(kmiddot2k middotmiddotmiddot (p-1)k) mod p

(kp-1)middot1middot2 middotmiddotmiddot (p-1) (mod p)

So by cancelling 1middot2 middotmiddotmiddot (p-1) on both sides applying

Claim 1

(we can cancel them because gcd(1middot2 middotmiddotmiddot (p-1) p)=1)

we have

1 kp-1 (mod p)

By 2 slides before

By the multiplication rule

Wilsonrsquos Theorem

Theorem p is a prime if and only if

(p-1) -1 (mod p)

First we consider the easy direction

If p is not a prime assume p gt= 5 (for p=4 3 2 (mod 4) )

Then p=qr for some 2 lt= q lt p and 2 lt= r lt p

If q ne r then both q and r appear in (p-1)

and so (p-1) 0 (mod p)

If q = r then p = q2 gt 2q (since we assume p gt 5 and thus q gt 2)

then both q and 2q are in (p-1)

and so again (p-1) 0 (mod p)

Wilsonrsquos Theorem

Theorem p is a prime if and only if

(p-1) -1 (mod p)

To prove the more interesting direction first we need a lemma

Lemma If p is a prime number

x2 1 (mod p) if and only if x 1 (mod p) or x -1 (mod p)

Proof x2 1 (mod p)

iff p | x2 - 1

iff p | (x ndash 1)(x + 1)

iff p | (x ndash 1) or p | (x+1)

iff x 1 (mod p) or x -1 (mod p)

Lemma p prime and p|amiddotb iff p|a or p|b

Wilsonrsquos Theorem

Theorem p is a prime if and only if

(p-1) -1 (mod p)

Letrsquos get the proof idea by considering a concrete example

10

1middot2middot3middot4middot5middot6middot7middot8middot9middot10 mod 11

1middot10middot(2middot6)middot(3middot4)middot(5middot9)middot(7middot8) mod 11

1middot-1middot(1)middot(1)middot(1)middot(1) mod 11

-1 mod 11

Besides 1 and 10 the remaining numbers are paired up into multiplicative inverse

Wilsonrsquos Theorem

Theorem p is a prime if and only if

(p-1) -1 (mod p)

ProofSince p is a prime every number from 1 to p-1 has a multiplicative inverse

By the Lemma every number 2 lt= k lt= p-2 has an inverse krsquo with knekrsquo

Since p is odd the numbers from 2 to p-2 can be grouped into pairs

(a1b1)(a2b2)hellip(a(p-3)2b(p-3)2) so that aibi 1 (mod p)

Therefore (p-1) 1middot(p-1)middot2middot3middotmiddotmiddotmiddotmiddot(p-3)middot(p-2) (mod p)

1middot(p-1)middot(a1b1)middot(a2b2)middotmiddotmiddotmiddotmiddot(a(p-3)2b(p-3)2) (mod p)

1middot(-1)middot(1)middot(1)middotmiddotmiddotmiddotmiddot(1) (mod p)

-1 (mod p)

This Lecture

bull Eulerrsquos phi function

Inclusion-Exclusion (n sets)

What is the inclusion-exclusion formula for the union of n sets

sum of sizes of all single setsndash sum of sizes of all 2-set intersections+ sum of sizes of all 3-set intersectionsndash sum of sizes of all 4-set intersectionshellip+ (ndash1)n+1 times sum of sizes of intersections of all n

sets

1 2 nA A A

1

12 1

( 1)n

ki

S nk i S

S k

A

Inclusion-Exclusion (n sets)

Inclusion-Exclusion (n sets)

sum of sizes of all single setsndash sum of sizes of all 2-set intersections+ sum of sizes of all 3-set intersectionsndash sum of sizes of all 4-set intersectionshellip+ (ndash1)n+1 times sum of sizes of intersections of all n

sets

|A1 [ A2 [ A3 [ hellip [ An|

We want to show that every element is counted exactly once

Consider an element which belongs to exactly k sets say A1 A2 A3 hellip Ak

In the formula such an element is counted the following number of times

Therefore each element is counted exactly once and thus the formula is correct

Euler Function

Given a number n how many numbers from 1 to n are relatively prime to n

When n is a prime number

then every number from 1 to n-1 is relatively prime to n

and so

When n is a prime power

then p 2p 3p 4p hellip n are not relatively prime to n

there are np = pc-1 of them

and other numbers are relatively prime to n

Therefore

Euler Function

Given a number n how many numbers from 1 to n are relatively prime to n

Suppose

Then p 2p 3p 4p hellip n are not relatively prime to n there are np of them

Also q 2q 3q 4q hellip n are not relatively prime to n and there are nq of them

Other numbers are relatively prime to n

Therefore

The numbers pq 2pq 3pq hellip n are subtracted twice and there are npq of them

So the correct answer is

Euler Function

Given a number n how many numbers from 1 to n are relatively prime to n

Let

Let S be the set of numbers from 1 to n that are not relatively prime to n

Let Ai be the set of numbers that are a multiple of piS = A1 [ A2 [ hellip [ An

For the intersection of k sets say A1 A2 A3hellip Ak

then every number in A1 Aring A2 Aring hellip Aring Ak is a multiple of p1p2hellippk

then |A1 Aring A2 Aring hellip Aring Ak| = np1p2hellippk

Euler Function

Given a number n how many numbers from 1 to n are relatively prime to n

Let

Let S be the set of numbers from 1 to n that are not relatively prime to n

Let Ai be the set of numbers that are a multiple of piS = A1 [ A2 [ hellip [ An

|A1 Aring A2 Aring hellip Aring Ak| = np1p2hellippk

When r=3 (only 3 distinct factors)

|A1 [ A2 [ A3|

= np1 + np2 + np3

- np1p2 ndash np1p3 ndash np2p3 + np1p2p3

|A1 [ A2 [ A3| = |A1| + |A2| + |A3|

ndash |A1 Aring A2| ndash |A1 Aring A3| ndash |A2 Aring A3|

+ |A1 Aring A2 Aring A3|

= n(1-p1)(1-p2)(1-p3)

Euler Function

Given a number n how many numbers from 1 to n are relatively prime to n

Let

Let S be the set of numbers from 1 to n that are not relatively prime to n

Let Ai be the set of numbers that are a multiple of piS = A1 [ A2 [ hellip [ An

|A1 Aring A2 Aring hellip Aring Ak| = np1p2hellippk

sum of sizes of all single setsndash sum of sizes of all 2-set intersections+ sum of sizes of all 3-set intersectionsndash sum of sizes of all 4-set intersectionshellip+ (ndash1)n+1 times sum of sizes of intersections of

n sets

|A1 [ A2 [ A3 [ hellip [ An|

|S| = |A1 [ A2 [ hellip [ An|

= n(1-p1)(1-p2)hellip(1-pn)

calculationshellip

  • Modular Arithmetic
  • PowerPoint Presentation
  • Slide 3
  • Slide 4
  • Slide 5
  • Slide 6
  • Slide 7
  • Slide 8
  • Slide 9
  • Slide 10
  • Slide 11
  • Slide 12
  • Slide 13
  • Slide 14
  • Slide 15
  • Slide 16
  • Slide 17
  • Slide 18
  • Slide 19
  • Slide 20
  • Slide 21
  • Slide 22
  • Slide 23
  • Slide 24
  • Slide 25
  • Slide 26
  • Slide 27
  • Slide 28
  • Slide 29
  • Slide 30
  • Slide 31
  • Slide 32
  • Slide 33
  • Slide 34
  • Slide 35
  • Slide 36
  • Slide 37
  • Slide 38
  • Slide 39
  • Slide 40
  • Slide 41
  • Slide 42
  • Slide 43
  • Slide 44
  • Slide 45
Page 32: Modular Arithmetic

Fermatrsquos Little Theorem

1 kp-1 (mod p)

Theorem If p is prime amp k not a multiple of p

Proof

1middot2middotmiddotmiddot(p-1) (k mod p middot 2k mod pmiddothellipmiddot(p-1)k mod p) mod p

(kmiddot2k middotmiddotmiddot (p-1)k) mod p

(kp-1)middot1middot2 middotmiddotmiddot (p-1) (mod p)

So by cancelling 1middot2 middotmiddotmiddot (p-1) on both sides applying

Claim 1

(we can cancel them because gcd(1middot2 middotmiddotmiddot (p-1) p)=1)

we have

1 kp-1 (mod p)

By 2 slides before

By the multiplication rule

Wilsonrsquos Theorem

Theorem p is a prime if and only if

(p-1) -1 (mod p)

First we consider the easy direction

If p is not a prime assume p gt= 5 (for p=4 3 2 (mod 4) )

Then p=qr for some 2 lt= q lt p and 2 lt= r lt p

If q ne r then both q and r appear in (p-1)

and so (p-1) 0 (mod p)

If q = r then p = q2 gt 2q (since we assume p gt 5 and thus q gt 2)

then both q and 2q are in (p-1)

and so again (p-1) 0 (mod p)

Wilsonrsquos Theorem

Theorem p is a prime if and only if

(p-1) -1 (mod p)

To prove the more interesting direction first we need a lemma

Lemma If p is a prime number

x2 1 (mod p) if and only if x 1 (mod p) or x -1 (mod p)

Proof x2 1 (mod p)

iff p | x2 - 1

iff p | (x ndash 1)(x + 1)

iff p | (x ndash 1) or p | (x+1)

iff x 1 (mod p) or x -1 (mod p)

Lemma p prime and p|amiddotb iff p|a or p|b

Wilsonrsquos Theorem

Theorem p is a prime if and only if

(p-1) -1 (mod p)

Letrsquos get the proof idea by considering a concrete example

10

1middot2middot3middot4middot5middot6middot7middot8middot9middot10 mod 11

1middot10middot(2middot6)middot(3middot4)middot(5middot9)middot(7middot8) mod 11

1middot-1middot(1)middot(1)middot(1)middot(1) mod 11

-1 mod 11

Besides 1 and 10 the remaining numbers are paired up into multiplicative inverse

Wilsonrsquos Theorem

Theorem p is a prime if and only if

(p-1) -1 (mod p)

ProofSince p is a prime every number from 1 to p-1 has a multiplicative inverse

By the Lemma every number 2 lt= k lt= p-2 has an inverse krsquo with knekrsquo

Since p is odd the numbers from 2 to p-2 can be grouped into pairs

(a1b1)(a2b2)hellip(a(p-3)2b(p-3)2) so that aibi 1 (mod p)

Therefore (p-1) 1middot(p-1)middot2middot3middotmiddotmiddotmiddotmiddot(p-3)middot(p-2) (mod p)

1middot(p-1)middot(a1b1)middot(a2b2)middotmiddotmiddotmiddotmiddot(a(p-3)2b(p-3)2) (mod p)

1middot(-1)middot(1)middot(1)middotmiddotmiddotmiddotmiddot(1) (mod p)

-1 (mod p)

This Lecture

bull Eulerrsquos phi function

Inclusion-Exclusion (n sets)

What is the inclusion-exclusion formula for the union of n sets

sum of sizes of all single setsndash sum of sizes of all 2-set intersections+ sum of sizes of all 3-set intersectionsndash sum of sizes of all 4-set intersectionshellip+ (ndash1)n+1 times sum of sizes of intersections of all n

sets

1 2 nA A A

1

12 1

( 1)n

ki

S nk i S

S k

A

Inclusion-Exclusion (n sets)

Inclusion-Exclusion (n sets)

sum of sizes of all single setsndash sum of sizes of all 2-set intersections+ sum of sizes of all 3-set intersectionsndash sum of sizes of all 4-set intersectionshellip+ (ndash1)n+1 times sum of sizes of intersections of all n

sets

|A1 [ A2 [ A3 [ hellip [ An|

We want to show that every element is counted exactly once

Consider an element which belongs to exactly k sets say A1 A2 A3 hellip Ak

In the formula such an element is counted the following number of times

Therefore each element is counted exactly once and thus the formula is correct

Euler Function

Given a number n how many numbers from 1 to n are relatively prime to n

When n is a prime number

then every number from 1 to n-1 is relatively prime to n

and so

When n is a prime power

then p 2p 3p 4p hellip n are not relatively prime to n

there are np = pc-1 of them

and other numbers are relatively prime to n

Therefore

Euler Function

Given a number n how many numbers from 1 to n are relatively prime to n

Suppose

Then p 2p 3p 4p hellip n are not relatively prime to n there are np of them

Also q 2q 3q 4q hellip n are not relatively prime to n and there are nq of them

Other numbers are relatively prime to n

Therefore

The numbers pq 2pq 3pq hellip n are subtracted twice and there are npq of them

So the correct answer is

Euler Function

Given a number n how many numbers from 1 to n are relatively prime to n

Let

Let S be the set of numbers from 1 to n that are not relatively prime to n

Let Ai be the set of numbers that are a multiple of piS = A1 [ A2 [ hellip [ An

For the intersection of k sets say A1 A2 A3hellip Ak

then every number in A1 Aring A2 Aring hellip Aring Ak is a multiple of p1p2hellippk

then |A1 Aring A2 Aring hellip Aring Ak| = np1p2hellippk

Euler Function

Given a number n how many numbers from 1 to n are relatively prime to n

Let

Let S be the set of numbers from 1 to n that are not relatively prime to n

Let Ai be the set of numbers that are a multiple of piS = A1 [ A2 [ hellip [ An

|A1 Aring A2 Aring hellip Aring Ak| = np1p2hellippk

When r=3 (only 3 distinct factors)

|A1 [ A2 [ A3|

= np1 + np2 + np3

- np1p2 ndash np1p3 ndash np2p3 + np1p2p3

|A1 [ A2 [ A3| = |A1| + |A2| + |A3|

ndash |A1 Aring A2| ndash |A1 Aring A3| ndash |A2 Aring A3|

+ |A1 Aring A2 Aring A3|

= n(1-p1)(1-p2)(1-p3)

Euler Function

Given a number n how many numbers from 1 to n are relatively prime to n

Let

Let S be the set of numbers from 1 to n that are not relatively prime to n

Let Ai be the set of numbers that are a multiple of piS = A1 [ A2 [ hellip [ An

|A1 Aring A2 Aring hellip Aring Ak| = np1p2hellippk

sum of sizes of all single setsndash sum of sizes of all 2-set intersections+ sum of sizes of all 3-set intersectionsndash sum of sizes of all 4-set intersectionshellip+ (ndash1)n+1 times sum of sizes of intersections of

n sets

|A1 [ A2 [ A3 [ hellip [ An|

|S| = |A1 [ A2 [ hellip [ An|

= n(1-p1)(1-p2)hellip(1-pn)

calculationshellip

  • Modular Arithmetic
  • PowerPoint Presentation
  • Slide 3
  • Slide 4
  • Slide 5
  • Slide 6
  • Slide 7
  • Slide 8
  • Slide 9
  • Slide 10
  • Slide 11
  • Slide 12
  • Slide 13
  • Slide 14
  • Slide 15
  • Slide 16
  • Slide 17
  • Slide 18
  • Slide 19
  • Slide 20
  • Slide 21
  • Slide 22
  • Slide 23
  • Slide 24
  • Slide 25
  • Slide 26
  • Slide 27
  • Slide 28
  • Slide 29
  • Slide 30
  • Slide 31
  • Slide 32
  • Slide 33
  • Slide 34
  • Slide 35
  • Slide 36
  • Slide 37
  • Slide 38
  • Slide 39
  • Slide 40
  • Slide 41
  • Slide 42
  • Slide 43
  • Slide 44
  • Slide 45
Page 33: Modular Arithmetic

Wilsonrsquos Theorem

Theorem p is a prime if and only if

(p-1) -1 (mod p)

First we consider the easy direction

If p is not a prime assume p gt= 5 (for p=4 3 2 (mod 4) )

Then p=qr for some 2 lt= q lt p and 2 lt= r lt p

If q ne r then both q and r appear in (p-1)

and so (p-1) 0 (mod p)

If q = r then p = q2 gt 2q (since we assume p gt 5 and thus q gt 2)

then both q and 2q are in (p-1)

and so again (p-1) 0 (mod p)

Wilsonrsquos Theorem

Theorem p is a prime if and only if

(p-1) -1 (mod p)

To prove the more interesting direction first we need a lemma

Lemma If p is a prime number

x2 1 (mod p) if and only if x 1 (mod p) or x -1 (mod p)

Proof x2 1 (mod p)

iff p | x2 - 1

iff p | (x ndash 1)(x + 1)

iff p | (x ndash 1) or p | (x+1)

iff x 1 (mod p) or x -1 (mod p)

Lemma p prime and p|amiddotb iff p|a or p|b

Wilsonrsquos Theorem

Theorem p is a prime if and only if

(p-1) -1 (mod p)

Letrsquos get the proof idea by considering a concrete example

10

1middot2middot3middot4middot5middot6middot7middot8middot9middot10 mod 11

1middot10middot(2middot6)middot(3middot4)middot(5middot9)middot(7middot8) mod 11

1middot-1middot(1)middot(1)middot(1)middot(1) mod 11

-1 mod 11

Besides 1 and 10 the remaining numbers are paired up into multiplicative inverse

Wilsonrsquos Theorem

Theorem p is a prime if and only if

(p-1) -1 (mod p)

ProofSince p is a prime every number from 1 to p-1 has a multiplicative inverse

By the Lemma every number 2 lt= k lt= p-2 has an inverse krsquo with knekrsquo

Since p is odd the numbers from 2 to p-2 can be grouped into pairs

(a1b1)(a2b2)hellip(a(p-3)2b(p-3)2) so that aibi 1 (mod p)

Therefore (p-1) 1middot(p-1)middot2middot3middotmiddotmiddotmiddotmiddot(p-3)middot(p-2) (mod p)

1middot(p-1)middot(a1b1)middot(a2b2)middotmiddotmiddotmiddotmiddot(a(p-3)2b(p-3)2) (mod p)

1middot(-1)middot(1)middot(1)middotmiddotmiddotmiddotmiddot(1) (mod p)

-1 (mod p)

This Lecture

bull Eulerrsquos phi function

Inclusion-Exclusion (n sets)

What is the inclusion-exclusion formula for the union of n sets

sum of sizes of all single setsndash sum of sizes of all 2-set intersections+ sum of sizes of all 3-set intersectionsndash sum of sizes of all 4-set intersectionshellip+ (ndash1)n+1 times sum of sizes of intersections of all n

sets

1 2 nA A A

1

12 1

( 1)n

ki

S nk i S

S k

A

Inclusion-Exclusion (n sets)

Inclusion-Exclusion (n sets)

sum of sizes of all single setsndash sum of sizes of all 2-set intersections+ sum of sizes of all 3-set intersectionsndash sum of sizes of all 4-set intersectionshellip+ (ndash1)n+1 times sum of sizes of intersections of all n

sets

|A1 [ A2 [ A3 [ hellip [ An|

We want to show that every element is counted exactly once

Consider an element which belongs to exactly k sets say A1 A2 A3 hellip Ak

In the formula such an element is counted the following number of times

Therefore each element is counted exactly once and thus the formula is correct

Euler Function

Given a number n how many numbers from 1 to n are relatively prime to n

When n is a prime number

then every number from 1 to n-1 is relatively prime to n

and so

When n is a prime power

then p 2p 3p 4p hellip n are not relatively prime to n

there are np = pc-1 of them

and other numbers are relatively prime to n

Therefore

Euler Function

Given a number n how many numbers from 1 to n are relatively prime to n

Suppose

Then p 2p 3p 4p hellip n are not relatively prime to n there are np of them

Also q 2q 3q 4q hellip n are not relatively prime to n and there are nq of them

Other numbers are relatively prime to n

Therefore

The numbers pq 2pq 3pq hellip n are subtracted twice and there are npq of them

So the correct answer is

Euler Function

Given a number n how many numbers from 1 to n are relatively prime to n

Let

Let S be the set of numbers from 1 to n that are not relatively prime to n

Let Ai be the set of numbers that are a multiple of piS = A1 [ A2 [ hellip [ An

For the intersection of k sets say A1 A2 A3hellip Ak

then every number in A1 Aring A2 Aring hellip Aring Ak is a multiple of p1p2hellippk

then |A1 Aring A2 Aring hellip Aring Ak| = np1p2hellippk

Euler Function

Given a number n how many numbers from 1 to n are relatively prime to n

Let

Let S be the set of numbers from 1 to n that are not relatively prime to n

Let Ai be the set of numbers that are a multiple of piS = A1 [ A2 [ hellip [ An

|A1 Aring A2 Aring hellip Aring Ak| = np1p2hellippk

When r=3 (only 3 distinct factors)

|A1 [ A2 [ A3|

= np1 + np2 + np3

- np1p2 ndash np1p3 ndash np2p3 + np1p2p3

|A1 [ A2 [ A3| = |A1| + |A2| + |A3|

ndash |A1 Aring A2| ndash |A1 Aring A3| ndash |A2 Aring A3|

+ |A1 Aring A2 Aring A3|

= n(1-p1)(1-p2)(1-p3)

Euler Function

Given a number n how many numbers from 1 to n are relatively prime to n

Let

Let S be the set of numbers from 1 to n that are not relatively prime to n

Let Ai be the set of numbers that are a multiple of piS = A1 [ A2 [ hellip [ An

|A1 Aring A2 Aring hellip Aring Ak| = np1p2hellippk

sum of sizes of all single setsndash sum of sizes of all 2-set intersections+ sum of sizes of all 3-set intersectionsndash sum of sizes of all 4-set intersectionshellip+ (ndash1)n+1 times sum of sizes of intersections of

n sets

|A1 [ A2 [ A3 [ hellip [ An|

|S| = |A1 [ A2 [ hellip [ An|

= n(1-p1)(1-p2)hellip(1-pn)

calculationshellip

  • Modular Arithmetic
  • PowerPoint Presentation
  • Slide 3
  • Slide 4
  • Slide 5
  • Slide 6
  • Slide 7
  • Slide 8
  • Slide 9
  • Slide 10
  • Slide 11
  • Slide 12
  • Slide 13
  • Slide 14
  • Slide 15
  • Slide 16
  • Slide 17
  • Slide 18
  • Slide 19
  • Slide 20
  • Slide 21
  • Slide 22
  • Slide 23
  • Slide 24
  • Slide 25
  • Slide 26
  • Slide 27
  • Slide 28
  • Slide 29
  • Slide 30
  • Slide 31
  • Slide 32
  • Slide 33
  • Slide 34
  • Slide 35
  • Slide 36
  • Slide 37
  • Slide 38
  • Slide 39
  • Slide 40
  • Slide 41
  • Slide 42
  • Slide 43
  • Slide 44
  • Slide 45
Page 34: Modular Arithmetic

Wilsonrsquos Theorem

Theorem p is a prime if and only if

(p-1) -1 (mod p)

To prove the more interesting direction first we need a lemma

Lemma If p is a prime number

x2 1 (mod p) if and only if x 1 (mod p) or x -1 (mod p)

Proof x2 1 (mod p)

iff p | x2 - 1

iff p | (x ndash 1)(x + 1)

iff p | (x ndash 1) or p | (x+1)

iff x 1 (mod p) or x -1 (mod p)

Lemma p prime and p|amiddotb iff p|a or p|b

Wilsonrsquos Theorem

Theorem p is a prime if and only if

(p-1) -1 (mod p)

Letrsquos get the proof idea by considering a concrete example

10

1middot2middot3middot4middot5middot6middot7middot8middot9middot10 mod 11

1middot10middot(2middot6)middot(3middot4)middot(5middot9)middot(7middot8) mod 11

1middot-1middot(1)middot(1)middot(1)middot(1) mod 11

-1 mod 11

Besides 1 and 10 the remaining numbers are paired up into multiplicative inverse

Wilsonrsquos Theorem

Theorem p is a prime if and only if

(p-1) -1 (mod p)

ProofSince p is a prime every number from 1 to p-1 has a multiplicative inverse

By the Lemma every number 2 lt= k lt= p-2 has an inverse krsquo with knekrsquo

Since p is odd the numbers from 2 to p-2 can be grouped into pairs

(a1b1)(a2b2)hellip(a(p-3)2b(p-3)2) so that aibi 1 (mod p)

Therefore (p-1) 1middot(p-1)middot2middot3middotmiddotmiddotmiddotmiddot(p-3)middot(p-2) (mod p)

1middot(p-1)middot(a1b1)middot(a2b2)middotmiddotmiddotmiddotmiddot(a(p-3)2b(p-3)2) (mod p)

1middot(-1)middot(1)middot(1)middotmiddotmiddotmiddotmiddot(1) (mod p)

-1 (mod p)

This Lecture

bull Eulerrsquos phi function

Inclusion-Exclusion (n sets)

What is the inclusion-exclusion formula for the union of n sets

sum of sizes of all single setsndash sum of sizes of all 2-set intersections+ sum of sizes of all 3-set intersectionsndash sum of sizes of all 4-set intersectionshellip+ (ndash1)n+1 times sum of sizes of intersections of all n

sets

1 2 nA A A

1

12 1

( 1)n

ki

S nk i S

S k

A

Inclusion-Exclusion (n sets)

Inclusion-Exclusion (n sets)

sum of sizes of all single setsndash sum of sizes of all 2-set intersections+ sum of sizes of all 3-set intersectionsndash sum of sizes of all 4-set intersectionshellip+ (ndash1)n+1 times sum of sizes of intersections of all n

sets

|A1 [ A2 [ A3 [ hellip [ An|

We want to show that every element is counted exactly once

Consider an element which belongs to exactly k sets say A1 A2 A3 hellip Ak

In the formula such an element is counted the following number of times

Therefore each element is counted exactly once and thus the formula is correct

Euler Function

Given a number n how many numbers from 1 to n are relatively prime to n

When n is a prime number

then every number from 1 to n-1 is relatively prime to n

and so

When n is a prime power

then p 2p 3p 4p hellip n are not relatively prime to n

there are np = pc-1 of them

and other numbers are relatively prime to n

Therefore

Euler Function

Given a number n how many numbers from 1 to n are relatively prime to n

Suppose

Then p 2p 3p 4p hellip n are not relatively prime to n there are np of them

Also q 2q 3q 4q hellip n are not relatively prime to n and there are nq of them

Other numbers are relatively prime to n

Therefore

The numbers pq 2pq 3pq hellip n are subtracted twice and there are npq of them

So the correct answer is

Euler Function

Given a number n how many numbers from 1 to n are relatively prime to n

Let

Let S be the set of numbers from 1 to n that are not relatively prime to n

Let Ai be the set of numbers that are a multiple of piS = A1 [ A2 [ hellip [ An

For the intersection of k sets say A1 A2 A3hellip Ak

then every number in A1 Aring A2 Aring hellip Aring Ak is a multiple of p1p2hellippk

then |A1 Aring A2 Aring hellip Aring Ak| = np1p2hellippk

Euler Function

Given a number n how many numbers from 1 to n are relatively prime to n

Let

Let S be the set of numbers from 1 to n that are not relatively prime to n

Let Ai be the set of numbers that are a multiple of piS = A1 [ A2 [ hellip [ An

|A1 Aring A2 Aring hellip Aring Ak| = np1p2hellippk

When r=3 (only 3 distinct factors)

|A1 [ A2 [ A3|

= np1 + np2 + np3

- np1p2 ndash np1p3 ndash np2p3 + np1p2p3

|A1 [ A2 [ A3| = |A1| + |A2| + |A3|

ndash |A1 Aring A2| ndash |A1 Aring A3| ndash |A2 Aring A3|

+ |A1 Aring A2 Aring A3|

= n(1-p1)(1-p2)(1-p3)

Euler Function

Given a number n how many numbers from 1 to n are relatively prime to n

Let

Let S be the set of numbers from 1 to n that are not relatively prime to n

Let Ai be the set of numbers that are a multiple of piS = A1 [ A2 [ hellip [ An

|A1 Aring A2 Aring hellip Aring Ak| = np1p2hellippk

sum of sizes of all single setsndash sum of sizes of all 2-set intersections+ sum of sizes of all 3-set intersectionsndash sum of sizes of all 4-set intersectionshellip+ (ndash1)n+1 times sum of sizes of intersections of

n sets

|A1 [ A2 [ A3 [ hellip [ An|

|S| = |A1 [ A2 [ hellip [ An|

= n(1-p1)(1-p2)hellip(1-pn)

calculationshellip

  • Modular Arithmetic
  • PowerPoint Presentation
  • Slide 3
  • Slide 4
  • Slide 5
  • Slide 6
  • Slide 7
  • Slide 8
  • Slide 9
  • Slide 10
  • Slide 11
  • Slide 12
  • Slide 13
  • Slide 14
  • Slide 15
  • Slide 16
  • Slide 17
  • Slide 18
  • Slide 19
  • Slide 20
  • Slide 21
  • Slide 22
  • Slide 23
  • Slide 24
  • Slide 25
  • Slide 26
  • Slide 27
  • Slide 28
  • Slide 29
  • Slide 30
  • Slide 31
  • Slide 32
  • Slide 33
  • Slide 34
  • Slide 35
  • Slide 36
  • Slide 37
  • Slide 38
  • Slide 39
  • Slide 40
  • Slide 41
  • Slide 42
  • Slide 43
  • Slide 44
  • Slide 45
Page 35: Modular Arithmetic

Wilsonrsquos Theorem

Theorem p is a prime if and only if

(p-1) -1 (mod p)

Letrsquos get the proof idea by considering a concrete example

10

1middot2middot3middot4middot5middot6middot7middot8middot9middot10 mod 11

1middot10middot(2middot6)middot(3middot4)middot(5middot9)middot(7middot8) mod 11

1middot-1middot(1)middot(1)middot(1)middot(1) mod 11

-1 mod 11

Besides 1 and 10 the remaining numbers are paired up into multiplicative inverse

Wilsonrsquos Theorem

Theorem p is a prime if and only if

(p-1) -1 (mod p)

ProofSince p is a prime every number from 1 to p-1 has a multiplicative inverse

By the Lemma every number 2 lt= k lt= p-2 has an inverse krsquo with knekrsquo

Since p is odd the numbers from 2 to p-2 can be grouped into pairs

(a1b1)(a2b2)hellip(a(p-3)2b(p-3)2) so that aibi 1 (mod p)

Therefore (p-1) 1middot(p-1)middot2middot3middotmiddotmiddotmiddotmiddot(p-3)middot(p-2) (mod p)

1middot(p-1)middot(a1b1)middot(a2b2)middotmiddotmiddotmiddotmiddot(a(p-3)2b(p-3)2) (mod p)

1middot(-1)middot(1)middot(1)middotmiddotmiddotmiddotmiddot(1) (mod p)

-1 (mod p)

This Lecture

bull Eulerrsquos phi function

Inclusion-Exclusion (n sets)

What is the inclusion-exclusion formula for the union of n sets

sum of sizes of all single setsndash sum of sizes of all 2-set intersections+ sum of sizes of all 3-set intersectionsndash sum of sizes of all 4-set intersectionshellip+ (ndash1)n+1 times sum of sizes of intersections of all n

sets

1 2 nA A A

1

12 1

( 1)n

ki

S nk i S

S k

A

Inclusion-Exclusion (n sets)

Inclusion-Exclusion (n sets)

sum of sizes of all single setsndash sum of sizes of all 2-set intersections+ sum of sizes of all 3-set intersectionsndash sum of sizes of all 4-set intersectionshellip+ (ndash1)n+1 times sum of sizes of intersections of all n

sets

|A1 [ A2 [ A3 [ hellip [ An|

We want to show that every element is counted exactly once

Consider an element which belongs to exactly k sets say A1 A2 A3 hellip Ak

In the formula such an element is counted the following number of times

Therefore each element is counted exactly once and thus the formula is correct

Euler Function

Given a number n how many numbers from 1 to n are relatively prime to n

When n is a prime number

then every number from 1 to n-1 is relatively prime to n

and so

When n is a prime power

then p 2p 3p 4p hellip n are not relatively prime to n

there are np = pc-1 of them

and other numbers are relatively prime to n

Therefore

Euler Function

Given a number n how many numbers from 1 to n are relatively prime to n

Suppose

Then p 2p 3p 4p hellip n are not relatively prime to n there are np of them

Also q 2q 3q 4q hellip n are not relatively prime to n and there are nq of them

Other numbers are relatively prime to n

Therefore

The numbers pq 2pq 3pq hellip n are subtracted twice and there are npq of them

So the correct answer is

Euler Function

Given a number n how many numbers from 1 to n are relatively prime to n

Let

Let S be the set of numbers from 1 to n that are not relatively prime to n

Let Ai be the set of numbers that are a multiple of piS = A1 [ A2 [ hellip [ An

For the intersection of k sets say A1 A2 A3hellip Ak

then every number in A1 Aring A2 Aring hellip Aring Ak is a multiple of p1p2hellippk

then |A1 Aring A2 Aring hellip Aring Ak| = np1p2hellippk

Euler Function

Given a number n how many numbers from 1 to n are relatively prime to n

Let

Let S be the set of numbers from 1 to n that are not relatively prime to n

Let Ai be the set of numbers that are a multiple of piS = A1 [ A2 [ hellip [ An

|A1 Aring A2 Aring hellip Aring Ak| = np1p2hellippk

When r=3 (only 3 distinct factors)

|A1 [ A2 [ A3|

= np1 + np2 + np3

- np1p2 ndash np1p3 ndash np2p3 + np1p2p3

|A1 [ A2 [ A3| = |A1| + |A2| + |A3|

ndash |A1 Aring A2| ndash |A1 Aring A3| ndash |A2 Aring A3|

+ |A1 Aring A2 Aring A3|

= n(1-p1)(1-p2)(1-p3)

Euler Function

Given a number n how many numbers from 1 to n are relatively prime to n

Let

Let S be the set of numbers from 1 to n that are not relatively prime to n

Let Ai be the set of numbers that are a multiple of piS = A1 [ A2 [ hellip [ An

|A1 Aring A2 Aring hellip Aring Ak| = np1p2hellippk

sum of sizes of all single setsndash sum of sizes of all 2-set intersections+ sum of sizes of all 3-set intersectionsndash sum of sizes of all 4-set intersectionshellip+ (ndash1)n+1 times sum of sizes of intersections of

n sets

|A1 [ A2 [ A3 [ hellip [ An|

|S| = |A1 [ A2 [ hellip [ An|

= n(1-p1)(1-p2)hellip(1-pn)

calculationshellip

  • Modular Arithmetic
  • PowerPoint Presentation
  • Slide 3
  • Slide 4
  • Slide 5
  • Slide 6
  • Slide 7
  • Slide 8
  • Slide 9
  • Slide 10
  • Slide 11
  • Slide 12
  • Slide 13
  • Slide 14
  • Slide 15
  • Slide 16
  • Slide 17
  • Slide 18
  • Slide 19
  • Slide 20
  • Slide 21
  • Slide 22
  • Slide 23
  • Slide 24
  • Slide 25
  • Slide 26
  • Slide 27
  • Slide 28
  • Slide 29
  • Slide 30
  • Slide 31
  • Slide 32
  • Slide 33
  • Slide 34
  • Slide 35
  • Slide 36
  • Slide 37
  • Slide 38
  • Slide 39
  • Slide 40
  • Slide 41
  • Slide 42
  • Slide 43
  • Slide 44
  • Slide 45
Page 36: Modular Arithmetic

Wilsonrsquos Theorem

Theorem p is a prime if and only if

(p-1) -1 (mod p)

ProofSince p is a prime every number from 1 to p-1 has a multiplicative inverse

By the Lemma every number 2 lt= k lt= p-2 has an inverse krsquo with knekrsquo

Since p is odd the numbers from 2 to p-2 can be grouped into pairs

(a1b1)(a2b2)hellip(a(p-3)2b(p-3)2) so that aibi 1 (mod p)

Therefore (p-1) 1middot(p-1)middot2middot3middotmiddotmiddotmiddotmiddot(p-3)middot(p-2) (mod p)

1middot(p-1)middot(a1b1)middot(a2b2)middotmiddotmiddotmiddotmiddot(a(p-3)2b(p-3)2) (mod p)

1middot(-1)middot(1)middot(1)middotmiddotmiddotmiddotmiddot(1) (mod p)

-1 (mod p)

This Lecture

bull Eulerrsquos phi function

Inclusion-Exclusion (n sets)

What is the inclusion-exclusion formula for the union of n sets

sum of sizes of all single setsndash sum of sizes of all 2-set intersections+ sum of sizes of all 3-set intersectionsndash sum of sizes of all 4-set intersectionshellip+ (ndash1)n+1 times sum of sizes of intersections of all n

sets

1 2 nA A A

1

12 1

( 1)n

ki

S nk i S

S k

A

Inclusion-Exclusion (n sets)

Inclusion-Exclusion (n sets)

sum of sizes of all single setsndash sum of sizes of all 2-set intersections+ sum of sizes of all 3-set intersectionsndash sum of sizes of all 4-set intersectionshellip+ (ndash1)n+1 times sum of sizes of intersections of all n

sets

|A1 [ A2 [ A3 [ hellip [ An|

We want to show that every element is counted exactly once

Consider an element which belongs to exactly k sets say A1 A2 A3 hellip Ak

In the formula such an element is counted the following number of times

Therefore each element is counted exactly once and thus the formula is correct

Euler Function

Given a number n how many numbers from 1 to n are relatively prime to n

When n is a prime number

then every number from 1 to n-1 is relatively prime to n

and so

When n is a prime power

then p 2p 3p 4p hellip n are not relatively prime to n

there are np = pc-1 of them

and other numbers are relatively prime to n

Therefore

Euler Function

Given a number n how many numbers from 1 to n are relatively prime to n

Suppose

Then p 2p 3p 4p hellip n are not relatively prime to n there are np of them

Also q 2q 3q 4q hellip n are not relatively prime to n and there are nq of them

Other numbers are relatively prime to n

Therefore

The numbers pq 2pq 3pq hellip n are subtracted twice and there are npq of them

So the correct answer is

Euler Function

Given a number n how many numbers from 1 to n are relatively prime to n

Let

Let S be the set of numbers from 1 to n that are not relatively prime to n

Let Ai be the set of numbers that are a multiple of piS = A1 [ A2 [ hellip [ An

For the intersection of k sets say A1 A2 A3hellip Ak

then every number in A1 Aring A2 Aring hellip Aring Ak is a multiple of p1p2hellippk

then |A1 Aring A2 Aring hellip Aring Ak| = np1p2hellippk

Euler Function

Given a number n how many numbers from 1 to n are relatively prime to n

Let

Let S be the set of numbers from 1 to n that are not relatively prime to n

Let Ai be the set of numbers that are a multiple of piS = A1 [ A2 [ hellip [ An

|A1 Aring A2 Aring hellip Aring Ak| = np1p2hellippk

When r=3 (only 3 distinct factors)

|A1 [ A2 [ A3|

= np1 + np2 + np3

- np1p2 ndash np1p3 ndash np2p3 + np1p2p3

|A1 [ A2 [ A3| = |A1| + |A2| + |A3|

ndash |A1 Aring A2| ndash |A1 Aring A3| ndash |A2 Aring A3|

+ |A1 Aring A2 Aring A3|

= n(1-p1)(1-p2)(1-p3)

Euler Function

Given a number n how many numbers from 1 to n are relatively prime to n

Let

Let S be the set of numbers from 1 to n that are not relatively prime to n

Let Ai be the set of numbers that are a multiple of piS = A1 [ A2 [ hellip [ An

|A1 Aring A2 Aring hellip Aring Ak| = np1p2hellippk

sum of sizes of all single setsndash sum of sizes of all 2-set intersections+ sum of sizes of all 3-set intersectionsndash sum of sizes of all 4-set intersectionshellip+ (ndash1)n+1 times sum of sizes of intersections of

n sets

|A1 [ A2 [ A3 [ hellip [ An|

|S| = |A1 [ A2 [ hellip [ An|

= n(1-p1)(1-p2)hellip(1-pn)

calculationshellip

  • Modular Arithmetic
  • PowerPoint Presentation
  • Slide 3
  • Slide 4
  • Slide 5
  • Slide 6
  • Slide 7
  • Slide 8
  • Slide 9
  • Slide 10
  • Slide 11
  • Slide 12
  • Slide 13
  • Slide 14
  • Slide 15
  • Slide 16
  • Slide 17
  • Slide 18
  • Slide 19
  • Slide 20
  • Slide 21
  • Slide 22
  • Slide 23
  • Slide 24
  • Slide 25
  • Slide 26
  • Slide 27
  • Slide 28
  • Slide 29
  • Slide 30
  • Slide 31
  • Slide 32
  • Slide 33
  • Slide 34
  • Slide 35
  • Slide 36
  • Slide 37
  • Slide 38
  • Slide 39
  • Slide 40
  • Slide 41
  • Slide 42
  • Slide 43
  • Slide 44
  • Slide 45
Page 37: Modular Arithmetic

This Lecture

bull Eulerrsquos phi function

Inclusion-Exclusion (n sets)

What is the inclusion-exclusion formula for the union of n sets

sum of sizes of all single setsndash sum of sizes of all 2-set intersections+ sum of sizes of all 3-set intersectionsndash sum of sizes of all 4-set intersectionshellip+ (ndash1)n+1 times sum of sizes of intersections of all n

sets

1 2 nA A A

1

12 1

( 1)n

ki

S nk i S

S k

A

Inclusion-Exclusion (n sets)

Inclusion-Exclusion (n sets)

sum of sizes of all single setsndash sum of sizes of all 2-set intersections+ sum of sizes of all 3-set intersectionsndash sum of sizes of all 4-set intersectionshellip+ (ndash1)n+1 times sum of sizes of intersections of all n

sets

|A1 [ A2 [ A3 [ hellip [ An|

We want to show that every element is counted exactly once

Consider an element which belongs to exactly k sets say A1 A2 A3 hellip Ak

In the formula such an element is counted the following number of times

Therefore each element is counted exactly once and thus the formula is correct

Euler Function

Given a number n how many numbers from 1 to n are relatively prime to n

When n is a prime number

then every number from 1 to n-1 is relatively prime to n

and so

When n is a prime power

then p 2p 3p 4p hellip n are not relatively prime to n

there are np = pc-1 of them

and other numbers are relatively prime to n

Therefore

Euler Function

Given a number n how many numbers from 1 to n are relatively prime to n

Suppose

Then p 2p 3p 4p hellip n are not relatively prime to n there are np of them

Also q 2q 3q 4q hellip n are not relatively prime to n and there are nq of them

Other numbers are relatively prime to n

Therefore

The numbers pq 2pq 3pq hellip n are subtracted twice and there are npq of them

So the correct answer is

Euler Function

Given a number n how many numbers from 1 to n are relatively prime to n

Let

Let S be the set of numbers from 1 to n that are not relatively prime to n

Let Ai be the set of numbers that are a multiple of piS = A1 [ A2 [ hellip [ An

For the intersection of k sets say A1 A2 A3hellip Ak

then every number in A1 Aring A2 Aring hellip Aring Ak is a multiple of p1p2hellippk

then |A1 Aring A2 Aring hellip Aring Ak| = np1p2hellippk

Euler Function

Given a number n how many numbers from 1 to n are relatively prime to n

Let

Let S be the set of numbers from 1 to n that are not relatively prime to n

Let Ai be the set of numbers that are a multiple of piS = A1 [ A2 [ hellip [ An

|A1 Aring A2 Aring hellip Aring Ak| = np1p2hellippk

When r=3 (only 3 distinct factors)

|A1 [ A2 [ A3|

= np1 + np2 + np3

- np1p2 ndash np1p3 ndash np2p3 + np1p2p3

|A1 [ A2 [ A3| = |A1| + |A2| + |A3|

ndash |A1 Aring A2| ndash |A1 Aring A3| ndash |A2 Aring A3|

+ |A1 Aring A2 Aring A3|

= n(1-p1)(1-p2)(1-p3)

Euler Function

Given a number n how many numbers from 1 to n are relatively prime to n

Let

Let S be the set of numbers from 1 to n that are not relatively prime to n

Let Ai be the set of numbers that are a multiple of piS = A1 [ A2 [ hellip [ An

|A1 Aring A2 Aring hellip Aring Ak| = np1p2hellippk

sum of sizes of all single setsndash sum of sizes of all 2-set intersections+ sum of sizes of all 3-set intersectionsndash sum of sizes of all 4-set intersectionshellip+ (ndash1)n+1 times sum of sizes of intersections of

n sets

|A1 [ A2 [ A3 [ hellip [ An|

|S| = |A1 [ A2 [ hellip [ An|

= n(1-p1)(1-p2)hellip(1-pn)

calculationshellip

  • Modular Arithmetic
  • PowerPoint Presentation
  • Slide 3
  • Slide 4
  • Slide 5
  • Slide 6
  • Slide 7
  • Slide 8
  • Slide 9
  • Slide 10
  • Slide 11
  • Slide 12
  • Slide 13
  • Slide 14
  • Slide 15
  • Slide 16
  • Slide 17
  • Slide 18
  • Slide 19
  • Slide 20
  • Slide 21
  • Slide 22
  • Slide 23
  • Slide 24
  • Slide 25
  • Slide 26
  • Slide 27
  • Slide 28
  • Slide 29
  • Slide 30
  • Slide 31
  • Slide 32
  • Slide 33
  • Slide 34
  • Slide 35
  • Slide 36
  • Slide 37
  • Slide 38
  • Slide 39
  • Slide 40
  • Slide 41
  • Slide 42
  • Slide 43
  • Slide 44
  • Slide 45
Page 38: Modular Arithmetic

Inclusion-Exclusion (n sets)

What is the inclusion-exclusion formula for the union of n sets

sum of sizes of all single setsndash sum of sizes of all 2-set intersections+ sum of sizes of all 3-set intersectionsndash sum of sizes of all 4-set intersectionshellip+ (ndash1)n+1 times sum of sizes of intersections of all n

sets

1 2 nA A A

1

12 1

( 1)n

ki

S nk i S

S k

A

Inclusion-Exclusion (n sets)

Inclusion-Exclusion (n sets)

sum of sizes of all single setsndash sum of sizes of all 2-set intersections+ sum of sizes of all 3-set intersectionsndash sum of sizes of all 4-set intersectionshellip+ (ndash1)n+1 times sum of sizes of intersections of all n

sets

|A1 [ A2 [ A3 [ hellip [ An|

We want to show that every element is counted exactly once

Consider an element which belongs to exactly k sets say A1 A2 A3 hellip Ak

In the formula such an element is counted the following number of times

Therefore each element is counted exactly once and thus the formula is correct

Euler Function

Given a number n how many numbers from 1 to n are relatively prime to n

When n is a prime number

then every number from 1 to n-1 is relatively prime to n

and so

When n is a prime power

then p 2p 3p 4p hellip n are not relatively prime to n

there are np = pc-1 of them

and other numbers are relatively prime to n

Therefore

Euler Function

Given a number n how many numbers from 1 to n are relatively prime to n

Suppose

Then p 2p 3p 4p hellip n are not relatively prime to n there are np of them

Also q 2q 3q 4q hellip n are not relatively prime to n and there are nq of them

Other numbers are relatively prime to n

Therefore

The numbers pq 2pq 3pq hellip n are subtracted twice and there are npq of them

So the correct answer is

Euler Function

Given a number n how many numbers from 1 to n are relatively prime to n

Let

Let S be the set of numbers from 1 to n that are not relatively prime to n

Let Ai be the set of numbers that are a multiple of piS = A1 [ A2 [ hellip [ An

For the intersection of k sets say A1 A2 A3hellip Ak

then every number in A1 Aring A2 Aring hellip Aring Ak is a multiple of p1p2hellippk

then |A1 Aring A2 Aring hellip Aring Ak| = np1p2hellippk

Euler Function

Given a number n how many numbers from 1 to n are relatively prime to n

Let

Let S be the set of numbers from 1 to n that are not relatively prime to n

Let Ai be the set of numbers that are a multiple of piS = A1 [ A2 [ hellip [ An

|A1 Aring A2 Aring hellip Aring Ak| = np1p2hellippk

When r=3 (only 3 distinct factors)

|A1 [ A2 [ A3|

= np1 + np2 + np3

- np1p2 ndash np1p3 ndash np2p3 + np1p2p3

|A1 [ A2 [ A3| = |A1| + |A2| + |A3|

ndash |A1 Aring A2| ndash |A1 Aring A3| ndash |A2 Aring A3|

+ |A1 Aring A2 Aring A3|

= n(1-p1)(1-p2)(1-p3)

Euler Function

Given a number n how many numbers from 1 to n are relatively prime to n

Let

Let S be the set of numbers from 1 to n that are not relatively prime to n

Let Ai be the set of numbers that are a multiple of piS = A1 [ A2 [ hellip [ An

|A1 Aring A2 Aring hellip Aring Ak| = np1p2hellippk

sum of sizes of all single setsndash sum of sizes of all 2-set intersections+ sum of sizes of all 3-set intersectionsndash sum of sizes of all 4-set intersectionshellip+ (ndash1)n+1 times sum of sizes of intersections of

n sets

|A1 [ A2 [ A3 [ hellip [ An|

|S| = |A1 [ A2 [ hellip [ An|

= n(1-p1)(1-p2)hellip(1-pn)

calculationshellip

  • Modular Arithmetic
  • PowerPoint Presentation
  • Slide 3
  • Slide 4
  • Slide 5
  • Slide 6
  • Slide 7
  • Slide 8
  • Slide 9
  • Slide 10
  • Slide 11
  • Slide 12
  • Slide 13
  • Slide 14
  • Slide 15
  • Slide 16
  • Slide 17
  • Slide 18
  • Slide 19
  • Slide 20
  • Slide 21
  • Slide 22
  • Slide 23
  • Slide 24
  • Slide 25
  • Slide 26
  • Slide 27
  • Slide 28
  • Slide 29
  • Slide 30
  • Slide 31
  • Slide 32
  • Slide 33
  • Slide 34
  • Slide 35
  • Slide 36
  • Slide 37
  • Slide 38
  • Slide 39
  • Slide 40
  • Slide 41
  • Slide 42
  • Slide 43
  • Slide 44
  • Slide 45
Page 39: Modular Arithmetic

sum of sizes of all single setsndash sum of sizes of all 2-set intersections+ sum of sizes of all 3-set intersectionsndash sum of sizes of all 4-set intersectionshellip+ (ndash1)n+1 times sum of sizes of intersections of all n

sets

1 2 nA A A

1

12 1

( 1)n

ki

S nk i S

S k

A

Inclusion-Exclusion (n sets)

Inclusion-Exclusion (n sets)

sum of sizes of all single setsndash sum of sizes of all 2-set intersections+ sum of sizes of all 3-set intersectionsndash sum of sizes of all 4-set intersectionshellip+ (ndash1)n+1 times sum of sizes of intersections of all n

sets

|A1 [ A2 [ A3 [ hellip [ An|

We want to show that every element is counted exactly once

Consider an element which belongs to exactly k sets say A1 A2 A3 hellip Ak

In the formula such an element is counted the following number of times

Therefore each element is counted exactly once and thus the formula is correct

Euler Function

Given a number n how many numbers from 1 to n are relatively prime to n

When n is a prime number

then every number from 1 to n-1 is relatively prime to n

and so

When n is a prime power

then p 2p 3p 4p hellip n are not relatively prime to n

there are np = pc-1 of them

and other numbers are relatively prime to n

Therefore

Euler Function

Given a number n how many numbers from 1 to n are relatively prime to n

Suppose

Then p 2p 3p 4p hellip n are not relatively prime to n there are np of them

Also q 2q 3q 4q hellip n are not relatively prime to n and there are nq of them

Other numbers are relatively prime to n

Therefore

The numbers pq 2pq 3pq hellip n are subtracted twice and there are npq of them

So the correct answer is

Euler Function

Given a number n how many numbers from 1 to n are relatively prime to n

Let

Let S be the set of numbers from 1 to n that are not relatively prime to n

Let Ai be the set of numbers that are a multiple of piS = A1 [ A2 [ hellip [ An

For the intersection of k sets say A1 A2 A3hellip Ak

then every number in A1 Aring A2 Aring hellip Aring Ak is a multiple of p1p2hellippk

then |A1 Aring A2 Aring hellip Aring Ak| = np1p2hellippk

Euler Function

Given a number n how many numbers from 1 to n are relatively prime to n

Let

Let S be the set of numbers from 1 to n that are not relatively prime to n

Let Ai be the set of numbers that are a multiple of piS = A1 [ A2 [ hellip [ An

|A1 Aring A2 Aring hellip Aring Ak| = np1p2hellippk

When r=3 (only 3 distinct factors)

|A1 [ A2 [ A3|

= np1 + np2 + np3

- np1p2 ndash np1p3 ndash np2p3 + np1p2p3

|A1 [ A2 [ A3| = |A1| + |A2| + |A3|

ndash |A1 Aring A2| ndash |A1 Aring A3| ndash |A2 Aring A3|

+ |A1 Aring A2 Aring A3|

= n(1-p1)(1-p2)(1-p3)

Euler Function

Given a number n how many numbers from 1 to n are relatively prime to n

Let

Let S be the set of numbers from 1 to n that are not relatively prime to n

Let Ai be the set of numbers that are a multiple of piS = A1 [ A2 [ hellip [ An

|A1 Aring A2 Aring hellip Aring Ak| = np1p2hellippk

sum of sizes of all single setsndash sum of sizes of all 2-set intersections+ sum of sizes of all 3-set intersectionsndash sum of sizes of all 4-set intersectionshellip+ (ndash1)n+1 times sum of sizes of intersections of

n sets

|A1 [ A2 [ A3 [ hellip [ An|

|S| = |A1 [ A2 [ hellip [ An|

= n(1-p1)(1-p2)hellip(1-pn)

calculationshellip

  • Modular Arithmetic
  • PowerPoint Presentation
  • Slide 3
  • Slide 4
  • Slide 5
  • Slide 6
  • Slide 7
  • Slide 8
  • Slide 9
  • Slide 10
  • Slide 11
  • Slide 12
  • Slide 13
  • Slide 14
  • Slide 15
  • Slide 16
  • Slide 17
  • Slide 18
  • Slide 19
  • Slide 20
  • Slide 21
  • Slide 22
  • Slide 23
  • Slide 24
  • Slide 25
  • Slide 26
  • Slide 27
  • Slide 28
  • Slide 29
  • Slide 30
  • Slide 31
  • Slide 32
  • Slide 33
  • Slide 34
  • Slide 35
  • Slide 36
  • Slide 37
  • Slide 38
  • Slide 39
  • Slide 40
  • Slide 41
  • Slide 42
  • Slide 43
  • Slide 44
  • Slide 45
Page 40: Modular Arithmetic

Inclusion-Exclusion (n sets)

sum of sizes of all single setsndash sum of sizes of all 2-set intersections+ sum of sizes of all 3-set intersectionsndash sum of sizes of all 4-set intersectionshellip+ (ndash1)n+1 times sum of sizes of intersections of all n

sets

|A1 [ A2 [ A3 [ hellip [ An|

We want to show that every element is counted exactly once

Consider an element which belongs to exactly k sets say A1 A2 A3 hellip Ak

In the formula such an element is counted the following number of times

Therefore each element is counted exactly once and thus the formula is correct

Euler Function

Given a number n how many numbers from 1 to n are relatively prime to n

When n is a prime number

then every number from 1 to n-1 is relatively prime to n

and so

When n is a prime power

then p 2p 3p 4p hellip n are not relatively prime to n

there are np = pc-1 of them

and other numbers are relatively prime to n

Therefore

Euler Function

Given a number n how many numbers from 1 to n are relatively prime to n

Suppose

Then p 2p 3p 4p hellip n are not relatively prime to n there are np of them

Also q 2q 3q 4q hellip n are not relatively prime to n and there are nq of them

Other numbers are relatively prime to n

Therefore

The numbers pq 2pq 3pq hellip n are subtracted twice and there are npq of them

So the correct answer is

Euler Function

Given a number n how many numbers from 1 to n are relatively prime to n

Let

Let S be the set of numbers from 1 to n that are not relatively prime to n

Let Ai be the set of numbers that are a multiple of piS = A1 [ A2 [ hellip [ An

For the intersection of k sets say A1 A2 A3hellip Ak

then every number in A1 Aring A2 Aring hellip Aring Ak is a multiple of p1p2hellippk

then |A1 Aring A2 Aring hellip Aring Ak| = np1p2hellippk

Euler Function

Given a number n how many numbers from 1 to n are relatively prime to n

Let

Let S be the set of numbers from 1 to n that are not relatively prime to n

Let Ai be the set of numbers that are a multiple of piS = A1 [ A2 [ hellip [ An

|A1 Aring A2 Aring hellip Aring Ak| = np1p2hellippk

When r=3 (only 3 distinct factors)

|A1 [ A2 [ A3|

= np1 + np2 + np3

- np1p2 ndash np1p3 ndash np2p3 + np1p2p3

|A1 [ A2 [ A3| = |A1| + |A2| + |A3|

ndash |A1 Aring A2| ndash |A1 Aring A3| ndash |A2 Aring A3|

+ |A1 Aring A2 Aring A3|

= n(1-p1)(1-p2)(1-p3)

Euler Function

Given a number n how many numbers from 1 to n are relatively prime to n

Let

Let S be the set of numbers from 1 to n that are not relatively prime to n

Let Ai be the set of numbers that are a multiple of piS = A1 [ A2 [ hellip [ An

|A1 Aring A2 Aring hellip Aring Ak| = np1p2hellippk

sum of sizes of all single setsndash sum of sizes of all 2-set intersections+ sum of sizes of all 3-set intersectionsndash sum of sizes of all 4-set intersectionshellip+ (ndash1)n+1 times sum of sizes of intersections of

n sets

|A1 [ A2 [ A3 [ hellip [ An|

|S| = |A1 [ A2 [ hellip [ An|

= n(1-p1)(1-p2)hellip(1-pn)

calculationshellip

  • Modular Arithmetic
  • PowerPoint Presentation
  • Slide 3
  • Slide 4
  • Slide 5
  • Slide 6
  • Slide 7
  • Slide 8
  • Slide 9
  • Slide 10
  • Slide 11
  • Slide 12
  • Slide 13
  • Slide 14
  • Slide 15
  • Slide 16
  • Slide 17
  • Slide 18
  • Slide 19
  • Slide 20
  • Slide 21
  • Slide 22
  • Slide 23
  • Slide 24
  • Slide 25
  • Slide 26
  • Slide 27
  • Slide 28
  • Slide 29
  • Slide 30
  • Slide 31
  • Slide 32
  • Slide 33
  • Slide 34
  • Slide 35
  • Slide 36
  • Slide 37
  • Slide 38
  • Slide 39
  • Slide 40
  • Slide 41
  • Slide 42
  • Slide 43
  • Slide 44
  • Slide 45
Page 41: Modular Arithmetic

Euler Function

Given a number n how many numbers from 1 to n are relatively prime to n

When n is a prime number

then every number from 1 to n-1 is relatively prime to n

and so

When n is a prime power

then p 2p 3p 4p hellip n are not relatively prime to n

there are np = pc-1 of them

and other numbers are relatively prime to n

Therefore

Euler Function

Given a number n how many numbers from 1 to n are relatively prime to n

Suppose

Then p 2p 3p 4p hellip n are not relatively prime to n there are np of them

Also q 2q 3q 4q hellip n are not relatively prime to n and there are nq of them

Other numbers are relatively prime to n

Therefore

The numbers pq 2pq 3pq hellip n are subtracted twice and there are npq of them

So the correct answer is

Euler Function

Given a number n how many numbers from 1 to n are relatively prime to n

Let

Let S be the set of numbers from 1 to n that are not relatively prime to n

Let Ai be the set of numbers that are a multiple of piS = A1 [ A2 [ hellip [ An

For the intersection of k sets say A1 A2 A3hellip Ak

then every number in A1 Aring A2 Aring hellip Aring Ak is a multiple of p1p2hellippk

then |A1 Aring A2 Aring hellip Aring Ak| = np1p2hellippk

Euler Function

Given a number n how many numbers from 1 to n are relatively prime to n

Let

Let S be the set of numbers from 1 to n that are not relatively prime to n

Let Ai be the set of numbers that are a multiple of piS = A1 [ A2 [ hellip [ An

|A1 Aring A2 Aring hellip Aring Ak| = np1p2hellippk

When r=3 (only 3 distinct factors)

|A1 [ A2 [ A3|

= np1 + np2 + np3

- np1p2 ndash np1p3 ndash np2p3 + np1p2p3

|A1 [ A2 [ A3| = |A1| + |A2| + |A3|

ndash |A1 Aring A2| ndash |A1 Aring A3| ndash |A2 Aring A3|

+ |A1 Aring A2 Aring A3|

= n(1-p1)(1-p2)(1-p3)

Euler Function

Given a number n how many numbers from 1 to n are relatively prime to n

Let

Let S be the set of numbers from 1 to n that are not relatively prime to n

Let Ai be the set of numbers that are a multiple of piS = A1 [ A2 [ hellip [ An

|A1 Aring A2 Aring hellip Aring Ak| = np1p2hellippk

sum of sizes of all single setsndash sum of sizes of all 2-set intersections+ sum of sizes of all 3-set intersectionsndash sum of sizes of all 4-set intersectionshellip+ (ndash1)n+1 times sum of sizes of intersections of

n sets

|A1 [ A2 [ A3 [ hellip [ An|

|S| = |A1 [ A2 [ hellip [ An|

= n(1-p1)(1-p2)hellip(1-pn)

calculationshellip

  • Modular Arithmetic
  • PowerPoint Presentation
  • Slide 3
  • Slide 4
  • Slide 5
  • Slide 6
  • Slide 7
  • Slide 8
  • Slide 9
  • Slide 10
  • Slide 11
  • Slide 12
  • Slide 13
  • Slide 14
  • Slide 15
  • Slide 16
  • Slide 17
  • Slide 18
  • Slide 19
  • Slide 20
  • Slide 21
  • Slide 22
  • Slide 23
  • Slide 24
  • Slide 25
  • Slide 26
  • Slide 27
  • Slide 28
  • Slide 29
  • Slide 30
  • Slide 31
  • Slide 32
  • Slide 33
  • Slide 34
  • Slide 35
  • Slide 36
  • Slide 37
  • Slide 38
  • Slide 39
  • Slide 40
  • Slide 41
  • Slide 42
  • Slide 43
  • Slide 44
  • Slide 45
Page 42: Modular Arithmetic

Euler Function

Given a number n how many numbers from 1 to n are relatively prime to n

Suppose

Then p 2p 3p 4p hellip n are not relatively prime to n there are np of them

Also q 2q 3q 4q hellip n are not relatively prime to n and there are nq of them

Other numbers are relatively prime to n

Therefore

The numbers pq 2pq 3pq hellip n are subtracted twice and there are npq of them

So the correct answer is

Euler Function

Given a number n how many numbers from 1 to n are relatively prime to n

Let

Let S be the set of numbers from 1 to n that are not relatively prime to n

Let Ai be the set of numbers that are a multiple of piS = A1 [ A2 [ hellip [ An

For the intersection of k sets say A1 A2 A3hellip Ak

then every number in A1 Aring A2 Aring hellip Aring Ak is a multiple of p1p2hellippk

then |A1 Aring A2 Aring hellip Aring Ak| = np1p2hellippk

Euler Function

Given a number n how many numbers from 1 to n are relatively prime to n

Let

Let S be the set of numbers from 1 to n that are not relatively prime to n

Let Ai be the set of numbers that are a multiple of piS = A1 [ A2 [ hellip [ An

|A1 Aring A2 Aring hellip Aring Ak| = np1p2hellippk

When r=3 (only 3 distinct factors)

|A1 [ A2 [ A3|

= np1 + np2 + np3

- np1p2 ndash np1p3 ndash np2p3 + np1p2p3

|A1 [ A2 [ A3| = |A1| + |A2| + |A3|

ndash |A1 Aring A2| ndash |A1 Aring A3| ndash |A2 Aring A3|

+ |A1 Aring A2 Aring A3|

= n(1-p1)(1-p2)(1-p3)

Euler Function

Given a number n how many numbers from 1 to n are relatively prime to n

Let

Let S be the set of numbers from 1 to n that are not relatively prime to n

Let Ai be the set of numbers that are a multiple of piS = A1 [ A2 [ hellip [ An

|A1 Aring A2 Aring hellip Aring Ak| = np1p2hellippk

sum of sizes of all single setsndash sum of sizes of all 2-set intersections+ sum of sizes of all 3-set intersectionsndash sum of sizes of all 4-set intersectionshellip+ (ndash1)n+1 times sum of sizes of intersections of

n sets

|A1 [ A2 [ A3 [ hellip [ An|

|S| = |A1 [ A2 [ hellip [ An|

= n(1-p1)(1-p2)hellip(1-pn)

calculationshellip

  • Modular Arithmetic
  • PowerPoint Presentation
  • Slide 3
  • Slide 4
  • Slide 5
  • Slide 6
  • Slide 7
  • Slide 8
  • Slide 9
  • Slide 10
  • Slide 11
  • Slide 12
  • Slide 13
  • Slide 14
  • Slide 15
  • Slide 16
  • Slide 17
  • Slide 18
  • Slide 19
  • Slide 20
  • Slide 21
  • Slide 22
  • Slide 23
  • Slide 24
  • Slide 25
  • Slide 26
  • Slide 27
  • Slide 28
  • Slide 29
  • Slide 30
  • Slide 31
  • Slide 32
  • Slide 33
  • Slide 34
  • Slide 35
  • Slide 36
  • Slide 37
  • Slide 38
  • Slide 39
  • Slide 40
  • Slide 41
  • Slide 42
  • Slide 43
  • Slide 44
  • Slide 45
Page 43: Modular Arithmetic

Euler Function

Given a number n how many numbers from 1 to n are relatively prime to n

Let

Let S be the set of numbers from 1 to n that are not relatively prime to n

Let Ai be the set of numbers that are a multiple of piS = A1 [ A2 [ hellip [ An

For the intersection of k sets say A1 A2 A3hellip Ak

then every number in A1 Aring A2 Aring hellip Aring Ak is a multiple of p1p2hellippk

then |A1 Aring A2 Aring hellip Aring Ak| = np1p2hellippk

Euler Function

Given a number n how many numbers from 1 to n are relatively prime to n

Let

Let S be the set of numbers from 1 to n that are not relatively prime to n

Let Ai be the set of numbers that are a multiple of piS = A1 [ A2 [ hellip [ An

|A1 Aring A2 Aring hellip Aring Ak| = np1p2hellippk

When r=3 (only 3 distinct factors)

|A1 [ A2 [ A3|

= np1 + np2 + np3

- np1p2 ndash np1p3 ndash np2p3 + np1p2p3

|A1 [ A2 [ A3| = |A1| + |A2| + |A3|

ndash |A1 Aring A2| ndash |A1 Aring A3| ndash |A2 Aring A3|

+ |A1 Aring A2 Aring A3|

= n(1-p1)(1-p2)(1-p3)

Euler Function

Given a number n how many numbers from 1 to n are relatively prime to n

Let

Let S be the set of numbers from 1 to n that are not relatively prime to n

Let Ai be the set of numbers that are a multiple of piS = A1 [ A2 [ hellip [ An

|A1 Aring A2 Aring hellip Aring Ak| = np1p2hellippk

sum of sizes of all single setsndash sum of sizes of all 2-set intersections+ sum of sizes of all 3-set intersectionsndash sum of sizes of all 4-set intersectionshellip+ (ndash1)n+1 times sum of sizes of intersections of

n sets

|A1 [ A2 [ A3 [ hellip [ An|

|S| = |A1 [ A2 [ hellip [ An|

= n(1-p1)(1-p2)hellip(1-pn)

calculationshellip

  • Modular Arithmetic
  • PowerPoint Presentation
  • Slide 3
  • Slide 4
  • Slide 5
  • Slide 6
  • Slide 7
  • Slide 8
  • Slide 9
  • Slide 10
  • Slide 11
  • Slide 12
  • Slide 13
  • Slide 14
  • Slide 15
  • Slide 16
  • Slide 17
  • Slide 18
  • Slide 19
  • Slide 20
  • Slide 21
  • Slide 22
  • Slide 23
  • Slide 24
  • Slide 25
  • Slide 26
  • Slide 27
  • Slide 28
  • Slide 29
  • Slide 30
  • Slide 31
  • Slide 32
  • Slide 33
  • Slide 34
  • Slide 35
  • Slide 36
  • Slide 37
  • Slide 38
  • Slide 39
  • Slide 40
  • Slide 41
  • Slide 42
  • Slide 43
  • Slide 44
  • Slide 45
Page 44: Modular Arithmetic

Euler Function

Given a number n how many numbers from 1 to n are relatively prime to n

Let

Let S be the set of numbers from 1 to n that are not relatively prime to n

Let Ai be the set of numbers that are a multiple of piS = A1 [ A2 [ hellip [ An

|A1 Aring A2 Aring hellip Aring Ak| = np1p2hellippk

When r=3 (only 3 distinct factors)

|A1 [ A2 [ A3|

= np1 + np2 + np3

- np1p2 ndash np1p3 ndash np2p3 + np1p2p3

|A1 [ A2 [ A3| = |A1| + |A2| + |A3|

ndash |A1 Aring A2| ndash |A1 Aring A3| ndash |A2 Aring A3|

+ |A1 Aring A2 Aring A3|

= n(1-p1)(1-p2)(1-p3)

Euler Function

Given a number n how many numbers from 1 to n are relatively prime to n

Let

Let S be the set of numbers from 1 to n that are not relatively prime to n

Let Ai be the set of numbers that are a multiple of piS = A1 [ A2 [ hellip [ An

|A1 Aring A2 Aring hellip Aring Ak| = np1p2hellippk

sum of sizes of all single setsndash sum of sizes of all 2-set intersections+ sum of sizes of all 3-set intersectionsndash sum of sizes of all 4-set intersectionshellip+ (ndash1)n+1 times sum of sizes of intersections of

n sets

|A1 [ A2 [ A3 [ hellip [ An|

|S| = |A1 [ A2 [ hellip [ An|

= n(1-p1)(1-p2)hellip(1-pn)

calculationshellip

  • Modular Arithmetic
  • PowerPoint Presentation
  • Slide 3
  • Slide 4
  • Slide 5
  • Slide 6
  • Slide 7
  • Slide 8
  • Slide 9
  • Slide 10
  • Slide 11
  • Slide 12
  • Slide 13
  • Slide 14
  • Slide 15
  • Slide 16
  • Slide 17
  • Slide 18
  • Slide 19
  • Slide 20
  • Slide 21
  • Slide 22
  • Slide 23
  • Slide 24
  • Slide 25
  • Slide 26
  • Slide 27
  • Slide 28
  • Slide 29
  • Slide 30
  • Slide 31
  • Slide 32
  • Slide 33
  • Slide 34
  • Slide 35
  • Slide 36
  • Slide 37
  • Slide 38
  • Slide 39
  • Slide 40
  • Slide 41
  • Slide 42
  • Slide 43
  • Slide 44
  • Slide 45
Page 45: Modular Arithmetic

Euler Function

Given a number n how many numbers from 1 to n are relatively prime to n

Let

Let S be the set of numbers from 1 to n that are not relatively prime to n

Let Ai be the set of numbers that are a multiple of piS = A1 [ A2 [ hellip [ An

|A1 Aring A2 Aring hellip Aring Ak| = np1p2hellippk

sum of sizes of all single setsndash sum of sizes of all 2-set intersections+ sum of sizes of all 3-set intersectionsndash sum of sizes of all 4-set intersectionshellip+ (ndash1)n+1 times sum of sizes of intersections of

n sets

|A1 [ A2 [ A3 [ hellip [ An|

|S| = |A1 [ A2 [ hellip [ An|

= n(1-p1)(1-p2)hellip(1-pn)

calculationshellip

  • Modular Arithmetic
  • PowerPoint Presentation
  • Slide 3
  • Slide 4
  • Slide 5
  • Slide 6
  • Slide 7
  • Slide 8
  • Slide 9
  • Slide 10
  • Slide 11
  • Slide 12
  • Slide 13
  • Slide 14
  • Slide 15
  • Slide 16
  • Slide 17
  • Slide 18
  • Slide 19
  • Slide 20
  • Slide 21
  • Slide 22
  • Slide 23
  • Slide 24
  • Slide 25
  • Slide 26
  • Slide 27
  • Slide 28
  • Slide 29
  • Slide 30
  • Slide 31
  • Slide 32
  • Slide 33
  • Slide 34
  • Slide 35
  • Slide 36
  • Slide 37
  • Slide 38
  • Slide 39
  • Slide 40
  • Slide 41
  • Slide 42
  • Slide 43
  • Slide 44
  • Slide 45